{"question": "A 64-year-old man presents to the emergency room with a headache and nausea. He reports that he was rocking his grandson to sleep when the symptoms began. He states the pain is constant and is primarily located on his right side. When asked to indicate the area of pain, he says that it surrounds his eye and upper forehead. He had one episode of vomiting. The patient also reports difficulty seeing out of his right eye, which he attributes to excessive tearing. The patient\u2019s past medical history is significant for hypertension. His medications include hydrochlorothiazide. His temperature is 98.6\u00b0F (37\u00b0C), blood pressure is 135/91 mmHg, pulse is 72/min, and respirations are 12/min. The patient\u2019s right eye is shown in Figure A. Upon physical examination, the right pupil is minimally responsive to light and the globe feels firm. A right-sided carotid bruit is appreciated. Which of the following is the most appropriate prophylaxis for this patient\u2019s condition?", "choicesA": "Acetazolamide", "choicesB": "Amitriptyline", "choicesC": "Clopidogrel", "choicesD": "Epinephrine", "choicesE": "Verapamil", "answer_idx": "A", "answer": "Acetazolamide", "explanation": "This patient is presenting with sudden-onset unilateral vision loss and an orbitofrontal headache with a dilated pupil and a hard ocular globe suggesting a diagnosis of acute angle-closure glaucoma. Long-term management of angle-closure glaucoma can include acetazolamide.\n\nExamination of the eye in a patient with acute-closure glaucoma will reveal a red eye that is rock-hard and a mid-dilated pupil which is minimally reactive to light. The fundoscopic exam will show an increased optic cup-to-disk ratio (>0.4) and tonometry will show increased intraocular pressure. Gonioscopy is the diagnostic gold standard. Acute treatment and long-term management involve the administration of beta-blockers, alpha-2-agonists, and carbonic anhydrase inhibitors such as acetazolamide to decrease intraocular pressure. The definitive treatment is iridotomy.\n\nAiraksinen et al. review the treatment of closed-angle glaucoma. They discuss how a combination of acetazolamide and beta-blockers can terminate an attack. They recommend using 1 drop of pilocarpine 3 hours after intravenous acetazolamide administration.\n\nFigure/Illustration A is a clinical photograph showing an eye with injected conjunctiva (red circles) and a mid-dilated pupil. These findings are consistent with angle-closure glaucoma.\n\nIncorrect Answers:\nAnswer B: Amitriptyline can be used as prophylaxis for migraines. Migraines present as unilateral, pulsating headaches that may be associated with nausea or photophobia. Patients with migraines can sometimes experience an aura with visual field changes; however, they would not have exam findings of a rock-hard eye or injection.\n\nAnswer C: Clopidogrel may be used as a conservative treatment for carotid atherosclerosis, which can be an embolic source for central retinal artery occlusion (CRAO). CRAO presents as acute, painless, monocular vision loss. A fundoscopic exam will demonstrate ischemia of the retina.\n\nAnswer D: Epinephrine is contraindicated in the treatment of angle-closure glaucoma as it increases pupillary dilation. It is used in the management of open-angle glaucoma. This would present with gradually patchy loss of vision and is generally painless.\n\nAnswer E: Verapamil is used as prophylaxis for cluster headaches. Cluster headaches present as unilateral, repetitive, brief headaches associated with severe peri-orbital pain, lacrimation, rhinorrhea, and Horner syndrome (miosis, ptosis, and anhidrosis).\n\nBullet Summary:\nPharmacologic management of acute angle-closure glaucoma involves beta-blockers, alpha-2 agonists, and carbonic anhydrase inhibitors.", "link": "https://step2.medbullets.com/testview?qid=108992"} {"question": "A 42-year-old woman is enrolled in a randomized controlled trial to study cardiac function in the setting of several different drugs. She is started on verapamil and instructed to exercise at 50% of her VO2 max while several cardiac parameters are being measured. During this experiment, which of the following represents the relative conduction speed through the heart from fastest to slowest?", "choicesA": "Atria > Purkinje fibers > ventricles > AV node", "choicesB": "AV node > ventricles > atria > Purkinje fibers", "choicesC": "Purkinje fibers > ventricles > atria > AV node", "choicesD": "Purkinje fibers > atria > ventricles > AV node", "choicesE": "Purkinje fibers > AV node > ventricles > atria", "answer_idx": "D", "answer": "Purkinje fibers > atria > ventricles > AV node", "explanation": "The conduction velocity of the structures of the heart are in the following order: Purkinje fibers > atria > ventricles > AV node. A calcium channel blocker such as verapamil would only slow conduction in the AV node.\n\nThe conduction velocity of cardiac tissue is determined by a wide variety of factors, including the number and identity of various ion channels, the number and type of gap junctions, and the size and composition of muscle fibers. Conduction through the Purkinje system is the fastest within the heart, allowing for coordinated contraction of the ventricles, and atrial muscle conducts faster than ventricular muscle. Conduction through the AV node is the slowest, to allow the ventricles enough time to fill with blood.\n\nBonke et al. review the evidence regarding the conduction velocity of parts of the heart. They discuss how the AV node is slower than the SA node. They recommend understanding how different parts of the heart conducts electricity.\n\nIncorrect Answers:\nAnswers A-C & E: These choices do not correspond to the correct conduction velocities in each part of the cardiac conduction system. A calcium channel blocker would not change the relative velocities of conduction in Purkinje fibers, atria, and ventricles.\n\nBullet Summary:\nThe conduction velocity through the heart in order of speed is Purkinje fibers > atria > ventricles > AV node.", "link": "https://bit.ly/47KxMQs"} {"question": "A 3-year-old girl is brought to the office for a routine well-child appointment. She was delivered preterm at 35 weeks, and her medical history is significant for several ear infections over the last year. Her older brother has been diagnosed with attention-deficit hyperactivity disorder (ADHD), but the rest of her family history is otherwise unremarkable. The patient has become more withdrawn lately and has sometimes been ignoring her parents when asked to perform tasks at home. This seems to have worsened over the last 8 weeks. Otherwise, the patient is energetic and started preschool 3 months ago. Her temperature is 98.6\u00b0F (37\u00b0C), blood pressure is 98/62 mmHg, pulse is 97/min, and respirations are 26/min. The patient successfully draws a square and can stand on 1 foot. Her language skills are unchanged from her appointment 6 months ago. Her neurological examination is normal, and she is at the 50th percentile for height and weight. She seems distracted during the interview and responds only intermittently to the physician\u2019s directions. Which of the following is the most appropriate next step in management?", "choicesA": "Audiometry testing", "choicesB": "Autism spectrum screening questionnaire", "choicesC": "Parent and teacher ADHD rating scales", "choicesD": "Refer to genetic testing", "choicesE": "Speech and language assessment", "answer_idx": "A", "answer": "Audiometry testing", "explanation": "This patient who presents with trouble following directions, inconsistent response to the physician's questions, and lack of progression in her language skills may have an acquired hearing impairment. The patient should undergo audiometry testing.\n\nHearing impairment in children can be genetic or acquired. In a patient with a history of recurrent ear infections, conductive hearing loss is the most common underlying cause of hearing impairment. Hearing impairment can often present similarly to behavioral or persistent developmental disorders. In a child with poor language skills, social skills, and self-isolation, hearing loss should be ruled out first before considering other diagnoses. Undetected hearing loss can progress with poor academic performance, personal-social maladjustment, and emotional difficulties as children progress through critical development stages. The treatment for hearing loss is the restoration of hearing through hearing aids, cochlear implants, and other supportive measures.\n\nDammeyer et al. studied the impact of childhood hearing loss on family life. They found that children with additional disabilities were more likely to be ignored by parents. They recommend understanding family dynamics in order to better support patients and families.\n\nIncorrect Answers\nAnswer B: Autism spectrum screening questionnaire is useful in the diagnosis of autism spectrum disorder in children presenting with social isolation and poor communication skills. Autism spectrum disorder often presents with repetitive behaviors and fixed interests, which are absent in this child. Hearing loss should be ruled out first before pursuing this diagnosis. Treatment of autism spectrum disorders is early intervention and counseling.\n\nAnswer C: Parent and teacher ADHD rating scales should be obtained to aid in the diagnosis of attention-deficit hyperactivity disorder (ADHD). While children with ADHD often appear inattentive and impulsive in multiple domains, poor language development, and social isolation are not features of ADHD. This patient should first undergo an audiometry test. ADHD can be treated with stimulants such as methylphenidate.\n\nAnswer D: Referral for genetic testing is an incorrect first step in the management of this patient. While delayed language can be seen with several genetic conditions, there are usually signs of other global motor, cognitive, and social delays. Additional genetic testing can be pursued if the patient\u2019s audiometry testing results are normal.\n\nAnswer E: Referral for speech and language assessment is an incorrect first step. A formal assessment should be obtained in the diagnosis of communication and language disorders. However, this should only be done after ruling out hearing loss. Therefore audiometric testing should be performed first.\n\nBullet Summary:\nChildren with a history of recurrent ear infections presenting with signs of social or language development delays should be evaluated for hearing impairment with an audiometry test.", "link": "https://step2.medbullets.com/testview?qid=216239"} {"question": "A 64-year-old man presents with nausea, vomiting, and weakness. He states that his wife made him come in when he fainted today while attempting to stand up. He otherwise states he has noticed some visual changes, including noting a yellow tint to objects. He has a history of heart failure with reduced ejection fraction, obesity, diabetes, hypertension, acute coronary syndrome, atrial fibrillation, and peripheral vascular disease. His temperature is 98.5\u00b0F (36.9\u00b0C), blood pressure is 153/91 mmHg, pulse is 40/min, respirations are 15/min, and oxygen saturation is 97% on room air. Physical exam reveals a frail man who is unable to ambulate secondary to lightheadedness. His neurological exam is otherwise non-focal. An ECG is performed as seen in Figure A. Which of the following is the most likely etiology of this patient\u2019s symptoms?", "choicesA": "Amiodarone", "choicesB": "Digoxin", "choicesC": "Diltiazem", "choicesD": "Procainamide", "choicesE": "Propranolol", "answer_idx": "B", "answer": "Digoxin", "explanation": "This patient with a history of heart failure with reduced ejection fraction and atrial fibrillation is presenting with nausea, vomiting, visual changes, bradycardia, and premature ventricular complexes, which are concerning for digoxin toxicity. Note that digoxin can be given in patients with poor cardiac function and atrial fibrillation; thus it is a plausible home medication for this patient.\n\nDigoxin is a cardiac glycoside that is used in conditions such as heart failure with reduced ejection fraction and atrial fibrillation. While it does not lower mortality, it may reduce symptoms of heart failure. Digoxin toxicity classically presents with gastrointestinal symptoms (nausea/vomiting), blurred vision or yellow halos, and cardiac dysrhythmias. While there is a myriad of dysrhythmias, digoxin can cause bradycardia, atrioventricular block, and premature ventricular contractions. ST-segment scooping can be seen with digoxin use both in toxicity but also with regular therapeutic use. Toxicity is treated with anti-digoxin antibodies.\n\nPatocka et al review digoxin toxicity. They note that this medication has a narrow therapeutic range and recommend close monitoring. Toxicity depends on severity and can be treated with anti-digoxin antibodies.\n\nFigure/Illustration A is an ECG demonstrating bradycardia and premature ventricular complexes (red circles). This is consistent with digoxin toxicity.\n\nIncorrect Answers:\nAnswer A: Amiodarone is an antidyshythmic agent with toxicities including bradycardia, heart block, a prolonged QT interval, hepatitis, interstitial lung disease, hypothyroidism, corneal deposits and optic neuropathy. This patient could use amiodarone for his atrial fibrillation, but the visual disturbances are more typical of digoxin toxicity.\n\nAnswer C: Diltiazem is a calcium channel blocker that would cause hypotension, bradycardia, and hyperglycemia. Treatment would involve immediate administration of calcium and vasopressors such as epinephrine. Other treatments in severe toxicity may include glucagon, dextrose, and insulin (to increase cardiac contractility).\n\nAnswer D: Procainamide is the preferred antidysrhythmic in tachydysrhythmias associated with Wolff Parkinson White syndrome. It may cause drug-induced lupus and other dysrhythmias in overdose but would not cause this patient\u2019s constellation of symptoms. Procainiamide is used rarely for atrial fibrillation and is only available for intravenous infusion in the US.\n\nAnswer E: Propranolol toxicity would present with bradycardia, hypotension, confusion, seizures, and hypoglycemia. Treatment is centered on glucagon, calcium, and vasopressors such as epinephrine. In severe toxicity, lipid emulsion therapy and insulin/dextrose should be administered.\n\nBullet Summary:\nDigoxin toxicity may present with nausea, vomiting, visual changes, bradycardia, and premature ventricular complexes.", "link": "https://step2.medbullets.com/testview?qid=216589"} {"question": "A 23-year-old woman presents to the emergency department complaining of a worsening headache. The headache started 1 month ago. It is constant and \u201call over\u201d but gets worse when she is lying down or in the setting of bright lights. A review of systems is significant for a low-grade fever, night sweats, cough, malaise, poor appetite, and unintentional weight loss of 12 pounds in the last 2 months. The patient is sexually active with multiple male partners and reports inconsistent condom use. She has a history of intravenous drug use and has not been to a doctor in the last 2 years. Her temperature is 100.4\u00b0F (38.0\u00b0C), blood pressure is 110/78 mmHg, pulse is 88/min, and respirations are 14/min with an oxygen saturation of 98% O2 on room air. On physical exam, pain is elicited upon passive flexion of the patient\u2019s neck. A CT scan shows ventricular enlargement. A CD4+ count is 57 cells/\u00b5L blood. A lumbar puncture is performed with the following findings:\n\nCerebrospinal fluid:\nOpening pressure: 210 mmH2O\nGlucose: 32 mg/dL\nProtein: 204 mg/dL\nIndia ink stain: Positive\n\nLeukocyte count and differential:\nLeukocyte count: 200/mm^3\nLymphocytes: 100%\nRed blood cell count: 2/mm^3\n\nWhich of the following treatments most directly addresses the underlying cause of this patient's symptoms?", "choicesA": "Acyclovir", "choicesB": "Amphotericin B and 5-flucytosine", "choicesC": "Dexamethasone", "choicesD": "Fluconazole", "choicesE": "Vancomycin and ceftriaxone", "answer_idx": "B", "answer": "Amphotericin B and 5-flucytosine", "explanation": "The patient with a history of unprotected sex and intravenous drug use who presents with sub-acute fever, meningismus, photophobia, a low CD4+ count, and a positive India ink-stained lumbar puncture most likely has cryptococcal meningitis. Acute treatment for cryptococcal meningitis is intrathecal amphotericin B and 5-flucytosine.\n\nCryptococcal meningitis is the most common fungal meningeal infection in patients with HIV, especially those with a CD4+ count < 100/mm^3. Symptoms are normally gradual in onset and can include headache, fever, neck stiffness, and photophobia. The most appropriate initial test is a lumbar puncture with an India ink stain. Cerebrospinal fluid in fungal meningitis would have low glucose, elevated protein, and an elevated lymphocyte-dominant leukocyte count. The most accurate diagnostic test is latex particle agglutination. Intrathecal amphotericin B and 5-flucytosine are the first-line treatments for acute management. Once the patient is treated, fluconazole is given for life or until the patient\u2019s CD4+ count is > 100/mm^3.\n\nBadali et al. present a case of a patient with cryptococcal meningitis. They discuss how despite immediate antifungal therapy, the patient died due to a rapidly progressive infection. They recommend aggressive intrathecal therapy to treat this disease.\n\nIncorrect Answers:\nAnswer A: Acyclovir is used to treat Herpes simplex virus encephalitis. Cerebrospinal fluid would show normal protein and glucose, an elevated lymphocyte-dominant leukocyte count, and an elevated red blood cell count. The Herpes virus can lead to a hemorrhage of the temporal lobes.\n\nAnswer C: Dexamethasone is given in addition to IV antibiotics if Streptococcus pneumoniae meningitis is suspected. This is the most common cause of meningitis in adults, elderly, and asplenic patients, and can spread to cause sinusitis, otitis media, or bacteremia.\n\nAnswer D: Fluconazole is used for prophylaxis against cryptococcal meningitis after a person is treated acutely. It is either given for life, or until the patient\u2019s CD4+ count is > 100/mm^3. This treatment is not sufficient for patients with acute meningitis with neurological changes.\n\nAnswer E: Vancomycin and ceftriaxone are given to a patient suspected of having bacterial meningitis until culture results confirm the bacterial organism. Bacterial meningitis would have decreased glucose, elevated protein, and an elevated neutrophil-dominant leukocyte count.\n\nBullet Summary:\nCryptococcal meningitis classically presents in a patient with a CD4+ count < 100/mm^3 and should be treated acutely with intrathecal amphotericin B and 5-flucytosine.", "link": "https://bit.ly/3MCzFqw"} {"question": "A 9-year-old girl presents to the emergency department with a fever and a change in her behavior. She presented with similar symptoms 6 weeks ago and was treated for an Escherchia coli infection. She also was treated for a urinary tract infection 10 weeks ago. Her mother says that last night her daughter felt ill, and her condition has been worsening. Her daughter experienced a severe headache and had a stiff neck. This morning she was minimally responsive, vomited several times, and produced a small amount of dark cloudy urine. The patient was born at 39 weeks and met all her developmental milestones. She is currently up to date on her vaccinations and did not have infections during early childhood. Her parents are divorced and her father has noted she does not seem to get sick when he takes care of her. Her temperature is 99.5\u00b0F (37.5\u00b0C), blood pressure is 60/35 mmHg, pulse is 190/min, respirations are 33/min, and oxygen saturation is 98% on room air. The patient is started on intravenous fluids, vasopressors, and broad-spectrum antibiotics. Which of the following is the most appropriate underlying explanation for this patient's presentation?", "choicesA": "Gastroenteritis", "choicesB": "Immunodeficiency", "choicesC": "Intentional contamination", "choicesD": "Meningitis", "choicesE": "Urinary tract infection", "answer_idx": "C", "answer": "Intentional contamination", "explanation": "This patient is presenting with repeat sepsis, which is uncommon in a pediatric patient with no significant medical history. This abnormal concentration of infections when the patient is with her mother suggests a diagnosis of intentional contamination.\n\nFactitious disorder by proxy (formerly Munchausen syndrome) occurs when 1 individual creates symptoms in another for attention. Typically, this is a parent feigning or creating symptoms in a child in order to receive the psychiatric gain of attention from healthcare personnel. The patient should be treated and removed from the parents as this is considered child abuse. Factitious disorder by proxy should be suspected in any pediatric patient who has repeat infections that are atypical or not expected epidemiologically. Patients should also be evaluated for organic causes of symptoms such as immunodeficiency syndromes.\n\nAbeln and Love review the evidence regarding the diagnosis of factitious disorder by proxy. They discuss how these disorders have high morbidity and mortality when not accurately recognized. They recommend paying attention to red flags such as abnormal constellations of symptoms.\n\nIncorrect Answers:\nAnswer A: Gastroenteritis would present with nausea, vomiting, and diarrhea; however, it would not present with sepsis routinely. Gastroenteritis does not best explain the most likely underlying cause of this patient's presentation.\n\nAnswer B: Immunodeficiency such as severe combined immunodeficiency (SCID) would present with lifelong infections from birth that are bacterial, viral, and fungal. Immunodeficiency would not present with repeat infections in a pediatric patient only when they are with one patient.\n\nAnswer D: Meningitis would present with headache, altered mental status, and meningeal signs. In the setting of repeat sepsis in a healthy pediatric patient, intentional contamination is more likely. Similarly, an absence of photophobia and no further history/physical exam supporting this diagnosis suggests an alternative diagnosis.\n\nAnswer E: Urinary tract infection would present with dysuria rather than repeat episodes of sepsis. Though a urinary tract infection could lead to sepsis, this would be rare and more likely to occur in an immunosuppressed patient, the elderly, or a neonate. UTIs are a common cause of infection and fever in pediatric patients.\n\nBullet Summary:\nFactitious disorder by proxy occurs when 1 individual feigns or creates symptoms in another.", "link": "https://bit.ly/3GZtkBx"} {"question": "A 55-year-old man presents to the emergency department with right knee pain. He woke last night with sudden-onset, sharp, 10/10 non-radiating pain in his right knee. He denies recent trauma and he has no known medical problems. He denies tobacco use and will often drink 6-8 beers or spirits on weekend nights. His temperature is 98.6\u00b0F (37.0\u00b0C), pulse is 90/min, blood pressure is 140/90 mmHg, respirations are 16/min, and oxygen saturation is 97% on room air. Physical exam reveals an erythematous, edematous right knee. Passive extension of the knee is limited by pain. Synovial fluid aspiration is performed and sent for analysis, which reveals 20,000 leukocytes/mm^3, and no organisms visualized. Microscopic examination of synovial fluid is shown in Figure A. Which of the following is the most likely diagnosis?", "choicesA": "Gout", "choicesB": "Lyme arthritis", "choicesC": "Pseudogout", "choicesD": "Rheumatoid arthritis", "choicesE": "Septic arthritis", "answer_idx": "A", "answer": "Gout", "explanation": "The patient\u2019s sudden-onset knee pain with effusion and synovial fluid findings are characteristic of gout.\n\nGout is caused by monosodium urate crystal deposition in joint spaces, most commonly in the big toe, ankle, or knee. Crystal deposition in the joint space leads to an intense inflammatory reaction that causes severe pain and joint effusion. Gout is most commonly due to lifestyle factors including a diet high in red meat and alcohol. Medications that cause elevated uric acid levels (such as thiazide diuretics) may also play a role. Conditions resulting in high cell turnover and therefore excessive purine release such as hematologic malignancies may also lead to the development of gout. Patients typically present with sudden-onset, severe pain, erythema, and joint effusion. Synovial fluid analysis will reveal an elevated leukocyte count and negatively birefringent, needle-shaped crystals. Treatment of acute gout is with NSAID medications, colchicine, or steroids depending on presentation and patient-specific factors. Prevention of further attacks is centered around lifestyle changes such as avoidance of red meat and alcohol. Xanthine oxidase inhibitors such as allopurinol are also often used for prevention of further episodes.\n\nNeogi discusses the management of acute gout. She comments that, while synovial fluid analysis is the gold standard of diagnosis, many physicians do not routinely perform it, instead relying on clinical judgment to diagnose gout. The differential diagnosis for a gout-like presentation includes pseudogout, septic arthritis, Lyme arthritis, reactive arthritis, and rheumatoid arthritis.\n\nFigure A shows monosodium urate crystals, which appear as negatively birefringent, needle-shaped crystals under polarized light.\n\nIncorrect Answers:\nAnswer B: Lyme disease may also presents as a monoarticular arthritis in later stages. However, a recent history of tick exposure and preceding flu-like illness would also be expected.\n\nAnswer C: Pseudogout presents similar to gout, but is caused by calcium pyrophosphate deposition. Crystals appear rhomboid shaped and are positively birefringent under polarized light.\n\nAnswer D: Septic arthritis typically has higher leukocyte counts (> 50,000/mm^3) in the synovial fluid with organisms visualized on gram stain. Fever would also be expected.\n\nAnswer E: Rheumatoid arthritis may present as a monoarticular arthritis with acute flairs. However, crystals visualized in synovial fluid are more suggestive of acute gout.\n\nBullet Summary:\nAcute gout presents with sudden-onset, severe joint pain and is characterized by needle-shaped negatively birefringent crystals visualized under polarized light.", "link": "https://bit.ly/47U1Ppp"} {"question": "A 44-year-old nurse presents to the emergency department with confusion. This has happened several times this past month with increasing frequency. Each time her symptoms improved with eating. She is otherwise healthy. She lives with her mother who has diabetes whose medications include metformin, insulin, lisinopril, amlodipine, and glyburide. Her temperature is 98.0\u00b0F (36.7\u00b0C), blood pressure is 132/81 mmHg, pulse is 85/min, respirations are 16/min, and oxygen saturation is 98% on room air. Physical exam reveals a confused woman. She is moving all her extremities but follows commands poorly. Laboratory studies are ordered as seen below.\n\nHemoglobin: 14 g/dL\nHematocrit: 41%\nLeukocyte count: 7,500/mm^3 with normal differential\nPlatelet count: 199,000/mm^3\n\nSerum:\nNa+: 140 mEq/L\nCl-: 102 mEq/L\nK+: 4.0 mEq/L\nHCO3-: 23 mEq/L\nBUN: 30 mg/dL\nGlucose: 29 mg/dL\nCreatinine: 1.4 mg/dL\nCa2+: 10.2 mg/dL\nSulfonylurea level: undetectable\nC-peptide level: 55 ng/mL (normal < 5 ng/mL)\n\nWhich of the following is the most likely etiology of this patient\u2019s symptoms?", "choicesA": "Alpha cell tumor", "choicesB": "Beta cell destruction", "choicesC": "Beta cell tumor", "choicesD": "Insulin overdose", "choicesE": "Sulfonylurea overdose", "answer_idx": "C", "answer": "Beta cell tumor", "explanation": "This patient is presenting with confusion, profound hypoglycemia, an elevated C-peptide level, and a negative sulfonylurea level, which are concerning for an insulinoma. An insulinoma is a beta cell tumor that secretes insulin.\n\nAn insulinoma is a beta cell tumor of the pancreas. The beta cells normally secrete insulin in response to increased circulating fuel substrate (such as glucose or fat). In the setting of an insulinoma, insulin is pathologically and continuously secreted. This leads to profound, recurrent episodes of hypoglycemia. When a patient presents with hypoglycemia of unclear etiology; a blood glucose level, C-peptide level, and sulfonylurea level should be ordered. An infectious workup may be indicated as sepsis can also cause hypoglycemia. In an insulinoma, patients will be hypoglycemic, have an elevated C-peptide level (as this is a marker of endogenous insulin release), and a negative sulfonylurea level (important to rule out as a sulfonylurea overdose can also cause hypoglycemia in the setting of an elevated C-peptide level). Further workup will involve imaging of the abdomen such as a CT scan or MRI (more accurate) to delineate this insulin-secreting mass.\n\nMathur et al. discuss insulinoma. They note that it is a rare endocrine tumor, and note its association with multiple endocrine neoplasia type 1. They recommend workup for an insulinoma should occur in patients with recurrent episodes of hypoglycemia without a clear cause.\n\nIncorrect Answers:\nAnswer A: Alpha cell tumor describes a glucagonoma, which pathologically secretes glucagon leading to profound hyperglycemia, a classic rash termed necrolytic migratory erythema (presents with painful, erythematous papules and plaques), and gastrointestinal symptoms including anorexia, abdominal pain, and diarrhea.\n\nAnswer B: Beta cell destruction describes the pathophysiology of type I diabetes mellitus, which presents in pediatric patients (most commonly) with hyperglycemia, weight loss, polydipsia, and polyuria. Treatment involves the administration of insulin. An initial presentation of type I diabetes may be diabetic ketoacidosis which presents with dehydration, abdominal pain, nausea, vomiting, Kussmaul respirations, and an anion gap acidosis.\n\nAnswer D: Insulin overdose would present with hypoglycemia, somnolence, confusion, and tremulousness. Note that exogenous insulin administration would not elevate the C-peptide level, which is a marker of endogenous insulin production.\n\nAnswer E: Sulfonylurea overdose would cause hypoglycemia that is often persistent given the long half-life of sulfonylureas. It would also elevate the C-peptide level as these medications increase the endogenous release of insulin. However, the sulfonylurea level was negative in this patient. While iatrogenic use of medications is plausible in this patient who has knowledge of medications (as she is a nurse), her repeat episodes and her workup suggesting against an iatrogenic etiology makes an insulinoma a more likely diagnosis.\n\nBullet Summary:\nInsulinomas are beta cell tumors of the pancreas and lead to severe hypoglycemia and an elevated insulin and C-peptide level.", "link": "https://step2.medbullets.com/testview?qid=216610"} {"question": "A 67-year-old man presents to the emergency department after fainting. He was outside tending to his lawn for several hours in the heat, when he stood up suddenly from pulling weeds and fainted. He denies any preceding symptoms and returned to baseline within 1 minute. The patient is not aware of any medical problems and just started seeing a primary care physician last week. He recently used a friend's prescription for ondansetron for nausea. His temperature is 99.3\u00b0F (37.4\u00b0C), blood pressure is 142/88 mmHg, pulse is 107/min, respirations are 14/min, and oxygen saturation is 99% on room air. Physical exam reveals intact cranial nerves, normal strength and sensation, and a stable gait. His abdomen is soft and nontender. An ECG is performed as seen in Figure A. Which of the following is the most likely diagnosis based on this patient\u2019s ECG?", "choicesA": "Acute myocardial infarction", "choicesB": "Hypokalemia", "choicesC": "Intermittent torsades des pointes", "choicesD": "Previous myocardial ischemia", "choicesE": "Pulmonary embolism", "answer_idx": "D", "answer": "Previous myocardial ischemia", "explanation": "This patient is presenting after syncope, likely secondary to dehydration and orthostatic hypotension given his outdoor gardening in the heat and symptoms when standing up rapidly. The patient\u2019s ECG reveals deep, pathologic Q waves, which in this case are an incidental finding indicative of a previous myocardial infarction or a previous ischemic event.\n\nThe Q wave is a negative deflection just prior to the R wave. Q waves are a normal finding when they are small and are seen on most ECGs. However, pathologic Q waves are indicative of myocardial ischemia. Pathologic Q waves are generally diagnosed when they are >2 mm deep, >1 mm wide, >25% of the QRS complex height, or are seen in leads V1-V3. The interpretation of Q waves depends on other findings on ECG and the patient's symptoms. Deep Q waves in the setting of ST elevation indicate an acute myocardial infarction. On the other hand, pathologic Q waves found incidentally indicate a previous myocardial infarction or previous ischemic event.\n\nRovai et al. review Q waves. They discuss how Q waves can be used to predict the location and size of a myocardial infarction as correlated with MRI. They recommend testing such as cardiac MRI to assess for previous infarction in the appropriate patient population.\n\nFigure/Illustration A is an ECG demonstrating pathologic Q waves (red arrows). Note the deep and wide Q waves; this raises concern for a past ischemic event.\n\nIncorrect Answers:\nAnswer A: Acute myocardial infarctions would present with ST elevation in a vascular distribution on ECG with possible reciprocal depressions. While T wave inversions may be indicative of new or acute ischemia, deep pathologic Q waves without ST elevation are indicative of previous infarction.\n\nAnswer B: Hypokalemia would cause U waves on ECG, which appear as a small, \"second T wave\" after the initial T wave. Note they are not U-shaped despite their name. Management is centered on repletion of potassium and often magnesium as well.\n\nAnswer C: Intermittent torsades de pointes can cause syncope and should be suspected in a patient with syncope in the setting of a prolonged QT interval. This patient\u2019s QT interval is not particularly prolonged, and his syncope seems to be secondary to orthostatic hypotension. Management is centered on cessation of QT-prolonging medications and magnesium administration. Note that while ondansetron does prolong the QT interval, it is unlikely to cause torsades given the patient took it a week ago.\n\nAnswer E: Pulmonary emboli would present on ECG with sinus tachycardia in addition to pleuritic chest pain and hypoxia. Note that the ECG is a nonspecific test for pulmonary emboli, and CT angiography is the preferred confirmatory test for this condition. Syncope is a possible presentation of larger pulmonary emboli (though the patient would also be hemodynamically unstable).\n\nBullet Summary:\nPathologic Q waves are indicative of a previous myocardial infarction.", "link": "https://step2.medbullets.com/testview?qid=216616"} {"question": "A 34-year-old man is brought to a rural emergency department by ambulance after being involved in a motor vehicle accident. Paramedics report that the patient was driving the car and crashed into a tree at roughly 25 miles per hour. There were no passengers and he was awake but disoriented at the scene. His temperature is 97.9\u00b0F (36.6\u00b0C), blood pressure is 131/88 mmHg, pulse is 89/min, and respirations are 14/min. He is speaking but is confused, opens his eyes to voice commands, and follows simple commands. He has multiple lacerations on his face and arms and smells of alcohol and marijuana. His cardiac exam is normal and his lungs are clear to auscultation bilaterally. He has bruising over his abdomen without any tenderness to palpation, distension, or rigidity. Which of the following is the most appropriate next step in management?", "choicesA": "Abdominal and chest CT", "choicesB": "Chest radiograph anterior-posterior and lateral", "choicesC": "Diagnostic laparoscopy", "choicesD": "Diagnostic peritoneal lavage", "choicesE": "Focused abdominal sonography for trauma exam", "answer_idx": "A", "answer": "Abdominal and chest CT", "explanation": "This patient presents following a motor vehicle accident with a tender abdomen and stable vital signs. Given his stability, the most appropriate next step is an abdominal and chest CT.\n\nIn blunt abdominal trauma, the next step in management depends upon whether the patient is hemodynamically stable. With a normal blood pressure and pulse within the normal range, patients can undergo imaging to evaluate for injury to the abdominal organs with the most appropriate study being an abdominal CT with contrast. This study can accurately localize bleeding and aid in operative planning. A bedside ultrasound known as the FAST exam is often performed in unstable patients. Unstable patients, patients with abdominal tenderness, and patients with a severe mechanism of trauma may have a bedside FAST exam performed. If the free fluid is localized, the patient can be transferred directly to the operating room. Note that in many hospitals, the FAST exam is done simultaneously with the primary and secondary survey; however, if asked to choose which exam to perform on a trauma patient who is stable with no signs of a surgical abdomen, a CT scan should be performed to more accurately assess the patient's injuries and assess for injuries that may be missed by a FAST exam.\n\nFeliciano reviewed the evidence surrounding the current diagnosis and management of abdominal trauma. He discusses how contrast-enhanced CT of the abdomen and pelvis is an important method for evaluating the etiology of intra-abdominal bleeding. He recommends considering damage control methods in the polytrauma patient.\n\nIncorrect Answers:\nAnswer B: Chest radiography has limited utility in the setting of blunt abdominal trauma; however, a portable anterior-posterior film is often taken in trauma, in particular, when the patient is intoxicated. However, an anterior-posterior and lateral film would involve transporting the patient to radiology and positioning the patient. Thus it would be both difficult to obtain and less useful when compared to a CT scan given his traumatic injuries. Findings on chest radiograph could suggest abdominal injury, such as lower rib fractures or free air under the diaphragm, which would suggest intestinal perforation.\n\nAnswer C: Diagnostic laparoscopy may play a role in evaluating for injury in penetrating traumas such as gunshot wounds or stabbing wounds but would not be indicated as the most appropriate initial step in management in a stable patient. Unstable patients, surgical abdomens, and positive FAST exams or CT scans warrant transfer to the operating room.\n\nAnswer D: Diagnostic peritoneal lavage is a historical test to assess for intra-abdominal injury that was typically used with ambiguous FAST exams or patients where it was unclear whether there was abdominal bleeding; however, it has almost entirely been supplanted by the FAST exam.\n\nAnswer E: FAST exams are often performed with the primary or secondary survey in large institutions where resources are available. However, in small hospitals with limited resources, the most appropriate next step in management when deciding between a FAST exam or a CT scan should be made based on the mechanism and concern for bleeding. This stable patient with a benign exam can undergo a CT scan, which will offer more data than a FAST exam and would be performed regardless of whether a FAST exam was positive or negative given this patient's intoxication and abdominal bruising.\n\nBullet Summary:\nIn a patient who has experienced blunt abdominal trauma the most appropriate initial step in a stable patient is an abdominal CT scan.", "link": "https://step2.medbullets.com/testview?qid=108735"} {"question": "A 26-year-old man presents to the emergency department with fatigue and dark urine over the past day. He was recently diagnosed with cellulitis of his left leg, for which he was prescribed trimethoprim-sulfamethoxazole. He denies having nausea, vomiting, or diarrhea. He has no significant medical history and does not currently take any medications. His temperature is 98.6\u00b0F (37\u00b0C), blood pressure is 115/72 mmHg, pulse is 98/min, and respirations are 14/min. Physical exam reveals conjunctival icterus, bilateral flank tenderness, and erythema of the left leg. Laboratory workup is performed, and the results are shown below: Serum: Hemoglobin: 7.2 g/dL Total bilirubin: 2.1 mg/dL Direct bilirubin: 0.8 mg/dL Lactate dehydrogenase: 470 U/L Glucose-6-phosphate dehydrogenase: 7 U/gHb (normal: 6-20 U/gHb) Which of the following would most likely be seen on a peripheral blood smear in this patient?", "choicesA": "Acanthocytes", "choicesB": "Codocytes", "choicesC": "Degmacytes", "choicesD": "Schistocytes", "choicesE": "Spherocytes", "answer_idx": "C", "answer": "Degmacytes", "explanation": "This patient presents with hemolytic anemia (low hemoglobin, elevated indirect bilirubin, and elevated lactate dehydrogenase) shortly after starting a sulfa medication (trimethoprim-sulfamethoxazole) in the setting of likely glucose-6-phosphate dehydrogenase deficiency. Degmacytes (\"bite cells\") will be seen on peripheral blood smears in these patients.\n\nPatients with glucose-6-phosphate dehydrogenase (G6PD) deficiency will experience hemolytic anemia when exposed to an oxidizing substance such as sulfa drugs or fava beans. Laboratory tests will show a decrease in hemoglobin and hematocrit levels. Since hemolysis releases intracellular enzymes into the bloodstream, serum levels of lactate dehydrogenase will be increased. Hemoglobin is then metabolized in the spleen into unconjugated bilirubin, which will cause an elevated bilirubin level that is primarily indirect. In the acute setting, G6PD levels will be normal because the most compromised cells have all been hemolyzed. Obtaining a G6PD level in between episodes of hemolysis is therefore required to observe the chronically decreased G6PD in these patients. A peripheral blood smear will reveal degmacytes (\"bite cells\") and Heinz bodies. Avoidance of oxidative stress to red blood cells is the most important component of management in these patients.\n\nBelfield and Tichy discuss the classic presentation of G6PD deficiency in patients. The authors review oxidizing medications such as rasburicase, primaquine, dapsone, pegloticase, and methylene blue. The authors recommend avoiding initiation of these medications whenever possible until a G6PD diagnostic test has been performed.\n\nIncorrect Answers:\nAnswer A: Acanthocytes can be seen in patients with liver disease due to defective production of lipoproteins; however, these cells do not undergo acute hemolytic destruction. Patients with abetalipoproteinemia usually present with symptoms consistent with fat-soluble vitamin deficiency such as bleeding (vitamin K) or osteoporosis (vitamin D).\n\nAnswer B: Codocytes are most commonly seen in patients with thalassemia but can also generally be seen in all microcytic anemias. Patients with thalassemia will present with chronic anemia without acute episodes of hemolysis. The increased lactate dehydrogenase and bilirubin levels in this patient indicate an acute hemolytic event.\n\nAnswer D: Schistocytes can be seen in thrombotic thrombocytopenic purpura (TTP) and disseminated intravascular coagulation (DIC). Patients with TTP present with thrombocytopenia, microangiopathic hemolytic anemia, fever, neurologic symptoms, and acute kidney injury. DIC is a common downstream pathway for many processes such as sepsis, malignancy, and eclampsia; however, it would present with diffuse clotting and bleeding from all sites.\n\nAnswer E: Spherocytes can be seen in hereditary spherocytosis, which may result in hemolysis during times of acute stress; however, this patient denies symptoms of nausea, vomiting, or diarrhea. Instead, the hemolysis in response to new foods is most likely due to exposure to oxidizing substances such as trimethoprim-sulfamethoxazole.\n\nBullet Summary:\nPatients with glucose-6-phosphate dehydrogenase deficiency will experience hemolytic anemia in response to oxidizing substances and will have degmacytes (bite cells) on peripheral blood smear.", "link": "https://bit.ly/3OhgJgC"} {"question": "A 45-year-old man with history of IV drug use presents to the outpatient clinic with a history of fever for 5 days. He also endorses sweats, headache behind the eyes, sore throat, muscle and joint pain, and a new rash seen in Figure A. He reports that he traveled to Africa last year and returned from a vacation to Thailand 10 days ago. His immunizations are up to date, although he missed the pre-departure travel medicine consult before his trip to Thailand. His temperature is 101.5\u00b0F (38.6\u00b0C), blood pressure is 125/80 mmHg, pulse is 105/min, and respirations are 14/min. Laboratory testing reveals the following:\n\nLeukocyte count: 2,400/mm^3\nHemoglobin: 13.4 g/dL\nHematocrit: 40%\nMean corpuscular volume: 81 \u00b5m^3\nPlatelets: 92,000/mm^3\n\nSerum:\nAST: 112 IU/L\nALT: 69 IU/L\nALP: 78 IU/L\nBilirubin: 0.9 mg/dL\n\nWhich of the following is the diagnostic test of choice for this disease?", "choicesA": "Blood culture", "choicesB": "Blood smear", "choicesC": "Serologic testing", "choicesD": "Sputum with acid fast stain", "choicesE": "Stool culture", "answer_idx": "C", "answer": "Serologic testing", "explanation": "This patient is presenting with fever, headache with retro-orbital pain, muscle aches, joint pain, rash, leukopenia and thrombocytopenia, which are classic findings for dengue fever. The diagnostic test of choice for dengue is serology.\n\nDengue is a viral infection endemic to South Asia, Central and South America, and the Caribbean. Although most infections are asymptomatic, notable characteristics include headache with retro-orbital pain, bone pain (\"break-bone fever\"), leukopenia, and thrombocytopenia (< 100,000 /\u00b5L). Symptoms typically develop several days to 2 weeks following the bite of an infected mosquito. The diagnostic test of choice is a serologic antibody test, although PCR can also be used in the acute (< 3 days) phase of illness. Treatment is with supportive care including rehydration.\n\nJasamai et al. present the current treatment options for dengue fever. They discuss how supportive therapy is the mainstay of treatment as no antibody or antiviral treatments have been effective at this time. They recommend using effective preventive measures such as mosquito control in endemic areas.\n\nFigure A is a clinical photograph demonstrating the maculopapular rash seen in dengue fever.\n\nIncorrect Answers:\nAnswer A: Blood culture is used for the detection of bacterial infections of the blood. This patient's IV drug use puts him at risk for bacterial endocarditis, but he does not have other expected findings, such as new murmur, Janeway lesions, Roth spots, or Osler nodes. Endocarditis should be treated with intravenous antibiotics.\n\nAnswer B: Blood smear is the diagnostic test of choice for other mosquito-borne diseases such as malaria. It is not used in the detection of dengue. Malaria presents with cyclical fevers and organisms on blood smear. It can be treated with chloroquine and artesunate.\n\nAnswer D: Sputum with acid fast stain is the test of choice for tuberculosis. This patient has a risk factor of TB as he recently traveled to Africa, but he does not have any pulmonary symptoms. Treatment for tuberculosis is rifampin, isoniazid, pyrazinamide, and ethambutol therapy (among other possible regimens).\n\nAnswer E: Stool culture may be used to diagnose typhoid fever. Typhoid fever presents with many of the same non-specific symptoms as dengue but would also present with prominent abdominal pain and gastrointestinal symptoms. Typhoid can be treated with antibiotics such as ciprofloxacin.\n\nBullet Summary:\nDengue fever is characterized by fever, headache with retro-orbital pain, muscle and joint pain, rash, leukopenia and thrombocytopenia that can be diagnosed through serologic testing.", "link": "https://bit.ly/3UV1eNW"} {"question": "A 30-year-old woman presents to the emergency department for palpitations. For 2 months, she has experienced intermittent palpitations along with menstrual irregularity and hair loss. At home, the air conditioner is at its coldest setting, but the patient still feels overheated. Her temperature is 99.5\u00b0F (37.5\u00b0C), blood pressure is 135/90 mmHg, pulse is 120/min, and respirations are 22/min. The patient is diaphoretic with tremulous hands, demonstrates thyromegaly, and protrusion of her eyes is noted. She is given atenolol in the emergency department with improvement of her palpitations and is instructed to follow up with her primary doctor for further management. Which of the following is most likely to be found in this patient after definitive treatment of her condition?", "choicesA": "Exophthalmos", "choicesB": "Hair loss", "choicesC": "Heat intolerance", "choicesD": "Menstrual irregularity", "choicesE": "Thyromegaly", "answer_idx": "A", "answer": "Exophthalmos", "explanation": "This patient presents with palpitations, tachycardia, tremor, heat intolerance, hair loss, menstrual irregularity, goiter, and exophthalmos concerning for hyperthyroidism due to Graves disease. Radioiodine ablation is the preferred definitive management for Graves disease, which can worsen exophthalmos.\n\nGraves disease is an autoimmune hyperthyroid disorder caused by antibodies to the thyroid stimulating hormone receptor (TSHR), which induce production of T4 and triiodothyronine (T3). Overstimulation of the thyroid gland by TSHR antibodies results in diffuse enlargement of the thyroid gland, termed \u201cgoiter.\u201d TSHR is also highly expressed in retro-ocular adipocytes and fibroblasts, where overstimulation by TSHR antibodies (and activated T-cells) leads to the expansion of retro-ocular connective tissue and extra-ocular muscle volume. This causes exophthalmos (bulging eyes) and is referred to as Graves ophthalmopathy or orbitopathy. Beta blockers are used to achieve acute sympathetic control in hyperthyroid Graves patients, and antithyroid medications (propylthiouracil or methimazole) are initiated to achieve a euthyroid state. Radioactive iodine ablation is first-line definitive management, particularly, for individuals with unsuccessful anti-thyroid medication management. Radioiodine ablation has been known to increase TSHR-antibodies, likely secondary to the leakage of thyroid antigens, and thus exophthalmos is often worsened after treatment. For this reason, corticosteroids are often co-administrated as a prophylactic measure.\n\nDavies et al. review Graves disease including the pathophysiology and management. They note that Graves orbitopathy may be worsened by radioiodine ablation. They recommend further studies and advancements in care given this lasting complication.\n\nIncorrect Answers:\nAnswer B: Hair loss can occur in hyperthyroid and hypothyroid states due to altered hair follicle stem cell function. However, this process is typically reversed, not worsened, after achieving a euthyroid state.\n\nAnswer C: Heat intolerance occurs in Graves disease due to hyperthyroid-induced sympathetic overactivity, which resolves after successful treatment.\n\nAnswer D: Menstrual irregularity occurs in hyperthyroidism through altered levels of sex hormone binding globulin, gonadotropin-releasing hormone, and prolactin. However, treatment of hyperthyroidism typically restores menstrual regularity. After ablation, patients will be hypothyroid; however, the standard of care is to start the patient on levothyroxine which should prevent this complication.\n\nAnswer E: Thyromegaly in Graves disease is caused by diffuse overactivation of the thyroid gland by the TSHR antibodies. Once treated with radioactive iodine (I-131), the thyroid gland gradually shrinks in size. Patients who undergo radioactive iodine ablation will later require life-long thyroxine supplementation due to resulting hypothyroidism.\n\nBullet Summary:\nDefinitive therapy for Graves disease is radioiodine ablation, which leads to increased TSHR antibodies and worsened exophthalmos from overactivation of TSHR-rich retro-ocular adipocytes and fibroblasts.", "link": "https://step2.medbullets.com/testview?qid=216266"} {"question": "A 1-year-old girl is brought to a neurologist due to increasing seizure frequency over the past 2 months. She recently underwent a neurology evaluation which revealed hypsarrhythmia on electroencephalography (EEG) with a mix of slow waves, multifocal spikes, and asynchrony. Her parents have noticed the patient occasionally stiffens and spreads her arms at home. She was born at 38-weeks gestational age without complications. She has no other medical problems. Her medications consist of lamotrigine and valproic acid. Her temperature is 98.3\u00b0F (36.8\u00b0C), blood pressure is 90/75 mmHg, pulse is 94/min, and respirations are 22/min. Physical exam reveals innumerable hypopigmented macules on the skin and an irregularly shaped, thickened, and elevated plaque on the lower back. Which of the following is most strongly associated with this patient's condition?", "choicesA": "Cardiac rhabdomyoma", "choicesB": "Glaucoma", "choicesC": "Optic glioma", "choicesD": "Polyostotic fibrous dysplasia", "choicesE": "Renal cell carcinoma", "answer_idx": "A", "answer": "Cardiac rhabdomyoma", "explanation": "This patient with a seizure disorder, ash-leaf spots (innumerable hypopigmented macules), Shagreen patch (elevated irregular plaque on the lower back), and West syndrome (hypsarrhythmia on EEG and movements consistent with infantile spasms) likely has tuberous sclerosis. Tuberous sclerosis is associated with cardiac rhabdomyomas.\n\nTuberous sclerosis is a neurocutaneous disorder that is inherited in an autosomal dominant fashion. Mutations in TSC1 and TSC2 lead to unregulated cell growth, leading to the formation of hamartomas in various locations. Manifestations of tuberous sclerosis include seizures (secondary to subependymal nodules and cortical dysplasia), mental retardation, renal angiomyolipomas, facial angiofibromas, mitral regurgitation, and hypomelanotic macules (ash-leaf spots). Cardiac rhabdomyoma is strongly associated with tuberous sclerosis and can lead to arrhythmias. West syndrome, which is characterized by infantile spasms, intellectual disability, and hypsarrhythmia (a chaotic mixture of high-amplitude slow waves, multifocal spikes, and intrahemispheric-interhemispheric asynchrony) on EEG, is also associated with tuberous sclerosis. Diagnosis of tuberous sclerosis is made with genetic testing. Advanced imaging (CT or MRI) may reveal cortical tubers and lesions in the third and fourth ventricles. Abdominal imaging may reveal renal angiomyolipomas. The management of seizures in tuberous sclerosis includes anti-seizure medications, though surgery may also be pursued in select cases.\n\nHinton et al. studied the cardiac manifestations of tuberous sclerosis. They found that cardiac rhabdomyomas can lead to arrhythmia later in life. They recommended that infantile spasms in tuberous sclerosis be treated with vigabatrin but other seizure types should be treated similar to other epileptic disorders.\n\nIncorrect Answers:\nAnswer B: Glaucoma may occur in patients with Sturge-Weber syndrome, which is characterized by \"tram-track\" calcifications in the brain, port-wine stains of the face, mental retardation, and epilepsy. Diagnosis is with MRI of the brain with contrast to demonstrate leptomeningeal vascular malformations that are characteristic of the disease.\n\nAnswer C: Optic glioma is associated with neurofibromatosis type 1 (NF1), which presents with hyperpigmented macules (cafe-au-lait spots), axillary freckling, and cutaneous neurofibromas. Neurofibromas in NF1 may also grow on peripheral nerves, leading to malignant peripheral nerve sheath tumors. Lisch nodules, or benign growths of the iris, may also be seen in NF1. Diagnosis of NF1 is clinical, but can be aided by genetic testing.\n\nAnswer D: Polyostotic fibrous dysplasia is associated with McCune-Albright syndrome. Polyostotic fibrous dysplasia causes fibrous tissue in bones, leading to growth abnormalities and pathologic fractures. McCune-Albright syndrome is also associated with cafe-au-lait macules, which have irregular borders, and endocrine abnormalities such as precocious puberty and hyperthyroidism. Diagnosis is supported with endocrine tests as well as genetic testing.\n\nAnswer E: Renal cell carcinoma is associated with Von-Hippel-Lindau syndrome (VHL), which presents with tumors arising in multiple organs. Patients with VHL present with hemangioblastomas in the brain, spinal cord, cerebellum, and retina, and pheochromocytoma. Pheochromocytomas release catecholamines, leading to episodic hypertension, diaphoresis, and palpitations. Diagnosis of VHL is with genetic testing.\n\nBullet Summary:\nTuberous sclerosis has numerous manifestations including seizures, angiofibromas, mitral regurgitation, renal angiomyolipoma, ash-leaf spots, mental retardation, and cardiac rhabdomyoma.", "link": "https://step2.medbullets.com/testview?qid=217175"} {"question": "A 17-year-old boy presents to his primary care physician with a chief concern of \"bad\" skin that has not improved despite home remedies. The patient has had lesions on his face that have persisted since he was 13 years of age. He has a diet high in refined carbohydrates and has gained 20 pounds since starting high school. Physical exam is notable for the findings in Figure A. The patient is started on benzoyl peroxide and topical retinoids. He returns 1 month later stating that his symptoms are roughly the same. Which of the following is the most appropriate next step in management?", "choicesA": "Continue current therapy for 1 more month", "choicesB": "Dietary intervention", "choicesC": "Isoretinoin", "choicesD": "Oral antibiotics", "choicesE": "Topical antibiotics", "answer_idx": "E", "answer": "Topical antibiotics", "explanation": "This patient is presenting with persistent acne vulgaris that is refractory to topical benzoyl peroxide and retinoids. The next step in management is topical antibiotics.\n\nAcne vulgaris is caused by blockage or outlet obstruction of the pilosebaceous unit. It presents with erythema, pustules, and comedones typically in young men going through puberty. Risk factors include stress, excessive sweating, greasy skin care products, and steroid use. The management of acne vulgaris, in order, is to start with topical benzoyl peroxide and topical retinoids, topical antibiotics, oral antibiotics, and isoretinoin. It is important to try a therapy for a proper duration before switching to a more invasive therapy.\n\nEichenfield et al. review the evidence regarding the treatment of patients with acne vulgaris. They discuss how this disease can be treated with benzoyl peroxide. They recommend using antibiotics or isotretinoin for refractory cases.\n\nFigure/Illustration A is a clinical photograph demonstrating diffuse comedones over the forehead (red circle). These findings are classically seen in patients with acne vulgaris.\n\nIncorrect Answers:\nAnswer A: Continuing current therapy for 1 more month is unnecessary as this patient's symptoms have not improved at all with 1 month of therapy. Continuing current therapy would be appropriate if the patient had only tried therapy for a few days.\n\nAnswer B: Dietary interventions have not been shown to improve outcomes in acne vulgaris. Stopping illicit substances such as performance enhancing steroids can be effective in reducing acne symptoms but there is no evidence this patient has been using steroids.\n\nAnswer C: Isoretinoin is the last-line therapy for acne vulgaris. It should not be given to pregnant women as it is teratogenic. Women taking isoretinoin should be on reliable birth control. Topical and oral antibiotics should be used first.\n\nAnswer D: Oral antibiotics would be indicated after topical antibiotics fail. Topical antibiotics should be tried first as these do not have systemic side effects.\n\nBullet Summary:\nThe management of acne vulgaris is to start with topical benzoyl peroxide and topical retinoids, followed by topical antibiotics, oral antibiotics, and isoretinoin in increasing order of severity.", "link": "https://bit.ly/40qTdn2"} {"question": "A 55-year-old woman is brought to the emergency department by her husband with a 1 hour history of an unremitting headache. The headache started suddenly while she was eating dinner and she says it feels like the \u201cworst headache of my life.\u201d An emergent CT scan of the head without contrast confirms the diagnosis, and a CT angiogram identifies the source of bleeding. The patient undergoes surgical management of her condition. On hospital day 3, she is found to be disoriented to person, place, and time. She also develops nausea and vomiting. Her medical problems consist of heart failure for which she takes furosemide, spironolactone, and metoprolol, which were continued at admission. Her temperature is 99.6\u00b0F (37.6\u00b0C), blood pressure is 100/60 mmHg, pulse is 112/min, and respirations are 16/min. Physical examination shows poor skin turgor. Capillary refill time is 4 seconds. Serum laboratory results are shown below: Na+: 120 mEq/L Cl-: 92 mEq/L K+: 3.9 mEq/L HCO3-: 26 mEq/L BUN: 32 mg/dL Creatinine: 1.0 mg/dL Serum osmolality is 265 mEq/L and urine osmolality is 340 mEq/L. Urine sodium is 44 mEq/L. Which of the following is the most likely diagnosis?", "choicesA": "Addison disease", "choicesB": "Cerebral salt wasting", "choicesC": "Diuretic overuse", "choicesD": "Primary polydipsia", "choicesE": "Syndrome of inappropriate anti-diuretic hormone", "answer_idx": "B", "answer": "Cerebral salt wasting", "explanation": "This patient presents with a subarachnoid hemorrhage (\u201cworst headache of my life\u201d) which is treated and then subsequently develops nausea, vomiting, and altered mental status with reduced serum sodium concentration, indicative of symptomatic hyponatremia. In the setting of serum hypo-osmolality, urine hyper-osmolality, urine sodium concentration > 20 mEq/L, and hypovolemia (low blood pressure, decreased skin turgor, and prolonged capillary refill), this is indicative of cerebral salt wasting.\n\nThe mechanism of cerebral salt wasting is unclear, although some hypotheses point to increased release of brain natriuretic peptide after injury leading to suppression of renal sodium reabsorption, or sympathetic nervous system injury resulting in an impaired renin response. Patients classically present several days after cerebral injury or neurosurgery with hypovolemic hyponatremia and hypochloremia. Water retention results in serum hypo-osmolality, hyponatremia, and hypochloremia with urine hyper-osmolality and increased urine sodium excretion. In patients with prolonged cerebral salt wasting, hypokalemia can also occur (potassium may be normal in acute states). Treatment in mild or moderate cases consists of normal saline infusion to correct both volume status and hyponatremia. Treatment in severe cases consists of hypertonic saline. During correction, serum sodium must be monitored closely to prevent central pontine myelinolysis.\n\nCui et al. describe the differences between syndrome of inappropriate anti-diuretic hormone (SIADH) and cerebral salt wasting. They discuss how volume status serves as the key differentiator between these conditions. They recommend paying closer attention to total body electrolyte balance in understanding these conditions.\n\nIncorrect Answers:\nAnswer A: Addison disease, also known as primary adrenal insufficiency, results in salt wasting due to decreased mineralocorticoid activity. Because of reduced aldosterone release, patients also have hyperkalemia and metabolic acidosis. In contrast, this patient\u2019s potassium and bicarbonate levels are normal. Treatment is with steroids.\n\nAnswer C: Diuretic overuse is occasionally observed in patients using loop diuretics. Overuse of loop diuretics can cause hyponatremia, hypokalemia, and hypochloremia. Metabolic alkalosis is commonly seen due to volume contraction. Cerebral salt wasting is more likely in this patient presenting after a recent cerebral injury. Treatment is the cessation of the causative medication.\n\nAnswer D: Primary polydipsia can cause symptomatic hyponatremia. The urine would be maximally diluted (urine osmolality < 100 mEq/L) due to an appropriate physiological response to hyponatremia and excess free water intake. Treatment is cessation of excessive water consumption.\n\nAnswer E: Syndrome of inappropriate anti-diuretic hormone (SIADH) presents with identical findings as cerebral salt wasting with the exception of euvolemia as opposed to hypovolemia. An assessment of volume status is essential to differentiate. This patient has clear signs of hypovolemia. Treatment of SIADH is fluid restriction followed by ADH inhibitors, if needed.\n\nBullet Summary:\nCerebral salt wasting presents with hyponatremia, hypochloremia, and hypovolemia in the setting of recent cerebral injury or neurosurgery.", "link": "https://bit.ly/3QOJp1p"} {"question": "A 69-year-old man passes away after a 3-week hospitalization for liver failure, complicated by a myocardial infarction. A medical student rotating in the intensive care unit asked if she can practice performing intubations during her rotation. The resident suggests performing one on the deceased patient, as the student cannot harm him. Which of the following is the most appropriate course of action for the medical student?", "choicesA": "Ask the resident if the patient\u2019s family or the patient has been consented for this procedure", "choicesB": "Consult the hospital ethics committee", "choicesC": "Contact the patient's family to ask for permission to perform the procedure", "choicesD": "Intubate the patient with the resident's supervision", "choicesE": "Report the resident to hospital administration", "answer_idx": "A", "answer": "Ask the resident if the patient\u2019s family or the patient has been consented for this procedure", "explanation": "This medical student wants to perform a procedure on a deceased patient for educational purposes. Before proceeding, the student should ensure that permission has been obtained from the patient's family or the patient when he was alive.\n\nPerforming procedures on deceased patients is ethically permissible with appropriate consent and consideration of risks and benefits for patients, families, learners, staff, and the field of medicine. Supervising physicians must obtain informed consent from the patient while alive and with capacity or from a newly deceased patient\u2019s healthcare proxy or family. During the consent process, the supervisor should outline the patient's and family's rights, wider educational and societal benefits, potential risks, and ensure that performing the procedure would be consistent with the patient's values and wishes. Additionally, the supervising physician has a responsibility to ensure that a medical examiner has cleared the patient and that the patient is not an organ donor before proceeding. Finally, the procedure should be documented in the patient's chart.\n\nBerger and Cassell review the ethics of practicing procedures on deceased patients. They note that some ethical norms may suggest this practice is not appropriate. They recommend performing procedures when consent is obtained and it is ethically acceptable to do so.\n\nIncorrect Answers:\nAnswer B: Consulting the hospital ethics committee is not necessary, for with proper permission, trainees may perform procedures on deceased patients for educational purposes. The ethics committee should only be consulted when there is no clear course of action (an example may be no advanced directive with family members disagreeing on what the patient's wishes would have been for a critically ill patient).\n\nAnswer C: Contacting the patient's family to ask for permission to perform the procedure is incorrect, as the supervising physician needs to verify and document consent. Medical students cannot consent patients and family members. Moreover, the family should not be contacted if the patient has already approved or refused this when he had capacity.\n\nAnswer D: Intubating the patient with the resident's supervision is incorrect because proper permission must be ensured prior to performing educational procedures on deceased patients.\n\nAnswer E: Reporting the resident to hospital administration is incorrect because it is ethically permissible to perform an intubation on a deceased patient provided there is proper permission. The resident is not asking the medical student to do anything illegal or against hospital policy. Reporting the resident may be indicated if the resident was performing procedures against the patient's will.\n\nBullet Summary:\nTrainees may perform procedures for educational purposes on deceased patients if permission is obtained from the patient prior to death, the patient's family, or a healthcare proxy.", "link": "https://step2.medbullets.com/testview?qid=216264"} {"question": "A 26-year-old man presents to his primary care physician with a 6-month history of increasing lower back pain. He first felt the pain while lifting boxes at work but thought that he had just strained a muscle. The pain appears to be worse in the mornings and after rest. Exercise and physical activity appear to temporarily make the pain better. He has taken acetaminophen and ibuprofen for the pain. He has no significant medical history and takes no medications. His temperature is 98.6\u00b0F (37.0\u00b0C), blood pressure is 115/70 mmHg, pulse is 70/min, and respirations are 12/min. On physical exam, he is found to have limited flexion and extension of his spine as well as tenderness to palpation over the insertion of his patellar tendons bilaterally. Results from laboratory tests are shown below:\n\nSerum:\nErythrocyte sedimentation rate (ESR): 61 mm/hr\nC-reactive protein (CRP): 36 mg/L (normal <10 mg/L)\n\nWhich of the following is the most accurate test for this patient's condition?", "choicesA": "Bone scan", "choicesB": "Computed tomography", "choicesC": "Magnetic resonance imaging", "choicesD": "Radiograph", "choicesE": "Ultrasound", "answer_idx": "C", "answer": "Magnetic resonance imaging", "explanation": "This young man presents with low back pain that is relieved by physical activity, decreased spinal mobility, enthesitis of his patellar tendons, and elevated ESR/CRP values most likely has ankylosing spondylitis (AS). The most accurate diagnostic test for early ankylosing spondylitis is MRI.\n\nMultiple imaging tests can diagnose ankylosing spondylitis. Radiographs of the spine will show squaring of vertebrae with vertical or marginal syndesmophytes resulting in a \"bamboo spine\" appearance. The earliest radiographic sign is the erosion of the iliac side of the sacroiliac joint, but this finding is not sensitive in the early stages of the disease. MRI is sensitive in detecting sacroiliac inflammation, which makes this the best modality for early detection of ankylosing spondylitis in young patients. Initial therapy for ankylosing spondylitis includes lifestyle modifications (exercise, smoking cessation, physical therapy) and non-steroidal anti-inflammatory drugs (NSAIDs). Therapy for patients with inadequate response to NSAIDs includes tumor necrosis factor inhibitors (etanercept, infliximab, adalimumab) and interleukin 17 inhibitors (secukinumab, ixekizumab).\n\nJung et al. studied whether signs of inflammation on MRI are correlated with bone quality in patients with ankylosing spondylitis. The authors find that bone marrow edema is negatively correlated with the trabecular bone score. The authors recommend control of active bone inflammation to prevent osteoporosis in patients with ankylosing spondylitis.\n\nIncorrect Answers:\nAnswer A: Bone scan will show inflammation in the sacroiliac joints, but this study lacks specificity for this disease compared with other inflammatory forms of arthritis.\n\nAnswer B: CT will show bony changes but not active inflammation, so it is most commonly used to diagnose cervical fractures in patients with ankylosing spondylitis. While it can support the diagnosis, it is less accurate than an MRI.\n\nAnswer D: Radiographs will show erosion of the iliac side of the sacroiliac joint, but this finding is not sensitive in the early stages of the disease when bony ankylosis has not yet occurred. Radiographs may be an appropriate initial test.\n\nAnswer E: Ultrasound is useful in evaluating muscle strains or tendon tears; however, it is not able to detect bony abnormalities and is therefore of limited use in ankylosing spondylitis.\n\nBullet Summary:\nMRI is the most sensitive and accurate test for the early detection of ankylosing spondylitis.", "link": "https://step2.medbullets.com/testview?qid=216501"} {"question": "A 62-year-old man presents to the emergency department with chest pain and shortness of breath that he experienced while barbecuing. The pain is over his chest and in his left arm. The patient has a medical history of obesity, type 2 diabetes mellitus, and hypertension. His current medications include insulin, metformin, lisinopril, and hydrochlorothiazide. An ECG is performed and is seen in Figure A. His temperature is 99.5\u00b0F (37.5\u00b0C), pulse is 112/min, blood pressure is 100/70 mmHg, respirations are 18/min, and oxygen saturation is 95% on room air. The patient is appropriately managed and is admitted to the step-down unit. 2 days later, he complains of chest pain. A repeat ECG is performed and is seen in Figure B. Which of the following is the most appropriate next step in management for this patient's underlying condition?", "choicesA": "Angiography", "choicesB": "Aspirin", "choicesC": "Ibuprofen", "choicesD": "Prednisone", "choicesE": "Stenting", "answer_idx": "B", "answer": "Aspirin", "explanation": "This patient is presenting with chest pain several days after sustaining a myocardial infarction and has an ECG suggestive of pericarditis. This presentation is consistent with Dressler syndrome and the most appropriate initial step in management is aspirin.\n\nDressler syndrome is pericarditis that can occur after a myocardial infarction (MI). It most commonly occurs weeks to months after the MI, but can also occur within 1 week from an MI in rare cases. It is believed to result from an autoimmune reaction that occurs after damage to the heart (surgery and MI). It presents with chest pain, fever, pleuritic pain, and a pericardial friction rub. ECG demonstrates diffuse ST elevation with PR depression consistent with pericarditis. The most appropriate initial step in the management of this condition is symptomatic management with aspirin.\n\nLeib et al. review the evidence regarding the diagnosis and treatment of Dressler syndrome. They discuss how this condition is secondary pericarditis with or without pericardial effusion resulting from injury to the heart or pericardium. They recommend considering this syndrome in all patients with chest pain after a myocardial infarction.\n\nFigure/Illustration A demonstrates an ECG with ST elevation in leads II, III, and aVF (blue circles) suggesting a diagnosis of right-sided MI.\n\nFigure/Illustration B demonstrates an ECG with diffuse ST elevation and PR depression (blue circles) suggesting a diagnosis of pericarditis.\n\nIncorrect Answers:\nAnswers 1 & 5: Angiography and stenting would be appropriate management if this patient were experiencing a repeat MI. The diffuse ST elevation and PR depression seen several days after his myocardial infarction are more indicative of pericarditis rather than a repeat MI. In addition, the chest pain the patient is experiencing can also occur in pericarditis and obfuscates the diagnosis of repeat MI.\n\nAnswers 3 & 4: Ibuprofen, other NSAIDs, and corticosteroids such as prednisone are not first-line therapy in post-MI pericarditis as they can hinder the healing and remodeling process of the myocardium. Current guidelines dictate that they may be used in cases that are refractory to initial therapy.\n\nBullet Summary:\nDressler syndrome (post-MI pericarditis) presents with chest pain, pericardial friction rub, fever, and diffuse ST elevation with PR depression on ECG, and should be treated initially with aspirin.", "link": "https://bit.ly/3BV4U9Y"} {"question": "A 6-year-old boy presents to the emergency department after falling from his scooter. He has dull, aching pain along his left side where he hit the ground. He fell sideways off the scooter as he rounded a curve in the road. He has never had any serious injuries but that he always seems to bruise easily, especially after he started playing soccer this fall. His parents deny that he has an abnormal number of nosebleeds or bleeding from the gums. They have never seen blood in his stool or urine. His mother notes that her brother has had similar problems. His temperature is 98.6\u00b0F (37\u00b0C), blood pressure is 112/74 mmHg, pulse is 82/min, and respirations are 11/min. On physical exam, the patient has extensive bruising of the lateral left thigh and tenderness to palpation. Laboratory tests are performed and reveal the following:\n\nHemoglobin: 14 g/dL\nHematocrit: 41%\nMean corpuscular volume: 89 \u00b5m3\nReticulocyte count: 0.8%\nLeukocyte count: 4,700/mm3\nProthrombin time (PT): 13 seconds\nPartial thromboplastin time (PTT): 56 seconds\nBleeding time (BT): 4 minutes\n\nWhich of the following is the most likely underlying pathophysiology?", "choicesA": "Anti-platelet antibodies", "choicesB": "Factor 8 deficiency", "choicesC": "Factor 9 deficiency", "choicesD": "GP1b deficiency", "choicesE": "Von Willebrand factor deficiency", "answer_idx": "B", "answer": "Factor 8 deficiency", "explanation": "This young patient presents with extensive bruising following an injury, a family history of blood disorder, and a prolonged partial thromboplastin time (PTT) with a normal prothrombin time (PT) and bleeding time. These findings suggest a diagnosis of hemophilia A caused by factor 8 deficiency.\n\nHemophilia is a deficiency of factor 8 (hemophilia A) or factor 9 (hemophilia B). Common symptoms of hemophilia include hemarthrosis, intramuscular hematoma, and gastrointestinal or genitourinary bleeding. Laboratory testing is helpful in confirming the diagnosis, as hemophilia is characterized by an isolated prolongation of PTT without a change in PT. The factors involved in hemophilia participate only in the indirect cascade of coagulation factor activation. Similarly, since bleeding time depends only upon platelet function and not coagulation factors, hemophilia presents with a normal bleeding time. Treatment of hemophilia includes desmopressin in mild cases and factor replacement in moderate to severe disease.\n\nBerntorp et al. present a review of the evidence regarding the diagnosis and treatment of hemophilia. They discuss how these patients now have normal life expectancies with the advent of factor replacement therapy. They recommend being vigilant for the development of anti-factor antibodies.\n\nIncorrect Answers:\nAnswer A: Anti-platelet antibodies are responsible for idiopathic thrombocytopenic purpura (ITP). This disease is caused by autoimmune antibody formation that damages platelets and causes them to be removed by splenic macrophages in the reticuloendothelial system. It usually occurs in children after an acute viral infection and it presents with bleeding from mucosal surfaces with laboratory findings showing an increased bleeding time, decreased platelets, and a normal PT/PTT.\n\nAnswer C: Factor 9 deficiency (hemophilia B) presents nearly identically to hemophilia A with a normal PT, prolonged PTT, and normal bleeding time with a history of bleeding in male relatives. However, because hemophilia B is epidemiologically less common than hemophilia A, hemophilia A is a more likely diagnosis.\n\nAnswer D: GP1b deficiency causes Bernard-Soulier syndrome, which is a bleeding disorder characterized by unusually large platelets and a low platelet count. Like von Willebrand disease (vWD), patients often present with cutaneous and mucosal bleeding, abnormal bleeding from minor trauma, and a prolonged bleeding time, which this patient does not have. It can be distinguished from vWD by a ristocetin assay. Treatment can include tranexamic acid.\n\nAnswer E: Von Willebrand factor deficiency results in vWD, which is a bleeding disorder characterized by cutaneous and mucosal bleeding, menorrhagia, and gastrointestinal bleeding. Unlike this patient, patients with vWD also present with a prolonged bleeding time. Patients with vWD will also have an increased PT because this factor is responsible for stabilizing coagulation factor 8.\n\nBullet Summary:\nHemophilia A is caused by factor 8 deficiency and presents with hemarthrosis, frequent bruising, intramuscular hematoma, and gastrointestinal bleeding with a prolonged PTT with a normal PT and bleeding time.", "link": "https://bit.ly/3BoKeXI"} {"question": "A 4-week-old boy is brought to the pediatrician by his parents for an initial evaluation. His parents are concerned that he is not feeding well and has lost weight over the last 2 weeks. He was born at home via spontaneous vaginal delivery at 37 weeks of gestation to a G1P1 mother. His mother did not receive prenatal care. His blood pressure is 70/44 mmHg, pulse is 151/min, and respirations are 41/min. His weight is 3 kg (4th percentile) compared with 3.5 kg at birth (45th percentile). On physical exam, he is jaundiced with an enlarged liver and spleen. He does not appear to respond to visual stimuli, and further examination reveals bilateral clouding of the lens. Which of the following interventions could have avoided this patient's symptoms?", "choicesA": "Avoiding fruit juice and sweetened foods", "choicesB": "Changing to a soy based formula", "choicesC": "Providing imiglucerase enzyme replacement", "choicesD": "Removing phenylalanine from maternal diet during pregnancy", "choicesE": "Vitamin B6 supplementation", "answer_idx": "B", "answer": "Changing to a soy based formula", "explanation": "This patient who presents with failure to thrive, hepatosplenomegaly, and bilateral cataracts most likely has classic galactosemia. Patients with this disorder should avoid lactose-containing products by changing to a soy-based formula.\n\nClassic galactosemia is an autosomal recessive defect in galactose-1-phosphate uridyltransferase. This enzyme is involved in the conversion of galactose to glucose, and a deficiency of this enzyme results in the accumulation of galactose 1-phosphate in the liver, kidney, and brain. This metabolite acts as a phosphate sink, meaning that it traps all free phosphate in the cytosol and inhibits the formation of other phosphate-dependent metabolites such as adenosine triphosphate. This results in failure to thrive, vomiting, lethargy, and hepatosplenomegaly after ingestion of galactose-containing products such as milk. Patients will also develop bilateral cataracts if the deficit is not detected in the first few weeks after birth. Avoiding galactose by changing to a soy-based formula will avoid the complications of this disorder.\n\nTurck discusses using soy protein for infant feeding. The author notes that there are safety concerns with regards to the high phyto-estrogenic isoflavone content of these formulas. The author recommends that despite these safety concerns, these formulas should still be used in patients with classic galactosemia.\n\nIncorrect Answers:\nAnswer A: Avoiding fruit juice and sweetened foods would be effective in the treatment of hereditary fructose intolerance. This disease presents similarly to classic galactosemia, but cataracts would not be seen in patients with this disease.\n\nAnswer C: Providing imiglucerase enzyme replacement would be effective in the treatment of patients with Gaucher disease, which can also present with failure to thrive. However, associated findings in this disease would be anemia and avascular necrosis of the bones.\n\nAnswer D: Removing phenylalanine from the maternal diet during pregnancy would be effective in the treatment of patients with phenylketonuria, which can also present with failure to thrive. However, these patients classically present with a \"musty smell\" and eczema.\n\nAnswer E: Vitamin B6 supplementation may be effective in mitigating the negative symptoms of homocystinuria. This disease may also present with cataracts or lens dislocations. Associated findings in this disorder would be marfanoid appearance and arachnodactyly.\n\nBullet Summary:\nPatients with classic galactosemia should not be breastfed and should receive a galactose-free and lactose-free diet such as soy-based formulas.", "link": "https://bit.ly/3D10TBk"} {"question": "A 25-year-old man presents to his gastroenterologist for trouble swallowing. Whenever he eats solids, he regurgitates them back up. Given this patient's suspected diagnosis, the gastroenterologist performs a diagnostic test. Several hours later, the patient presents to the emergency department with chest pain and shortness of breath. His temperature is 99.5\u00b0F (37.5\u00b0C), blood pressure is 130/85 mmHg, pulse is 60/min, respirations are 12/min, and oxygen saturation is 99% on room air. On physical exam, the patient demonstrates a normal cardiopulmonary exam. His physical exam demonstrates no tenderness of the neck, a normal oropharynx, palpable crepitus above the clavicles, and minor lymphadenopathy. Which of the following is the most appropriate next step in management?", "choicesA": "Barium swallow", "choicesB": "Gastrografin swallow", "choicesC": "Magnetic resonance imaging", "choicesD": "Ultrasound", "choicesE": "Urgent surgery", "answer_idx": "B", "answer": "Gastrografin swallow", "explanation": "This patient is presenting with chest pain and palpable crepitus after endoscopy suggesting a diagnosis of esophageal perforation. The most appropriate next step in management is a gastrografin swallow.\n\nEsophageal perforation presents with severe chest pain and palpable crepitus typically occurring after endoscopy or repeated forceful vomiting. The most appropriate initial step in management is a water-soluble contrast image (gastrografin) in order to confirm the diagnosis. Alternatively, a CT scan or radiograph may also be appropriate initially. Once the diagnosis is confirmed, surgical correction is necessary in order to prevent gastric contents from continuing to enter the mediastinum. Broad-spectrum antibiotics and proton pump inhibitors should also be administered.\n\nKim reviews the factors associated with esophageal perforation and rupture. They discuss how causes include foreign body, iatrogenic, trauma, and forceful vomiting. They recommend urgent surgical treatment.\n\nIncorrect Answers:\nAnswer A: Barium swallow would be inappropriate as it is an irritating substance and could travel out of the perforated esophagus. This substance is only used in rare cases when gastrografin is not revealing, per the discretion of the surgeon.\n\nAnswer C: MRI would appropriately diagnose the condition but acquiring such imaging would take too long for such an urgent presentation. A gastrografin swallow is a more appropriate initial test.\n\nAnswer D: Ultrasound is inappropriate as it does not demonstrate the location of the perforation and the information obtained is dependent on the habitus of the patient and the operator.\n\nAnswer E: Urgent surgery would be necessary once the diagnosis is confirmed. Performing a rapid diagnostic step is necessary in order to establish the diagnosis first.\n\nBullet Summary:\nThe most appropriate initial step in the management of esophageal perforation is a gastrografin swallow followed by surgery.", "link": "https://bit.ly/3ZLKJGM"} {"question": "A 25-year-old woman is brought to the emergency department by her family. They found her somnolent next to her computer with an empty pill bottle around 3 hours ago. The family cannot recall the types of pill bottles that they found and did not bring them to the ED. The patient's medical history is notable for anxiety, obesity, hypertension, and polycystic ovarian syndrome. She is currently not on any medications. She was given an exercise program but she has not been compliant with these exercises. Her temperature is 99.5\u00b0F (37.5\u00b0C), pulse is 82/min, blood pressure is 125/85 mmHg, respirations are 11/min, and oxygen saturation is 97% on room air. Physical exam is within normal limits. The patient begins communicating with the physician and states that she did take acetaminophen but it was only a few pills. Her parents are certain the bottle was new. Which of the following is the most appropriate next step in management?", "choicesA": "Blood acetaminophen levels", "choicesB": "Charcoal", "choicesC": "Lactated ringer bolus", "choicesD": "N-acetylcysteine", "choicesE": "Syrup of ipecac", "answer_idx": "D", "answer": "N-acetylcysteine", "explanation": "This patient is presenting with acute acetaminophen intoxication. The most appropriate next step in management is to administer the antidote N-acetyl cysteine (NAC).\n\nIn a toxicology case where the ingestion is unknown, the most appropriate initial step in management is to begin treatment empirically with IV fluids, charcoal, NAC, naloxone, and glucose if needed. These agents are effective against the most common causes of intoxication. In cases where there is known acetaminophen ingestion, the most important treatment is prompt administration of NAC. This medication works by increasing the hepatic production of glutathione, which detoxifies the toxic metabolite of acetaminophen N-acetyl-p-benzoquinone imine (NAPQI). NAC is a safe, cheap, and effective treatment that could be life-saving if administered in a timely fashion. In massive overdoses, it is appropriate to start treatment empirically as it can be discontinued later. Otherwise, a 4 hour acetaminophen level should be collected and the level should be plotted on the Rumack Matthew nomogram to determine treatment. In chronic acetaminophen toxicity, the acute serum level play no role in guiding treatment.\n\nChiew et al. performed a meta-analysis regarding the treatment of acetaminophen overdose. They found that the administration of charcoal immediately after ingestion can decrease the rate of absorption of acetaminophen. They recommend early administration of N-acetyl cysteine in order to decrease the hepatotoxicity of this substance.\n\nIncorrect Answers:\nAnswer A: Blood acetaminophen levels should be ordered, but it is much less dire to confirm the diagnosis than it is to begin treatment given the high fatality rate with untreated acetaminophen overdose. Testing for all common intoxication should proceed while treatment is being administered. In this case, the concern for consumption of an entire bottle of acetaminophen would make it reasonable to treat first.\n\nAnswer B: Charcoal is an appropriate intervention if it is given in a timely manner; however, this patient consumed acetaminophen over 2 hours ago. In general, if the patient's airway is secure, it does no harm and could offer some benefit by decreasing the absorption of the substance of interest.\n\nAnswer C: IV fluid administration is the most appropriate initial step in the management of trauma when there is suspected bleeding and vitals are unstable (hypotension and tachycardia). Fluids are not urgently needed in this toxicity case with a patient who has stable vitals.\n\nAnswer E: Syrup of ipecac induces emesis in patients and was previously used in an attempt to have the patient vomit the toxic substance. This is no longer a preferred method of managing toxicities as patients can aspirate the vomitus or the toxic substance can induce more tissue damage during vomiting. In general, syrup of ipecac is the wrong answer choice for all toxicity cases.\n\nBullet Summary:\nAcetaminophen toxicity should be treated immediately with N-acetyl cysteine followed by further diagnostic workup including serum acetaminophen levels.", "link": "https://bit.ly/3om7HFU"} {"question": "A 22-year-old woman presents to the emergency department with shortness of breath. She was hiking when she suddenly felt unable to breathe and had to take slow deep breaths to improve her symptoms. The patient is a Swedish foreign exchange student and does not speak any English. Her medical history and current medications are unknown. Her temperature is 99.5\u00b0F (37.5\u00b0C), blood pressure is 127/68 mmHg, pulse is 120/min, respirations are 22/min, and oxygen saturation is 90% on room air. Physical exam is notable for poor air movement bilaterally and tachycardia. The patient is started on treatment. Which of the following parameters including forced expiratory volume in 1 second (FEV1), forced vital capacity (FVC), and diffusing capacity of carbon monoxide (DLCO) most appropriately describes this patient's underlying pathology?", "choicesA": "Decreased airway tone", "choicesB": "Increased FEV1", "choicesC": "Increased FEV1/FVC", "choicesD": "Increased FVC", "choicesE": "Normal DLCO", "answer_idx": "E", "answer": "Normal DLCO", "explanation": "This patient who presents with dyspnea and poor air movement bilaterally most likely has an acute asthma exacerbation. DLCO is the one parameter that is normal in these episodes.\n\nAsthma is an obstructive lung disease that results in intermittent episodes of respiratory compromise. Asthma presents with shortness of breath typically secondary to an allergen/antigen or with exposure to exercise or cold air that causes bronchospasm. Pulmonary function testing will typically reveal a decreased FEV1, a decreased FVC, and a decreased FEV1/FVC in both asthma and emphysema. The DLCO will be normal in asthma in contrast to emphysema where the DLCO is decreased. Patients should be treated with inhaled beta-adrenergic agonists and corticosteroids for acute exacerbations.\n\nPeng et al. review the evidence regarding patients who have COPD versus asthma. They discuss how patients with COPD have reduced DLCO. They recommend measuring parameters to distinguish between these entities.\n\nIncorrect Answers:\nAnswer A: Decreased airway tone does not describe asthma. Rather, hyperreactive airways with increased tone describes an asthma attack. Decreased airway tone may be seen in patients with connective tissue diseases such as Ehlers-Danlos syndrome.\n\nAnswers 2-4: Increased FEV1, FEV1/FVC, and FVC do not describe asthma. All of these laboratory values are decreased in asthma and emphysema. Increased respiratory performance can be seen in patients who are athletes or training for marathons.\n\nBullet Summary:\nBoth asthma and emphysema present with a decreased FEV1, FVC, and FEV1/FVC; however, asthma has a normal DLCO while COPD has a reduced DLCO.", "link": "https://bit.ly/46AmGNy"} {"question": "A 7-year-old boy presents to the emergency department after an episode of jerking his extremities followed by confusion. This has happened in the past, but his mother thought he was fabricating his symptoms. She has taken him to a free health clinic to have him seen by a doctor who prescribed medication; however, she has been unable to fill the prescription. He has had an MRI of the brain with no acute findings in the past. The mother states this episode lasted about 3 minutes and involved full-body jerking. After the episode, the child was non-responsive but was breathing on his own. The child has a history of migraine headaches with aura that resolve with ibuprofen. He is otherwise healthy and has no medical problems or recent illness and is up to date on his vaccinations. His temperature is 97.6\u00b0F (36.4\u00b0C), blood pressure is 100/64 mmHg, pulse is 90/min, respirations are 12/min, and oxygen saturation is 99% on room air. The patient is protecting his airway but is unable to answer questions. He is able to move the right upper and lower extremity. He initially is unable to move his left upper and lower extremity and has a left facial droop; however, after a few minutes, he has 4/5 strength in the affected side with resolution of his facial weakness. The mother notes that this is similar to his past episodes. Which of the following is the most appropriate management?", "choicesA": "Alteplase", "choicesB": "CT head", "choicesC": "Diazepam", "choicesD": "Observation", "choicesE": "Sumatriptan", "answer_idx": "D", "answer": "Observation", "explanation": "This patient is presenting after a seizure (tonic-clonic jerking of his extremities) and is currently postictal given that he is obtunded. Given that the patient is not moving the left side of his body, he is likely suffering from Todd paralysis, which can occur after seizures and resolves on its own with only observation.\n\nA generalized tonic-clonic seizure may present with full-body tonic-clonic activity with altered mental status. After a generalized tonic-clonic seizure, patients can be postictal and appear to be obtunded. Other than ensuring adequate oxygenation/ventilation and evaluating for the cause of the seizure, there is no further intervention needed as this postictal state will improve with time. After seizures, some patients present with Todd paralysis, which is a focal paralysis after a seizure. This may mimic a stroke, and thus the patient\u2019s risk factors and known seizure history should be factored into the decision. Todd paralysis does not require any intervention and will improve with time and observation only.\n\nXu et al. review Todd paralysis. They note that Todd paralysis is characterized by temporary limb weakness or hemiplegia and usually occurs following a seizure. They recommend that it can be difficult to differentiate Todd paralysis from a stroke in certain patient populations.\n\nBullet Summary:\nAnswer A: Alteplase is a thrombolytic that could be used in an acute ischemic stroke if given within 4.5 hours of symptom onset without any contraindications to its use. It would only be given after the CT scan of the head has been performed and other reversible causes have been addressed in the setting of acute ischemic stroke. Alteplase is not given when neurologic symptoms are thought to be secondary to Todd paralysis. In fact, seizure and Todd paralysis would be a contraindication to thrombolytic use in stroke.\n\nAnswer B: CT head, in addition to a fingerstick blood glucose, should immediately be performed in all patients with sudden onset focal neurologic deficits thought to be secondary to a stroke. A stroke would present with sudden onset focal neurologic deficits. However, it is common for patients to be obtunded and have focal neurologic deficits after a seizure and this would not warrant a stroke workup unless clinical suspicion exists. This patient has no risk factors for stroke and his improving strength suggests against a stroke. Moreover, he has a history of a normal MRI of the brain.\n\nAnswer C: Diazepam would only be given for a patient who is actively seizing as it is a benzodiazepine with a long half-life. Note that it is not indicated in a postictal patient as there is no ongoing seizure. Benzodiazepines are not given as preventive measures for seizures.\n\nAnswer E: Sumatriptan may be indicated to treat a migraine, which presents with a pulsatile, unilateral headache and is more common in women. Hemiplegic migraine can cause focal neurologic deficits, and it would be reasonable to treat a migraine headache if a patient had focal neurologic deficits and a history of known hemiplegic migraines. Patients will either have a frequent history of hemiplegic migraines themselves or in their family.\n\nBullet Summary:\nFocal neurologic deficits (Todd paralysis) can be seen after a seizure and will resolve with observation.", "link": "https://step2.medbullets.com/testview?qid=216507"} {"question": "A 44-year-old man presents to the emergency department acutely confused. The patient\u2019s wife states she found him lethargic at home. He has been sick the past week with diarrhea and has been staying home from work. He is otherwise healthy and does not take any medications. His temperature is 97.5\u00b0F (36.4\u00b0C), blood pressure is 62/32 mmHg, pulse is 185/min, respirations are 25/min, and oxygen saturation is 98% on room air. The patient has profuse, bloody diarrhea while in the emergency department. The patient is given 3L of ringer lactate and subsequently appears less confused, with a blood pressure of 100/70 mmHg. He is able to drink oral fluids and protect his airway. An ECG is performed as seen in Figure A. He continues to have diarrhea while in the emergency department. Which of the following is contraindicated in the management of this patient?", "choicesA": "Ceftriaxone", "choicesB": "Loperamide", "choicesC": "Magnesium", "choicesD": "Norepinephrine", "choicesE": "Normal saline", "answer_idx": "B", "answer": "Loperamide", "explanation": "This patient is presenting with infectious diarrhea (given his bloody diarrhea and hypotension) with unstable vitals that improve with fluids. In bloody diarrhea, loperamide is contraindicated and could worsen illness.\n\nBacterial diarrhea is commonly caused by organisms such as Campylobacter, Shigella, and Escherichia coli. It may present with bloody or purulent diarrhea and fever and may progress to sepsis or septic shock. Immediate management is centered on hydration. Unstable patients or those who cannot tolerate PO require IV fluids immediately. Otherwise, patients can rehydrate orally. Loperamide is an antimotility agent that decreases diarrheal output. It is contraindicated in bloody diarrhea as it can decrease the rate of fecal shedding and lead to bacteremia and a worsening clinical picture. Loperamide may be appropriate in controlling diarrhea in milder infectious forms of diarrhea, or in non-infectious conditions such as irritable bowel syndrome.\n\nBaker reviews loperamide. He discusses its indications, contraindications, and mechanism of action. They recommend that loperamide should not be given in invasive diarrhea.\n\nFigure/Illustration A demonstrates U waves on ECG (red arrows) that are significant for hypokalemia, which would be expected in a dehydrated patient with profuse diarrhea.\n\nIncorrect Answers:\nAnswer A: Ceftriaxone would be appropriate in this patient who has bacterial diarrhea and appears septic as his critically ill state warrants antibiotics. Note that this patient may receive broader antibiotics than ceftriaxone given how ill he appears. Antibiotics should not be routinely used for bacterial diarrhea in young, healthy patients with stable vitals.\n\nAnswer C: Magnesium could be appropriate in this patient even though his magnesium level is not stated as he is hypokalemic given his ECG is demonstrating U waves. The serum magnesium level does not adequately reflect intracellular magnesium levels. It requires replacement in many cases of hypokalemia.\n\nAnswer D: Norepinephrine could be appropriate in this patient if he remained persistently hypotensive despite fluid administration. Note that this patient could receive more fluids and is not frankly unstable. While norepinephrine may not be appropriate at this time, it is not contraindicated.\n\nAnswer E: Normal saline is generally avoided in diarrheal illness in patients who can tolerate PO as oral hydration is preferred to IV. However, IV fluids are not contraindicated and would be appropriate in this persistently hypotensive patient. Normal saline has a very high sodium and chloride load and can cause a hyperchloremic metabolic acidosis. For this reason, a lower-chloride solution may be preferred (e.g, Lactated Ringer).\n\nBullet Summary:\nLoperamide is contraindicated in bloody diarrhea.", "link": "https://step2.medbullets.com/testview?qid=216585"} {"question": "A 14-year-old boy is brought to the pediatrician for behavioral issues. At school, he interrupts class by making barking noises. His mother notes that he has been making throat-clearing noises daily for the past 2 years. He has no medical problems and takes no medications. He gets along with his 2 brothers, plays turn-taking games with his friends, and completes his chores. His temperature is 98.4\u00b0F (36.9\u00b0C), blood pressure is 118/72 mmHg, pulse is 72/min, and respirations are 16/min. On exam, he grimaces several times and blinks his eyes rapidly. He does not appear to be reacting to internal stimuli. His mood and affect are normal. Which of the following is the most appropriate treatment for this patient?", "choicesA": "Clozapine", "choicesB": "Dextroamphetamine", "choicesC": "Fluoxetine", "choicesD": "Haloperidol", "choicesE": "Methylphenidate", "answer_idx": "D", "answer": "Haloperidol", "explanation": "This patient presents with multiple motor tics (grimacing, eye blinking) and phonic tics (barking, throat-clearing) on a daily basis for more than 1 year, which are consistent with Tourette syndrome. Tourette syndrome can be medically treated with antipsychotic agents such as haloperidol.\n\nTourette syndrome presents with multiple motor tics (e.g., grimacing, sniffing) and 1 or more phonic tics (e.g., throat-clearing, barking) that occur for at least 1 year. Tics must begin before the age of 18 years, and their location, frequency, type, complexity, or severity must change over time. Attention-deficit hyperactivity disorder (ADHD) and obsessive compulsive disorder (OCD) are often co-morbid conditions. Common first-line treatments for Tourette syndrome-related tics include habit reversal training and alpha-2 agonists such as guanfacine or clonidine. However, the only pharmacologic treatments approved for Tourette syndrome in the United States are the antipsychotics haloperidol, pimozide, and aripiprazole. Risperidone is also used due to its favorable side effect profile.\n\nSeideman and Seideman review the current treatments for Tourette syndrome. They note it is a hyperkinetic movement disorder that often occurs in children and presents with motor and phonic tics. They recommend treating with both non-pharmacological and pharmacological treatments for optimal care.\n\nIncorrect Answers:\nAnswer A: Clozapine is an atypical antipsychotic that blocks dopamine D2 receptors but is not approved for treatment of Tourette syndrome. It is typically avoided due to the risk for agranulocytosis and is mainly used for treatment-resistant schizophrenia as a last-line agent. Weight gain is another significant side-effect.\n\nAnswer B: Dextroamphetamine is a stimulant commonly used for the treatment of ADHD. ADHD can be comorbid with Tourette syndrome but presents with hyperactivity (e.g., fidgeting, difficulty taking turns, blurting out answers) and/or inattentive symptoms (e.g., daydreaming, difficulty completing tasks) in at least 2 settings. While this patient\u2019s phonic tics are disruptive, he has no difficulty waiting his turn and completing his chores, making Tourette syndrome more likely.\n\nAnswer C: Fluoxetine is a selective serotonin reuptake inhibitor (SSRI) that may be used for the treatment of OCD. OCD is often co-morbid with Tourette syndrome but compulsions are present, which manifest as repetitive counting, hoarding, tapping, or error checking. This patient\u2019s behavior is better attributed to his tics rather than compulsions.\n\nAnswer E: Methylphenidate is a stimulant used for the treatment of ADHD. ADHD would present with hyperactivity and/or inattentive symptoms. The patient's vocal interruptions are better attributed to his phonic tics.\n\nBullet Summary:\nTourette syndrome is treated with habit reversal training, alpha-2 agonists (e.g., guanfacine, clonidine), and/or antipsychotics (e.g., haloperidol, risperidone).", "link": "https://step2.medbullets.com/testview?qid=216269"} {"question": "A 38-year-old man presents to the emergency department with nasal congestion, blurry vision, and diplopia. His nasal congestion started about 1 week ago and he noticed the blurry vision this morning when he struggled to read the words on his television screen. His medical history is significant for IV drug use, HIV (CD4: 47/mm^3), hypertension, hyperlipidemia, diabetes mellitus, and seasonal allergies. His home medications include hydrochlorothiazide, atorvastatin, metformin, cetirizine, darunavir, tenofovir, and emtricitabine. He denies recent IV drug use. His temperature is 100.8\u00b0F (38.2\u00b0C), blood pressure is 127/85 mmHg, pulse is 78/min, and respirations are 12/min. He has injected conjunctiva and rhinorrhea. His cranial nerves are intact, and his motor and sensory neurologic exam is normal. A MRI of the brain is performed and can be seen in Figure A. Which of the following is the most appropriate next step in management?", "choicesA": "Brain biopsy", "choicesB": "Empiric treatment with pyrimethamine-sulfadiazine", "choicesC": "Empiric treatment with dexamethasone", "choicesD": "Lumbar puncture", "choicesE": "Serology for anti-John Cunningham (JC) virus antibodies", "answer_idx": "B", "answer": "Empiric treatment with pyrimethamine-sulfadiazine", "explanation": "This patient with a past medical history of immunosuppression due to HIV (CD4 < 100/mm^3), new focal neurologic findings, and ring-enhancing lesions on head MRI should be treated empirically with pyrimethamine-sulfadiazine for 10-14 days.\n\nThe differential diagnosis for ring-enhancing lesions in patients with HIV is determined by the degree of immunosuppression. For patients with CD4 counts > 200/mm^3, the most likely diagnoses include primary brain tumors and metastases. In patients with CD4 counts < 200/mm^3, the differential includes toxoplasma encephalitis, primary CNS lymphoma, progressive multifocal leukoencephalopathy (PML), and other infections. Patients with CD4 counts < 100/mm^3 should have empiric treatment for toxoplasma encephalitis with pyrimethamine-sulfadiazine with follow-up head imaging after 10-14 days. If the patient fails to improve clinically or the size of the lesion does not change, the next step would be a biopsy of the lesion.\n\nRobert-Gangneux and Darde present a review of toxoplasmosis diagnosis and treatment. They discuss how the incidence of these infections has increased dramatically since the advent of organ transplantation and HIV. They recommend increasing vigilance and prophylaxis for this disease.\n\nFigure/Illustration A is an MRI of the brain that demonstrates a ring-enhancing lesion (red circle) in the right occipital lobe. This lesion is likely causing the patient's visual complaints and is consistent with toxoplasmosis.\n\nIncorrect Answers:\nAnswer A: Brain biopsy should be performed if the patient has no response clinically or on follow-up head imaging to empiric treatment with pyrimethamine-sulfadiazine for 10-14 days. Biopsy in these cases may reveal a CNS lymphoma or other neoplastic process.\n\nAnswer C: Empiric treatment with dexamethasone is indicated for patients with substantial mass effect on imaging and severely depressed mental status, neither of which is present in this patient.\n\nAnswer D: Lumbar puncture is contraindicated in patients with focal neurologic findings due to the risk of transtentorial herniation. Lumbar puncture would be indicated in the diagnosis of meningitis which presents with fever, headache, photophobies, and meningeal signs.\n\nAnswer E: There is no role for serology for anti-John Cunningham (JC) virus antibodies in the workup of ring-enhancing lesions. If it were safe to perform a lumbar puncture (if the patient had no focal findings and no evidence of mass effect on head imaging), it would be reasonable to perform PCR of the CSF for JC virus.\n\nBullet Summary:\nFor patients with a past medical history of HIV with CD4 <100 cells/\u00b5L, focal neurologic findings, and ring-enhancing lesions on head imaging, the next step in management is empiric treatment for toxoplasma encephalitis with pyrimethamine-sulfadiazine for 10-14 days.", "link": "https://step2.medbullets.com/testview?qid=108601"} {"question": "A 2-week-old boy is evaluated by his pediatrician for abnormal feet. The patient was born at 39 weeks via vaginal delivery to a G1P1 29-year-old woman. The patient has been breastfeeding and producing 5 stools/day. He is otherwise healthy. His temperature is 99.5\u00b0F (37.5\u00b0C), blood pressure is 60/38 mmHg, pulse is 150/min, respirations are 24/min, and oxygen saturation is 98% on room air. A cardiopulmonary exam is notable for a benign flow murmur. A musculoskeletal exam reveals the findings shown in Figure A. Which of the following is the most appropriate next step in management?", "choicesA": "Botulinum toxin injections", "choicesB": "Reassurance and reassessment in 1 month", "choicesC": "Serial casting", "choicesD": "Surgical pinning", "choicesE": "Surgical soft tissue release", "answer_idx": "C", "answer": "Serial casting", "explanation": "This patient is presenting with talipes equinovarus (congenital clubfoot) for which the most appropriate initial step in management is serial casting using the Ponseti method.\n\nTalipes equinovarus is described clinically as a foot that is cavus, adductus, varus, and equinus. Cavus occurs when the forefoot is pronated relative to the hindfoot. Adductus occurs when the forefoot is medially deviated compared to the hindfoot. Varus occurs when the heel is medially deviated compared to the ankle. Finally, equinus occurs when the ankle rests in a plantarflexed position. The most appropriate initial step in management for this condition is serial casting using the Ponseti method. Cases that are refractory to this intervention are treated with surgical interventions such as split tibialis anterior transfer.\n\nCady et al. review the evidence regarding the treatment of clubfeet. They discuss how the Ponseti method is now the standard of care in the treatment of this disorder. They recommend early diagnosis and treatment of these patients.\n\nFigure/Illustration A is a clinical photograph that demonstrates cavus, adductus, equinus, and varus positioning of the foot (red circles). This appearance is classically seen in congenital talipes equinovarus.\n\nIncorrect Answers:\nAnswer A: Botulinum toxin injections are often used to treat spasticity and tension headaches; however, they are not indicated for the treatment of talipes equinovarus.\n\nAnswer B: Reassurance and reassessment in 1 month would be inappropriate management and could lead to permanent sequelae. Patients require immediate treatment with serial casting in order to correct the deformity.\n\nAnswers 4 & 5: Surgical interventions are reserved for cases of clubfoot that are refractory to stretching and serial casting. A percutaneous Achilles tenotomy is part of the Ponseti method but tendon transfers and releases are reserved for recurrent cases.\n\nBullet Summary:\nThe most appropriate initial step in management for congenital clubfoot (talipes equinovarus) is serial casting using the Ponseti method.", "link": "https://bit.ly/3QrrmQ1"} {"question": "A 56-year-old woman presents to the emergency department with muscle weakness. Her symptoms have progressively worsened over the course of 2 weeks and are most significant in her lower extremities. She also notices increased urinary frequency. Approximately 1 month ago, she was diagnosed with calcium phosphate nephrolithiasis. Medical history is significant for rheumatoid arthritis diagnosed approximately 10 years ago and treated with methotrexate as well as type 2 diabetes mellitus treated with metformin. Her temperature is 98.6\u00b0F (37\u00b0C), blood pressure is 138/92 mmHg, pulse is 92/min, and respirations are 17/min. On physical exam, there is mild tenderness to palpation of the metacarpophalangeal and proximal interphalangeal joints. There is 4/5 power throughout the lower extremity. Laboratory testing is shown. Serum: Na+: 137 mEq/L Cl-: 106 mEq/L K+: 2.9 mEq/L HCO3-: 18 mEq/L Glucose: 115 mg/dL Creatinine: 1.0 mg/dL Urine pH: 5.6 Which of the following is the most appropriate next step in management?", "choicesA": "Administer hydrochlorothiazide", "choicesB": "Administer intravenous insulin", "choicesC": "Administer intravenous sodium bicarbonate", "choicesD": "Begin potassium replacement therapy with dextrose", "choicesE": "Increase methotrexate dose", "answer_idx": "C", "answer": "Administer intravenous sodium bicarbonate", "explanation": "This patient's non-anion gap metabolic acidosis, hypokalemia, alkaline urine, and history of a calcium phosphate kidney stone and rheumatoid arthritis are suggestive of type 1 (distal) renal tubular acidosis. The treatment of choice is alkali replacement (sodium bicarbonate).\n\nType 1 (distal) renal tubular acidosis results from impaired acid (H+) secretion in the distal nephron. Impaired acid secretion decreases the kidney's ability to acidify the urine, thus increasing the urine pH. Impaired acid secretion also results in potassium wasting resulting in hypokalemia. Acidemia causes the bone to release calcium phosphate which buffers the acid. The alkaline urine along with decreased citrate excretion (caused by acidemia) leads to a favorable environment to form calcium phosphate stones. Therefore, correction of metabolic acidosis is essential in managing this condition.\n\nPalmer et al. review the evidence regarding the treatment of renal tubular acidosis. They discuss how this disease can be divided into types based on the location of the deficit. They recommend using alkali therapy in patients with distal renal tubular acidosis.\n\nIncorrect Answers:\nAnswer A: Administering hydrochlorothiazide would be a treatment option for type 2 (proximal) renal tubular acidosis. Patients with this condition can have associated hypokalemia but would be expected to have an acidic urine pH (less than 5.5) after significant bicarbonate excretion.\n\nAnswer B: Administering intravenous insulin would further decrease the serum potassium and place the patient at risk of developing cardiac arrhythmia. This patient's most important concern is her renal tubular acidosis that is resulting in muscle weakness from hypokalemia. Her polyuria is caused by hypokalemia, as it decreases the kidney's responsiveness to antidiuretic hormone. Her serum glucose is also not significantly elevated to cause osmotic diuresis.\n\nAnswer D: The dextrose (in the replacement therapy with potassium) would trigger the pancreas to secrete insulin and would lead to an intracellular shift of potassium. This would not particularly improve extracellular potassium levels. Treatment should be aimed at addressing the root of the patient's symptoms. Potassium citrate can be administered in cases of severe hypokalemia and hypokalemia that persists after correcting the serum bicarbonate level.\n\nAnswer E: Increasing the methotrexate dose is not appropriate in this case. The patient's muscle weakness is more suggestive of hypokalemia secondary to type 1 renal tubular acidosis. She does not report symptoms of inflammatory arthritis (joint pain/stiffness that is worse in the morning and improves with use).\n\nBullet Summary:\nType 1 (distal) renal tubular acidosis is treated with alkali replacement (sodium bicarbonate or sodium citrate).", "link": "https://bit.ly/3OXaArd"} {"question": "A 3-month-old boy presents to the pediatrician with his mother for a well-child visit. The patient drinks 4 ounces of cow\u2019s milk formula every 3 hours. He usually stools 1 time per day and urinates up to 6 times per day. He regurgitates a moderate amount of formula through his nose and mouth after most feeds. He does not seem interested in additional feeding after these episodes of regurgitation and he has become progressively more irritable around meal times. The patient is starting to refuse some feeds. His mother denies seeing blood or streaks of red in his stool and she denies any family history of food allergies or dermatological problems. The patient\u2019s weight was in the 62nd percentile 4 weeks ago and he is now in the 48th percentile. His height and head circumference have followed similar trends. His temperature is 98.2\u00b0F (36.8\u00b0C), blood pressure is 72/47 mmHg, pulse is 125/min, and respirations are 27/min. On physical exam, the patient smiles reciprocally and can lift his head and chest when in the prone position. His abdomen is soft, non-tender, and non-distended. Which of the following is the most appropriate next step in management?", "choicesA": "Counsel on positioning and thickening feeds", "choicesB": "Initiate proton pump inhibitor", "choicesC": "Obtain abdominal ultrasound", "choicesD": "Provide reassurance", "choicesE": "Switch to hydrolyzed formula", "answer_idx": "A", "answer": "Counsel on positioning and thickening feeds", "explanation": "This patient presents with frequent regurgitation after feeding, worsening irritability, feeding aversion, and poor weight gain, which is concerning for gastroesophageal reflux disease (GERD). The most appropriate next step in management is counseling on proper positioning and thickening feeds.\n\nInfants with GERD may present with regurgitation or vomiting after feeding, significant irritability, feeding aversion, and poor weight gain or failure to thrive. Symptoms can usually be managed with lifestyle changes including more frequent, thicker, and lower volume feeds and positioning the patient upright during and after feeding. If symptoms continue, patients should undergo a trial of a dairy-free diet to address possible underlying food protein intolerance. Pharmacotherapy with proton pump inhibitors (PPI) is appropriate for patients with persistent symptoms despite these changes, although PPIs can increase the risk of pneumonia and infection with C. difficile.\n\nChabra and Peeples review the evidence regarding the treatment of neonatal GERD. They discuss how alterations in the infant's environment and feeding pattern are effective in most cases. They recommend proton pump inhibitors if conservative treatments are not effective.\n\nIncorrect Answers:\nAnswer B: Initiating medical therapy with a proton pump inhibitor is appropriate for patients who have already failed lifestyle changes, including positioning therapy, adjusting feeding patterns, and eliminating cow\u2019s milk and soy from the diet.\n\nAnswer C: Obtaining an abdominal ultrasound would help diagnose pyloric stenosis. Patients with pyloric stenosis typically present at a younger age (between 3 and 6 weeks of life) with non-bilious, projectile vomiting and a palpable \u201colive-shaped\u201d mass in the epigastrium. Treatment is with a pyloromyotomy.\n\nAnswer D: Providing reassurance would not be appropriate due to this patient\u2019s poor weight gain. Although most cases of gastroesophageal reflux are uncomplicated and self-resolve by 1 year of age, this patient\u2019s symptoms should be addressed to ensure he is meeting his nutritional requirements.\n\nAnswer E: Switching to a hydrolyzed formula to eliminate cow\u2019s milk and soy protein from the patient\u2019s diet would be appropriate if the patient\u2019s symptoms did not improve with changing position and thickening feeds.\n\nBullet Summary:\nThe most appropriate initial step in managing gastroesophageal reflux disease in infants is counseling on proper positioning and thickening feeds.", "link": "https://bit.ly/3PAOhbu"} {"question": "A 1-month-old girl presents to her pediatrician with her mother. The patient was born at 38 weeks gestation via Caesarean section for cervical incompetence. The patient\u2019s mother has no complaints. The child had a runny nose and cough for a few days last week. The patient\u2019s mother endorses decreased appetite during the aforementioned illness which has now returned to baseline. The patient\u2019s family history is significant for an older brother with glucose-6-phosphate dehydrogenase (G6PD) deficiency and a maternal uncle with cirrhosis secondary to chronic hepatitis B. On physical exam, the patient has scleral icterus and dark urine staining her diaper. Laboratory testing reveals the following: Serum: Na+: 137 mEq/L Cl-: 102 mEq/L K+: 4.2 mEq/L HCO3-: 24 mEq/L Urea nitrogen: 12 mg/dL Glucose: 96 mg/dL Creatinine: 0.36 mg/dL Alkaline phosphatase: 146 U/L Aspartate aminotransferase (AST): 86 U/L Alanine aminotransferase (ALT): 76 U/L Total bilirubin: 4.6 mg/dL Direct bilirubin: 3.8 mg/dL Which of the following is the most likely diagnosis?", "choicesA": "Decreased metabolism of bilirubin", "choicesB": "Increased enterohepatic circulation of bilirubin", "choicesC": "Increased production of bilirubin", "choicesD": "Obstruction of the extrahepatic biliary tree", "choicesE": "Obstruction of the intrahepatic biliary tree", "answer_idx": "D", "answer": "Obstruction of the extrahepatic biliary tree", "explanation": "This 2-month-old patient presents with jaundice, dark urine, mild transaminitis, and conjugated hyperbilirubinemia, which suggests the diagnosis of biliary atresia. Biliary atresia is characterized by obstruction of the extrahepatic biliary tree.\n\nBiliary atresia is a progressive, obliterative disease of the extrahepatic biliary tree. It presents in the first 8 weeks of life with jaundice, acholic stools, dark urine, hepatomegaly, mild transaminitis, and conjugated hyperbilirubinemia. The definitive diagnosis is made by a cholangiogram that will demonstrate the obliteration of these ducts. Patients who are untreated experience fatal neurological defects due to kernicterus and accumulation of bilirubin in the brain. Treatment usually involves hepatoportoenterostomy (Kasai procedure), in which the duodenum is directly anastomosed to the liver. Most patients also eventually require a liver transplant.\n\nHartley et al. review the evidence regarding the diagnosis and treatment of patients with biliary atresia. They discuss how researchers are trying to reverse the liver fibrosis associated with this disease. They recommend early referral for a Kasai procedure or liver transplant as needed.\n\nIncorrect Answers:\nAnswer A: Decreased metabolism of bilirubin occurs with enzymatic deficiencies such as those seen in Crigler-Najjar or Gilbert syndrome. Both Crigler-Najjar and Gilbert syndromes lead to unconjugated hyperbilirubinemia (much more severe in Crigler-Najjar syndrome) rather than the conjugated hyperbilirubinemia seen in this patient.\n\nAnswer B: Increased enterohepatic circulation describes the pathophysiology underlying breast milk jaundice. Breast milk jaundice classically peaks in the first 2 weeks of life and leads to unconjugated rather than conjugated hyperbilirubinemia. Treatment is usually supportive and patients can typically continue to breastfeed.\n\nAnswer C: Increased production of bilirubin can occur in hemolytic diseases such as G6PD deficiency or hereditary spherocytosis. These conditions would similarly lead to an unconjugated hyperbilirubinemia. Treatment is with addressing the underlying anemia with possible transfusion or splenectomy.\n\nAnswer E: Obstruction of the intrahepatic biliary tree describes Alagille syndrome, in which patients have a paucity of interlobular bile ducts. Alagille syndrome is less common than biliary atresia and usually presents with other associated features, such as short stature, dysmorphic facies, ocular findings, and cardiac or renal anomalies. Treatment is with ursodiol.\n\nBullet Summary:\nBiliary atresia presents in the first 8 weeks of life with jaundice and conjugated hyperbilirubinemia.", "link": "https://bit.ly/3skGIMT"} {"question": "A 59-year-old man presents to his primary care physician for abdominal pain and foul-smelling diarrhea that has persisted for the past 2 years. The patient has a medical history of alcoholism and has been admitted to the hospital multiple times for withdrawal, abdominal pain, and traumatic injuries from his drinking. The patient states that his last drink was more than 1 year ago. His temperature is 99.3\u00b0F (37.4\u00b0C), blood pressure is 115/78 mmHg, pulse is 78/min, respirations are 15/min, and oxygen saturation is 99% on room air. Physical exam reveals a non-tender, non-distended abdomen. Laboratory studies are notable for a serum calcium of 7.0 mg/dL. A computed tomography (CT) scan of the abdomen is performed as seen in Figure A. A Sudan Black stain of the patient\u2019s stool is positive. Which of the following is the most appropriate treatment for the underlying cause of this patient\u2019s diarrhea?", "choicesA": "Ciprofloxacin", "choicesB": "Loperamide", "choicesC": "Oral rehydration", "choicesD": "Pantoprazole", "choicesE": "Rifaximin", "answer_idx": "D", "answer": "Pantoprazole", "explanation": "This patient with a history of alcoholism is presenting with steatorrhea (foul-smelling diarrhea that stains with Sudan Black, a stain that detects fat in the stool), hypocalcemia, and findings suggestive of chronic pancreatitis on CT scan (inflammatory changes and calcification). His steatorrhea can be improved with the administration of proton pump inhibitors such as pantoprazole.\n\nChronic pancreatitis is a sequela that occurs after repeated bouts of acute pancreatitis. It can occur secondary to alcoholism or gallstones, and it may present with chronic abdominal pain, hypocalcemia, and signs of malnutrition and weight loss. Malnutrition and weight loss occur secondary to the pancreas\u2019 inability to produce pancreatic enzymes which leads to poor nutrient absorption. Lack of lipase can lead to fat wasting in the stool, steatorrhea, and fat-soluble vitamin deficiency (A, D, E, and K). The management of chronic pancreatitis is the cessation of the underlying cause, pancreatic enzyme replacement, and the administration of proton pump inhibitors. Proton pump inhibitors help with steatorrhea by increasing the pH as low pH inactivates pancreatic enzymes.\n\nSingh et al. review the diagnosis and management of chronic pancreatitis. The authors find that the diagnosis can be secured by typical findings on CT or magnetic resonance imaging (MRI) including pancreatic calcifications, ductal dilatation, and pancreatic atrophy. The authors recommend the use of endoscopic ultrasound in securing the diagnosis in patients with equivocal imaging findings and a high index of suspicion, such as patients with repeated episodes of acute pancreatitis.\n\nFigure/Illustration A is a CT scan of the abdomen demonstrating calcifications and chronic inflammatory injury (red arrows) suggesting a diagnosis of chronic pancreatitis.\n\nIncorrect Answers:\nAnswer A: Ciprofloxacin or ceftriaxone is the appropriate management of infectious diarrhea from a bacterial organism. This usually presents with fever, as well as bloody or purulent diarrhea. In general, young and healthy patients who are able to hydrate themselves should not be given antibiotics, even for infectious diarrhea. Elderly patients, immunosuppressed patients, or patients with symptoms that do not improve spontaneously should be managed with a stool culture and antibiotics.\n\nAnswer B: Loperamide is an anti-diarrheal agent that could be used for irritable bowel syndrome to decrease bowel motility and improve symptoms of diarrhea. Loperamide and other antidiarrheals should never be used for infectious causes of diarrhea as they will delay the fecal shedding of the organism or virus and could prolong symptoms.\n\nAnswer C: Oral rehydration is recommended for any patient with diarrhea and signs of volume depletion and is preferred to IV fluids as the integrity of the gut is protected. This patient\u2019s diarrhea is likely caused by his chronic pancreatitis; thus, oral rehydration may be indicated but does not address the underlying cause of this patient\u2019s diarrhea.\n\nAnswer E: Rifaximin is an antibiotic that targets gut flora and can decrease enteric flora and treat conditions such as small bowel bacterial overgrowth. Small bowel bacterial overgrowth presents in patients with conditions such as diabetic gastroparesis or scleroderma secondary to decreased bowel motility causing bacterial overgrowth and profuse diarrhea. Treating the underlying cause in addition to rifaximin will eliminate the excess bacteria.\n\nBullet Summary:\nProton pump inhibitors should be given in patients with persistent diarrhea in the setting of chronic pancreatitis to prevent the inactivation of pancreatic enzymes.", "link": "https://step2.medbullets.com/testview?qid=215171"} {"question": "A newborn boy is evaluated in the hospital nursery 2 hours after birth. The patient was born at 39 weeks of gestation to a 30-year-old primigravid via vaginal delivery. The patient\u2019s mother received routine prenatal care and the pregnancy was uncomplicated. The patient\u2019s anatomy ultrasound at 20 weeks of gestation was unremarkable. The patient\u2019s mother denies any family history of genetic diseases. The patient\u2019s APGAR scores were notable for poor muscle tone at both 1 and 5 minutes of life. The patient\u2019s birth weight is 2.6 kg (5 lb 11 oz), which is at the 5th percentile. His height and head circumference are in the 15th and 3rd percentile, respectively. His temperature is 97.1\u00b0F (36.2\u00b0C), blood pressure is 57/42 mmHg, pulse is 140/min, and respirations are 38/min. On physical exam, the patient has a wide nasal bridge, down slanting palpebral fissures, and widely spaced eyes. He has good respiratory effort with a high-pitched cry. This patient is most likely to have experienced a deletion on which of the following chromosomes?", "choicesA": "4p", "choicesB": "5p", "choicesC": "5q", "choicesD": "7q", "choicesE": "15q", "answer_idx": "B", "answer": "5p", "explanation": "This patient presents with a characteristic mew, microcephaly, hypertelorism, wide nasal bridge, and hypotonia, which suggests a diagnosis of cri-du-chat syndrome. Cri-du-chat syndrome is caused by a deletion on chromosome 5p.\n\nCri-du-chat syndrome is most commonly caused by a de novo partial deletion of chromosome 5p. Other cases may be caused by a parental translocation involving the relevant section of chromosome 5p. Patients present with a mew-like cry, microcephaly, hypertelorism, down slanting palpebral fissures, a broad nasal bridge, low-set ears, low birth weight, hypotonia, and intellectual disability. The cat-like cry usually resolves by 2 years of age. Treatment is supportive though some patients may have cardiac abnormalities that require surgical intervention.\n\nCerruti Mainardi reviews the evidence regarding the etiology and diagnosis of patients with Cri-du-chat syndrome. She discusses how the diagnosis is mostly made clinically at this time. She recommends karyotype and FISH studies for those with ambiguous features.\n\nIncorrect Answers:\nAnswer A: A deletion on chromosome 4p causes Wolf-Hirshhorn syndrome, which presents with growth restriction, microcephaly, congenital heart disease, severe intellectual disability and dysmorphic \"Greek warrior helmet\" facies of a high forehead, prominent glabella, arched eyebrows, hypertelorism, and epicanthal folds. Treatment is supportive though some patients may require surgical intervention for heart defects.\n\nAnswer C: A deletion on chromosome 5q results in Sotos syndrome, a pediatric overgrowth disorder characterized by an increase in head circumference, hypotonia, delayed acquisition of motor skills, and mild intellectual disability.\n\nAnswer D: A deletion on chromosome 7q may results in Williams syndrome, which presents with dysmorphic, \u201celfin\u201d facies, global developmental delay with a hypersocial personality, and supravalvular aortic stenosis. Treatment is supportive though some patients will require surgical intervention for aortic stenosis.\n\nAnswer E: A deletion on chromosome 15q may cause either Prader-Willi syndrome or Angelman syndrome, depending on whether the deletion affects the maternal or paternal chromosome. Prader-Willi syndrome classically presents with dysmorphic facies, hyperphagia, and hypogonadism. Angelman syndrome classically presents with intellectual disability, frequent smiling or laughing, and gait ataxia. Treatment is supportive with seizure control for those who develop seizures.\n\nBullet Summary:\nCri-du-chat syndrome (presents with a characteristic mew, microcephaly, hypertelorism, wide nasal bridge, and hypotonia) is most commonly caused by a deletion on chromosome 5p.", "link": "https://step2.medbullets.com/testview?qid=109452"} {"question": "A 55-year-old woman presents to a primary care physician with persistent nausea and abdominal discomfort. She has experienced these symptoms daily for the past 3 months. She feels bloated and has episodic loose and watery stools. She has a history of hypertension and hyperlipidemia for which she takes amlodipine and atorvastatin. She immigrated from Vietnam 6 months ago. Her temperature is 98.6\u00b0F (37.0\u00b0C), blood pressure is 110/60 mmHg, pulse is 70/min, and respirations are 18/min. Physical exam reveals a thin-appearing woman in no acute distress. Her mucous membranes are moist. Cardiac and lung examinations are unremarkable. Laboratory results are as follows: Leukocyte count: 13,000/mm^3 Segmented neutrophils: 54% Bands: 2% Eosinophils: 6% Basophils: 0.4% Lymphocytes: 30% Monocytes: 5% A vitamin D level is 26 ng/mL (reference: >= 30 ng/mL). Which of the following is the most appropriate test in making the diagnosis?", "choicesA": "Bowel wall biopsy", "choicesB": "CT scan of the abdomen", "choicesC": "Lower endoscopy", "choicesD": "Stool ova and parasite assay", "choicesE": "Stool toxin assay", "answer_idx": "D", "answer": "Stool ova and parasite assay", "explanation": "This patient who presents with persistent abdominal discomfort, nausea, bloating, diarrhea, eosinophilia, and signs of malabsorption (thin-appearing, vitamin D deficiency) in the setting of immigration from a resource-limited country most likely has a gastrointestinal parasite infection (Ascaris lumbricoides). The most appropriate next step in management is to obtain a stool ova and parasite assay.\n\nAscaris lumbricoides is a roundworm that is transmitted via the fecal-oral route (egg ingestion). While most patients are asymptomatic, there can be pulmonary and intestinal manifestations in the early and late phases, respectively. Pulmonary signs include dry cough, dyspnea, fever, and wheezing. Intestinal signs include abdominal discomfort, diarrhea, nausea, vomiting, malabsorption, and/or anorexia. The diagnosis should be suspected in patients with vague abdominal symptoms with travel history to an endemic area. The diagnosis is secured with stool microscopy for eggs or worms. Complications of this condition include intestinal obstruction or hepatobiliary/pancreatic involvement, presenting as biliary colic, obstructive jaundice, and/or ascending cholangitis. Treatment in patients with uncomplicated infection is with anti-helminth therapy (albendazole or mebendazole).\n\nParija et al. review the epidemiology of helminth infections globally. The authors note that many cases are caused by Ascaris species. They recommend prompt identification of these infections as they can cause lack of school attendance, anemia, and cognitive deficits if left untreated.\n\nIncorrect Answers:\nAnswer A: Bowel wall biopsy is used in the diagnostic work-up of suspected Whipple disease, which presents with chronic diarrhea, malabsorption, weight loss, and joint symptoms (arthralgias). This disease is progressive over years and would be less likely in this patient with disease onset of 3 months.\n\nAnswer B: CT scan of the abdomen is used in the diagnostic work-up of small bowel obstruction, which presents as crampy abdominal pain, constipation, obstipation, and/or vomiting. Bowel obstruction is an acute process that would be unlikely in this patient with chronic symptoms. Although bowel obstruction is a complication of Ascaris infection, this patient has no signs of obstruction. Diarrhea may occur secondary to infection of the colon as well including colitis or diverticulitis; however, these conditions typically cause pain and low grade fever.\n\nAnswer C: Lower endoscopy (colonoscopy) is used in the diagnosis of inflammatory bowel disease, consisting of Crohn disease and ulcerative colitis. Ulcerative colitis presents with bloody diarrhea, abdominal pain, weight loss, and fatigue. Eosinophilia would be unexpected, and this patient does not have bloody diarrhea.\n\nAnswer E: Stool toxin assay is used to diagnose Clostridioides difficile infection. This presents after recent antibiotic use with watery diarrhea, dehydration, fever, and nausea. It presents acutely and generally without eosinophilia.\n\nBullet Summary:\nParasitic gastrointestinal infections present with chronic diarrhea, abdominal discomfort, malabsorption, and eosinophilia, and are diagnosed using stool ova and parasite assays.", "link": "https://step2.medbullets.com/testview?qid=217603"} {"question": "A 55-year-old male bodybuilder presents to the emergency department with weakness of his right arm. The patient states he has experienced these symptoms for a few weeks; however, today his hand felt so weak he dropped his cup of tea. The patient has a medical history of diabetes. He drinks 2-7 alcoholic drinks per day and has smoked 2 packs of cigarettes per day since he was 25. The patient uses anabolic steroids. He has lost 17 pounds since he last came to the emergency department 1 month ago. His temperature is 99.5\u00b0F (37.5\u00b0C), blood pressure is 177/108 mmHg, pulse is 90/min, respirations are 17/min, and oxygen saturation is 98% on room air. Physical exam reveals decreased sensation in the right arm and 2/5 strength in the right arm and 5/5 strength in in the left arm. The patient states that he is experiencing a dull aching and burning pain in his right arm during the exam. Symptoms are not changed with changes in head and neck position. Which of the following is the most likely diagnosis?", "choicesA": "Apical lung tumor", "choicesB": "Brachial plexopathy", "choicesC": "Cerebral infarction", "choicesD": "Scalenus anticus syndrome", "choicesE": "Subclavian steal syndrome", "answer_idx": "A", "answer": "Apical lung tumor", "explanation": "This patient is presenting with unilateral upper extremity paresthesia and weakness suggestive of compression of the neurovascular structures of the upper extremity secondary to an apical lung tumor.\n\nAn apical lung tumor can compress the neurovascular structures in its proximity and cause pain, numbness, and weakness of the ipsilateral arm. A Horner syndrome can also be observed with compression of the sympathetic chain leading to ipsilateral miosis, ptosis, and anhidrosis in some cases. These malignancies typically occur in older male smokers and can be associated with symptoms such as fatigue, cough, and weight loss. Chest radiography will demonstrate an apical opacity and CT scan can confirm the size and location of the mass. Treatment is with chemotherapy and surgical excision of the tumor.\n\nDavis and Knight review the evidence regarding the diagnosis and treatment of patients with apical lung tumors. They discuss how these tumors can be associated with brachial plexus injuries. They recommend surgical excision of these tumors if they are resectable.\n\nIncorrect Answers:\nAnswer B: Brachial plexopathy could present with weakness and numbness of the ipsilateral arm but is a less likely diagnosis (and a diagnosis of exclusion). Treatment is with identification and surgical release of any underlying anatomic defects (such as compressive bands) and supportive therapy if none are found.\n\nAnswer C: Cerebral infarction (stroke) would present with sudden onset of acute neurological deficits. These would typically only affect one side of the body and would not necessarily be associated with a dull pain or numbness that is chronic. Treatment is with thrombolytics if patients meet criteria for acute therapy.\n\nAnswer D: Scalenus anticus syndrome occurs in bodybuilders or athletes with hypertrophied neck muscles that compress the brachial plexus. Symptoms are typically worsened by certain positions. Treatment is with physical therapy and surgical decompression in refractory cases.\n\nAnswer E: Subclavian steal syndrome presents with numbness, tingling, weakness in a limb, and CNS symptoms (vertigo, slurred speech, or stroke-like symptoms) that are worsened with activity and relieved with rest. Treatment is with vascular intervention to restore blood flow.\n\nBullet Summary:\nApical lung tumors can compress neurovascular structures in the upper extremity and cause pain, numbness, and tingling in the ipsilateral arm.", "link": "https://step2.medbullets.com/testview?qid=109967"} {"question": "A 27-year-old nurse presents to the emergency department with a 30-minute history of palpitations and chest pain. She has been experiencing nausea, vomiting, and diarrhea after eating leftovers 2 days ago. Her husband tells the physician that he is concerned that the patient has been increasingly obsessed about her weight recently. She has a history of major depressive disorder for which she takes escitalopram. Her temperature is 98.6\u00b0F (37.0\u00b0C), blood pressure is 110/72 mmHg, pulse is 110/min, and respirations are 11/min. Physical examination is notable for dry mucous membranes and tachycardia with regular rhythm. Laboratory testing shows the following results:\n\nSerum:\nNa+: 135 mEq/L\nCl-: 96 mEq/L\nK+: 2.9 mEq/L\nHCO3-: 36 mEq/L\n\nVenous blood gas shows a pH of 7.52. Which of the following tests is most appropriate in differentiating between different etiologies of this patient's lab abnormalities?", "choicesA": "Arterial blood gas", "choicesB": "Complete blood count", "choicesC": "Serum glucose", "choicesD": "Urine chloride", "choicesE": "Urine toxicology", "answer_idx": "D", "answer": "Urine chloride", "explanation": "This patient who presents with hypokalemia and metabolic alkalosis may either be vomiting or engaging in diuretic abuse. A urine chloride test can be used to distinguish between these possibilities because urine chloride will be high in diuretic abuse and will be low/normal in surreptitious vomiting.\n\nMetabolic alkalosis is an acid-base disorder that is due to an increased level of serum bicarbonate. This increase can be secondary to increased bicarbonate intake or loss of hydrogen ions. Etiologies consistent with increased intake include antacid use or milk-alkali syndrome. Etiologies consistent with hydrogen ion loss include gastrointestinal losses such as vomiting or renal losses such as hyperaldosteronism, Bartter syndrome, or diuretic abuse. Urine chloride can distinguish between gastrointestinal and renal losses because urine chloride levels will be high in patients with renal tubule dysfunction or diuretic abuse. This is due to increased urinary excretion of ammonium chloride in cases of diarrhea. On the other hand, urine chloride will generally be low in hyperaldosteronism due to loss of chloride with vomiting.\n\nWu et al. studied the causes of chronic hypokalemia as well as laboratory findings that can be used to distinguish between different etiologies. They show how urine sodium and chloride can be helpful for identifying the surreptitious use of diuretics. The authors recommend the use of the sodium-to-chloride excretion ratio to help diagnose patients with chronic normotensive hypokalemia.\n\nIncorrect Answers:\nAnswer A: Arterial blood gas is useful for determining the acid-base state of a patient because it simultaneously measures carbon dioxide and bicarbonate levels. This test is not as useful in determining different causes of metabolic alkalosis because all of these etiologies will have similar findings (high bicarbonate and compensatory high carbon dioxide), especially in patient who has already received a venous blood gas.\n\nAnswer B: Complete blood count is useful for determining whether a patient is anemic resulting in decreased oxygen delivery to end organs due to decreased carrying capacity for oxygen. In cases of severe anemia and hypoperfusion, peripheral tissues produce lactic acid which will result in an anion gap metabolic acidosis rather than metabolic alkalosis.\n\nAnswer C: Serum glucose elevation can result in increased diuresis as well as pseudohyponatremia (decreased measured sodium due to increased glucose). Alternatively, acidosis may also be caused by insulin deficiency in diabetic ketoacidosis. Neither of these findings is consistent with the lab results seen in this patient.\n\nAnswer E: Urine toxicology can be used to determine whether the patient is taking any substances of abuse. There is no pathognomonic acid-base finding that can aid in diagnosing overdose and/or substance abuse. However, most standard urine toxicology screens do not detect the use of diuretics.\n\nBullet Summary:\nUrine chloride can distinguish between gastrointestinal and renal causes of metabolic alkalosis.", "link": "https://step2.medbullets.com/testview?qid=216419"} {"question": "A 2-year-old boy is brought to the emergency room by his mother, who states that her son has been coughing uncontrollably for 3 hours. The child has a medical history of obesity and atopic dermatitis and uses topical steroid cream. His birth history is unremarkable. His family history is notable for asthma in his older sister and irritable bowel syndrome in his father. His temperature is 98.5\u00b0F (36.9\u00b0C), pulse is 120/min, blood pressure is 90/52 mmHg, and respirations are 32/min. On exam, he appears uncomfortable, is coughing continuously, and is drooling. Cardiopulmonary and abdominal exams are unremarkable. A chest radiograph is performed and the results are shown in Figures A and B. Which of the following is the most appropriate next step in management?", "choicesA": "Balloon catheter removal", "choicesB": "Endoscopic removal", "choicesC": "Ipecac administration", "choicesD": "Observation", "choicesE": "Surgical extraction", "answer_idx": "B", "answer": "Endoscopic removal", "explanation": "This child presents with foreign body ingestion with a button battery seen in the esophagus on radiography. The most appropriate next step in management is urgent endoscopic removal of the battery.\n\nButton batteries are commonly used in watches and small electronic devices. If these objects are ingested (usually by children), they can become lodged in the esophagus. Coughing, drooling, and chest discomfort are classic symptoms but a lack of symptoms does not negate the need for evaluation if suspicion is high. The location of the battery can be confirmed by radiograph and orthogonal views can localize the object in the esophagus versus the trachea. Endoscopy is the preferred method of removal as direct visualization of any associated esophageal injury is possible. The battery can cause ulceration or perforation if left in the esophagus due to chemical extravasation.\n\nBaharudin et al. review a number of cases of pediatric ingestions resulting in esophageal lodging. They discuss how a stack of coins can mimic the appearance of a battery. They recommend urgent intervention in cases of suspected battery ingestion.\n\nFigure/Illustration A is a chest radiograph depicting a round object lodged in the esophagus, with a \u201cdouble halo\u201d sign (red circle). This sign is an indication that this object is a battery rather than a coin. Figure/Illustration B is a lateral chest radiograph demonstrating that the battery is posterior to the trachea (red circle). This view confirms that the location of the battery is in the esophagus.\n\nIncorrect Answers:\nAnswer A: Balloon catheter removal can be attempted for ingested button batteries but is typically used only if endoscopy is not available. There is more potential for tissue damage without direct visualization, so ingestion time greater than 2 hours is generally a contraindication for this method.\n\nAnswer B: Ipecac administration can induce emesis and is rarely used for toxic ingestions. Vomiting can cause further lodging in the esophagus and even more tissue damage. Ipecac is therefore not recommended in the modern management of toxic ingestions.\n\nAnswer D: Observation would be appropriate if there is an ingested object that has already traveled further into the gastrointestinal tract (if they are small and smooth), as these objects would likely pass on their own. A classic example would be a coin ingestion that would appear as a single round object rather than a double ring appearance. A button battery in the esophagus is an emergency and should be removed urgently.\n\nAnswer E: Surgical extraction of a battery with associated repair may be required if there is suspicion of an esophageal-vascular fistula, mediastinitis, or other such complications. It may also be necessary if a magnet was simultaneously ingested. In the absence of these complications, endoscopic removal should be attempted first.\n\nBullet Summary:\nButton batteries lodged in the esophagus require urgent endoscopic removal.", "link": "https://bit.ly/3DIAhW0"} {"question": "A 45-year-old man presents to a physician due to difficulty swallowing and slurred speech. For the past 3 months, the patient has been coughing when he drinks liquids. His speech also sounds different according to his wife. The patient has no significant medical history. His temperature is 98.6\u00b0F (37.0\u00b0C), blood pressure is 110/80 mmHg, pulse is 60/min, and respirations are 12/min. On physical exam, he is unable to fully extend his tongue, and there are fasciculations on his tongue. When tapping the mandible at the chin while the mouth is held open, there is pronounced upward movement of the lower jaw. Sensory exam reveals no deficits. Which of the following therapies is most likely to improve survival for this patient's disease process?", "choicesA": "Baclofen", "choicesB": "Dextromethorphan and quinidine", "choicesC": "Edaravone", "choicesD": "Riluzole", "choicesE": "Tizanidine", "answer_idx": "D", "answer": "Riluzole", "explanation": "This patient with dysphagia (difficulty swallowing water), dysarthria (altered and slurred speech), lower motor neuron deficits (tongue fasciculations and weakness), upper motor neuron deficits (exaggerated jaw jerk reflex), and preserved sensory function most likely has amyotrophic lateral sclerosis (ALS). Riluzole, a glutamate inhibitor, is currently the only Food and Drug Administration (FDA) approved medication that improves survival in patients with ALS.\n\nALS is a neurodegenerative disease that affects both upper and lower motor neurons. Upper motor neuron signs include spasticity and hyperreflexia, while lower motor neuron signs include muscle atrophy and fasciculations (spontaneous, involuntary muscle contraction and relaxation). In patients with bulbar-onset ALS, the initial clinical presentation is dysarthria, dysphagia, tongue atrophy, and abnormal jaw jerk reflex. Riluzole decreases the rate of glutamate release from presynaptic neurons, which is thought to decrease excitatory neurotoxicity. Riluzole improves survival by 2-6 months in patients with ALS and may have a greater survival benefit in patients with bulbar-onset ALS.\n\nMasori and Van Damme review the genetics, pathophysiology, classic clinical presentation, extra-motor manifestations, diagnostic workup, and treatment for ALS. They discuss how riluzole provides a survival benefit in patients with this disease. They recommend treating patients with ALS using riluzole as tolerated.\n\nIncorrect Answers:\nAnswer A: Baclofen is a gamma-aminobutyric acid (GABA) agonist that reduces the release of excitatory neurotransmitters to decrease spasticity in ALS. Along with tizanidine, baclofen is a first-line symptomatic treatment for spasticity in ALS. Baclofen does not improve survival in ALS.\n\nAnswer B: Dextromethorphan and quinidine is a combination of a sigma opioid receptor agonist and a sodium channel blocker used to control pseudobulbar affect in patients with ALS. Pseudobulbar affect is a sudden uncontrollable outburst of laughter or tearfulness that affects almost half of patients with ALS. Dextromethorphan and quinidine has not been shown to improve survival in ALS.\n\nAnswer C: Edaravone is an intravenous antioxidant that may decrease the progression of early ALS. Although it is now FDA approved, studies have not yet shown that edaravone improves survival in patients with ALS. Further studies will be needed in order to determine the risks and benefits of using this drug.\n\nAnswer E: Tizanidine is a central alpha-2-adrenergic receptor agonist that decreases spasticity in ALS by increasing presynaptic inhibition of motor neurons. Excess spasticity in ALS can negatively impact coordination, ambulation, and other activities of daily living. Tizanidine and other antispastic medications do not improve survival in patients with ALS.\n\nBullet Summary:\nRiluzole is a glutamate inhibitor that improves survival in patients with amyotrophic lateral sclerosis.", "link": "https://bit.ly/3QdXJSe"} {"question": "A 55-year-old male bodybuilder presents to the emergency department with weakness of his right arm. The patient states he has experienced these symptoms for a few weeks; however, today his hand felt so weak he dropped his cup of tea. The patient has a medical history of diabetes. He drinks 2-7 alcoholic drinks per day and has smoked 2 packs of cigarettes per day since he was 25. The patient uses anabolic steroids. He has lost 17 pounds since he last came to the emergency department 1 month ago. His temperature is 99.5\u00b0F (37.5\u00b0C), blood pressure is 177/108 mmHg, pulse is 90/min, respirations are 17/min, and oxygen saturation is 98% on room air. Physical exam reveals decreased sensation in the right arm and 2/5 strength in the right arm and 5/5 strength in in the left arm. The patient states that he is experiencing a dull aching and burning pain in his right arm during the exam. Symptoms are not changed with changes in head and neck position. Which of the following is the most likely diagnosis?", "choicesA": "Apical lung tumor", "choicesB": "Brachial plexopathy", "choicesC": "Cerebral infarction", "choicesD": "Scalenus anticus syndrome", "choicesE": "Subclavian steal syndrome", "answer_idx": "A", "answer": "Apical lung tumor", "explanation": "This patient is presenting with unilateral upper extremity paresthesia and weakness suggestive of compression of the neurovascular structures of the upper extremity secondary to an apical lung tumor.\n\nAn apical lung tumor can compress the neurovascular structures in its proximity and cause pain, numbness, and weakness of the ipsilateral arm. A Horner syndrome can also be observed with compression of the sympathetic chain leading to ipsilateral miosis, ptosis, and anhidrosis in some cases. These malignancies typically occur in older male smokers and can be associated with symptoms such as fatigue, cough, and weight loss. Chest radiography will demonstrate an apical opacity and CT scan can confirm the size and location of the mass. Treatment is with chemotherapy and surgical excision of the tumor.\n\nDavis and Knight review the evidence regarding the diagnosis and treatment of patients with apical lung tumors. They discuss how these tumors can be associated with brachial plexus injuries. They recommend surgical excision of these tumors if they are resectable.\n\nIncorrect Answers:\nAnswer B: Brachial plexopathy could present with weakness and numbness of the ipsilateral arm but is a less likely diagnosis (and a diagnosis of exclusion). Treatment is with identification and surgical release of any underlying anatomic defects (such as compressive bands) and supportive therapy if none are found.\n\nAnswer C: Cerebral infarction (stroke) would present with sudden onset of acute neurological deficits. These would typically only affect one side of the body and would not necessarily be associated with a dull pain or numbness that is chronic. Treatment is with thrombolytics if patients meet criteria for acute therapy.\n\nAnswer D: Scalenus anticus syndrome occurs in bodybuilders or athletes with hypertrophied neck muscles that compress the brachial plexus. Symptoms are typically worsened by certain positions. Treatment is with physical therapy and surgical decompression in refractory cases.\n\nAnswer E: Subclavian steal syndrome presents with numbness, tingling, weakness in a limb, and CNS symptoms (vertigo, slurred speech, or stroke-like symptoms) that are worsened with activity and relieved with rest. Treatment is with vascular intervention to restore blood flow.\n\nBullet Summary:\nApical lung tumors can compress neurovascular structures in the upper extremity and cause pain, numbness, and tingling in the ipsilateral arm.", "link": "https://bit.ly/3sB1jwE"} {"question": "A 41-year-old man presents to his primary care provider after seeing bright red blood in the toilet bowl after his last 2 bowel movements. He reports that he also noticed some blood mixed with his stool. The patient denies abdominal pain or any changes in his stool habits. He notes a weight loss of 8 pounds in the last 2 months with no changes in his diet or exercise habits. His medical history is significant for an episode of pancreatitis 2 years ago for which he was hospitalized for several days. He drinks 2-3 beers on the weekend and he has never smoked. He has no family history of colon cancer. His temperature is 97.6\u00b0F (36.4\u00b0C), blood pressure is 135/78 mmHg, pulse is 88/min, and respirations are 14/min. On physical exam, his abdomen is soft and non-tender to palpation. Bowel sounds are present, and there is no hepatomegaly. Which of the following is the most appropriate next step in management?", "choicesA": "Abdominal CT", "choicesB": "Anoscopy", "choicesC": "Barium enema", "choicesD": "Colonoscopy", "choicesE": "Ultrasound of abdomen", "answer_idx": "D", "answer": "Colonoscopy", "explanation": "This patient presents with hematochezia and weight loss, which raises concern for colorectal cancer. The most appropriate next step in management is performing a colonoscopy.\n\nThe differential diagnosis of hematochezia includes colorectal cancer, inflammatory bowel disease, diverticulosis, and hemorrhoids. The evaluation depends in large part on the age of the patient and whether the patient presents with alarm symptoms such as weight loss, abdominal pain, change in bowel habits, or iron-deficiency anemia. If the patient is over the age of 50, has alarm symptoms, or has a family history of colorectal cancer, the evaluation should begin with a colonoscopy to rule out malignancy. If the patient is between the ages of 40 and 50 and without other red flags, the patient can elect to undergo sigmoidoscopy as a less invasive procedure than colonoscopy. If the patient is under the age of 40 and without alarm symptoms, the workup may begin with an anoscopy to first evaluate for hemorrhoids. Once cancer has been ruled out, additional evaluation for chronic inflammatory diseases such as ulcerative colitis can be conducted.\n\nChurch presents evidence regarding the current use of colonoscopy. He discusses how the detection of neoplasia during colonoscopy is suboptimal despite its use as the current screening tool of choice. He recommends understanding which patients are missed and how to better detect neoplasias in these patients.\n\nIncorrect Answers:\nAnswer A: Abdominal CT might be used to evaluate for metastases if the diagnosis of colorectal cancer is confirmed but it would not be used to diagnose the primary malignancy. A CT scan would be the most appropriate initial step in diagnosis if diffuse metastasis were already suspected such as a presentation of severe weight loss and an enlarged, palpable, and nodular liver.\n\nAnswer B: Anoscopy is used for the evaluation of hemorrhoids in a patient < 40 years of age who presents with bright red blood per rectum in the absence of alarm symptoms. It would not be the most appropriate next step for a patient who presents with unexplained weight loss as it cannot be used to diagnose colorectal cancer.\n\nAnswer C: Barium enema is sometimes used to screen for colorectal cancer in conjunction with other tests in asymptomatic patients, but it would not be used to diagnose malignancy in a patient with alarm symptoms such as hematochezia and unexpected weight loss.\n\nAnswer E: Ultrasound of the abdomen would be effective in diagnosing conditions such as intraperitoneal bleeding and cholecystitis. There is no evidence of abdominal trauma that would suggest intraperitoneal bleeding in this patient. A positive ultrasound with free fluid would be treated with diagnostic laparoscopy.\n\nBullet Summary:\nIf a patient is over the age of 50 or has alarm symptoms of weight loss, abdominal pain, change in bowel habits, or iron-deficiency anemia, the patient should undergo a colonoscopy to rule out colorectal cancer.", "link": "https://bit.ly/42tvFxa"} {"question": "A 57-year-old man presents to the emergency department for shortness of breath. He was riding the train home from a business conference when his symptoms began. The patient has a medical history of obesity, diabetes, diabetic nephropathy, hypertension, and a 40-pack-year smoking history. His current medications include atorvastatin, lisinopril, insulin, metformin, and nicotine gum. His temperature is 99.5\u00b0F (37.5\u00b0C), blood pressure is 130/87 mmHg, pulse is 120/min, respirations are 15/min, and oxygen saturation is 93% on room air. An ECG is ordered as seen in Figure A. A chest radiograph is ordered as seen in Figure B. Laboratory values are ordered as seen below: Serum: Na+: 137 mEq/L Cl-: 105 mEq/L K+: 4.1 mEq/L HCO3-: 24 mEq/L BUN: 22 mg/dL Glucose: 129 mg/dL Creatinine: 1.2 mg/dL Ca2+: 10.1 mg/dL AST: 11 U/L ALT: 12 U/L On physical exam, the patient appears to be in distress. His cardiac exam is notable for tachycardia. Pulmonary exam reveals bilateral air movement without any wheezes or crackles. The patient's right lower extremity appears mildly inflamed, and palpation of the calf elicits pain. Which of the following is the most appropriate next step in management?", "choicesA": "Aspirin", "choicesB": "Cardiac troponins", "choicesC": "CT scan of the chest without contrast", "choicesD": "Heparin", "choicesE": "Ventilation perfusion scan", "answer_idx": "D", "answer": "Heparin", "explanation": "This patient is presenting with a score of 7.5 on Wells criteria suggesting a diagnosis of pulmonary embolism. The most appropriate initial step in management is heparin administration.\n\nPatients with suspected pulmonary embolism (PE) can be evaluated with Wells criteria which consists of: clinical signs and symptoms of DVT (3 points), PE is the most likely diagnosis (3 points), pulse > 100 (1.5 points), immobilization for 3 days, or surgery within 4 weeks (1.5 points), history of DVT or PE (1.5 points), hemoptysis (1 point), and malignancy (1 point). Scores higher than 3 suggest a high probability of PE. Patients will often present with shortness of breath, with ECG findings of sinus tachycardia, and a chest radiograph that is typically normal. With a high pre-test probability, the most important initial step in management is anticoagulation with heparin versus further diagnostic workup.\n\nDoherty reviews the evidence regarding the diagnosis and treatment of pulmonary embolism. He discusses how clinical criteria such as Wells criteria can be helpful in assessing the likelihood of a PE. He recommends immediate treatment in patients who have a PE.\n\nFigure/Illustration A is an ECG demonstrating sinus tachycardia with a P wave, a QRS complex, and a T wave during every beat (red circle). This is the most common ECG presentation in PE, though right heart strain can also be seen.\n\nFigure B is a chest radiograph that is within normal limits without evidence of fluid in the dependent portions of the lung (red circles). This is the most common radiological presentation in PE.\n\nIncorrect Answers:\nAnswer A: Aspirin would be the most appropriate initial step in the management of MI and can reduce mortality by up to 25% if administered promptly. PE is a more likely diagnosis in this patient given the ECG that only demonstrates sinus tachycardia.\n\nAnswer B: Cardiac troponins could be used to confirm the diagnosis of myocardial infarction (MI). In the presence of this patient's ECG (which does not demonstrate ST elevation), MI is a less likely diagnosis. Treatment would be with aspirin in patients who had an MI followed by emergent revascularization.\n\nAnswer C: A CT scan of the chest without contrast would not confirm the diagnosis in this patient; however, a CT angiogram would be the most accurate test. It is more important to begin treatment with heparin when there is a very high clinical suspicion of PE.\n\nAnswer E: A ventilation perfusion scan is the most accurate test for PE in a patient who cannot tolerate contrast. This test has a low specificity.\n\nBullet Summary:\nThe most appropriate initial step in the management of a pulmonary embolism in patients with a high pre-test probability is anticoagulation with heparin.", "link": "https://step2.medbullets.com/testview?qid=109453"} {"question": "An 18-year-old man presents to his primary care provider with his parents for a sports physical. He was last seen in the clinic several months ago when he was diagnosed with attention deficit hyperactivity disorder (ADHD). He was started on methylphenidate at that time and the patient now reports improvement in his ability to concentrate in school and at home. He hopes to play baseball in college and has begun lifting weights daily in preparation for baseball season. The patient has a healthy diet to fuel his exercise regimen. His parents have no concerns and are pleased with the recent improvement in his grades. His temperature is 98.6\u00b0F (37\u00b0C), blood pressure is 115/71 mmHg, pulse is 72/min, and respirations are 12/min. On physical exam, the patient has tall stature with average muscle mass for his age. He has no dysmorphic features. His chest has a normal appearance other than mild gynecomastia. The patient has sparse facial hair and a moderate amount of coarse pubic hair that extends across the pubis and spares the medial thighs. His testes are small and firm. Laboratory testing is performed and reveals the following:\n\nFollicle-stimulating hormone (FSH): 42 mIU/mL\nLuteinizing hormone (LH): 38 mIU/mL\n\nWhich of the following is the most likely etiology of this patient\u2019s presentation?", "choicesA": "Anabolic steroid use", "choicesB": "CGG trinucleotide repeat disorder", "choicesC": "CTG trinucleotide repeat disorder", "choicesD": "Failure of neuronal migration", "choicesE": "Meiotic nondisjunction", "answer_idx": "E", "answer": "Meiotic nondisjunction", "explanation": "This patient presents with tall stature, gynecomastia, and small testes with elevated FSH and LH, which suggests a diagnosis of Klinefelter syndrome. Klinefelter syndrome is usually caused by meiotic nondisjunction that results in a 47,XXY genotype.\n\nKlinefelter syndrome is the most common cause of primary hypogonadism. Patients with Klinefelter syndrome present with tall stature, neurocognitive difficulties (ADHD), and features of hypogonadism including gynecomastia, small testes, small phallus, hypospadias, underdeveloped secondary sex characteristics, and cryptorchidism. Patients without hypospadias or cryptorchidism are often not diagnosed until after puberty when the symptoms of gynecomastia and small testes become more prominent. Because the hypogonadism in Klinefelter syndrome is caused by testicular fibrosis, laboratory results demonstrate low testosterone and elevated FSH and LH levels. Treatment for this condition includes androgen replacement therapy.\n\nLanfranco et al. review the evidence regarding the treatment of Klinefelter syndrome. They discuss how modern fertility augmentation therapy has allowed some of these patients to produce viable offspring. They recommend having a detailed discussion with these patients and their partners regarding possible outcomes.\n\nIncorrect Answers:\nAnswer A: Anabolic steroid use causes decreased levels of FSH and LH due to the suppression of GnRH release by the hypothalamus. This results in decreased FSH and LH release by the pituitary gland. Anabolic steroid use would not present with signs of hypogonadism.\n\nAnswer B: The CGG trinucleotide repeat disorder characterizes fragile X syndrome. Fragile X presents with macroorchidism rather than hypogonadism and patients typically have dysmorphic features of a long, narrow face with large ears, a prominent forehead, and a prominent chin. Fragile X is the most common cause of inherited intellectual disability.\n\nAnswer C: The CTG trinucleotide repeat disorder characterizes myotonic dystrophy. Although myotonic dystrophy presents with hypogonadism, patients would also present with symptoms of progressive weakness, such as facial weakness, dysphagia, or hand grip weakness. Treatment is with cardiac pacemakers and ventilatory support.\n\nAnswer D: Failure of neuronal migration characterizes Kallmann syndrome. Kallmann syndrome presents with the classic symptoms of anosmia and hypogonadism, but patients with Kallmann syndrome have a low FSH and LH. Treatment is with hormone replacement therapy.\n\nBullet Summary:\nKlinefelter syndrome results in primary hypogonadism and presents with tall stature, gynecomastia, small testes, a small phallus, hypospadias, and cryptorchidism.", "link": "https://bit.ly/3BSwAMD"} {"question": "A 57-year-old immigrant from Nigeria presents to the emergency department for sudden, severe pain and swelling in her lower extremity. She was at a rehabilitation hospital when her symptoms started. The patient has a medical history of obesity, diabetes, bipolar disorder, and tonic-clonic seizures. Her current medications include metformin, insulin, lisinopril, and valproic acid. The patient has IV drug and alcohol use disorder and has presented to the ED many times for intoxication. Her temperature is 98.6\u00b0F (37\u00b0C), blood pressure is 115/71 mmHg, pulse is 72/min, and respirations are 12/min. Physical exam reveals anasarca and asymmetric lower extremity swelling. Based on the results of a doppler ultrasound of her swollen lower extremity, heparin is started. The patient is then transferred to the general medicine floor for continued management. Laboratory studies are shown below.\n\nSerum:\nNa+: 137 mEq/L\nK+: 5.5 mEq/L\nCl-: 100 mEq/L\nHCO3-: 24 mEq/L\nUrea nitrogen: 22 mg/dL\nCa2+: 5.7 mg/dL\nCreatinine: 1.7 mg/dL\nGlucose: 70 mg/dL\n\nWhich of the following is the most likely diagnosis?", "choicesA": "Antithrombin III deficiency", "choicesB": "Factor V Leiden", "choicesC": "Liver failure", "choicesD": "Nephrotic syndrome", "choicesE": "Prothrombin gene mutation", "answer_idx": "D", "answer": "Nephrotic syndrome", "explanation": "This patient who presents with a deep venous thrombosis (lower extremity swelling, ultrasound followed by treatment with heparin), anasarca, and asymptomatic hypocalcemia most likely has a diagnosis of nephrotic syndrome.\n\nNephrotic syndrome occurs when large amounts of protein are lost in the urine. This can lead to hyperlipidemia (due to loss of lipoproteins), hypercoagulable state (due to loss of antithrombin III and protein C/S), and hypoalbuminemia. Most serum calcium is bound to albumin, so when albumin is lost in the urine, it can lead to profound hypocalcemia. The ionized or free portion of calcium remains the same; therefore, these patients do not exhibit symptoms of hypocalcemia. Furthermore, the loss of albumin will reduce the oncotic pressure in the vessels resulting in massive extravasation of fluid and anasarca.\n\nAmbler et al. report a case of a patient who presented with a DVT and was found to have nephrotic syndrome. They discuss how the patient had asymptomatic changes in lab values including calcium. They recommend performing a urine dipstick for protein in cases of suspected nephrotic syndrome.\n\nIncorrect Answers:\nAnswer A: Antithrombin III deficiency is a common cause of a hypercoagulable state because antithrombin III normally helps to neutralize coagulation factors. This disease is a possible explanation for this patient's DVT as it predisposes patients to thrombotic events; however, it does not explain her profound hypocalcemia. Asymptomatic antithrombin III deficiency does not need to be treated; however, patients should have anticoagulation if they undergo surgery.\n\nAnswer B: Factor V Leiden is the most common cause of a hypercoagulable state because this mutation renders factor V unable to be inactivated by proteins C and S. This is a possible explanation for this patient's DVT; however, it does not explain her hypocalcemia. Asymptomatic factor V Leiden mutation does not need to be treated; however, patients should have anticoagulation if they undergo surgery.\n\nAnswer D: Liver dysfunction is possible given this patient's history of alcohol use and IV drug abuse, placing her at risk for chronic hepatitis C infection. Despite the anasarca, this patient has no other stigmata of liver failure such as jaundice or ascites, making nephrotic syndrome more likely.\n\nAnswer E: Prothrombin gene mutation could explain this patient's hypercoagulable state because high levels of this gene can result in unprovoked thrombotic events. This disease would not explain why the patient has anasarca or hypocalcemia. Asymptomatic patients with this mutation do not require treatment.\n\nBullet Summary:\nNephrotic syndrome can present with a hypercoagulable state due to loss of antithrombin III and protein C/S and hypocalcemia from loss of albumin in the urine.", "link": "https://step2.medbullets.com/testview?qid=108996"} {"question": "A 57-year-old woman presents to the emergency department for a headache. She was at home when a sudden headache began with greater severity than she had experienced in the past. She presented within 30 minutes of symptom onset. The patient has a medical history of sinusitis, constipation, diabetes, and pelvic inflammatory disease. Her current medications include fexofenadine, oral contraceptive pills, and metformin. She is currently taking amoxicillin for a sinus infection. She smokes 1 pack of cigarettes per day and drinks 2-3 alcoholic beverages daily. Her temperature is 99.5\u00b0F (37.5\u00b0C), blood pressure is 187/118 mmHg, pulse is 120/min, respirations are 17/min, and oxygen saturation is 98% on room air. On physical exam, the patient is noted to be uncomfortable with the appearance shown in Figure A. Neurological exam reveals a confused woman who is now complaining of \"seeing double.\" Cardiopulmonary exam reveals mild bibasilar crackles and tachycardia. Abdominal exam is within normal limits. Which of the following is the most appropriate initial step in management?", "choicesA": "Amoxicillin-clavulinic acid", "choicesB": "CT of the head", "choicesC": "Dexamethasone", "choicesD": "MRI of the head", "choicesE": "Piperacillin-tazobactam", "answer_idx": "E", "answer": "Piperacillin-tazobactam", "explanation": "This patient is presenting with symptoms suggestive of septic cavernous sinus thrombosis (CST). The most appropriate initial step in management is urgent broad-spectrum antibiotics and surgical drainage.\n\nCavernous sinus thrombosis is an emergency that requires immediate intervention. It typically occurs in a patient with a history of sinusitis, oral contraceptive pill usage, smoking, or other risk factors that could cause a hypercoagulable state. These patients will often present with severe pain, a sudden headache, and diplopia. A distinct appearance is typically noted with periorbital edema. These patients should be immediately treated with broad-spectrum antibiotics and undergo surgical drainage of the inciting source of infection. In some cases, thrombolytics can be used as well.\n\nCaranfa and Yoon review the evidence regarding the diagnosis and treatment of septic cavernous sinus thrombosis. They discuss how this diagnosis was almost universally fatal in the pre-antibiotic era. They recommend using broad-spectrum antibiotics and debridement to address this disease.\n\nFigure/Illustration A is a clinical photograph demonstrating a face with swelling and periorbital edema (red circles). This appearance is classically seen in patients with CST.\n\nIncorrect Answers:\nAnswer A: Amoxicillin-clavulanic acid would be appropriate management of acute otitis media or sinusitis; however, it is insufficient coverage for CST. Broad-spectrum antibiotics and surgical intervention are needed to prevent loss of vision.\n\nAnswer B: A CT scan of the head could be indicated potentially for surgical planning in a more stable patient; however, in this situation, immediate intervention is more dire. After antibiotics and stabilization, CNS imaging will be needed.\n\nAnswer C: Dexamethasone would be appropriate treatment of a relapse of multiple sclerosis, which can also present with visual changes; however, it would not present with sudden headache and periorbital edema.\n\nAnswer D: An MRI of the brain would be appropriate for further characterizing neurologic deficits of unclear etiology. In this case, the patient's acute presentation with neurologic changes secondary to CST requires immediate treatment rather than additional imaging.\n\nBullet Summary:\nCavernous sinus thrombosis should be immediately managed with broad-spectrum antibiotics and neurosurgery.", "link": "https://bit.ly/3qNCi0x"} {"question": "A 43-year-old man presents to the emergency department with bright red blood in his stool this morning. He noticed this yesterday as well, but it was much scanter. The patient is otherwise healthy. He was celebrating his birthday last night and admits having \"too much to drink\" and vomited profusely overnight. He currently feels well and is not nauseous. He has no other significant medical history and does not take any medications. His temperature is 99.2\u00b0F (37.3\u00b0C), blood pressure is 110/75 mmHg, pulse is 82/min, respirations are 16/min, and oxygen saturation is 100% on room air. Physical exam reveals a nontender abdomen with normal bowel sounds. Rectal exam reveals grossly bloody stool. Laboratory studies are ordered as shown below.\n\nHemoglobin: 12 g/dL\nHematocrit: 36%\nLeukocyte count: 5,500/mm^3 with normal differential\nPlatelet count: 179,000/mm^3\n\nWhich of the following is the most likely cause of this patient's symptoms?", "choicesA": "Erosion into a gastric vessel", "choicesB": "Inflammation and infection of colonic outpouching", "choicesC": "Malignancy", "choicesD": "Submucosal esophageal tear", "choicesE": "Vascular malformation", "answer_idx": "E", "answer": "Vascular malformation", "explanation": "This patient is presenting with bright red blood per rectum with a non-tender abdomen. Of the answer choices given, angiodysplasia is the most likely to cause such profuse and sudden bleeding.\n\nAngiodysplasia is a vascular malformation found in the gastrointestinal (GI) tract, most commonly in the colon. The exposed vessels can bleed, leading to bright red blood per rectum. The diagnosis can be confirmed with colonoscopy. Ablation can be performed to stop the bleeding. Any patient who is unstable or anemic with a hemoglobin < 7.0 g/dL requires a transfusion with blood products. Angiodysplasia is a common cause of bright red blood per rectum but is not more common than diverticulosis.\n\nGarcia-Compeon et al. review the presentation and treatment of gastrointestinal angiodysplasia. The authors note that angiodysplasias are the cause of many \"occult\" GI bleeds not visualized on upper endoscopy or colonoscopy. The authors recommend the use of capsule endoscopy in occult GI bleeds to identify angiodysplasias as a potential source.\n\nIncorrect Answers:\nAnswer A: Erosion into a gastric vessel is the pathophysiology of a bleeding gastric vessel secondary to peptic ulcer disease. This usually presents with melena (black tarry stools) rather than bright red blood per rectum. Only in cases of extremely brisk upper GI bleeding would patients present with bright red blood per rectum. Treatment involves treating Helicobacter pylori infection if present, proton pump inhibitors, and sucralfate. Endoscopy is needed to confirm and treat the bleeding vessel.\n\nAnswer B: Inflammation and infection of colonic outpouching is the pathophysiology of diverticulitis which presents with a history of constipation with a fever and left lower quadrant abdominal pain. Management involves a computed tomography (CT) scan of the abdomen/pelvis and administration of antibiotics (such as ciprofloxacin and metronidazole or ceftriaxone and metronidazole). Diverticulosis presents with bright red blood and is possible in this patient, but it does not involve inflammation or infection, which defines diverticulitis.\n\nAnswer C: Malignancy or colon cancer can present with bright red blood per rectum, malaise, weight loss, and microcytic anemia. While it is possible in this patient, his age, lack of risk factors for colon cancer, and the sudden onset of his symptoms make this diagnosis less likely.\n\nAnswer D: Submucosal esophageal (Mallory-Weiss) tear presents after profuse vomiting (as in this patient) with bloody vomitus. Treatment is supportive and involves antiemetics.\n\nBullet Summary:\nAngiodysplasia is a vascular malformation that presents with bright red blood per rectum.", "link": "https://step2.medbullets.com/testview?qid=215172"} {"question": "A 5-year-old girl presents to the emergency room due to difficulty walking. She has been complaining of pain in her right leg for the last few days. Her neighbor\u2019s cat bit her last week and her parents attributed the pain to her healing bite. At the time of the bite, they cleaned the wound and irrigated it with sterile saline water from a first aid kit. She has no medical history and has never been hospitalized. Her temperature is 102.2\u00b0F (39\u00b0C), blood pressure is 118/78 mmHg, pulse is 90/min, respirations are 21/min, and pulse oximetry is 99% on room air. The open wound remains present on the thigh with surrounding erythema and edema. MRI is consistent with osteomyelitis. Which of the following is the most appropriate next step in management?", "choicesA": "Amoxicillin and clavulanate", "choicesB": "Ampicillin and sulbactam", "choicesC": "Doxycycline", "choicesD": "Flucanozole", "choicesE": "Vancomycin", "answer_idx": "B", "answer": "Ampicillin and sulbactam", "explanation": "This young girl with a recent history of a cat bite and difficulty walking most likely has likely Pasteurella multocida osteomyelitis. Empiric IV antibiotic therapy with ampicillin-sulbactam is the most appropriate initial management.\n\nPasteurella multocida is a gram-negative rod that is associated with cat and dog bites. Infection can lead to cellulitis and osteomyelitis. Treatment of minor infection is with amoxicillin-clavulanate; however, patients suspected of having osteomyelitis should be treated empirically with IV antibiotics (e.g., ampicillin and sulbactam) and taken for biopsy/culture to identify the pathogen. Antibiotics can then be narrowed down accordingly. Patients will often require between 4-8 weeks of antibiotic treatment. MRSA coverage may initially be started and tapered later once cultures result.\n\nMollitt reviews the evidence regarding the microbiology of various injuries. He discusses how Pasteurella multocida is a common complication of animal bites. He recommends early effective antibiotic treatment.\n\nIncorrect Answer:\nAnswer A: Amoxicillin and clavulanate can be used to treat Pasteurella infection. Patients with osteomyelitis should not be treated with oral antibiotics initially, in particular, when there are systemic manifestations as is present in this patient. IV antibiotics such as ampicillin and sulbactam are the mainstay for empiric osteomyelitis treatment after an animal bite in a patient with systemic manifestations.\n\nAnswer C: Doxycycline is used to treat uncomplicated chlamydia infections. Gonococcal osteomyelitis may present in sexually active individuals and may be associated with septic arthritis. Sexually active patients may be suspected of having this disease.\n\nAnswer D: Fluconazole is an antifungal that is used to treat Candida osteomyelitis. Candida osteomyelitis would be highly unusual in patients with no other risk factors. Instead, this disease is more common in immunodeficient patients or those who use IV drugs.\n\nAnswer E: Vancomycin is used for MRSA infections. Although S. aureus is the most common overall cause of osteomyelitis, this patient with a recent animal bite most likely has Pasteurella infection and this organism should be covered accordingly.\n\nBullet Summary:\nPatients with suspicion of Pasteurella osteomyelitis infection after an animal bite should be treated with empiric IV antibiotics such as ampicillin and sulbactam.", "link": "https://bit.ly/45a5Zb7"} {"question": "A 14-year-old boy presents to his pediatrician with a 2-month history of intermittent arm and leg spasms. He says that the spasms usually occur after he wakes up as he is getting ready for school, and consist of disorganized jerking motions that stop after a few minutes. He has also noticed that they occur when he stays up late to cram last minute for exams. He has not noticed any other symptoms. He has no medical history and takes no medications. He recently started experimenting with alcohol and marijuana that one of his friends has been stealing from their parents. His temperature is 98.6\u00b0F (37.0\u00b0C), blood pressure is 115/70 mmHg, pulse is 70/min, and respirations are 12/min. Physical exam reveals a healthy child. Which of the following is the most likely cause of this patient's symptoms?", "choicesA": "Absence seizures", "choicesB": "Alcohol intoxication", "choicesC": "Cannabinoid use", "choicesD": "Juvenile myoclonic epilepsy", "choicesE": "West syndrome", "answer_idx": "D", "answer": "Juvenile myoclonic epilepsy", "explanation": "This patient who presents with intermittent jerking of his extremities that occur primarily after waking up or when sleep-deprived most likely has juvenile myoclonic epilepsy.\n\nJuvenile myoclonic epilepsy (JME) is a poorly understood condition that presents with intermittent myoclonic jerks of the extremities. These myoclonic jerks usually have an onset occurring in early adolescence in otherwise healthy children. Myoclonic jerks are most frequently seen right after waking from sleep, but they may also be triggered by lack of sleep, stress, or alcohol consumption. Patients with this disease rarely have associated symptoms but may occasionally also experience absence seizures or generalized tonic-clonic seizures. The diagnosis can be secured with a history of seizures alongside an interictal electroencephalogram (EEG) that shows 4 to 6 Hz bilateral polyspikes. Valproate is the first-line treatment.\n\nJohannessen et al. discuss the treatment of juvenile myoclonic epilepsy. They find that standard antiepileptic treatment with agents such as valproate leads to only 33% of patients remaining seizure-free. They recommend trials of alternative antiepileptic medications such as levetiracetam, lamotrigine, and topiramate in patients that do not respond to valproate.\n\nIncorrect Answers:\nAnswer A: Absence seizures commonly present during childhood in otherwise healthy children; however, these seizures are typically characterized by short episodes of loss of awareness known as \"staring spells\" without physical manifestations.\n\nAnswer B: Alcohol intoxication is a risk factor for precipitating juvenile myoclonic epilepsy, but alcohol use does not cause this condition. Seizure activity may also occur during alcohol withdrawal, but the timing of this patient's symptoms is more suggestive of seizure symptoms. Alcohol withdrawal requires chronic drinking of large amounts leading to an increase in central nervous system N-methyl-D-aspartate receptors and a decrease in CNS gamma-aminobutyric acid type A receptors. When alcohol cessation occurs, seizures ensue.\n\nAnswer C: Cannabinoid use is not associated with seizure activity either during intoxication or withdrawal. Patients may have altered mental status, but this does not manifest as myoclonic jerks.\n\nAnswer E: West syndrome is an infantile seizure disorder that usually presents between 3 months and 12 months of age. These patients will present with recurrent flexion spasms of the extremities.\n\nBullet Summary:\nJuvenile myoclonic epilepsy presents in adolescents with recurrent spasms after waking from sleep or when sleep-deprived.", "link": "https://bit.ly/3LZVzD7"} {"question": "A 4-week-old boy presents to the pediatrician for vomiting. The patient is breastfed and typically feeds for 20-30 minutes every 2 hours. One week ago, he began vomiting and regurgitating breastmilk through his nose after most feedings. His stools over the past week have also become blood-streaked. The patient does not seem to be in any distress when he passes these bloody stools. The patient surpassed his birth weight by 2 weeks of age but has fallen 1 standard deviation on the growth curve since then. His temperature is 98.6\u00b0F (37\u00b0C), blood pressure is 78/47 mmHg, pulse is 115/min, and respirations are 28/min. On physical exam, the patient is well-appearing. His face and back have the physical exam finding seen in Figure A. The patient\u2019s abdomen is soft, non-tender, and non-distended. A digital rectal exam reveals a small amount of blood in the rectal vault. Which of the following is the most appropriate next step in management?", "choicesA": "Initiate a proton pump inhibitor", "choicesB": "Modify mother's diet", "choicesC": "Obtain abdominal ultrasound", "choicesD": "Provide reassurance", "choicesE": "Switch to hydrolyzed formula", "answer_idx": "B", "answer": "Modify mother's diet", "explanation": "This infant presents with vomiting, bloody stools, and poor weight gain, which suggests a diagnosis of milk-protein allergy. The most appropriate next step in management is the modification of the mother\u2019s diet to eliminate cow\u2019s milk and soy.\n\nInfant food protein-induced proctocolitis is caused by distal colon inflammation secondary to an immune reaction to certain food proteins. Milk-protein allergy is the most common food allergy in children and presents with vomiting, bloody stools, and poor weight gain or failure to thrive. In an infant that is breastfed, the most appropriate next step in management is the elimination of cow\u2019s milk and soy from the mother\u2019s diet. In an infant who is formula fed, the most appropriate next step would be switching to either an extensively hydrolyzed or amino acid-based infant formula.\n\nGiannetti et al. review the evidence regarding the diagnosis and treatment of milk protein allergy in infants. They discuss how the selective elimination of certain proteins from the diet is usually effective. They recommend considering oral immunotherapy as an adjuvant in the treatment of this disease.\n\nFigure/Illustration A is a clinical photograph that demonstrates the finding of eczema (red circle). This finding is commonly found in patients with milk-protein allergy.\n\nIncorrect Answers:\nAnswer A: Initiating a proton pump inhibitor would be appropriate in patients with refractory gastroesophageal reflux disease, which presents with regurgitation or vomiting and symptoms of feeding aversion or failure to thrive. Because this patient presents with bloody stools, he is more likely to have a milk protein allergy.\n\nAnswer B: Obtaining an abdominal ultrasound would help evaluate for pyloric stenosis, which presents in infants between 3-6 months of age with non-bilious, projectile vomiting. Pyloric stenosis does not cause bloody stools. Treatment is with surgical pyloromyotomy.\n\nAnswer C: Providing reassurance would not be appropriate for an infant presenting with bloody stools and evidence of poor weight gain. The cause of his blood-streaked stools should be investigated to ensure this patient is getting the nutrition he needs.\n\nAnswer E: Switching to a hydrolyzed formula would be appropriate if this patient were formula fed, but it is not necessary to discontinue breastfeeding if the mother is willing to modify her diet to exclude cow\u2019s milk and soy.\n\nBullet Summary:\nA milk-protein allergy in a breastfed infant should be treated with modification of the mother\u2019s diet to eliminate cow\u2019s milk and soy.", "link": "https://bit.ly/3PCWiwF"} {"question": "A 4-year-old boy comes to the pediatrician with his mother with a 3-day history of cough and runny nose with decreased oral intake over the past 24 hours. His activity level has been normal and he has been attending school until this morning. He has no medical history or known allergies, but he has not received any of the recommended vaccines due to parental refusal. His temperature is 100.3\u00b0F (37.9\u00b0C), blood pressure is 98/62 mmHg, pulse is 85/min, respirations are 22/min, and oxygen saturation is 98% on room air. On examination, the child is fussy. Bilateral conjunctivae are injected, and there is redness on and around the tonsils. There is no lymphadenopathy. Faint rhonchi are heard at the lung bases bilaterally. Which of the following is the most appropriate next step in management?", "choicesA": "Administer antitoxin and toxoid vaccine", "choicesB": "Amoxicillin", "choicesC": "Influenza testing and oseltamivir", "choicesD": "Obtain chest radiography and respiratory virus panel", "choicesE": "Supportive care with return precautions", "answer_idx": "E", "answer": "Supportive care with return precautions", "explanation": "This boy presenting with 3 days of progressive fever, rhinorrhea, cough, conjunctivitis, and tonsilar erythema most likely has viral pharyngitis. The appropriate treatment for this is supportive care with return precautions (instructing the mother to bring the child back to a physician if he becomes short of breath).\n\nViral pharyngitis in children is most often caused by adenovirus, though other viruses can cause the condition as well. The diagnosis is clinical, with a presentation of fever, rhinorrhea, conjunctivitis, sore throat, and cough. Features that can help distinguish viral pharyngitis from pneumonia include a lack of tachypnea, increased work of breathing, hypoxemia, or adventitial lung sounds. No additional testing or intervention is necessary. The illness is typically self-limited and resolves with supportive care.\n\nShapiro et al. study features predictive of group A streptococcal (GAS) pharyngitis in children. The authors found that among patients with features of viral illness, patients older than 11 years of age who did not have a fever, cervical lymphadenopathy, or tonsillar exudates were at the lowest risk for GAS. The authors recommend against testing low-risk children for GAS to avoid antibiotic treatment in patients merely colonized with GAS.\n\nIncorrect Answers:\nAnswer A: Administering antitoxin and toxoid vaccine is the treatment for Corynebacterium diphtheriae infection. Although this child has not received his age-appropriate vaccinations, his presentation is not concerning for infection with diphtheria. C. diphtheriae infection presents with cervical lymphadenopathy and gray pseudomembranes on pharyngeal exam, neither of which this child has.\n\nAnswer B: Amoxicillin should be given in cases of group A streptococcal (GAS) pharyngitis. Before amoxicillin is prescribed, testing with Streptococcal rapid antigen detection testing should be performed. Additionally, rapid antigen detection testing is not indicated in this patient as she has a cough and does not have cervical lymphadenopathy, tonsillar exudates, or palatal petechiae. All of these findings are not typical of GAS pharyngitis.\n\nAnswer C: Influenza testing and oseltamivir are indicated if the child had myalgias or high fevers that would make influenza more likely and if he presented within 2 days of symptom onset. Oseltamivir is only approved for treating influenza in uncomplicated cases if started within this 2-day window from symptom onset. Exceptions to this 2-day guideline include adults greater than 65 years of age, pregnant women, residents of long-term care facilities, American Indians, persons with BMI greater than 40 kg/m^2, and those receiving immunosuppressive medications.\n\nAnswer D: Obtaining a chest radiograph and respiratory virus panel can be done when a lower respiratory infection such as bacterial pneumonia is expected. However, viral infections are much more common than bacterial pneumonia in this age group. Additionally, this patient is not tachypneic, does not have evidence of increased work of breathing, and is not hypoxemic, all of which make bacterial pneumonia less likely.\n\nBullet Summary:\nViral pharyngitis in children is a clinical diagnosis consisting of fever, rhinorrhea, conjunctivitis, cough, sore throat, and pharyngeal erythema, and is treated with supportive care.", "link": "https://step2.medbullets.com/testview?qid=215042"} {"question": "A 37-year-old woman presents to the emergency department in cardiac arrest. She was found to have altered mental status at home, and during transport to the hospital, she became pulseless. Cardiopulmonary resuscitation is ongoing and 2 intravenous lines are placed. The patient is obese, and her only medication is an oral contraceptive pill. She recently underwent Achilles tendon repair. She otherwise is known to smoke cigarettes on occasion. Physical exam is notable for a critically ill patient with chest compressions ongoing. The monitor shows pulseless electrical activity. Her right lower extremity is in a post-operative splint. The patient regains her pulse after several rounds of CPR and epinephrine. A bedside echocardiograph is notable for a dilated and hypokinetic right ventricle with septal bowing into a hyperkinetic left ventricle. Which of the following is the most likely diagnosis?", "choicesA": "Cardiogenic shock", "choicesB": "Hemorrhagic shock", "choicesC": "Neurogenic shock", "choicesD": "Obstructive shock", "choicesE": "Septic shock", "answer_idx": "D", "answer": "Obstructive shock", "explanation": "This obese patient on oral contraceptive pills with recent orthopedic surgery (Achilles tendon repair) is presenting in cardiac arrest. Given the bedside echocardiograph that is notable for a dilated and hypokinetic right ventricle with septal bowing into a hyperkinetic left ventricle, the most likely diagnosis is a pulmonary embolism that is causing obstructive shock.\n\nPulmonary embolism is most commonly caused by a deep vein thrombosis in the veins of the lower extremity that travels to the pulmonary circulation. Patients will present with pleuritic chest pain, hypoxia, sinus tachycardia, and hemoptysis. Massive pulmonary emboli can cause hemodynamic instability. Patients may become hypotensive, tachycardic, and ultimately pulseless. The pulmonary embolism can obstruct blood flow from the right ventricle to the pulmonary artery, leading to a dilated and hypokinetic right ventricle with septal bowing and a hyperdynamic left ventricle. This is a type of obstructive shock. Treatment of a massive pulmonary embolism causing hemodynamic instability may involve thrombolytics or thrombectomy.\n\nStandl et al. review obstructive shock. They note that obstructive shock is caused by an obstruction of blood flow out of (or in the case of tamponade, into) the heart. They recommend that treating the underlying cause is the only effective management option.\n\nIncorrect Answers:\nAnswer A: Cardiogenic shock is caused by cardiac dysfunction in conditions such as myocardial infarction or heart failure. Cardiac dysfunction then leads to impaired systemic perfusion. Initial management includes treating the underlying cause (e.g., cardiac catheterization for an ST-elevation myocardial infarction) and vasopressors (e.g., norepinephrine) to maintain the patient\u2019s blood pressure.\n\nAnswer B: Hemorrhagic shock is a common cause of shock in trauma and presents with hypotension and tachycardia secondary to blood loss. A narrowed pulse pressure may be the first sign of hemorrhagic shock. The mainstay of management is blood transfusion and an intervention to stop the bleeding.\n\nAnswer C: Neurogenic shock occurs when there is central nervous system trauma (often to the cervical spine) leading to decreased sympathetic tone systemically. Patients will present with hypotension and bradycardia (contrast with tachycardia in hemorrhagic shock), and management is centered on vasopressors to maintain blood pressure and neurosurgical intervention for the underlying etiology. IV fluids can also be given to maintain blood pressure.\n\nAnswer E: Septic shock occurs when a systemic infection leads to vasodilation and endothelial dysfunction with hypotension refractory to fluid resuscitation. The mainstay of management includes fluids, blood cultures, vasopressors, and broad-spectrum antibiotics. Early administration of antibiotics improves outcomes.\n\nBullet Summary:\nPulmonary embolism is a common cause of obstructive shock.", "link": "https://step2.medbullets.com/testview?qid=216473"} {"question": "A 23-year-old woman presents to the emergency room for a self-inflicted laceration of her distal volar forearm. The patient states she knew her boyfriend was having sexual thoughts about the woman from the grocery store, prompting her decision to cut her own wrist. In the emergency department, the bleeding has stopped and the patient is currently medically stable. When interviewing the patient, she is teary and apologizes for her behavior. She is grateful to you for her care and regrets her actions. Of note, the patient has presented to the emergency department before for a similar reason when she was struggling with online dating. The patient states that she struggles with her romantic relationships though she deeply desires them. Her temperature is 98.6\u00b0F (37\u00b0C), blood pressure is 112/71 mmHg, pulse is 73/min, and respirations are 14/min. On physical exam, you note a frightened young woman who is wearing a revealing dress that prominently displays her breasts. You tell the patient that she will have to stay in the psychiatric emergency department for the night which makes her furious. Which of the following personality disorders is the most likely diagnosis?", "choicesA": "Antisocial", "choicesB": "Avoidant", "choicesC": "Borderline", "choicesD": "Dependent", "choicesE": "Histrionic", "answer_idx": "C", "answer": "Borderline", "explanation": "This patient is presenting with a suicide attempt, unstable relationships, and emotional lability suggesting a diagnosis of borderline personality disorder.\n\nBorderline personality disorder classically presents with emotional lability, and intense, unstable relationships. These patients fear abandonment and will often engage in self-harming behavior like cutting and suicide attempts. The main defense mechanism these patients employ is splitting, in which patients divide the world into dichotomous categories where people are either all good or all bad. This personality disorder is associated with mood disorders and substance use. The treatment of choice for these patients is long-term dialectical behavioral therapy and mood stabilizers.\n\nLeichsenring et al. review the evidence regarding the treatment of patients with borderline personality disorder. They discuss no specific form of psychotherapy is superior to others. They recommend starting any form of psychotherapy for these patients to improve outcomes.\n\nIncorrect Answers:\nAnswer A: Antisocial personality disorder presents with aggressive, criminal behavior. These individuals do not conform to social norms or rules and violate the rights of others. Patients often are not amenable to behavioral modification and therefore incarceration may be necessary in order to protect the rights of others.\n\nAnswer B: Avoidant personality disorder presents with excessive shyness and hypersensitivity to rejection. These patients fear social interaction though they strongly desire relationships. Cognitive behavioral therapy can be used in order to encourage these patients to be more open to seeking out relationships.\n\nAnswer D: Dependent personality disorder presents with submissive and clingy behavior. They often have a need to be cared for and will be very agreeable and dependent on others. This can be harmful as patients may stay in abusive or degrading relationships due to a fear of being alone. Treatment is with cognitive behavioral therapy.\n\nAnswer E: Histrionic personality disorder presents with colorful and exaggerated attention-seeking behavior. These patients will often employ sexuality to draw attention to themselves. Though this patient is displaying her breasts and wearing seductive clothing, her history of self-harm behavior and unstable relationships suggests a diagnosis of borderline personality disorder.\n\nBullet Summary:\nBorderline personality disorder presents with emotional instability, unstable relationships, and self-harming behavior.", "link": "https://bit.ly/3HvRZyr"} {"question": "An 8-week-old boy presents with his mother to the pediatrician for a well visit. The patient has been breastfed since birth, and usually feeds for 30 minutes every 2-3 hours. The patient\u2019s mother is concerned that her milk production is not keeping up with the patient\u2019s nutritional requirements. She reports that about 2 weeks ago the patient began regurgitating breastmilk through his nose and mouth after some feeds. He seems mildly upset during the episodes of regurgitation but usually settles down quickly and is hungry again soon afterwards. His mother has already tried limiting the volume of each feed, which seems to have reduced the frequency of regurgitation. She denies any diarrhea, hematochezia, or family history of food allergies. Her older son had a similar problem with vomiting that resolved around 12 months of age. Four weeks ago, the patient\u2019s height and weight were in the 40th and 34th percentiles, respectively. His height and weight are now respectively in the 37th and 36th percentiles. His temperature is 98.6\u00b0F (37\u00b0C), blood pressure is 72/49 mmHg, pulse is 121/min, and respirations are 32/min. On physical exam, the patient is cooing in his mother\u2019s lap and smiles reciprocally with her. He lifts his head and shoulders off the examination table when placed in the supine position. His abdomen is soft, non-tender and non-distended. Bowel sounds are normoactive. Which of the following is the most appropriate next step in management?", "choicesA": "Initiate proton pump inhibitor", "choicesB": "Obtain abdominal ultrasound", "choicesC": "Reassurance and counseling on positioning", "choicesD": "Recommend modification of mother\u2019s diet", "choicesE": "Switch to hydrolyzed formula", "answer_idx": "C", "answer": "Reassurance and counseling on positioning", "explanation": "This patient presents with regurgitation after feeding, appropriate appetite after episodes of regurgitation, and good weight gain, which suggests a diagnosis of uncomplicated gastroesophageal reflux. The most appropriate next step in management is reassurance and counseling on proper positioning.\n\nPhysiologic or uncomplicated gastroesophageal reflux usually presents in patients in the first few months of life with frequent regurgitation or vomiting, sustained appetite after regurgitation, and minimal irritability. The most appropriate first step in management is reassurance and counseling parents to position patients upright for 20-30 minutes after feeding. For patients with persistent symptoms, parents should be encouraged to provide lower volume feeds more frequently throughout the day, and consider a milk-free diet to address possible underlying cow\u2019s milk allergy. Treatment with a proton pump inhibitor is reserved for intractable cases. For most patients, gastroesophageal reflux self-resolves by 1 year of age.\n\nChabra and Peeples review the evidence regarding the treatment of neonatal GERD. They discuss how alterations in the infant's environment and feeding pattern are effective in most cases. They recommend proton pump inhibitors if conservative treatments are not effective.\n\nIncorrect Answers:\nAnswer A: Initiating a proton pump inhibitor is appropriate for infants with gastroesophageal reflux disease who have failed management with lifestyle changes. This patient has uncomplicated gastroesophageal reflux without any signs of feeding refusal, worsening irritability, or nutritional compromise.\n\nAnswer B: Obtaining an abdominal ultrasound would evaluate for pyloric stenosis, which presents with non-bilious, projectile vomiting rather than the regurgitation that this patient\u2019s mother is describing. An abdominal ultrasound would be the appropriate next step in management for patients who present with a description of more forceful vomiting or have a classic \u201colive-shaped\u201d mass in the epigastrium on physical exam.\n\nAnswer D: Recommending modification of the mother\u2019s diet would be appropriate if this patient had more severe symptoms that were refractory to lifestyle changes. It would also be appropriate if the patient had signs of an underlying allergy to cow\u2019s milk or soy protein, such as blood-streaked stools.\n\nAnswer E: Switching to a hydrolyzed formula would be appropriate for formula-fed patients with refractory symptoms of gastroesophageal reflux disease or signs of an allergy to cow\u2019s milk or soy protein. Unless the patient has evidence of severe nutritional compromise, continued breastfeeding should be encouraged for breastfed patients with possible modification of the mother\u2019s diet to eliminate cow\u2019s milk or soy if an allergy is suspected.\n\nBullet Summary:\nUncomplicated gastroesophageal reflux in infants should be treated with reassurance and counseling to position the infant upright for 20-30 minutes after feeding.", "link": "https://step2.medbullets.com/testview?qid=109261"} {"question": "A 28-year-old G1P0 woman presents to initiate prenatal care after a positive home pregnancy test. The first day of her last menstrual period was 7 weeks prior to presentation. The patient has no medical conditions and takes no medications. Her temperature is 98.6\u00b0F (37.0\u00b0C), blood pressure is 110/70 mmHg, pulse is 64/min, and respirations are 17/min. Physical exam is notable for a nontender abdomen. Pelvic exam reveals a closed cervix with no adnexal or uterine tenderness. A urine pregnancy test is positive, and an intrauterine singleton pregnancy measuring approximately 9 weeks gestational age is visualized on transvaginal ultrasound. The patient asks if she should modify her current exercise regimen due to her pregnancy. She typically swims or uses an exercise bike for 45 minutes, 5 days a week, and plays in an office soccer league for 1 hour on 1 day per week. Which of the following is the most appropriate recommendation regarding exercise for this patient?", "choicesA": "Avoid exercise during pregnancy", "choicesB": "Avoid soccer during pregnancy, continue other activities", "choicesC": "Continue the current exercise regimen", "choicesD": "Limit moderate exercise to a maximum of 30 minutes per day during pregnancy", "choicesE": "Reduce to walking only, beginning in the third trimester", "answer_idx": "B", "answer": "Avoid soccer during pregnancy, continue other activities", "explanation": "This healthy primigravida woman presents to the clinic inquiring about her current exercise regimen involving 285 minutes of moderate to exercise weekly. With the exception of soccer, a contact sport, her exercise regimen is appropriate throughout her pregnancy.\n\nThe American College of Obstetricians and Gynecologists (ACOG), recommends that healthy pregnant women engage in moderate-intensity exercise for at least 30 minutes or more for 5-7 days weekly provided there are no contraindications to doing so (certain cardiac or pulmonary conditions, history of cerclage placement, multiple pregnancies, preeclampsia, or severe anemia). Highly active patients may continue their more vigorous pre-pregnancy regimens after consulting with a physician. Exercise in pregnancy can ease gastrointestinal symptoms and may decrease the risk of gestational diabetes, preeclampsia, and cesarean delivery. However, activities that may cause fetal or maternal injury, such as contact sports or activities with a significant fall risk should be avoided. Examples of contact sports include football, basketball, hockey, lacrosse, and wrestling.\n\nNascimento et al. review exercise in pregnancy. They note that exercise is beneficial in pregnancy and in the postpartum period and recommend engaging in appropriate exercise in this patient population.\n\nIncorrect Answers:\nAnswer A: Avoiding exercise during pregnancy is not recommended, as at least 150 minutes of moderate exercise weekly is suggested during pregnancy for the health of the mother and the fetus.\n\nAnswer C: Continuing the current exercise regimen is not recommended, as soccer is a high-contact sport that could cause injury to the fetus or placenta.\n\nAnswer D: Limiting moderate exercise to 30 minutes per day maximum during pregnancy is not recommended, as 30 minutes daily is the minimum recommended time for moderate-exercise. There is no reason for healthy pregnant patients without high-risk pregnancies to significantly limit their exercise.\n\nAnswer E: Reducing to walking-only beginning in the third trimester is not necessary, as moderate activities are healthy throughout pregnancy.\n\nBullet Summary:\nHealthy pregnant patients should perform at least 30 minutes of exercise 5-7 days per week and avoid high-contact sports or sports with fall risk.", "link": "https://step2.medbullets.com/testview?qid=216262"} {"question": "A 39-year-old man presents to his doctor for a wellness checkup. He is concerned about a rash that does not seem to be improving. He was recently exposed to his grandfather who has vesicular lesions on his skin and is being treated. He has a family history of skin cancer, colon cancer, and ovarian cancer. The patient has a medical history of asthma and seasonal allergies. His temperature is 98.6\u00b0F (37.0\u00b0C), blood pressure is 137/98 mmHg, pulse is 90/min, respirations are 14/min, and oxygen saturation is 98% on room air. Physical exam reveals the finding in Figure A. Which of the following describes this patient's most likely diagnosis?", "choicesA": "Benign capillary proliferation", "choicesB": "Edema of the epidermis", "choicesC": "Healthy dermatologic development", "choicesD": "Malignant blood vessel proliferation", "choicesE": "Viral infection", "answer_idx": "A", "answer": "Benign capillary proliferation", "explanation": "This patient who presents with red raised papules is presenting with a cherry angioma. These lesions are composed of benign capillary proliferations.\n\nCherry angiomas present with cherry red macules/papules that occur secondary to benign capillary and venule proliferation. The lesions are entirely benign and are cosmetic. These lesions are common in middle-aged adults. As the patient ages, more lesions will appear. In the absence of symptoms, observation and reassurance can be used for conservative management. They can be removed surgically if the patient desires.\n\nNazer et al. study the risk factors associated with the development of cherry angioma. They found that the use of clopidogrel and tamsulosin is associated with these lesions. They recommend ruling out underlying conditions.\n\nFigure/Illustration A demonstrates cherry red macules/papules classically seen in patients with cherry angioma.\n\nIncorrect Answers:\nAnswer B: Edema of the epidermis describes eczema or atopic dermatitis. This disease would present with pruritic, erythematous, and vesicular lesions. Topical creams and hygiene can be used for symptomatic management of these lesions.\n\nAnswer C: Healthy dermatologic development misses the diagnosis of a cherry angioma, which although benign, is a more accurate diagnosis for this case.\n\nAnswer D: Malignant blood vessel proliferation does not describe a cherry angioma which is benign and purely cosmetic. An angiosarcoma usually occurs in deep tissues and requires surgical excision.\n\nAnswer E: Viral infection describes herpes simplex virus which would present with vesicular lesions and neurogenic pain. These patients can be treated with valacyclovir as well as gabapentin for neuropathic pain.\n\nBullet Summary:\nCherry angiomas are benign capillary proliferations that appear as cherry red macules/papules.", "link": "https://bit.ly/3N80FxZ"} {"question": "A 46-year-old man presents with his wife to his primary care provider for depression and strange movements. His wife reports that her husband has not been himself for the last 2 months. Whereas he was previously outgoing and \u201cthe life of the party,\u201d the patient is now irritable and withdrawn. He is a partner at an accounting firm, but his colleagues are threatening to fire him if he continues to perform poorly at work. The patient cannot explain the recent changes to his mood and tearfully admits he fears there is something seriously wrong with him. His wife says that he is getting worse. The patient\u2019s medical history is significant for hypertension, for which he takes lisinopril. His family history is unknown as he was adopted. The patient met his mother once and never knew his father but was told he died in his 50s. He drinks a few glasses of wine per week and has never smoked. His temperature is 98.6\u00b0F (37\u00b0C), blood pressure is 141/87 mmHg, pulse is 81/min, and respirations are 12/min. On physical exam, the patient has a flat affect with a facial grimace and sudden jerky movements of his upper extremities. Which of the following is most likely to be seen on further workup?", "choicesA": "Alpha-synuclein aggregates on brain biopsy", "choicesB": "Dorsal striatum atrophy on head CT", "choicesC": "Frontotemporal atrophy on head CT", "choicesD": "Neurofibrillary tangles on brain biopsy", "choicesE": "Positive 14-3-3 CSF assay", "answer_idx": "B", "answer": "Dorsal striatum atrophy on head CT", "explanation": "This patient presents with recent mood changes, jerky movements of the upper extremities, and a history of early death in his father, which suggests a diagnosis of Huntington disease. Huntington disease is characterized by atrophy of the caudate nucleus, which is a structure of the dorsal striatum.\n\nHuntington disease (HD) is an autosomal dominant trinucleotide (CAG) repeat disorder located on chromosome 4. HD typically presents with mood changes and abnormal movements (particularly chorea or myoclonus) over weeks to months. Although executive function can be affected early on, memory is usually preserved until relatively late in the disease. Frontotemporal dementia, in contrast, usually presents with behavioral changes, and Alzheimer disease has memory impairment as an early sign. Caudate atrophy is typically seen on head imaging in Huntington disease. This disease cannot be cured but supportive treatment includes benzodiazepines and tetrabenazine for symptomatic relief of muscle spasms.\n\nWalker presents a review of the evidence regarding the etiology of Huntington disease. He discusses that typically the onset of symptoms is in middle age after affected individuals have had children. He recommends a better understanding of the function of the mutant protein in this disease.\n\nIncorrect Answers:\nAnswer A: Alpha-synuclein aggregates would be found on brain biopsy in Parkinson disease and Lewy body dementia. Lewy body dementia is characterized by parkinsonism and visual hallucinations. Patients would present with cogwheel rigidity, bradykinesia, and a resting tremor. Treatment is with dopamine agonists such as carbidopa/levodopa.\n\nAnswer C: Frontotemporal atrophy is consistent with a diagnosis of frontotemporal dementia. Frontotemporal dementia presents primarily with early behavioral changes, and it would not present with myoclonus. Treatment is supportive as there is no cure for this disease. Patients and their spouses may require psychiatric support for changes in behavior.\n\nAnswer D: Neurofibrillary tangles on brain biopsy would suggest a diagnosis of Alzheimer disease. This disease presents with a slowly progressive cognitive decline in elderly patients. Patients have early loss of memory but have maintained motor function and behavior until later in the course of the disease.\n\nAnswer E: A positive 14-3-3 CSF assay would confirm the diagnosis of Creutzfeldt-Jakob disease (CJD). Although CJD may also present with mood changes and myoclonus, CJD most prominently causes rapidly progressive dementia early in life. Treatment is supportive and death occurs quickly.\n\nBullet Summary:\nHuntington disease is an autosomal dominant trinucleotide repeat disorder located on chromosome 4 that causes caudate atrophy and presents with mood changes, abnormal movements, and subcortical dementia.", "link": "https://bit.ly/3nctYpc"} {"question": "A 69-year-old man presents to the emergency department with shortness of breath. The patient has presented 3 times this month with similar complaints. He states his shortness of breath started when he was walking from his car to a local restaurant. He does not see a primary care physician and is not currently taking any medications. He drinks alcohol socially and does not smoke. His temperature is 99.5\u00b0F (37.5\u00b0C), pulse is 100/min, blood pressure is 130/90 mmHg, respirations are 18/min, and oxygen saturation is 96% on room air. On physical exam, he appears fatigued and the cardiovascular exam reveals an additional heart sound after S2. A pulmonary exam is notable for bilateral crackles and an abdominal exam reveals an obese abdomen without pain in any of the quadrants. Lower extremity pitting edema is noted bilaterally. Which of the sets of lab values shown in Figure A would most likely be seen in this patient?", "choicesA": "A", "choicesB": "B", "choicesC": "C", "choicesD": "D", "choicesE": "E", "answer_idx": "C", "answer": "C", "explanation": "This patient is presenting with symptoms of heart failure (CHF). The most likely laboratory abnormalities are elevated brain natriuretic peptide (BNP), high anti-diuretic hormone (ADH), low sodium, and low potassium.\n\nHeart failure is characterized by the decreased performance of the cardiac ventricles resulting in ventricular dilation. As the ventricles dilate, they release BNP as a marker of stretching of myocardial tissue. BNP levels can therefore serve as a marker of CHF. The kidneys will also have decreased perfusion levels in this disease leading to several key physiologic changes. First, is the activation of the renin-angiotensin-aldosterone system, which serves to increase perfusion to the kidneys by increasing blood pressure. This system achieves increased blood pressure by maintaining sodium and fluid retention at the expense of decreased potassium (hypokalemia) and decreased hydronium ions (metabolic alkalosis). In addition, ADH is also increased to further increase perfusion, resulting in water retention, hemodilution, and hyponatremia.\n\nMa et al. studied the correlation between BNP levels and cardiovascular outcomes. They found that patients with CHF and high levels of BNP had increased rates of cardiovascular complications. They recommend assessing BNP levels in patients who are suspected of having CHF.\n\nFigure A is a chart with changes in brain natriuretic peptide, anti-diuretic hormone, sodium, and potassium that would be seen in various diseases.\n\nIncorrect Answers:\nAnswer A: High BNP, high ADH, high sodium, and high potassium does not reflect the changes that would be seen in CHF. Though aldosterone serves to retain sodium, it also absorbs water. Thus, hypernatremia would not be seen. This pattern could be seen in patients with heart failure who have high free water losses such as after a burn or in desert environments.\n\nAnswer B: High BNP, low ADH, normal sodium, and low potassium does not reflect the appropriate increase in ADH and subsequent decrease in sodium that would be seen in CHF. Patients who have diabetes insipidus can have low ADH in this setting resulting in loss of free water.\n\nAnswer D: Low BNP, high ADH, low sodium, and low potassium does not reflect the finding of elevated BNP that is classically found in the dilated ventricles of CHF. This pattern would be seen in patients with the syndrome of inappropriate ADH where excessive resorption of free water results in hemodilution.\n\nAnswer E: Low BNP, low ADH, normal sodium, and normal potassium reflect the findings in a healthy patient. This patient most likely has CHF, which would result in increased aldosterone and ADH function.\n\nBullet Summary:\nCommon lab findings in heart failure include elevated BNP, elevated ADH, low sodium, and low potassium.", "link": "https://bit.ly/3Mqa3f2"} {"question": "A 19-year-old woman presents to her primary care physician with headaches, blurry vision, and tinnitus over the past month. She started treatment for a skin condition with a new oral medication 2 months prior. She has not noticed any changes in her complexion as of yet. Her temperature is 98.2\u00b0F (36.8\u00b0C), blood pressure 100/65 mmHg, pulse 87/min, and respiratory rate 14/min. She is oriented to person, place, and time. Neurological exam reveals papilledema upon examination of both eyes. Skin examination is notable for the findings in Figure A. Which of the following is the most likely diagnosis?", "choicesA": "Cerebrovascular accident", "choicesB": "Giant cell arteritis", "choicesC": "Idiopathic intracranial hypertension", "choicesD": "Meniere disease", "choicesE": "Migraine", "answer_idx": "C", "answer": "Idiopathic intracranial hypertension", "explanation": "This young patient with severe nodulocytic acne who was started on a new oral medication (likely isotretinoin) is likely suffering from idiopathic intracranial hypertension, which can present with papilledema, headache, blurry vision, and diplopia.\n\nIdiopathic intracranial hypertension is characterized by an increase in intracranial pressure without an identifiable cause. Patients often present with symptoms that include headache, blurry vision, and tinnitus. Fundoscopic examination typically reveals bilateral papilledema. Vision loss is a complication of this condition with high morbidity. While there is no identifiable cause, common associations include obesity or a recent increase in weight as well as medications such as isotretinoin - a vitamin A derivative that is often used in the management of severe nodulocystic acne vulgaris. Management should first involve imaging of the brain (CT or MRI) to exclude elevated cerebrospinal fluid pressure due to other causes such as brain tumor, dural sinus thrombosis, or hydrocephalus. The diagnosis is made via lumbar puncture revealing increased opening pressure. Lumbar puncture can also result in therapeutic relief. The specimen should be sent for gram stain, culture, and viral PCR to rule out other etiologies. Long term treatment is centered on weight loss and avoiding causative agents. Acetazolamide and serial lumbar punctures are other treatments.\n\nBoyter reviews the clinical presentation and management of patients with idiopathic intracranial hypertension. They found the condition is most common among obese women in their 20's, but it also may occur in children, men, and older adults. They recommend referral for patients to see a neurologist for ongoing management of headaches and other symptoms, as well as an ophthalmologist or neuro-ophthalmologist for monitoring papilledema, given the risk of vision loss.\n\nFigure/Illustration A: The figure depicts a patient with severe nodules and cysts (red arrows) consistent with severe inflammatory nodulocystic acne that would be suitable for treatment with isotretinoin.\n\nIncorrect Answers:\nAnswer A: Cerebrovascular accidents can be embolic or hemorrhagic in nature and often result in persistent neurologic deficits or resolution of an acute neurologic deficit in a patient with cerebrovascular risk factors (diabetes, hypertension, hyperlipidemia). This young, healthy patient with intermittent headache and neurologic symptoms without a family history of hypercoagulable state is unlikely to have suffered from a cerebrovascular accident.\n\nAnswer B: Giant cell arteritis (GCA) is a vasculitis that is typically seen in elderly patients (>60 years of age) and presents with tenderness of the ipsilateral temple and elevated inflammatory markers. Many patients with GCA have one or more constitutional symptoms (weight loss, fever, fatigue, anorexia, malaise) in addition to symptoms such as new onset headache or jaw claudication.\n\nAnswer D: Meniere disease affects the inner ear and is characterized by severe episodes of vertigo, tinnitus, hearing loss, and fullness in the ear. Patients are often women between the ages of 40 and 60. Headaches and blurry vision are not a classic part of the clinical presentation.\n\nAnswer E: Migraines present with severe headaches that can be incapacitating with associated photophobia and phonophobia. In some cases, patients may present with an aura that precedes or occurs during the migraine. The presence of papilledema suggests elevated intracranial pressure and is not typically seen in migraines.\n\nBullet Summary:\nIdiopathic intracranial hypertension may be a side effect of isotretinoin.", "link": "https://bit.ly/436MoX6"} {"question": "A 52-year-old man presents to the emergency department by ambulance for evaluation of hematemesis. His wife states that he began vomiting bright red blood this evening while eating dinner, and has produced about 5 cups total. He has a history of alcohol use disorder and has vomited small streaks of blood in the past, but has never undergone formal medical workup for it. On exam, he has bright red blood in his mouth and pharynx, and also displays a distended abdomen with a fluid wave, jaundice, scattered spider angiomas, and multiple ecchymoses. His temperature is 98.4\u00b0F (36.9\u00b0C), blood pressure is 98/68 mmHg, pulse is 125/min, and respirations are 22/min. A rapid point-of-care hemoglobin and platelet count is 8.8 g/dL and 80,000/mm^3, respectively. The patient is given 1L of normal saline. Which of the following is the most appropriate initial step in management?", "choicesA": "Balloon tamponade", "choicesB": "Ceftriaxone", "choicesC": "Nadolol", "choicesD": "Octreotide", "choicesE": "Platelet transfusion", "answer_idx": "D", "answer": "Octreotide", "explanation": "This patient with a history of alcohol use disorder and signs of liver failure (jaundice, ascites, spider angiomas, and ecchymoses) presents with acute hematemesis and hypovolemic shock (hypotension, tachycardia, tachypnea) most concerning for a variceal hemorrhage. The first step in the resuscitation of patients with variceal hemorrhage is rapid administration of intravenous (IV) fluids and octreotide.\n\nEsophageal and gastric varices, most commonly seen in patients with cirrhosis, are abnormally distended veins that are a product of venous congestion from portal hypertension. As these varices can project into the gastrointestinal lumen, they are subject to rupture either spontaneously or by mechanical disruption (e.g., tearing by food particles). Variceal hemorrhage is a life-threatening complication of cirrhosis as it can quickly progress to hemorrhagic shock or airway loss (if profuse vomiting is present). Patients should immediately receive 2 large-bore peripheral IVs (or central IV access) with which to administer IV fluid boluses to increase circulating volume. Blood should be transfused as needed, and a massive transfusion protocol may be needed. Patients should also be given octreotide, a splanchnic vasoconstrictor, in an effort to divert blood away from the hemorrhaging varices. Other treatments that should be given include ceftriaxone (to reduce bacteremia and mortality), proton pump inhibitors, and possibly interventions such as tranexamic acid. Upper GI endoscopy and banding are the definitive therapy.\n\nStanley et al review the management of acute upper gastrointestinal bleeding with initial IV fluid resuscitation as the first measure to achieve hemodynamic stability. They also report that for patients with cirrhosis, vasoactive drugs such as octreotide should be started as soon as variceal hemorrhage is suspected. Red blood cell transfusion is indicated at a hemoglobin transfusion of 7.0-8.0 g/dL. Definitive therapy is endoscopy with variceal ligation.\n\nIncorrect Answers:\nAnswer A: Balloon tamponade is often performed in the emergency department or intensive care unit as a temporizing measure to achieve hemostasis for patients with severe and persistent variceal hemorrhage. The balloon is inserted into the esophagus or stomach and inflated to apply direct pressure to the bleeding varices; this buys valuable time in the transition to definitive endoscopic management.\n\nAnswer B: Ceftriaxone is administered to patients with cirrhosis who present with an upper gastrointestinal bleed as a prophylactic measure against spontaneous bacterial peritonitis. This patient is hemodynamically unstable, and antibiotics should not delay initial intravenous fluid resuscitation as well as other measures to reduce and/or stop the bleeding. While this patient should receive ceftriaxone, it is more dire to first give fluids, blood, and octreotide.\n\nAnswer C: Nadolol or other nonselective beta-blocker therapy is indicated as secondary prophylaxis to prevent future bleeding episodes for patients with a history of variceal hemorrhage. Non-selective beta blockers such as nadolol decrease portal pressure, but are contraindicated in the acute setting as they could exacerbate hypovolemic shock by reducing cardiac output.\n\nAnswer D: Platelet transfusion is required for patients with variceal hemorrhage with an initial platelet count < 50,000/mm^3, and for those who receive a significant amount of blood products during their resuscitation (significant dilutional thrombocytopenia can occur after 10-12 units of transfused red blood cells). This patient's platelet count is above 50,000/mm^3, and he has not yet received blood products.\n\nBullet Summary:\nThe most appropriate initial step in the management of a cirrhosis patient with acute variceal hemorrhage is the rapid administration of intravenous fluids and octreotide, a splanchnic vasoconstrictor.", "link": "https://step2.medbullets.com/testview?qid=216270"} {"question": "A 38-year-old woman presents to the emergency department with difficulty eating. Over the past week, she has had increasing difficulty chewing her food and progressive pain in the left side of her jaw. This morning, she was unable to close her mouth for several minutes after taking a bite of her breakfast. Two months ago, the patient had a root canal on a left molar. Her medical history is significant for hyperlipidemia, mild intermittent asthma, and type 2 diabetes mellitus. She has never smoked and has 10-15 alcoholic drinks per week. She works as an aide at a nursing home. Her temperature is 100.8\u00b0F (38.2\u00b0C), blood pressure is 133/74 mmHg, pulse is 105/min, and respirations are 14/min. On physical exam, there is a bluish hue to the skin on the lower left side of the patient\u2019s face. A 3x4 cm non-tender mass can be palpated inferior to the angle of the left mandible. Thick exudate is draining from an opening in the skin. The gram stain of the exudate can be seen in Figure A. Which of the following is the most appropriate treatment for this patient?", "choicesA": "Amphotericin B", "choicesB": "Clindamycin", "choicesC": "Metronidazole", "choicesD": "Penicillin", "choicesE": "Trimethoprim-sulfamethoxazole", "answer_idx": "D", "answer": "Penicillin", "explanation": "This patient presents with fever, trismus, and cutaneous drainage of sulfur granules in the setting of a recent dental procedure, which points to a diagnosis of cervicofacial actinomycosis. The most appropriate treatment for this disease is penicillin.\n\nActinomyces is a gram-positive, filamentous rod that causes a cervicofacial infection after direct inoculation during a dental procedure or trauma. It typically presents in an indolent fashion over a time course of weeks to months in which a hard, indurated mass evolves into multiple abscesses that drain through sinus tracts through the skin. The exudate is classically thick with small yellow \u201csulfur\u201d granules, although it may also be serosanguinous. Since the infection spreads by direct extension, patients may complain of pain when the infection infiltrates or compresses nearby structures. Trismus may occur for the same reasons. The treatment of choice is penicillin.\n\nValour et al. review the evidence regarding the treatment of Actinomyces infection. They discuss how prolonged (6- to 12-month) high-dose penicillin or amoxicillin is required for the treatment of this disease. They recommend preventive measures such as improvement of dental hygiene to prevent this disease.\n\nFigure/Illustration A demonstrates the histologic appearance of a basophilic \u201csulfur granule\u201d (red circle). These granules are found in the draining exudate in patients with actinomycosis.\n\nIncorrect Answers:\nAnswer A: Amphotericin B is the treatment of choice for fungal infections such as mucormycosis. This disease generally presents with a rapidly progressive soft tissue infection that can be fatal if left untreated. Patients would present with black fungal lesions generally in immunocompromised or diabetic hosts.\n\nAnswer B: Clindamycin may be used in the treatment of empyema as it has activity against many anaerobes. Although this patient has a substantial weekly alcohol intake and has a chief complaint of \u201cdifficulty eating,\u201d she has no true pulmonary symptoms and her presentation is more likely related to her recent dental procedure.\n\nAnswer C: Metronidazole may be used in the treatment of empyema. Although this patient has a substantial weekly alcohol intake and has a chief complaint of \u201cdifficulty eating,\u201d she has no true pulmonary symptoms and her presentation is more likely related to her recent dental procedure.\n\nAnswer E: Trimethoprim-sulfamethoxazole is the treatment of choice for Nocardiosis, which may cause cutaneous lesions in immunocompromised hosts. This disease more commonly causes pulmonary or central nervous system disease. Nocardia would not explain the granule seen in histology.\n\nBullet Summary:\nActinomyces is a gram-positive anaerobic rod that presents as an indurated mass with draining sinus tracts and should be treated with penicillin.", "link": "https://bit.ly/41Hqt8O"} {"question": "A 3-year-old boy presents with his parents to a pediatrician for a new patient visit. The child was recently adopted and little is known about his medical or family history. The child seems to be doing well, but the patient is much larger than any of the other 3-year-olds in his preschool class. He eats a varied diet at home and with limited juice and snack foods. His temperature is 98.6\u00b0F (37\u00b0C), blood pressure is 101/67 mmHg, pulse is 110/min, and respirations are 24/min. On physical exam, the patient is in the 73rd percentile for weight, 99th percentile for height, and 86th percentile for head circumference. He appears mildly developmentally delayed. He has a fair complexion and tall stature with a prominent sternum. The patient also has joint hypermobility and hyperelastic skin. He appears to have poor visual acuity and is referred to an ophthalmologist, who finds downward lens subluxation of the right eye. This child is most likely to develop which of the following complications?", "choicesA": "Aortic dissection", "choicesB": "Medullary thyroid cancer", "choicesC": "Osteoarthritis", "choicesD": "Thromboembolic stroke", "choicesE": "Wilms tumor", "answer_idx": "D", "answer": "Thromboembolic stroke", "explanation": "This patient presents with Marfanoid habitus, joint hypermobility, hyperelastic skin, developmental delay, and downward lens subluxation, which suggests a diagnosis of homocystinuria. Homocystinuria predisposes to thromboembolic events such as stroke.\n\nAlthough homocystinuria shares many features with Marfan syndrome, patients with homocystinuria also present with a fair complexion, intellectual disability, and downward subluxation of the lens (instead of the upward subluxation seen in Marfan syndrome). Patients with homocystinuria are also at an increased risk of thromboembolic events that disproportionally affect the cerebral vessels. Similar to Marfan syndrome, patients have phenotypic features of a Marfanoid habitus, including tall stature, long limbs, arachnodactyly, joint hypermobility, skin hyperelasticity, pectus deformity, and scoliosis. Treatment includes vitamin B6 supplementation.\n\nHasan et al. review the evidence regarding diseases associated with homocystinuria. They discuss how this disease is associated with cancer and Alzheimer disease. They recommend screening for this disease.\n\nIncorrect Answers:\nAnswer A: Aortic dissection is a complication of Marfan syndrome, an autosomal dominant mutation of the fibrillin-1 gene. It is also associated with elevated homocysteine levels. Although Marfan syndrome shares many features with homocystinuria, the fair complexion, and intellectual disability seen in this patient are specific to homocystinuria. Treatment involves avoidance of contact sports and beta blockers.\n\nAnswer B: Medullary thyroid cancer is part of the syndrome of multiple endocrine neoplasia type 2B (MEN2B), which is caused by a mutation in the RET proto-oncogene. Patients with MEN2B present with Marfanoid habitus, but they do not present with the other features described in this vignette, including fair complexion, intellectual disability, and downward lens subluxation. Treatment is with surgical excision of tumors.\n\nAnswer C: Osteoarthritis is a complication of Ehlers-Danlos syndrome (EDS). Although patients with EDS may present with many of the phenotypic features of both Marfan syndrome and homocystinuria, patients are much less likely to have lens subluxation. Osteoarthritis may be seen with higher incidence in patients with homocystinuria. Treatment is with brace support of unstable joints.\n\nAnswer E: Wilms tumor is associated with Beckwith-Wiedemann syndrome (BWS), a pediatric overgrowth disorder usually caused by a mutation at chromosome 11p15. BWS presents with macrosomia (\u201cgigantism\u201d), which is defined as height and weight > 97th percentile. BWS additionally presents with omphalocele, hemihyperplasia, visceromegaly, neonatal hypoglycemia, and other embryonal tumors, including hepatoblastoma and neuroblastoma. Treatment is with surgical correction of defects.\n\nBullet Summary:\nHomocystinuria presents with Marfanoid habitus, fair complexion, intellectual disability, downward lens subluxation, and an increased risk of thromboembolic events.", "link": "https://bit.ly/45Vk2SF"} {"question": "A 67-year-old woman presents from a nursing home with altered mental status. She was last known to be normal the night before. The patient has dementia at baseline and is minimally interactive. This morning, she was found to be obtunded. No further history was provided from the nursing home, and the patient is unable to provide any history. Her temperature is 95.0\u00b0F (35.0\u00b0C), blood pressure is 84/54 mmHg, pulse is 50/min, respirations are 9/min, and oxygen saturation is 98% on room air. Physical exam reveals an obtunded woman who only withdraws her extremities to pain. Her skin is cool and pale, and her hair is thin. There are no signs of trauma noted on exam. Laboratory values are notable for a whole blood sodium of 120 mEq/L. The patient is given several intravenous boluses of fluids and her blood pressure improves to 100/60 mmHg. Which of the following is most likely to confirm the underlying cause of this patient's symptoms?", "choicesA": "Administer glucagon", "choicesB": "Check serum TSH and free T4 levels", "choicesC": "Obtain blood cultures and a serum lactate", "choicesD": "Perform a CT scan of the head", "choicesE": "Perform an echocardiogram", "answer_idx": "B", "answer": "Check serum TSH and free T4 levels", "explanation": "This patient is presenting with obtundation, bradycardia, hypotension, hypothermia, thinning of her hair, and hyponatremia, which are concerning for myxedema coma. A serum TSH and free T4 would be most informative in confirming this diagnosis.\n\nMyxedema coma (decompensated hypothyroidism) is the most life-threatening manifestation of hypothyroidism. Key clinical features include hypothermia (often the temperature is <95.9\u00b0F (35.5\u00b0C)), bradycardia, hypotension, bradypnea (causing a respiratory acidosis), hypoxia, and obtundation. Immediate management is centered on warming the patient, warmed IV fluids, and protecting the patient's airway, if needed. If there is high clinical suspicion for myxedema coma, then IV levothyroxine can be given empirically. Otherwise, the TSH and free T4 should be checked, which will reveal an elevated TSH and a low free T4. Patients are often critically ill and require active rewarming and close monitoring in an ICU setting.\n\nWall reviews myxedema coma. Wall notes that myxedema coma is a life-threatening manifestation of hypothyroidism. She recommends that generally IV T4 (levothyroxine) is preferred over IV T3 in initial management.\n\nIncorrect Answers:\nAnswer A: Administering glucagon would be appropriate in a beta-blocker overdose, which presents with bradycardia, hypotension, and altered mental status. There is no indication that this patient intentionally overdosed, and her hyponatremia and thinning hair supports a diagnosis of hypothyroidism. Other treatments that can be given in a beta-blocker overdose including epinephrine, calcium, insulin, dextrose, and lipid emulsion therapy.\n\nAnswer C: Obtaining blood cultures and a serum lactate is the appropriate management of sepsis/septic shock. Generally, patients with septic shock present with hypotension (refractory to fluid resuscitation), tachycardia, and fever or hypothermia. Though myocardial depression may happen in critically ill patients who are about to undergo cardiac arrest, hypothyroidism better explains this patient's constellation of symptoms. It would be appropriate to obtain cultures and a serum lactate in this patient while the differential is being narrowed.\n\nAnswer D: Performing a CT scan of the head would be indicated at some point in the workup of patients with altered mental status if there was a suspected central nervous system etiology such as a subdural hematoma. However, hemodynamically unstable patients should never be sent to the CT scanner prior to stabilization and resuscitation. A subdural hematoma is more common in older patients and patients with alcohol use disorder who experience head trauma as the atrophy of their brain makes the bridging veins more vulnerable.\n\nAnswer E: Performing an echocardiogram may show reduced cardiac function; however, it is unlikely to reveal the underlying cause of this patient's hypotension and bradycardia other than showing poor cardiac function.\n\nBullet Summary:\nMyxedema coma presents as a critically ill, obtunded patient who is hypothermic, bradycardic, and hypotensive, and the diagnosis can be supported with a high TSH and a low free T4.", "link": "https://step2.medbullets.com/testview?qid=216396"} {"question": "A 43-year-old man presents to the emergency department acutely with altered mental status. He uses intravenous drugs and is unable to offer a history. He was found unconscious in a park. His temperature is 99.0\u00b0F (37.2\u00b0C), blood pressure is 80/40 mmHg, pulse is 156/min, and respirations are 26/min. Physical exam reveals an obtunded man covered in vomit, with track marks on his arm and several skin abscesses. The patient is given 4 liters of Ringer lactate and intravenous antibiotics. As the fourth liter of fluids is finishing, the patient is noted to have jugular venous distension and an oxygen saturation of 92% on nonrebreather. He is subsequently intubated. An ECG is obtained as seen in Figure A. The patient is started on norepinephrine, and his blood pressure is subsequently found to be 85/44 mmHg. Which of the following is the most appropriate next step in management?", "choicesA": "Cardioversion", "choicesB": "Epinephrine", "choicesC": "Hydrocortisone", "choicesD": "Normal saline", "choicesE": "Phenylephrine", "answer_idx": "B", "answer": "Epinephrine", "explanation": "This patient who uses intravenous drugs with skin abscesses (the likely source of his infection) is hypotensive and tachycardic, with a blood pressure that is not responding to IV fluids; this is concerning for septic shock. Given his persistent hypotension, he should be started on vasopressors of which norepinephrine is first line and epinephrine is second line.\n\nSeptic shock is a life-threatening, systemic bacterial infection that leads to vasodilation and endothelial dysfunction. Early management is centered on IV fluids, blood cultures, broad-spectrum antibiotics, and workup to uncover the source. If the patient's blood pressure does not respond to IV fluids (by definition, their blood pressure will not respond to fluids in septic shock), vasopressors should be started with a goal mean arterial pressure > 65 mmHg. The first line vasopressor is norepinephrine, which can be titrated up for maximal effect. If this vasopressor fails to reach blood pressure goals, then the next vasopressor is either vasopressin or epinephrine (either is acceptable). If the second line vasopressor fails to increase blood pressure, then the third line vasopressor is vasopressin or epinephrine (whichever was not used as the second line pressor). Finally, the fourth line vasopressor for septic shock is phenylephrine.\n\nLevy et al. review sepsis. They update the Surviving Sepsis campaign which gives the most up to date management guidelines of sepsis including adequate fluid resuscitation, broad-spectrum antibiotics, and vasopressors. They recommend early antibiotics and resuscitation to minimize morbidity and mortality.\n\nFigure/Illustration A is an ECG demonstrating sinus tachycardia. Note the presence of P waves (red arrows highlighting several examples) that are present throughout all leads.\n\nIncorrect Answers:\nAnswer A: Cardioversion would be appropriate in an unstable patient with unstable vital signs in the setting of atrial fibrillation with rapid ventricular response to convert the patient to normal sinus rhythm. Atrial fibrillation would present with an irregularly irregular rhythm without P waves. Cardioversion would not be indicated in sinus tachycardia.\n\nAnswer C: Hydrocortisone is a potent glucocorticoid that also has mineralocorticoid properties. While giving an IV steroid may have a permissive effect on blood pressure (via upregulating alpha-1 receptors on the vasculature) there is no reason that this patient would have a low cortisol level (such as in chronic steroid use). Epinephrine is a better answer in this case, though steroids can be considered. Steroids are typically reserved for patients failing at least 2 vasopressors (unless they have a known cortisol-deficient state).\n\nAnswer D: Normal saline is unlikely to benefit this patient as he has already been sufficiently fluid resuscitated. Moreover, it seems the patient may be developing pulmonary edema (hypoxia, jugular venous distension), a sign of fluid overload.\n\nAnswer E: Phenylephrine is the fourth line vasopressor and is an alpha-1 agonist. It is much less potent than norepinephrine and epinephrine, and would not be started in the management of septic shock until the patient has received epinephrine, norepinephrine, and vasopressin.\n\nBullet Summary:\nIn septic shock, the vasopressors given (in order) are norepinephrine, epinephrine/vasopressin, then phenylephrine.", "link": "https://step2.medbullets.com/testview?qid=216495"} {"question": "A 53-year-old man presents to the clinic with a non-tender mass in his right anterior neck. The mass has been slowly enlarging over the past 1 year. He reports no dysphagia, difficulty breathing, or changes to his voice. He also denies heat intolerance, palpitations, tremors, or diarrhea. He has hypertension and hyperlipidemia, for which he takes lisinopril and atorvastatin. He has never smoked cigarettes. His temperature is 37.1\u00b0C (98.8\u00b0F), blood pressure is 130/84 mmHg, pulse is 86/min, respirations are 12/min, and oxygen saturation is 99% on room air. Bedside ultrasound reveals a 1x2 cm hypoechogenic nodule with microcalcifications in the right thyroid lobe, which is biopsied. Pathology results are shown in Figure A. Which of the following is the most appropriate next step in management?", "choicesA": "Levothyroxine", "choicesB": "Methimazole", "choicesC": "Radioactive iodine", "choicesD": "Repeat ultrasound in 6 months", "choicesE": "Thyroidectomy followed by radioactive iodine", "answer_idx": "E", "answer": "Thyroidectomy followed by radioactive iodine", "explanation": "This patient who presents with a slowly enlarging neck mass has ultrasound (hypoechogenic nodule with microcalcifications) and histology findings (concentrically laminated calcifications called psammoma bodies) consistent with papillary thyroid carcinoma. The most appropriate next step in management would be thyroidectomy followed by radioiodine ablation.\n\nPapillary thyroid carcinoma represents the majority of malignant thyroid cancers. They are slow-growing and are found in the hormone-producing follicular cells of the thyroid. Typically, these tumors are found incidentally on imaging. Risk factors for thyroid malignancy include a history of radiation exposure, female sex, and positive family history. Nodules with minimal uptake on radioactive iodine scan, firm and immobile solitary nodules, and rapidly growing nodules with associated hoarseness are more suggestive of thyroid malignancy. Findings on histology include empty-appearing nuclei with central clearing, psammoma bodies, and nuclear grooves. The treatment is thyroidectomy followed by radioactive iodine ablation of any remaining thyroid cells. Long-term care should include levothyroxine at a dose sufficient to suppress thyroid-stimulating hormone (TSH) release that may stimulate any remaining malignant cells.\n\nOakley et al. studied if relatives of patients diagnosed as having papillary thyroid carcinoma are at increased risk using a retrospective cohort study. The authors found first-degree relatives were at a 5-fold increased risk, and second-degree relatives were at a 2-fold increased risk. The authors recommend more research into optimal approaches to screening for thyroid cancers.\n\nFigure/Illustration A is a histology section showing a psammoma body (blue circle), which are round, concentrically laminated calcifications that are formed after necrosis of papillary structures. Psammoma bodies and nuclei with central clearing (yellow circle) and nuclear grooves are common histology findings for papillary thyroid carcinoma.\n\nIncorrect Answers:\nAnswer A: Levothyroxine would be the treatment of choice in hypothyroidism and is important in the long-term care of papillary thyroid carcinoma to suppress the TSH release that may stimulate any remaining malignant cells after treatment (thyroidectomy and radioactive iodine).\n\nAnswer B: Methimazole is an agent that blocks thyroid peroxidase, thus inhibiting the synthesis of thyroid hormones. It is used in the treatment of hyperthyroidism and does not have a role in the treatment of papillary thyroid carcinoma.\n\nAnswer C: Radioactive iodine is used as an adjunct after thyroidectomy in the treatment of papillary thyroid carcinoma in order to target any thyroid cells that may remain even after surgical excision. Surgery is the primary therapy for papillary thyroid carcinoma. Radioactive iodine is the primary method of therapy in cases of Grave disease, toxic adenoma, or multinodular goiter.\n\nAnswer D: Repeating an ultrasound in 6 months to assess the growth of the nodule or the development of suspicious characteristics is appropriate in cases where the ultrasound and biopsy findings are consistent with a benign thyroid nodule. The ultrasound findings of hypoechogenicity, microcalcifications, hypervascularity, and taller-than-wide nodules suggest malignancy, and together with a biopsy consistent with thyroid malignancy would warrant surgical treatment with subsequent radioactive iodine ablation.\n\nBullet Summary:\nPapillary thyroid carcinoma is managed with thyroidectomy followed by radioactive iodine ablation and long-term levothyroxine to suppress thyroid-stimulating hormone stimulation of any remaining malignant cells.", "link": "https://bit.ly/44GDl0l"} {"question": "An 84-year-old man presents to the physician with numbness and tingling in the right hand for the last 6 months. He reports these symptoms started intermittently in the middle finger after returning from a golfing trip but has progressed to being nearly constant. The thenar eminence, thumb, and other digits are spared. He reports no other symptoms. His medical problems include hypertension, hyperlipidemia, hypothyroidism, and chronic neck pain for which he takes amlodipine, hydrochlorothiazide, atorvastatin, levothyroxine, and duloxetine. Surgical history consists of posterior lumbar fusion and elective left total hip arthroplasty. He has a 90-pack-year smoking history and quit 10 years ago. The patient\u2019s temperature is 99.6\u00b0F (37.6\u00b0C), blood pressure is 134/80 mmHg, pulse is 90/min, and respirations are 16/min. Physical exam reveals 4/5 strength in elbow extension and wrist flexion on the right. He has 5/5 strength in the remainder of the motor exam on the right upper extremity and on the left upper extremity. The triceps reflex is 1+ on the right compared to 2+ on the left. There is no spasticity. His neck pain, which radiates down his right arm, is reproduced with axial loading during neck extension with simultaneous rightward rotation and lateral bending. Which of the following is the most likely diagnosis in this patient?", "choicesA": "Amyotrophic lateral sclerosis", "choicesB": "Carpal tunnel syndrome", "choicesC": "Cervical spondylosis", "choicesD": "Syringomyelia", "choicesE": "Transverse myelitis", "answer_idx": "C", "answer": "Cervical spondylosis", "explanation": "This patient presents with chronic middle finger paresthesia, weakness in elbow extension and wrist flexion, and a diminished triceps reflex in the setting of a positive Spurling test (pain with axial loading during neck extension, rotation, and lateral bending towards the affected side), indicative of C7 cervical radiculopathy. Cervical spondylosis is a common cause of cervical radiculopathy in elderly patients with chronic neck pain, especially in patients with findings suggestive of degenerative joint diseases (history of lumbar fusion and hip arthroplasty).\n\nCervical spondylosis is caused by degeneration of the intervertebral disc with possible disc herniation and facet joints with osteophyte formation. These osteophytes and cervical disc herniation can cause compression of exiting nerve roots, leading to radiculopathy. Radicular symptoms depend on the nerve root affected. For example, C6 radiculopathy causes weakness in wrist extension and a decreased brachioradialis reflex while C7 radiculopathy causes weakness in elbow extension and wrist flexion with a decreased triceps reflex. Sensory symptoms may be present depending on the dermatome affected. The diagnosis is confirmed with MRI. Treatment consists of non-steroidal anti-inflammatory drugs (NSAIDs), muscle relaxants, and/or corticosteroids for pain. Surgical treatments for severe or refractory cases include anterior cervical discectomy and fusion.\n\nYoung outlines the diagnosis and management of cervical spondylotic myelopathy. The discusses its pathophysiology, including osteophyte formation and spinal cord dysfunction. They recommend considering decompression and fusion operations in selected patients.\n\nIncorrect Answers:\nAnswer A: Amyotrophic lateral sclerosis (ALS) is a motor neuron disease of the brain and spinal cord that leads to upper and lower motor neuron signs. ALS presents with hyperreflexia (as opposed to hyporeflexia), spasticity, weakness, and fasciculations. Treatment includes riluzole (to improve survival) and symptomatic management.\n\nAnswer B: Carpal tunnel syndrome is caused by compression of the median nerve as it travels through the carpal tunnel at the wrist. While the thenar eminence is typically spared due to take-off of the palmar cutaneous branch proximal to the carpal tunnel, the thumb and index finger are also affected. This patient\u2019s symptoms primarily involve the middle finger, which is more indicative of a radiculopathy. Treatment is activity modification, splinting, and ultimately, surgical decompression of the carpal tunnel if refractory.\n\nAnswer D: Syringomyelia is caused by a fluid-filled cavity (syrinx) within the spinal cord that leads to central cord syndrome, characterized by weakness that is greater in the upper extremities compared to lower extremities and loss of pain and temperature sensation in a cape-like distribution in the upper back and upper extremities. This patient\u2019s sensory loss is unilateral and in a dermatomal distribution. Treatment of syringomyelia can include surgical decompression.\n\nAnswer E: Transverse myelitis is characterized by segmental inflammation of the spinal cord that would present with acute, bilateral flaccid paresis based on the level affected and sensory loss within a sensory level. Autonomic symptoms such as incontinence may also be present. This patient\u2019s symptoms are unilateral and localizable to nerve root compression as opposed to the spinal cord.\n\nBullet Summary:\nCervical spondylosis is caused by degeneration of cervical discs and facet joints with osteophyte formation and can present with symptoms of cervical radiculopathy or myelopathy in elderly patients with chronic neck pain.", "link": "https://bit.ly/47eGzd4"} {"question": "A 2-year-old boy is brought to the pediatrician\u2019s office for a well-child visit. The parents have no concerns at this time and note he is starting to run around and speak in 3-word sentences. He has no remarkable medical history and his mother had an uncomplicated full-term pregnancy and delivery. He is appropriately meeting the developmental milestones for his age. The patient\u2019s temperature is 98.4\u00b0F (36.9\u00b0C), blood pressure is 96/52 mmHg, pulse is 102/min, and respirations are 25/min. He is in the 55th percentile for height and 62nd percentile for weight, which is consistent with his documented growth curves. Fundoscopic exam shows the finding in Figure A. His pediatrician continues the ocular examination and further notes that the pupils are equal, round, and reactive, extraocular movements are full, and ocular alignment appears normal. Which of the following is the most appropriate next step in management?", "choicesA": "Biopsy", "choicesB": "Genetic testing", "choicesC": "Magnetic resonance imaging of the brain and orbits", "choicesD": "Ophthalmology referral", "choicesE": "Reevaluation at the next well-child visit", "answer_idx": "D", "answer": "Ophthalmology referral", "explanation": "This otherwise healthy patient presents with leukocoria, or a white pupillary reflex, and requires an urgent referral to ophthalmology for further evaluation given the concern for retinoblastoma.\n\nThe differential diagnosis for leukocoria includes retinoblastoma, cataract, vitreous hemorrhage, retinal detachment, and other intraocular/systemic disease processes. Newly diagnosed leukocoria should be urgently referred to ophthalmology to exclude retinoblastoma. Retinoblastoma is the most common intraocular cancer of childhood caused by mutations in the RB1 tumor suppressor gene that encodes the retinoblastoma protein. Retinoblastoma typically presents as leukocoria in a child under the age of 3 with other signs including strabismus, nystagmus, or a red, inflamed eye. Treatment options include chemotherapy, laser photocoagulation, radiation, cryotherapy, and enucleation. Prompt diagnosis and treatment are important for optimal visual outcome and survival.\n\nFrancis et al. study the risk of extraocular extension in eyes with retinoblastoma that received intravitreous chemotherapy injections. The authors find that the risk of extraocular extension is small. The authors recommend the regular use of precautionary injection methods including lowering of intraocular pressure, cryotherapy, ocular surface irrigation, and ultrasonic biomicroscopy surveillance of the injection site.\n\nFigure/Illustration A shows a child with leukocoria (blue circle) in the left eye.\n\nIncorrect Answers:\nAnswer A: Biopsy of the eye is contraindicated in retinoblastoma due to the risk of tumor seeding.\n\nAnswer B: Genetic testing can be used to determine if a patient has the hereditary form of retinoblastoma, characterized by germline RB1 mutations, as opposed to the nonheritable form, which is associated with somatic RB1 mutations. An ophthalmological evaluation is required first to confirm the diagnosis.\n\nAnswer C: While magnetic resonance imaging of the brain and orbits should be performed in all children with suspected retinoblastoma to assess disease extent, an ophthalmological evaluation should be completed first.\n\nAnswer E: Reassurance or reevaluation at the next well-child visit is not appropriate management of newly diagnosed leukocoria given that undiagnosed retinoblastoma can be vision- and life-threatening.\n\nBullet Summary:\nLeukocoria, or a white pupillary reflex, in a child requires urgent ophthalmologic referral to rule out retinoblastoma.", "link": "https://bit.ly/3RhK2Tp"} {"question": "A 17-year-old boy presents to the emergency department after a car accident. He drove his car into a pole while speeding at night. He has no past medical history and takes no medications. The patient is unable to offer a history and repetitively asks the nurses what happened. He knows his name and address and is able to remember other information about himself. He is complaining of a headache. His temperature is 97.6\u00b0F (36.4\u00b0C), blood pressure is 122/74 mmHg, pulse is 90/min, respirations are 12/min, and oxygen saturation is 98% of room air. Physical exam reveals a confused young man. Cranial nerves 2-12 are intact and the patient has normal strength and sensation. He demonstrates a stable gait. Throughout the exam, he continuously asks what happened to him. A head CT scan is ordered. Which of the following is the most likely diagnosis?", "choicesA": "Cerebral contusion", "choicesB": "Concussion", "choicesC": "Diffuse axonal injury", "choicesD": "Epidural hematoma", "choicesE": "Subarachnoid hemorrhage", "answer_idx": "B", "answer": "Concussion", "explanation": "This patient is presenting after a motor vehicle accident with confusion, repeated questioning, and a normal neurologic exam aside from his inability to remember the events that occurred. This constellation of symptoms is concerning for a concussion.\n\nA concussion occurs after blunt trauma to the head and is common in athletics and motor vehicle collisions. Patients may present initially with symptoms including confusion, repeat questioning, repeating themselves, and amnesia surrounding the event. A thorough neurological exam is important as there should be no neurologic deficits in a concussion. Patients with a classic history and reassuring exam require rest from physical and cognitive activity. Post-concussive syndrome may present with chronic headaches, dizziness, trouble sleeping, trouble focusing, nausea, and difficulties with learning and concentrating. Mood changes such as depression and anxiety are common. Any patient with a concerning mechanism or symptoms should have a head CT to rule out other more severe diagnoses.\n\nPertab et al. review concussion and its presentation. They note that concussion can cause many symptoms including trouble focusing, mood changes, headaches, and also autonomic nervous system anomalies. They recommend reviewing the constellation of symptoms to make the diagnosis.\n\nIncorrect Answers:\nAnswer A: Cerebral contusions are common after head trauma and may present with confusion, obtundation, neurologic deficits, and amnesia. While a severe concussion and a cerebral contusion have overlap, given this patient\u2019s age, mechanism, and symptoms, concussion is more likely. A head CT would demonstrate contusion (hyperdense intraparenchymal lesion) with a possible contrecoup injury.\n\nAnswer C: Diffuse axonal injury occurs after a severe CNS injury with an obtunded patient who is typically unresponsive. CT imaging will show blurring of the gray-white interface with punctate hemorrhages. These patients have a very poor prognosis. They often are obtunded with minimal neurologic function.\n\nAnswer D: Epidural hematoma presents with trauma and the classic \u201ctalk and die\u201d syndrome including a lucid interval followed by progressive obtundation and fixed, dilated pupils. Symptoms are rapidly progressive as this is an arterial bleed. CT scan would demonstrate a hyperdense lens-shaped lesion (composed of blood). A burr hole may relieve pressure and can be life-saving.\n\nAnswer E: Subarachnoid hemorrhage may present after trauma with a severe headache and meningeal signs. Patients may demonstrate focal neurologic deficits in severe cases. CT scan will demonstrate blood in the subarachnoid space and lumbar puncture would demonstrate blood in the CSF. A CTA may also localize the site of bleeding and if any aneurysm is present.\n\nBullet Summary:\nA severe concussion can present with confusion, repeat questioning, and amnesia.", "link": "https://step2.medbullets.com/testview?qid=216509"} {"question": "A 47-year-old man presents to the emergency department with a headache. He suddenly developed a throbbing, bitemporal headache about 5 hours ago \"out of nowhere.\" He has a history of migraine headaches, but he feels that this headache is significantly more painful than his typical migraines. The patient took his prescribed sumatriptan with no relief of his symptoms. He also endorses nausea and he reports that he vomited once before arrival in the emergency department. The patient denies any recent trauma to the head. His medical history is significant for migraines and hypertension. He has a 20-pack-year smoking history and a history of cocaine use. He drinks 5-6 beers per week. His temperature is 98.6\u00b0F (37\u00b0C), blood pressure is 147/91 mmHg, pulse is 62/min, and respirations are 12/min. On physical exam, he appears to be in moderate distress and has pain with neck flexion. He has no focal neurologic deficits. A head CT is performed and can be seen in Figure A. This patient\u2019s condition affects the space in between which of the following brain layers?", "choicesA": "Arachnoid mater and pia mater", "choicesB": "Dura mater and arachnoid mater", "choicesC": "Periosteum and galea aponeurosis", "choicesD": "Periosteum and skull", "choicesE": "Skull and dura mater", "answer_idx": "A", "answer": "Arachnoid mater and pia mater", "explanation": "This patient presents with a sudden, severe headache and a head CT showing bleeding in the subarachnoid space, which confirms a diagnosis of subarachnoid hemorrhage. The bleeding in a subarachnoid hemorrhage occurs between the arachnoid and pia mater.\n\nSubarachnoid hemorrhages present with a sudden, severe headache referred to as a \u201cthunderclap\u201d headache. Patients typically complain that the headache is the \"worst headache of their life.\" Some patients also present with symptoms of meningeal irritation such as nausea and vomiting, nuchal rigidity, or focal neurologic deficits. Although most cases of subarachnoid hemorrhage are secondary to trauma, patients with risk factors such as hypertension and cocaine use can have non-traumatic presentations. Patients with an expanding bleed or acute neurologic changes require surgical clipping or embolization of the bleeding vessels.\n\nMacdonald and Schweizer review the evidence regarding the treatment of patients with subarachnoid hemorrhage. They discuss how survival rates have improved with early aneurysm repair, prescription of nimodipine, and advanced intensive care support. They recommend early diagnosis in order to improve outcomes in these patients.\n\nFigure/Illustration A is an axial slice of a head CT showing hyperintensity in the basal cisterns (red circle). This finding is consistent with bleeding in the subarachnoid space.\n\nIncorrect Answers:\nAnswer B: Bleeding between the dura and arachnoid mater describes a subdural hematoma. A subdural hematoma presents as a headache with progressive confusion. It appears as a crescent-shaped lesion on head CT. Patients with an expanding bleed will require surgical decompression to avoid herniation syndromes.\n\nAnswer C: Bleeding between the periosteum and galea aponeurosis describes a subgaleal hemorrhage. Subgaleal hemorrhages occur in newborns and are caused by vacuum-assisted deliveries. Treatment is usually supportive as the bleeding will resolve by itself. Some patients with severe bleeds may require transfusion due to excessive blood loss.\n\nAnswer D: Bleeding between the periosteum and skull describes a cephalohematoma. Cephalohematomas occur in newborns and are caused by rupture of the blood vessels crossing the periosteum. They are usually secondary to a prolonged second stage of labor. Treatment is supportive as the blood will resolve and be resorbed.\n\nAnswer E: Bleeding between the skull and dura mater describes an epidural hematoma. An epidural hematoma presents as a brief loss of consciousness followed by a lucid interval and then progressive confusion or somnolence. It appears as a biconvex lesion on head CT. Treatment is with urgent surgical decompression as patients will rapidly decompensate after the lucid interval.\n\nBullet Summary:\nSubarachnoid hemorrhage classically presents as a thunderclap headache with a brief loss of consciousness and meningismus.", "link": "https://step2.medbullets.com/testview?qid=109022"} {"question": "An 82-year-old man with a history of atrial fibrillation presents to the emergency room complaining of sudden-onset, painless vision loss in his right eye starting 2 days ago. When it started, he felt like a \u201ccurtain was pulled down\u201d over his right eye but wasn't able to seek care immediately because it was snowing outside and he didn't want to travel in inclement weather. A review of systems is significant for occasional palpitations and presyncope. The patient\u2019s medications include aspirin and carvedilol. He reports that he is a smoker with a 50-pack-year history. The patient\u2019s temperature is 99\u00b0F (37.2\u00b0C), blood pressure is 112/72 mmHg, pulse is 84/min, and respirations are 14/min with an oxygen saturation of 97% on room air. Physical exam reveals 20/800 vision in the right eye. A fundoscopic exam of the right eye is shown in Figure A. Physical and fundoscopic examinations of the left eye are unremarkable. Which of the following is the most appropriate next step in management?", "choicesA": "Administer ranibizumab", "choicesB": "Administer tissue plasma activator", "choicesC": "Continue aspirin and start statin therapy", "choicesD": "Start retinal photocoagulation", "choicesE": "Start timolol drops", "answer_idx": "C", "answer": "Continue aspirin and start statin therapy", "explanation": "This patient with a history of atrial fibrillation is presenting with acute-onset, painless, monocular vision loss and a fundoscopic exam positive for retinal whitening. This suggests a diagnosis of central retinal artery occlusion and management should focus on control of underlying risk factors given that he is 2 days out from the event.\n\nSymptoms of central retinal artery occlusion (CRAO) include sudden, painless, complete, and normally unilateral vision loss. A fundoscopic exam can show pallor of the optic disc, diffuse ischemic whitening of the retina, cherry-red macula, retinal edema, and constricted/bloodless retinal arterioles. The center of the macula appears red because the photoreceptor layer at the fovea is perfused by the posterior choroidal vasculature rather than the central retinal artery. Treatment involves surgical decompression of the retinal anterior chamber, ideally within 1 hour of the occlusion. Tissue plasminogen activator should be administered if the patient presents within an acceptable time frame. If surgical intervention is not readily available, additional steps with minimal evidence to support their use include ocular massage (to dislodge the embolus) and either carbogen therapy or hyperbaric oxygen. Chronic management includes control of underlying risk factors with aspirin and statin therapy.\n\nDattilo et al. review the evidence regarding the treatment of central retinal artery occlusion. They discuss how treatments have not been particularly effective in the treatment of this disorder. They recommend engaging in secondary prevention of vascular events by controlling risk factors.\n\nFigure/Illustration A is a fundoscopic photograph showing retinal pallor surrounding the superior temporal arcade (red circle). This finding is characteristically seen in central retinal artery occlusion.\n\nIncorrect Answers:\nAnswer A: Administering ranibizumab is not appropriate because this medication is an anti-VEGF antibody, which can be used to treat wet age-related macular degeneration. Symptoms of wet age-related macular degeneration include acute visual distortion and loss of central vision that occurs over months. A fundoscopic exam would show neovascularization.\n\nAnswer B: Administration of tissue plasma activator has not been shown to be beneficial in central retinal artery occlusion if the event has occurred several days ago. This medication is used in patients who sustain an ischemic stroke within 4.5 hours of developing symptoms. A stroke in the posterior lobe can present with cortical hemianopsia.\n\nAnswer D: Retinal photocoagulation is used to treat diabetic retinopathy. In diabetic retinopathy, symptoms of painless vision loss present gradually. A retinal exam would show retinal hemorrhage and possibly neovascularization.\n\nAnswer E: Timolol is a beta-blocker used to treat glaucoma. Assessment of the optic disk would show an enlarged cup-to-disk ratio. Glaucoma can present with pain and pressure in the affected eye. Acute angle closure glaucoma would present with a rock-hard eye and may lead to permanent vision loss if not treated promptly.\n\nBullet Summary:\nChronic management of central retinal artery occlusion involves control of underlying risk factors with aspirin and statin therapy.", "link": "https://step2.medbullets.com/testview?qid=108988"} {"question": "A 34-year-old nulliparous woman presents to the clinic with 2 weeks of copper-colored discharge from her right nipple. Her breast has not been tender, and she has not noticed any changes to the skin. She is otherwise healthy and does not take any medications. Her temperature is 36.5\u00b0C (97.7\u00b0F), blood pressure is 110/82 mmHg, pulse is 68/min, respirations are 12/min, and oxygen saturation is 99% on room air. Exam of the right breast reveals no mass on palpation. Bloody discharge is expressed from the right nipple. There is no axillary lymphadenopathy. Which of the following is the most likely diagnosis?", "choicesA": "Ductal carcinoma in situ", "choicesB": "Intraductal papilloma", "choicesC": "Invasive ductal carcinoma", "choicesD": "Mammary duct ectasia", "choicesE": "Paget disease of the breast", "answer_idx": "B", "answer": "Intraductal papilloma", "explanation": "This young patient who presents with non-painful unilateral bloody nipple discharge and no palpable mass on breast exam most likely has an intraductal papilloma.\n\nIntraductal papilloma is a benign epithelial tumor that develops in the lactiferous ducts of the breast and is the most common cause of bloody nipple discharge in premenopausal women. Patients typically present with painless unilateral bloody nipple discharge, and often there is no mass palpable on breast exam. Ultrasound is the initial method for evaluation and may reveal an intraductal mass, but if results are inconclusive, further imaging with mammography is indicated. If the results of both still remain inconclusive, galactography (radiography after injecting radiopaque contrast into the duct system) is the most accurate imaging method for diagnosing intraductal papilloma. A core biopsy of the intraductal mass should be obtained, and if results show papilloma with atypia, treatment would be surgical excision of the involved duct to rule out a concomitant malignant neoplasm.\n\nKhan et al. review the treatment of intraductal papilloma and note that in lesions with core biopsy showing papilloma with atypia, surgical excision revealed the diagnosis of a concomitant invasive or ductal in situ cancer is common. The authors also find that in patients whose core biopsies showed papilloma without atypia, the likelihood of cancer was much lower. The authors recommend that since the diagnosis of intraductal papilloma carries an increased risk of breast cancer compared to the general population, these patients should be monitored accordingly.\n\nIncorrect Answers:\nAnswer A: Ductal carcinoma in situ (DCIS) is a neoplastic lesion confined to the breast duct that if untreated develops into invasive ductal carcinoma. Typically, patients are asymptomatic, and lesions are detected via screening mammography. Though it can sometimes present with nipple discharge, this is less common. Unilateral painless bloody nipple discharge is more likely to be due to intraductal papilloma, though no mass may be palpable for either diagnosis.\n\nAnswer C: Invasive ductal carcinoma is the most common type of breast cancer, which typically presents as a firm, immobile, painless lump in the breast, sometimes associated with nipple discharge, overlying skin changes, inverted nipple, and/or axillary lymphadenopathy. As with DCIS, nipple discharge may be present but is usually not bloody as in cases of intraductal papilloma.\n\nAnswer D: Mammary duct ectasia is a benign condition in which a milk duct in the breast widens and its walls thicken, leading to blockage and build-up of fluid in the duct. This is usually asymptomatic but can present with off-white or greenish nipple discharge. It may also result in infection of the affected duct, leading to periductal mastitis and a tender, erythematous breast. It tends to occur in perimenopausal women and often resolves without treatment, though if symptoms persist it can be treated with antibiotics and duct excision.\n\nAnswer E: Paget disease of the breast often presents as a scaly, raw, vesicular, or ulcerated lesion that begins on the nipple and then spreads to the areola, sometimes associated with bloody nipple discharge. Pain, burning, and pruritis are also common presenting symptoms. There is often an underlying breast carcinoma; thus, any patient with Paget disease should be evaluated with a mammogram. Although this patient has bloody nipple discharge, she does not present with a painful, scaly, raw, or vesiculated lesion that would be concerning for Paget disease of the breast.\n\nBullet Summary:\nIntraductal papilloma often manifests in patients as painless unilateral bloody nipple discharge, and typically no mass is palpable on exam.", "link": "https://bit.ly/3IgfxrD"} {"question": "A 21-year-old man presents to the emergency room with right lower quadrant pain that started this afternoon at his wrestling match. He also endorses loss of appetite and subjective fever. He has had vague, mild discomfort in the right upper quadrant since the start of this wrestling season. He endorses a history of unprotected sex with multiple partners and intravenous drug use during a recent vacation to Mexico but has no other significant medical history. He drinks socially and has never smoked. His temperature is 98.6\u00b0F (37\u00b0C), blood pressure is 119/72 mmHg, pulse is 102/min, and respirations are 12/min. Physical exam demonstrates pain on palpation of the right lower quadrant and mild pain in the right upper quadrant. He has significant acne on his face, chest, and back. A CT of the abdomen confirms appendicitis. The scan also shows a large liver mass measuring 6 cm as seen in Figure A. Aside from addressing his appendicitis, which of the following is the most appropriate next step in management?", "choicesA": "Administer metronidazole", "choicesB": "Administer sofosbuvir", "choicesC": "Obtain follow-up CT of the abdomen in 3 months", "choicesD": "Perform liver mass biopsy", "choicesE": "Perform liver mass resection", "answer_idx": "E", "answer": "Perform liver mass resection", "explanation": "This patient with a history of possible anabolic steroid use (wrestler with acne) who presents with right upper quadrant pain and an enhancing lesion in the liver most likely has a hepatic adenoma. The treatment of hepatic adenomas is surgical removal and discontinuation of precipitating agents.\n\nHepatic adenomas are benign epithelial tumors that typically arise in the right hepatic lobe. They are typically solitary masses but multiple lesions can be seen in patients with prolonged contraceptive or anabolic steroid use and those who have glycogen storage disorders. Patients present with abdominal pain that is localized in the epigastrium or in the right upper quadrant. Sudden, severe abdominal pain with hypotension suggests rupture of the hepatic adenoma into the peritoneum. The lesions can be confirmed on imaging such as ultrasound, CT, or MRI. For hepatic adenomas that are < 5 cm and asymptomatic, simply discontinuing the causative drug and observing are acceptable. Large or symptomatic lesions requiring surgical resection in addition to discontinuation of the offending agent. Because these tumors are highly vascular, failure to resect may result in rupture, hemoperitoneum, and shock. Biopsy is contraindicated to avoid rupturing the mass and causing hemorrhage.\n\nKlompenhouwer et al. studied the current management strategies in patients with hepatocellular adenoma. They found that various molecular groups have been identified allowing for subtyping of these lesions. They recommend performing a CT scan to better characterize these lesions.\n\nFigure/Illustration A is a CT with an enhancing mass with a hypoattenuating fatty component. This is characteristically seen in large hepatic adenomas, which are solitary well-marginated lesions.\n\nIncorrect Answers:\nAnswer A: Metronidazole would be an effective treatment if this patient had contracted Entamoeba histolytica from contaminated food or water on his recent trip to Mexico. While this amoeba can lead to a liver abscess presenting as a mass on CT, it often also causes abdominal pain and dysentery due to intestinal involvement.\n\nAnswer B: Sofosbuvir is an antiviral medication used for hepatitis C. This patient does have risk factors for hepatitis C (risky sexual behavior and intravenous drug use). Although hepatitis C could lead to hepatocellular carcinoma and cause a liver mass, this patient\u2019s presentation is not consistent with that diagnosis. Even if he did have hepatocellular carcinoma, treating hepatitis C alone without surgical resection or adjuvant therapies would be insufficient.\n\nAnswer C: Follow-up CT in 3 months would be dangerous for this man with a large hepatic adenoma. In the interim, the mass will likely continue to grow if he continues to use anabolic steroids, increasing the risk of rupture. Observation and repeat imaging may be suitable for asymptomatic hepatic lesions that are of uncertain but likely benign etiology.\n\nAnswer D: Biopsying the liver mass is appropriate if malignancy is suspected and there is no concern for hepatic adenoma. If an adenoma is on the differential, biopsy is contraindicated due to the risk of rupture and hemorrhage intraoperatively.\n\nBullet Summary:\nHepatic adenomas are benign, vascular liver masses associated with oral contraceptives and anabolic steroids that should be promptly resected if large and symptomatic.", "link": "https://step2.medbullets.com/testview?qid=108583"} {"question": "A 19-year-old man presents to his primary care physician. He is asymptomatic and has no acute complaints, has no medical problems, and takes no medications. He was born in the United States but moved to El Salvador at age 11 and has only recently moved back to the United States in the past year. He was up to date on his vaccinations prior to his move to El Salvador. He last received a diphtheria, tetanus, and acellular pertussis (DTaP) vaccine 14 years ago at age 5, the second dose of a meningococcal vaccine 3 years ago, and an influenza vaccine 2 months ago. The patient\u2019s temperature is 98.2\u00b0F (36.8\u00b0C), blood pressure is 120/76 mmHg, pulse is 82/min, and respirations are 16/min. There are no rashes noted on physical exam. Which of the following vaccine regimens is indicated in this patient?", "choicesA": "Influenza", "choicesB": "Tetanus and diphtheria", "choicesC": "Tetanus, diphtheria, acellular pertussis, and influenza", "choicesD": "Tetanus, diphtheria, and acellular pertussis", "choicesE": "Tetanus, diphtheria, and influenza", "answer_idx": "D", "answer": "Tetanus, diphtheria, and acellular pertussis", "explanation": "This patient has recently moved back to the United States after living in another country for several years and is behind on his tetanus, diphtheria, and pertussis vaccine series. The tetanus, diphtheria, and acellular pertussis (Tdap) vaccine is indicated at least once in adults who have not previously received Tdap.\n\nThe standard vaccination schedule for tetanus, diphtheria, and pertussis includes a 5-dose childhood vaccine series consisting of DTaP at ages 2 months, 4 months, 6 months, 15-18 months, and 4-6 years. The first booster dose then consists of Tdap at age 11-12, with a tetanus and diphtheria (Td) vaccine every 10 years afterwards. In adults who have not previously received Tdap, a single Tdap dose should be administered regardless of time interval since the last dose of Td. Subsequent Td booster doses should then be given at 10-year intervals. Contraindications to Tdap include prior anaphylaxis after a tetanus, diphtheria, or pertussis vaccine or a history of seizures or encephalopathy within 7 days of receiving a previous pertussis-containing vaccine.\n\nKerr et al. studied the risk of major birth defects after administering the Tdap vaccine in pregnant patients. The authors found no increase in the risk of malformations in the offspring of pregnant patients that received the Tdap vaccine. The authors recommend the administration of Tdap during pregnancy.\n\nIncorrect Answers:\nAnswer A: Influenza is incorrect because the influenza vaccine is recommended only once per flu season. This patient already received the flu vaccine during this flu season, so the influenza vaccine is not indicated.\n\nAnswer B: Tetanus and diphtheria only are incorrect because the Td vaccine booster is indicated every 10 years in adults who have already received a prior dose of Tdap. In adults who have never received a previous Tdap dose, Tdap is indicated unless contraindications exist.\n\nAnswer C: Tetanus, diphtheria, acellular pertussis, and influenza are incorrect because influenza vaccine is not indicated in this patient. This patient has already received a dose of the influenza vaccine during this flu season.\n\nAnswer E: Tetanus, diphtheria, and influenza are incorrect because neither the Td nor influenza vaccines are indicated in this patient. The Td vaccine is indicated every 10 years in adults who have already received a prior dose of Tdap. The influenza vaccine is only indicated once per flu season.\n\nBullet Summary:\nThe tetanus, diphtheria, and acellular pertussis (Tdap) vaccine is usually administered to adolescents at age 11-12 and is indicated at least once in adults who have not previously received Tdap.", "link": "https://step2.medbullets.com/testview?qid=215185"} {"question": "A 7-week-old boy presents to the pediatrician for vomiting. Three weeks ago, the patient began vomiting after meals. The vomitus appears to be normal stomach contents without streaks of red or green. His parents have already tried repositioning him during mealtimes and switching his formula to eliminate cow\u2019s milk and soy. Despite these adjustments, the vomiting has become more frequent and forceful. The patient is voiding about 4 times per day and his urine appears dark yellow. He has fallen 1 standard deviation off of the growth curve. The patient's mother reports that the pregnancy was uncomplicated other than an episode of sinusitis in the 3rd trimester, for which she was treated with azithromycin. The patient's temperature is 98.7\u00b0F (37.1\u00b0C), blood pressure is 58/41 mmHg, pulse is 166/min, and respirations are 16/min. On physical exam, the patient looks small for his age. His abdomen is soft, non-tender, and non-distended. Which of the following is the most appropriate next step in management?", "choicesA": "Abdominal ultrasound", "choicesB": "Intravenous hydration", "choicesC": "MRI of the head", "choicesD": "Pyloromyotomy", "choicesE": "Thickening feeds", "answer_idx": "B", "answer": "Intravenous hydration", "explanation": "This patient presents with nonbilious, forceful vomiting, poor weight gain, signs of dehydration, and a history of exposure to azithromycin, which suggests a diagnosis of pyloric stenosis. The most appropriate next step in management is intravenous hydration.\n\nPyloric stenosis classically presents in a male infant between 4 to 6 weeks of age with nonbilious, projectile vomiting after meals. Risk factors for pyloric stenosis include bottle feeding and exposure to macrolide antibiotics in the patient or mother. Protracted vomiting frequently leads to hypochloremic, hypokalemic metabolic alkalosis. Patients should be resuscitated with intravenous hydration and evaluated for electrolyte abnormalities that require repletion. After a patient is adequately resuscitated, patients can undergo an ultrasound to confirm the diagnosis and then be treated surgically for definitive therapy.\n\nAdams and Stanton review the evidence regarding the diagnosis and treatment of neonatal bowel obstruction. They discuss how malrotation, jejunoileal atresia, duodenal atresia, and colonic atresia are common causes of this finding. They recommend surgical management of these conditions.\n\nIncorrect Answers:\nAnswer A: Abdominal ultrasound is the test of choice to diagnose pyloric stenosis, but evaluation for electrolyte derangements and subsequent intravenous hydration and electrolyte repletion should be initiated first in this patient with signs of dehydration and malnutrition. Further evaluation is reserved after a patient is adequately resuscitated.\n\nAnswer B: MRI of the head would be appropriate if this patient had additional signs of a mass in the head, such as neurological changes or worsening of the vomiting in the morning. In the absence of these signs and the presence of a mass in the epigastrium, this patient is more likely to have pyloric stenosis.\n\nAnswer C: Pyloromyotomy is the definitive management of pyloric stenosis, but this patient should undergo fluid replacement and have any electrolyte derangements detected and managed before undergoing surgery. Surgical intervention can carry significant risks if patients are not adequately resuscitated.\n\nAnswer E: Thickening feeds are one of the techniques used to manage gastroesophageal reflux. In the setting of this patient\u2019s epigastric mass and clinical picture, pyloric stenosis is the more likely etiology of his vomiting. Reflex presents with non-projectile vomiting that is dependent on position.\n\nBullet Summary:\nPatients with pyloric stenosis should first undergo intravenous rehydration and electrolyte repletion, as protracted vomiting may lead to severe hypochloremic, hypokalemic metabolic alkalosis.", "link": "https://step2.medbullets.com/testview?qid=109242"} {"question": "A 34-year-old man presents to the emergency room with pain behind his eyes. He had a similar episode about 1 month ago which resolved after a few hours. The pain feels like a stabbing sensation behind his right eye. He denies any symptoms on the left side. The last time he had an episode like this, his eye was tearing. His temperature is 98.2\u00b0F (36.8\u00b0C), blood pressure is 120/90 mmHg, pulse is 110/min, and respirations are 12/min. Physical exam reveals a 2 mm pupil on the right and a 5 mm pupil on the left. Extraocular movements are intact bilaterally. The remainder of the neurologic exam is unremarkable. Which of the following is the most appropriate prophylactic treatment?", "choicesA": "Ibuprofen", "choicesB": "Oxygen", "choicesC": "Propranolol", "choicesD": "Sumatriptan", "choicesE": "Verapamil", "answer_idx": "E", "answer": "Verapamil", "explanation": "This patient with a history of retro-orbital pain, unilateral miosis (2 mm pupil on the right), and history of lacrimation during a similar episode is likely experiencing cluster headaches. Verapamil may be used as prophylaxis in the treatment of cluster headaches.\n\nCluster headaches occur more commonly in men and present with severe, unilateral, retro-orbital pain. Typical episodes last 15 minutes to 3 hours. Attacks may be associated with the circadian rhythm, as patients may notice that they get headaches at night. Because of the extreme pain, cluster headaches are sometimes colloquially referred to as \"suicide\" headaches. These headaches may present with autonomic symptoms, such as lacrimation and rhinorrhea. Horner syndrome may also occur, defined by a unilateral triad of ptosis, miosis, and anhidrosis. Acute management of cluster headaches includes administration of 100% oxygen. Other options include triptans. The first-line medication for prophylaxis of cluster headaches is verapamil. Studies have shown that lithium, topiramate, and ergotamine may also be effective as prophylaxis for cluster headaches; however, these therapies are less established than verapamil.\n\nBlau and Engel review the use of medications for cluster headaches. They note that most of patients with episodic cluster headaches had complete suppression of attacks with the use of verapamil. It is recommended to give verapamil in this patient population.\n\nIncorrect Answers:\nAnswer A: Ibuprofen is a non-steroidal anti-inflammatory drug (NSAID) that is used in the treatment of migraine and tension headaches. Migraine headaches present with unilateral, pulsating pain with photophobia and phonophobia, whereas tension headaches present with constant bilateral pain. Ibuprofen should not be used as prophylaxis for any type of headache as it has side effects including kidney dysfunction, ulcers, and even premature aging.\n\nAnswer B: Oxygen is first-line treatment for acute cluster headaches; however, it does not have any role in the prophylaxis of cluster headaches.\n\nAnswer C: Propranolol is a beta-blocker that is a prophylactic medication for migraine headaches. Migraines are more common in women and typically present as a unilateral headache with photophobia or phonophobia. They may also be associated with an aura, which presents as a visual, auditory, olfactory, or motor disturbance.\n\nAnswer D: Sumatriptan is a triptan that is used as an abortive treatment for migraine headaches and cluster headaches. It should not be used as prophylaxis for headaches.\n\nBullet Summary:\nVerapamil may be used as prophylaxis for cluster headaches.", "link": "https://step2.medbullets.com/testview?qid=217746"} {"question": "A 65-year-old man presents to his primary care physician for a routine appointment. He has no concerns. His past medical history is significant for human immunodeficiency virus (HIV) infection, diagnosed 15 years ago. The patient is intermittently compliant with his antiretroviral therapy (ART). His temperature is 99.0\u00b0F (37.2\u00b0C), blood pressure is 130/84 mmHg, pulse is 92/min, and respirations are 11/min. His most recent lab work showed a cluster of differentiation CD4 count of 150 cells/\u00b5L. He received a dose of the 13-valent pneumococcal vaccine and a dose of the 23-valent pneumococcal vaccine 15 years ago, as well as a second dose of the 23-valent pneumococcal vaccine 10 years ago. In addition to encouraging greater compliance with his ART, which of the following is indicated in this patient?", "choicesA": "Azithromycin and pneumococcal vaccine", "choicesB": "Azithromycin and zoster vaccine", "choicesC": "Trimethoprim-sulfamethoxazole and pneumococcal vaccine", "choicesD": "Trimethoprim-sulfamethoxazole and varicella vaccine", "choicesE": "Trimethoprim-sulfamethoxazole and zoster vaccine", "answer_idx": "C", "answer": "Trimethoprim-sulfamethoxazole and pneumococcal vaccine", "explanation": "This patient with HIV and a CD4 count of 150 cells/\u03bcL requires trimethoprim-sulfamethoxazole (TMP-SMX) prophylaxis and another dose of the 23-valent pneumococcal vaccine.\n\nAll HIV patients with a CD4 count < 200 cells/\u03bcL require TMP-SMX as prophylaxis against Pneumocystis jirovecci pneumonia. Patients with CD4 counts < 100 cells/\u03bcL also require TMP-SMX as prophylaxis against toxoplasmosis. HIV patients require a dose of the 23-valent pneumococcal vaccine at age 65. Patients should initially receive the 13-valent pneumococcal vaccine, followed by the 23-valent pneumococcal vaccine 8 weeks later, 5 years later, and at age 65. Compliance with ART is also critical in patients with HIV.\n\nKaplan et al. discuss recommendations for the prevention of opportunistic infections in HIV-infected adults. The authors find that daily prophylaxis with TMP-SMX is an appropriate regimen for the prevention of Pneumocystis pneumonia. The authors recommend dapsone, atovaquone, or pentamidine for patients that cannot receive TMP-SMX.\n\nIncorrect Answers:\nAnswer A: Azithromycin is used as prophylaxis against Mycobacterium avium complex (MAC) in patients with a CD4 count < 50 cells/\u03bcL that has not been started on ART. Since this patient has a CD4 count of 150 cells/\u03bcL, azithromycin is not indicated at this time.\n\nAnswer B: Azithromycin is used as prophylaxis against Mycobacterium avium complex (MAC) in patients with a CD4 count < 50 cells/\u03bcL that has not been started on ART. This patient should not receive the zoster vaccine as it is a live vaccine and is contraindicated in immunocompromised patients.\n\nAnswer D: Although this patient does require TMP-SMX as prophylaxis against Pneumocystis pneumonia, he should not receive the varicella vaccine as it is a live vaccine and therefore contraindicated in immunocompromised patients.\n\nAnswer E: Although this patient does require TMP-SMX as prophylaxis against Pneumocystis pneumonia, he should not receive the zoster vaccine as it is a live vaccine and therefore contraindicated in immunocompromised patients.\n\nBullet Summary:\nPatients with a CD4 count < 200 cells/\u03bcL require prophylaxis with TMP-SMX and all patients with HIV should receive a dose of the 23-valent pneumococcal vaccine at age 65.", "link": "https://step2.medbullets.com/testview?qid=215026"} {"question": "A 55-year-old woman is brought to the emergency department by her husband with a 1 hour history of an unremitting headache. The headache started suddenly while she was eating dinner and she says it feels like the \u201cworst headache of my life.\u201d An emergent CT scan of the head without contrast confirms the diagnosis, and a CT angiogram identifies the source of bleeding. The patient undergoes surgical management of her condition. On hospital day 3, she is found to be disoriented to person, place, and time. She also develops nausea and vomiting. Her medical problems consist of heart failure for which she takes furosemide, spironolactone, and metoprolol, which were continued at admission. Her temperature is 99.6\u00b0F (37.6\u00b0C), blood pressure is 100/60 mmHg, pulse is 112/min, and respirations are 16/min. Physical examination shows poor skin turgor. Capillary refill time is 4 seconds. Serum laboratory results are shown below: Na+: 120 mEq/L Cl-: 92 mEq/L K+: 3.9 mEq/L HCO3-: 26 mEq/L BUN: 32 mg/dL Creatinine: 1.0 mg/dL Serum osmolality is 265 mEq/L and urine osmolality is 340 mEq/L. Urine sodium is 44 mEq/L. Which of the following is the most likely diagnosis?", "choicesA": "Addison disease", "choicesB": "Cerebral salt wasting", "choicesC": "Diuretic overuse", "choicesD": "Primary polydipsia", "choicesE": "Syndrome of inappropriate anti-diuretic hormone", "answer_idx": "B", "answer": "Cerebral salt wasting", "explanation": "This patient presents with a subarachnoid hemorrhage (\u201cworst headache of my life\u201d) which is treated and then subsequently develops nausea, vomiting, and altered mental status with reduced serum sodium concentration, indicative of symptomatic hyponatremia. In the setting of serum hypo-osmolality, urine hyper-osmolality, urine sodium concentration > 20 mEq/L, and hypovolemia (low blood pressure, decreased skin turgor, and prolonged capillary refill), this is indicative of cerebral salt wasting.\n\nThe mechanism of cerebral salt wasting is unclear, although some hypotheses point to increased release of brain natriuretic peptide after injury leading to suppression of renal sodium reabsorption, or sympathetic nervous system injury resulting in an impaired renin response. Patients classically present several days after cerebral injury or neurosurgery with hypovolemic hyponatremia and hypochloremia. Water retention results in serum hypo-osmolality, hyponatremia, and hypochloremia with urine hyper-osmolality and increased urine sodium excretion. In patients with prolonged cerebral salt wasting, hypokalemia can also occur (potassium may be normal in acute states). Treatment in mild or moderate cases consists of normal saline infusion to correct both volume status and hyponatremia. Treatment in severe cases consists of hypertonic saline. During correction, serum sodium must be monitored closely to prevent central pontine myelinolysis.\n\nCui et al. describe the differences between syndrome of inappropriate anti-diuretic hormone (SIADH) and cerebral salt wasting. They discuss how volume status serves as the key differentiator between these conditions. They recommend paying closer attention to total body electrolyte balance in understanding these conditions.\n\nIncorrect Answers:\nAnswer A: Addison disease, also known as primary adrenal insufficiency, results in salt wasting due to decreased mineralocorticoid activity. Because of reduced aldosterone release, patients also have hyperkalemia and metabolic acidosis. In contrast, this patient\u2019s potassium and bicarbonate levels are normal. Treatment is with steroids.\n\nAnswer C: Diuretic overuse is occasionally observed in patients using loop diuretics. Overuse of loop diuretics can cause hyponatremia, hypokalemia, and hypochloremia. Metabolic alkalosis is commonly seen due to volume contraction. Cerebral salt wasting is more likely in this patient presenting after a recent cerebral injury. Treatment is the cessation of the causative medication.\n\nAnswer D: Primary polydipsia can cause symptomatic hyponatremia. The urine would be maximally diluted (urine osmolality < 100 mEq/L) due to an appropriate physiological response to hyponatremia and excess free water intake. Treatment is cessation of excessive water consumption.\n\nAnswer E: Syndrome of inappropriate anti-diuretic hormone (SIADH) presents with identical findings as cerebral salt wasting with the exception of euvolemia as opposed to hypovolemia. An assessment of volume status is essential to differentiate. This patient has clear signs of hypovolemia. Treatment of SIADH is fluid restriction followed by ADH inhibitors, if needed.\n\nBullet Summary:\nCerebral salt wasting presents with hyponatremia, hypochloremia, and hypovolemia in the setting of recent cerebral injury or neurosurgery.", "link": "https://step2.medbullets.com/testview?qid=217155"} {"question": "A 23-year-old woman presents to labor and delivery at 40 weeks gestation with abdominal contractions and leakage of fluid from her vagina. She subsequently undergoes an uncomplicated vaginal delivery. The child is healthy and is heated, suctioned, and stimulated. While holding her newborn, the mother endorses a headache, blurry vision, and abdominal pain. Her temperature is 98.0\u00b0F (36.7\u00b0C), blood pressure is 194/104 mmHg, pulse is 100/min, respirations are 18/min, and oxygen saturation is 98% on room air. Physical exam reveals an uncomfortable woman. There is blood in her vagina and her uterus is contracted. Her cranial nerve exam is unremarkable and she has normal strength and sensation. Laboratory studies and a urinalysis are pending. Which of the following is the most appropriate next step in management?", "choicesA": "CT head", "choicesB": "Labetalol", "choicesC": "Magnesium", "choicesD": "MRV head", "choicesE": "Ondansetron and morphine", "answer_idx": "C", "answer": "Magnesium", "explanation": "This postpartum patient is presenting with hypertension, headache, blurry vision, and abdominal pain, which are concerning for a diagnosis of preeclampsia. The most appropriate initial intervention is magnesium.\n\nPreeclampsia classically presents after 20 weeks gestation (or earlier in the setting of a molar pregnancy) with hypertension and proteinuria. Other symptoms that may be indicative of preeclampsia with severe features include headaches, visual changes, and abdominal pain. Other features consistent with preeclampsia include thrombocytopenia, kidney insufficiency, impaired liver function, and pulmonary edema. The diagnosis can be supported when a pregnant patient is both hypertensive and demonstrates an elevated protein in her urine. It is possible for postpartum patients to experience preeclampsia. Though the fetus and placenta have been delivered, which is the definitive treatment of preeclampsia, further medical management is still needed. The most important initial step in management is to administer magnesium which is neuroprotective and also modestly lowers blood pressure. Further care involves blood pressure control and benzodiazepines as needed to abort seizure episodes (though note that if a patient with preeclampsia is having seizures, by definition, they have eclampsia).\n\nAl-Safi et al. discuss preeclampsia and eclampsia. They note that postpartum preeclampsia and eclampsia can occur within the first week after delivery. They recommend that mothers should be educated on what symptoms to look for.\n\nIncorrect Answers:\nAnswer A: CT head may be indicated, in particular, if an intracranial hemorrhage was thought to be the cause of this patient\u2019s headache and vision changes. A subarachnoid hemorrhage would present with the classic \u201cthunderclap\u201d headache with blood in the subarachnoid space on head CT. A CTA may localize a bleeding aneurysm and a lumbar puncture would show an elevated red blood cell count in the CSF. It would not explain this patient\u2019s hypertension and abdominal pain. Note that a CT head is not a CTV which is specific venous imaging.\n\nAnswer B: Labetalol is an appropriate blood pressure medication to give in pregnancy; however, it is more appropriate to first give magnesium as this agent is neuroprotective and is the first-line agent of choice in preeclampsia.\n\nAnswer D: MRV head or a CTV head would be appropriate in the workup of cerebral venous thrombosis which is common in the setting of hypercoagulability (such as in the postpartum period) and presents with headache, blurry vision, and seizures. Note that it would not explain this patient\u2019s profound hypertension. It would be more reasonable to first empirically treat this patient's possible preeclampsia.\n\nAnswer E: Ondansetron and morphine would be appropriate symptomatic management of this patient but would not treat her underlying preeclampsia. It is more appropriate to first treat the life-threatening diagnosis before symptom management. If this patient\u2019s hypertension was thought to be due to pain, then morphine could be helpful.\n\nBullet Summary:\nPreeclampsia can occur in the postpartum period and should be managed with magnesium sulfate.", "link": "https://step2.medbullets.com/testview?qid=216512"} {"question": "A 57-year-old man is brought to the emergency department by his wife with confusion. He was seen in the emergency department 2 days ago for profuse epistaxis. His primary care provider was concerned and had advised him to go to the hospital, as the patient is on warfarin for atrial fibrillation. However, his bleeding resolved with pressure and nasal packing. Upon presentation to the ED today, his temperature is 97.6\u00b0F (36.4\u00b0C), blood pressure is 64/33 mmHg, pulse is 160/min, respirations are 32/min, and oxygen saturation is 100% on room air. Physical exam reveals an obtunded man who cannot answer questions. A dark, purpuric rash is noted on his torso and extremities. The patient\u2019s nasal packing is removed with no further bleeding. The patient is given IV fluids and blood cultures are drawn. Which of the following should be administered first?", "choicesA": "Clindamycin", "choicesB": "Meropenem", "choicesC": "Penicillin", "choicesD": "Piperacillin-tazobactam", "choicesE": "Vancomycin", "answer_idx": "A", "answer": "Clindamycin", "explanation": "This patient is presenting with nasal packing, hypotension, tachycardia, a dark, purpuric rash, and confusion, which are concerning for toxic shock syndrome. The most important initial step in management is to administer clindamycin.\n\nToxic shock syndrome is a life-threatening diagnosis caused by Streptococcus pyogenes or Staphylococcus aureus and occurs commonly secondary to retained tampons or nasal packing. Patients are critically ill and require immediate management with fluids, vasopressors, and blood cultures. Source control is imperative, and the foreign body causing the infection must be removed. Next, broad-spectrum antibiotics should be started immediately. Clindamycin is often preferred as the first agent as this lincosamide antibiotic not only covers Staphylococcus aureus and Streptococcus pyogenes, but also has the theoretical benefit of suppressing toxin production. Further management is centered on fluid administration, vasopressors, and monitoring and care in an ICU setting as these patients are typically critically ill.\n\nRussell and Pachorek review the treatments for toxic shock syndrome. They note that clindamycin is particularly important in toxic shock syndrome that is complicated by toxin production. They recommend that administration may reduce toxin production and could improve outcomes.\n\nIncorrect Answers:\nAnswer B: Meropenem is a broad-spectrum antibiotic that covers most organisms except for MRSA. It is generally not used as a first-line agent unless the patient has allergies to other antibiotics or the infectious organism is only sensitive to this agent. Even in toxic shock syndrome, it would not be used as a first-line agent.\n\nAnswer C: Penicillin may be used for infections such as streptococcal pharyngitis, which presents with a sore throat, fever, and tonsillar exudates. It is a relatively weaker antibiotic and would not be used in a critically ill patient with toxic shock syndrome.\n\nAnswers 4 & 5: Piperacillin-tazobactam and vancomycin represent a powerful broad-spectrum regimen of antibiotics that covers gram-positives, gram-negatives, Pseudomonas aeruginosa, and anaerobes (piperacillin-tazobactam) as well as further gram-positive coverage and MRSA (vancomycin). Though this is a potent antibiotic combination, neither of these takes priority over clindamycin given the possible benefit of toxin suppression from clindamycin. In this patient, clindamycin and piperacillin-tazobactam would be an appropriate antibiotic regimen.\n\nBullet Summary:\nClindamycin may suppress toxin production and should be given early in toxic shock syndrome.", "link": "https://step2.medbullets.com/testview?qid=216474"} {"question": "A 69-year-old woman presents with pain in her hip and groin. The pain is present in the morning but is nearly unbearable by the end of the day. Her medical history is notable for a treated episode of acute renal failure, diabetes mellitus, obesity, and hypertension. Her current medications include losartan, metformin, insulin, and ibuprofen. The patient recently started taking high doses of vitamin D as she believes that it could help her symptoms. She also recently fell off the treadmill while exercising at the gym. Her temperature is 98.6\u00b0F (37\u00b0C), blood pressure is 135/91 mmHg, pulse is 72/min, and respirations are 12/min. On exam, an obese woman in no distress is noted. There is pain, decreased range of motion, and crepitus on exam of her right hip. The patient points to the areas that cause her pain stating that it is mostly over the groin. Which of the following is most likely to be found on plain radiographic imaging of this patient?", "choicesA": "Femoral neck fracture", "choicesB": "Hyperdense foci in the ureters", "choicesC": "Loss of joint space and osteophytes", "choicesD": "Normal radiography", "choicesE": "Posterior displacement of the femoral head", "answer_idx": "C", "answer": "Loss of joint space and osteophytes", "explanation": "This patient is presenting with pain worse on exertion, decreased range of motion, and crepitus in her hip, suggesting a diagnosis of osteoarthritis. A loss of joint space and osteophytes (reactive bone formation) would be seen on radiography.\n\nOsteoarthritis occurs due to wear and tear of the joint typically from repeat trauma in the setting of risk factors such as obesity. Pain can be located in the hip, and can be referred to the groin or knee. The classic presentation is pain that worsens with activity. Crepitus and decreased range of motion can be found on physical exam. Common radiological findings include a loss of joint space and osteophytes. Treatment should start with conservative measures such as activity modification, weight loss, and an unloading brace. A corticosteroid injection may be appropriate as a second-line measure. Refractory cases can be treated with a total joint replacement.\n\nGlyn-Jones et al. review the evidence regarding the diagnosis and treatment of osteoarthritis. They discuss how total joint arthroplasty operations can be limited by the lifestyle of the prosthesis and other complications. They recommend considering joint preserving options such as lifestyle modification.\n\nIncorrect Answers:\nAnswer A: Femoral neck fracture would present acutely with pain in the hip and a shortened and externally rotated leg. This diagnosis is possible given the patient's recent fall; however, her other symptoms and less acute presentation suggest a diagnosis of osteoarthritis. Treatment is with a hemiarthroplasty.\n\nAnswer B: Hyperdense foci in the ureters suggest a diagnosis of renal calculi. Though this patient's referred pain to the groin, history of renal failure, and possible hypercalcemia (due to increased vitamin D intake) could suggest this diagnosis, her other symptoms are more suggestive of osteoarthritis. Treatment is lithotripsy or stone removal for larger kidney stones.\n\nAnswer D: Normal radiography would be found with a minor sprain/muscular injury which presents with a self-limited course of pain and tenderness over the affected muscle. Treatment is with rest, ice, compression, and elevation.\n\nAnswer E: Posterior displacement of the femoral head suggests a diagnosis of posterior hip dislocation. Posterior hip dislocation would present with a shortened and internally rotated leg and a more acute presentation. Treatment is with closed reduction of the hip under sedation.\n\nBullet Summary:\nOsteoarthritis presents with joint pain that is worse with exertion and radiographical findings of decreased joint space and osteophytes.", "link": "https://step2.medbullets.com/testview?qid=109238"} {"question": "A 7-year-old adopted boy presents to the emergency department after fainting. He just switched from being home-schooled to public school and joined the basketball team. He has fainted several times during basketball practices. He did not sustain any injuries or have any prodromes prior to these episodes. He is usually unconscious for less than a minute. He is otherwise healthy and has met all developmental milestones. He does not currently take any medications. His temperature is 97.6\u00b0F (36.4\u00b0C), blood pressure is 104/54 mmHg, pulse is 100/min, respirations are 19/min, and oxygen saturation is 98% on room air. Physical exam reveals a well-appearing and playful boy. An ECG is performed as seen in Figure A. An echocardiogram is ordered and pending. Which of the following is the most likely diagnosis?", "choicesA": "Arrhythmogenic right ventricular dysplasia", "choicesB": "Brugada syndrome", "choicesC": "Hypertrophic obstructive cardiomyopathy", "choicesD": "QT prolongation", "choicesE": "Wolff Parkinson White syndrome", "answer_idx": "A", "answer": "Arrhythmogenic right ventricular dysplasia", "explanation": "This adopted pediatric patient is presenting with recurrent syncope and an ECG demonstrating an epsilon wave and T wave inversions, which are concerning for arrhythmogenic right ventricular dysplasia.\n\nAn epsilon wave is a small positive deflection at the end of the QRS complex. This is an ominous finding and suggests a diagnosis of arrhythmogenic right ventricular dysplasia (ARVD) though it may also be seen in other infiltrative diseases such as sarcoidosis or after ischemic events. ARVD is an autosomal dominant condition where there is fatty infiltration of the right ventricular wall predisposing the patient (classically younger patients) to sudden cardiac death from lethal ventricular dysrhythmias. The epsilon wave is the most specific finding; however, other findings include T wave inversion in V1-V3 or a left bundle branch appearance with frequent premature ventricular complexes among other possibilities. This is a potentially lethal diagnosis and requires management with antiarrhythmics (such as beta-blockers or amiodarone) and the placement of an implantable cardioverter-defibrillator.\n\nKayser et al. review AVRD. They note the presentation and management of ARVD and how it may present initially only with syncope. Cardiac arrest is a possible presentation as well. They recommend early detection and treatment with an implantable cardioverter-defibrillator to prevent death in this population.\n\nFigure/Illustration A is an ECG with an epsilon wave (red arrow) and inverted T waves in V1/V2 (blue arrows) concerning for ARVD.\n\nIncorrect Answers:\nAnswer B: Brugada syndrome is an autosomal dominant mutation in cardiac ion channels that presents with a possible right bundle branch pattern with ST segment elevation in V1-V3 or ST coving in these same leads. It may present with syncope.\n\nAnswer C: Hypertrophic obstructive cardiomyopathy presents with syncope, sudden death, or cardiac arrest in a young patient associated with exertion or dehydration. It can cause obstruction of the left ventricular outflow tract. A systolic murmur can be heard on cardiac auscultation and echocardiography can confirm the diagnosis. ECG will demonstrate large voltages in V1-V5.\n\nAnswer D: QT prolongation can lead to torsades des pointes which presents with twisting of the QRS complexes around an isoelectric baseline. It may present with syncope, hemodynamic instability, or cardiac arrest. Medications, infection, and congenital conditions may prolong the QT interval.\n\nAnswer E: Wolff Parkinson White syndrome presents with a delta wave on ECG or a slurred upstroke of the QRS complex. It may cause syncope and evolve into other dysrhythmias including supraventricular tachycardia and ventricular tachycardia.\n\nBullet Summary:\nArrhythmogenic right ventricular dysplasia (ARVD) may cause syncope or cardiac arrest, and presents on ECG with an epsilon wave.", "link": "https://step2.medbullets.com/testview?qid=216634"} {"question": "A 48-year-old woman, gravida 3 para 3, presents to the clinic with a 5-month history of intermittent loss of urine. Her urinary leaking occurs throughout the day and night, and she often has to get up to use the bathroom while asleep. Exercising does not affect the loss of urine. She states that she often cannot make it to the bathroom in time once she senses a need to void. Her last menstrual period was 2 weeks ago. Her urinalysis is unremarkable, and her postvoid residual volume is 40 mL. On pelvic examination, there is no dribbling of urine when the patient coughs. Physical exam is otherwise unremarkable. Which of the following is the most likely diagnosis?", "choicesA": "Genitourinary syndrome of menopause", "choicesB": "Mixed urinary incontinence", "choicesC": "Overflow incontinence", "choicesD": "Stress incontinence", "choicesE": "Urge incontinence", "answer_idx": "E", "answer": "Urge incontinence", "explanation": "This patient with a sudden need to void followed by loss of urine with urgency (unable to make it to the bathroom in time) most likely has urinary urge incontinence.\n\nUrge incontinence is a form of urinary incontinence characterized by a sudden urge to urinate, resulting in the involuntary leakage of urine. Normally, bladder contraction occurs due to the stimulation of muscarinic receptors in the detrusor muscle. Urinary urge incontinence is due to detrusor overactivity, leading to a sudden and frequently overwhelming need to void that occurs throughout the day and night. The most common cause is idiopathic, though it may be secondary to neurologic disorders (e.g., spinal cord injury or multiple sclerosis), bladder abnormalities, or chronic bladder inflammation. Workup is first centered on ruling out a urinary tract infection with a urinalysis. The diagnosis is made with urodynamic studies, and first-line treatment involves bladder training and pelvic floor exercises. Should first-line treatment of urge incontinence fail, antimuscarinic medications (such as oxybutynin) which reduce acetylcholine activity on muscarinic receptors can decrease detrusor contraction and thus the sense of urgency.\n\nNandy and Ranganathan review urge incontinence. They note that it presents with leakage of urine associated with a sudden urge to urinate. They recommend an appropriate history and workup to first rule out other causes.\n\nIncorrect Answers\nAnswer A: Genitourinary syndrome of menopause represents urinary symptoms (stress and/or urge incontinence) due to vaginal dryness and atrophy from menopause. This patient still has menstrual cycles and no symptoms of ovarian follicle depletion (narrowed introitus, vulvovaginal dryness and irritation, irregular vaginal bleeding), making this an unlikely diagnosis.\n\nAnswer B: Mixed urinary incontinence presents with features of both stress and urge incontinence. This patient lacks symptoms of stress incontinence given a lack of incontinence with increases in intrabdominal pressure (such as exercise, coughing, laughing, or sneezing).\n\nAnswer C: Overflow incontinence is caused by impaired detrusor muscle activity or bladder outlet obstruction. It presents with constant involuntary dribbling or urine and incomplete bladder emptying. Post-void residual volume is increased in patients with overflow incontinence.\n\nAnswer D: Stress incontinence is caused by urethral hypermobility and otherwise insufficient urethral support. It presents with urinary leakage with activities that increase intraabdominal pressure (such as coughing, sneezing, laughing, and exercising). This patient\u2019s symptoms are unrelated to exercise.\n\nBullet Summary:\nUrge incontinence presents with sudden urinary urgency with a normal post-void residual volume and absence of findings consistent with urinary tract infection.", "link": "https://step2.medbullets.com/testview?qid=216248"} {"question": "A 51-year-old man presents for his annual wellness visit with his primary care physician. His wife has noticed a tremor while he is working on his car and knows that his father died of Parkinson disease. He says that the tremor is nothing to worry about and only occurs during a few activities such as pouring drinks, pointing the remote at the television, or fixing his car. The patient states his wife is just overreacting. His medical problems include hypertension, diabetes, and hyperlipidemia for which he takes lisinopril, metformin, and atorvastatin. He has 3-5 drinks of scotch per night before working on his car. On physical exam, the tremor is replicated during finger to nose testing. Romberg sign is negative. Cranial nerves II-XII are intact. The rest of the exam is benign. Which of the following is the most appropriate treatment for this patient's tremor?", "choicesA": "Alcoholics anonymous", "choicesB": "Aripiprazole", "choicesC": "Carbidopa", "choicesD": "Phenobarbital", "choicesE": "Propranolol", "answer_idx": "E", "answer": "Propranolol", "explanation": "This patient who presents with a tremor that only occurs during terminal activity and is likely improved by alcohol (hence why he drinks prior to working on the car) has benign essential tremor. This is best treated with propranolol.\n\nBenign essential tremor is a type of rhythmic oscillatory movement that occurs with posturing and action. The pathophysiology of this disorder is unclear; however, it classically gets worse with actions that require terminal positioning of the extremities such as pouring a drink or pointing a television remote. The tremor improves with alcohol consumption so patients will often self-medicate with alcoholic drinks. There is a strong genetic component, with an autosomal dominant pattern of inheritance with incomplete penetrance. The best method of replicating the tremor is with the finger-nose-finger test because this requires the patient to perform a precise activity at a terminal distance. The diagnosis is made clinically. Treatment is with propranolol or primidone, which are both considered first-line agents although the latter has less evidence for its efficacy.\n\nPaparella et al. studied the effects of propranolol on head and upper extremity movements in patients with benign essential tremors. They found that this medication has no effect on head movements but reduces the number of upper extremity tremors in these patients. They recommended that agents other than propranolol should be used for the treatment of head tremors in these patients.\n\nIncorrect Answers:\nAnswer A: Alcoholics anonymous is the first-line treatment for patients with alcohol use disorder; however, the tremors that occur during alcohol withdrawal are not isolated to movements during terminal activities. Alcohol withdrawal also presents with other signs including restlessness, diaphoresis, and insomnia. More severe cases can present with hallucinations and seizures. The response of this patient's tremor to alcohol consumption is typical of patients with benign essential tremor.\n\nAnswer B: Aripiprazole and other atypical antipsychotic medications can be used in patients with Tourette syndrome; however, these patients will present with involuntary tics rather than a tremor that only occurs during a voluntary activity at the terminal range of motion. Moreover, for the diagnosis of Tourette syndrome, both motor and phonic tics must be present.\n\nAnswer C: Carbidopa can be used in the treatment of Parkinson disease; however, the tremor associated with this disease classically occurs at rest and is described in character as a \"pill-rolling\" tremor. The other features of Parkinson disease include bradykinesia or akinesia, rigidity, and postural instability.\n\nAnswer D: Phenobarbital can be used in the treatment of Huntington disease; however, the involuntary movements seen in this disease are classically choreiform rather than intention tremor. Huntington disease presents with chorea, psychiatric illness, and depression. There is commonly a strong genetic predisposition, as it is a trinucleotide repeat disorder that exhibits anticipation.\n\nBullet Summary:\nPropranolol can be used in the treatment of patients with benign essential tremor, which presents with an action tremor that is alleviated by alcohol use.", "link": "https://bit.ly/3Q3WlAi"} {"question": "A 26-year-old woman presents to the clinic with diarrhea, bloating, flatulence, and abdominal cramps. These symptoms have been going on for 2 months. She has lost 6 pounds (2.7 kg) over these 2 months. Her medical history is significant for mild intermittent asthma for which she uses an albuterol inhaler as needed. She is sexually active with 1 male partner. She recently returned from South India following a mission trip for 6 weeks. Her temperature is 98.6\u00b0F (37.0\u00b0C), blood pressure is 105/70 mmHg, pulse is 95/min, and respirations are 14/min. On examination, the patient\u2019s skin is pale. Labs are obtained and show the following results:\n\nLeukocyte count: 4,500/mm^3\nHemoglobin: 10.5 g/dL\nPlatelets: 110,000/mm^3\nMean corpuscular volume (MCV): 116 \u00b5m^3\nReticulocyte count: 0.5%\n\nA biopsy of the small bowel reveals blunting of villi and a mixed infiltration of lymphocytes, plasma cells, and eosinophils. Which of the following is the most likely diagnosis?", "choicesA": "Celiac disease", "choicesB": "Crohn disease", "choicesC": "Giardia", "choicesD": "Tropical sprue", "choicesE": "Whipple disease", "answer_idx": "D", "answer": "Tropical sprue", "explanation": "The patient is presenting with chronic diarrhea, weight loss, megaloblastic anemia (low hemoglobin and elevated MCV), and a biopsy showing blunted villi and mixed infiltrate following significant travel to South India. This is consistent with tropical sprue.\n\nTropical sprue is characterized by chronic diarrhea that occurs after travel to an endemic area. It is thought to be due to a bacterial overgrowth/infection that then damages the intestinal mucosa. Endemic areas include the Caribbean, Southeast Asia, and South India. Patients present with chronic diarrhea, abdominal discomfort/cramps, progressive weight loss, and clinical features of malabsorption. Patients with tropical sprue can develop megaloblastic anemia (low hemoglobin and elevated MCV) from a deficiency in folate and vitamin B12 absorption. A small bowel biopsy would show blunting of the villi and infiltration of lymphocytes, plasma cells, and eosinophils. Microorganisms may also be seen. Treatment is an oral tetracycline and folic acid for 3-6 months.\n\nMcCarroll et al. study the relationship between infectious gastroenteritis and tropical sprue. In a case-control study, the authors found that prior infectious gastroenteritis increased the risk of tropical sprue. The authors recommend treatment of suspected tropical sprue with oral folic acid replacement.\n\nIncorrect Answers:\nAnswer A: Celiac disease can present similarly to tropical sprue with chronic diarrhea, abdominal pain, and weight loss. A small intestine biopsy can similarly show villous atrophy and a lymphocytic infiltrate, as well as crypt hyperplasia. Celiac disease is an immune response to gluten, while symptoms of tropical sprue are not diet-associated. The development of symptoms following significant travel (lasting at least several weeks) to an endemic country should raise suspicion for tropical sprue.\n\nAnswer B: Crohn disease presents with chronic diarrhea, abdominal pain, weight loss, skip lesions anywhere from mouth to anus, extraintestinal manifestations, and signs of malabsorption. A biopsy will show transmural inflammation with noncaseating granulomas, giant cells, and aggregates of lymphocytes in the lamina propria.\n\nAnswer C: Giardia is a parasite that is endemic in the tropics and North American mountain regions. Patients present with foul-smelling diarrhea, excessive gas, and nausea. Diagnosis can be confirmed with a stool analysis or immunoassay. Though a biopsy is normally not indicated, it would show mild lymphocytic infiltration and Giardia trophozoites on the intestinal surface.\n\nAnswer E: Whipple disease is caused by the bacteria Tropheryma whipplei. Patients present with diarrhea, abdominal pain, and signs of malabsorption. Extraintestinal symptoms such as arthritis are common. A biopsy of the small intestine would show Periodic acid-Schiff (PAS)-positive foamy macrophages in the lamina propria.\n\nBullet Summary:\nTropical sprue presents with diarrhea, abdominal cramping, weight loss, megaloblastic anemia due to malabsorption, and a small bowel biopsy showing villi blunting following travel to an endemic area such as South Asia or the Caribbean.", "link": "https://step2.medbullets.com/testview?qid=215056"} {"question": "A 33-year-old woman presents to the emergency department with muscle spasms. She has noticed \"twitching\" in both her upper and lower extremities over the past 3 days that have interfered with her training for a marathon. She has felt more fatigued over the past month. She has no significant medical history. She takes no medication other than whey protein supplements, a multivitamin, and fish oil. Her temperature is 98.5\u00b0F (36.9\u00b0C), blood pressure is 103/71 mmHg, pulse is 105/min, respirations are 11/min, and oxygen saturation is 100% on room air. Physical exam reveals a thin woman with thinning hair. She appears pale and her mucous membranes are dry. Her neurological and cardiac exams are unremarkable. Laboratory studies are ordered as seen below. Hemoglobin: 10 g/dL Hematocrit: 30% Leukocyte count: 6,100/mm^3 with normal differential Platelet count: 187,500/mm^3 Serum: Na+: 130 mEq/L Cl-: 100 mEq/L K+: 2.3 mEq/L HCO3-: 28 mEq/L BUN: 40 mg/dL Glucose: 79 mg/dL Creatinine: 0.9 mg/dL Ca2+: 8.2 mg/dL Mg2+: 1.8 mg/dL Thyroid stimulating hormone (TSH): 4.0 mIU/L Free T4: 0.4 ng/dL (normal: 0.7-1.53 ng/dL) The patient is given 4 liters of lactated ringer solution and 40 mEq of potassium. Her repeat whole blood potassium 3 hours later is 2.4 mEq/L, and her repeat BUN is 20 mEq/L. Another 40 mEq of potassium is administered. Another repeat whole blood potassium is 2.5 mEq/L. The patient\u2019s muscle spasms persist. Which of the following is the most likely underlying etiology for this patient\u2019s chief concern?", "choicesA": "Hypocalcemia", "choicesB": "Hypomagnesemia", "choicesC": "Hyponatremia", "choicesD": "Hypothyroidism", "choicesE": "Hypovolemia", "answer_idx": "B", "answer": "Hypomagnesemia", "explanation": "This patient is presenting with what appears to be anorexia nervosa (thin habitus, possible excessive exercise in the setting of marathon running, thinning hair, dry mucous membranes, decreased free T4 level, and multiple electrolyte derangements). The lack of improvement in either her muscle spasms with fluid repletion or her hypokalemia with adequate repletion, even in the setting of normal serum magnesium, suggests depleted magnesium stores are the underlying etiology of her symptoms.\n\nHypomagnesemia typically occurs in alcoholics, malnourished patients, and patients with eating disorders such as anorexia or bulimia nervosa. Patients may present with a wide array of symptoms including weakness, confusion, dysrhythmias, hypotension, and overall malaise. Hypomagnesemia can cause hypocalcemia as magnesium is needed for proper parathyroid hormone secretion. Another pathognomonic finding associated with hypomagnesemia is hypokalemia which is refractory to repletion. The pathophysiology is thought to be due to increased potassium wasting in the kidney due to increased activity of renal outer medullary potassium (ROMK) channels. The serum magnesium level should be checked and repletion should occur accordingly. Serum magnesium levels represent 1% or less of total body magnesium stores and do not reflect if a patient truly has hypomagnesemia, particularly if there is intracellular depletion (such as in chronically malnourished patients). Thus, any patient with findings consistent with hypomagnesemia with normal serum magnesium should have their other electrolytes repleted and be given magnesium. Magnesium repletion is a safe intervention with a wide therapeutic index, and the serum level often does not correlate with total body stores.\n\nRazzaque discusses the relationship between serum magnesium levels and total body magnesium status. The author finds that inadequate magnesium balance is associated with myriad disorders including skeletal deformities, cardiovascular diseases, and metabolic syndrome. The author recommends the use of the magnesium loading test to provide more accurate reflections of total body magnesium status.\n\nIncorrect Answers:\nAnswer A: Hypocalcemia may present with perioral numbness, paresthesias, and muscle spasms. It can occur secondary to poor nutrition, vitamin D deficiency, and hypoparathyroidism. Low serum calcium may also be seen in patients with low albumin and in states of hyperventilation (which lowers the free calcium but not the total calcium due to a respiratory alkalosis that causes albumin to bind free calcium and release protons). Hypomagnesemia can also cause hypocalcemia as magnesium is needed for proper parathyroid hormone release.\n\nAnswer C: Hyponatremia may present with malaise, nausea, vomiting, headache, and confusion. Patients may also be asymptomatic in mild cases. Asymptomatic patients with hyponatremia and functioning kidneys require only fluid restriction. On the other hand, severe hyponatremia presents with very low sodium (often < 120 mEq/L) and neurological symptoms such as profound confusion or seizures. Treatment with hypertonic saline would be indicated in this situation.\n\nAnswer D: Hypothyroidism presents with fatigue, malaise, weight gain, bradycardia, scarce lanugo, and depression. This patient\u2019s TSH is on the high end of normal and her T4 is lower than the cutoff value. However, this is likely a compensatory response to her low BMI and excessive exercise in order to slow down her metabolism to conserve energy.\n\nAnswer E: Hypovolemia was certainly present in this patient given her dry mucous membranes and prerenal pattern of labs (BUN:Cr ratio > 20:1). After receiving 4L of fluids, it is clear that dehydration is not the cause of her symptoms as her BUN corrects and her symptoms persist. In general, hypovolemia should be treated with oral rehydration, if possible.\n\nBullet Summary:\nHypomagnesemia can cause hypocalcemia and hypokalemia refractory to repletion (a normal serum level does not indicate normal intracellular levels of magnesium).", "link": "https://bit.ly/48uDHdl"} {"question": "A 70-year-old woman is brought to the emergency department by ambulance. She was found on the floor of her apartment after her neighbor called 911. She is confused and is unable to provide any history, but complains of generalized pain. Her temperature is 99.2\u00b0F (37.3\u00b0C), blood pressure is 129/64 mmHg, pulse is 63/min, respirations are 13/min, and oxygen saturation is 99% on room air. Physical exam reveals, a confused, ill-appearing woman. Lungs are clear to auscultation bilaterally. An electrocardiogram is obtained as shown in Figure A. Dipstick urinalysis is notable for 4+ blood and dark colored urine. Which of the following is the most appropriate next step in management?", "choicesA": "Albuterol and IV fluid resuscitation", "choicesB": "Calcium gluconate", "choicesC": "Insulin, glucose, and IV fluid resuscitation", "choicesD": "IV fluid resuscitation", "choicesE": "Sodium polystyrene sulfonate", "answer_idx": "B", "answer": "Calcium gluconate", "explanation": "This patient with confusion, dark urine, and peaked T waves on electrocardiogram in the setting of a prolonged time spent down likely has developed rhabdomyolysis with associated hyperkalemia. For patients with suspected hyperkalemia and electrocardiogram changes, the most appropriate next step in management is treatment with calcium gluconate\n\nRhabdomyolysis occurs when muscle cells lyse and release their intracellular contents. This can occur secondary to intense athletics, seizures, stimulant drug use, or a prolonged period of immobility. Lysis of muscle cells leads to release of potassium, myoglobin, purines, and other intracellular contents. Release of large amounts of myoglobin can result in renal impairment and myoglobinuria. On dipstick urinalysis, myoglobin causes a positive result for blood. Renal impairment further increases the risk for development of hyperkalemia. On ECG, hyperkalemia manifests with a \"peaked\" appearance of T waves, especially in the precordial leads. As hyperkalemia worsens, electrocardiogram changes progress to loss of P waves, QRS complex widening, and eventual sine wave rhythm. For patients with suspected hyperkalemia and characteristic changes noted on the electrocardiogram, the most appropriate immediate step in management is treatment with intravenous calcium gluconate. Calcium gluconate acts to stabilize cardiac myocyte membranes and prevent development of life-threatening arrhythmias.\n\nGupta et. al review rhabdomyolysis. They discuss the pathophysiology, clinical manifestations and diagnosis. They further detail complications that may arise such as hyperkalemia, and discuss respective management strategies.\n\nFigure A demonstrates an electrocardiogram with features characteristic of hyperkalemia. Note the peaked appearance of the T waves in the precordial leads.\n\nIncorrect Answers:\nAnswer A: Albuterol and IV fluid resuscitation may eventually be appropriate. Albuterol (a beta agonist) will act to shift potassium to the intracellular compartment. This patient's electrocardiogram changes warrant immediate treatment with calcium gluconate in order to stabilize cardiac myocyte membranes.\n\nAnswer C: Insulin, glucose, and IV fluid resuscitation are indicated in this patient. Insulin acts to shift potassium to the intracellular compartment, and fluid resuscitation is warranted for the patient's likely renal impairment. Calcium gluconate is the more immediate priority to prevent development of arrhythmia.\n\nAnswer D: IV fluid resuscitation is indicated for this patient with likely renal impairment. However, calcium gluconate is the more immediate priority to address this patient's hyperkalemia and prevent development of arrhythmia.\n\nAnswer E: Sodium polystyrene is an oral potassium binder that increases GI excretion of potassium. It may be appropriate to treat mild hyperkalemia in some patients. It takes a prolonged amount of time to have any significant effect.\n\nBullet Summary:\nFor patients with suspected hyperkalemia and characteristic changes noted on ECG, the most appropriate immediate step in management is treatment with intravenous calcium gluconate.", "link": "https://bit.ly/3R0iH7u"} {"question": "A 3-week-old girl presents with her parents to her pediatrician for a routine visit. The child is sleeping at regular 2-hour intervals and feeding and stooling well. The parents have noticed a \u201cswelling of the belly button.\u201d Cord separation occurred at 7 days of age. The swelling seems to come and go but is never larger than the size of a blueberry. They deny any drainage from the swelling. Her temperature is 98.6\u00b0F (37\u00b0C), blood pressure is 68/43 mmHg, pulse is 128/min, and respirations are 32/min. On physical exam, the child is in no acute distress and appears developmentally appropriate for her age. Her abdomen is soft and non-tender with a soft, 1 cm bulge at the umbilicus. The bulge increases in size when the child cries and can be easily reduced inside the umbilical ring without apparent pain. Which of the following is the most appropriate next step in management?", "choicesA": "Elective surgical management", "choicesB": "Expectant management", "choicesC": "Histopathologic evaluation", "choicesD": "Immediate surgical management", "choicesE": "Thyroid hormone replacement", "answer_idx": "B", "answer": "Expectant management", "explanation": "This infant presents with a soft, 1 cm bulge at the umbilicus that is easily reducible inside the umbilical ring, which suggests a diagnosis of an umbilical hernia. Most umbilical hernias in children resolve spontaneously and can be managed expectantly.\n\nAn umbilical hernia is a protrusion of bowel, omentum, or preperitoneal fat through the umbilical ring that can change in size with increases in intra-abdominal pressure (e.g., crying). The majority of umbilical hernias in patients under 5 years of age resolve spontaneously. The indications for surgery on umbilical hernias in that age group are limited to incarcerated hernias or hernias > 1.5 cm in diameter. Incarcerated hernias present with an irreducible mass that is associated with pain and erythema. Surgical closure may be considered in patients whose defects fail to close by the age of 5.\n\nChirdan et al. review the evidence regarding the diagnosis and treatment of pediatric patients with an incarcerated umbilical hernia. They discuss how most patients with hernias resolve spontaneously before 5 years of age but a small number will have incarcerated hernias that require surgery earlier. They recommend active observation of patients to diagnose incarceration earlier.\n\nIncorrect Answers:\nAnswer A: Elective surgical management later on in childhood is usually not required to manage umbilical hernias as most umbilical hernias in patients less than 5 years of age close spontaneously. Hernias that persist past age 5 can be closed surgically in an elective fashion.\n\nAnswer C: Histopathologic evaluation would be the most appropriate next step in management if the patient had an unknown umbilical mass, such as an umbilical granuloma or polyp. Umbilical granulomas form shortly after birth from excess tissue at the base of the umbilicus and can be treated topically, whereas umbilical polyps are firm masses of omphalomesenteric duct or urachal embryologic remnants and require surgical excision.\n\nAnswer D: Immediate surgical management is required in patients with incarcerated hernias, but this patient\u2019s hernia is easily reducible and < 1.5 cm. Incarcerated hernias present with an irreducible mass that is associated with pain and erythema.\n\nAnswer E: Thyroid hormone replacement would be indicated in newborns that suffer from hypothyroidism which could present with an umbilical hernia. This patient has no other symptoms of hypothyroidism such as dry skin, hypoactive reflexes, constipation, poor muscle tone, and coarse facial features.\n\nBullet Summary:\nUmbilical hernias are common in children and usually resolve spontaneously in children under 5 years of age.", "link": "https://bit.ly/468jb0I"} {"question": "A 72-year-old man presents to his primary care physician for a general checkup. The patient works as a farmer and has no concerns about his health. He has a medical history of hypertension and obesity. His current medications include lisinopril and metoprolol. His temperature is 99.5\u00b0F (37.5\u00b0C), blood pressure is 177/108 mmHg, pulse is 90/min, respirations are 17/min, and oxygen saturation is 98% on room air. Physical exam is notable for a murmur after S2 over the left sternal border. The patient demonstrates a stable gait and 5/5 strength in his upper and lower extremities. Which of the following is another possible finding in this patient?", "choicesA": "Audible click heard at the cardiac apex", "choicesB": "Femoral artery murmur", "choicesC": "Murmur that radiates to the carotids during systole", "choicesD": "Rumbling heard at the cardiac apex", "choicesE": "Wedge pressure lower than expected", "answer_idx": "B", "answer": "Femoral artery murmur", "explanation": "This patient is presenting with aortic regurgitation which could be associated with Duroziez sign (femoral artery murmur).\n\nAortic regurgitation occurs when the aortic valve closes improperly resulting in retrograde flow during diastole. This is auscultated as a diastolic murmur heard best over the left upper sternal border and the left lower sternal border. This murmur typically occurs in elderly patients and the retrograde flow through the aorta cause a number of physical exam findings. These include a wide pulse pressure, fingernail pulsations, head bobbing, and femoral artery murmur. Treatment is with surgical valve replacement.\n\nSapira reviews the evidence regarding the physical exam findings in aortic regurgitation. He discusses how Duroziez sign can often be seen in patients with this disease. He recommends using these signs to properly identify patients.\n\nIncorrect Answers:\nAnswer A: An audible click heard at the cardiac apex describes mitral valve prolapse which typically is asymptomatic and presents in younger women. Treatment is with surgical valve replacement in symptomatic patients.\n\nAnswer C: A murmur that radiates to the carotids during systole describes aortic stenosis which is possible in this patient except for the fact that his murmur is diastolic. Treatment is with valvuloplasty or with surgical valve replacement.\n\nAnswer D: A rumbling heard at the cardiac apex describes mitral valve stenosis which presents with a diastolic murmur best heard at the cardiac apex. Treatment is with valvuloplasty or with surgical valve replacement.\n\nAnswer E: A wedge pressure lower than expected would be seen in tricuspid regurgitation. In tricuspid stenosis, less blood flows to the lungs and thus the left heart, resulting in a low wedge pressure. Treatment is with identification of the underlying cause and valve replacement.\n\nBullet Summary:\nAortic regurgitation is associated with a wide pulse pressure, fingernail pulsations, head bobbing, and a femoral artery murmur.", "link": "https://bit.ly/47rczKM"} {"question": "A 69-year-old man presents to clinic due to shortness of breath, worsening pain in his right shoulder, and episodes of hemoptysis. His symptoms began 3 months ago. He has also lost 18 pounds. He has a history of coronary artery disease and underwent an uncomplicated coronary angioplasty with stent placement 2 years ago. He routinely travels both domestically and internationally as a senior shipyard worker and has a 30-pack-year smoking history. He used to drink 4 cocktails a week. He has not smoke or drank alcohol in over 7 years. His temperature is 99.5\u00b0F (37.7\u00b0C), blood pressure is 140/60 mmHg, pulse is 97/min, and respirations are 13/min. Physical exam is notable for right pupillary constriction as well as paresthesias in his right fourth and fifth digits. There is no pain with active shoulder rotation. His chest imaging is shown in Figure A. Which of the following is most likely responsible for the patient\u2019s symptoms?", "choicesA": "Lung adenocarcinoma", "choicesB": "Mesothelioma", "choicesC": "Rotator cuff tendinopathy", "choicesD": "Syringomyelia", "choicesE": "Tuberculosis", "answer_idx": "A", "answer": "Lung adenocarcinoma", "explanation": "This elderly patient with an extensive smoking history is presenting with hemoptysis, weight loss, and shortness of breath. Combined with neurological findings in the ulnar nerve distribution (paresthesias in fourth and fifth digits) and imaging evidence of a superior right lung mass, the most likely diagnosis is a Pancoast tumor, which is most commonly a lung adenocarcinoma.\n\nA Pancoast tumor is a peripheral lung cancer often caused by lung adenocarcinoma or squamous cell carcinoma and is often located in the superior sulcus of the lung. The most important risk factor is smoking. This tumor causes compression of adjacent structures. Patients can present with severe, localized axilla pain (due to invasion of the brachial plexus), weakness of intrinsic hand muscles with radicular pain and paresthesias in the fourth and fifth digits (due to invasion of the C8-T1 portion of the brachial plexus), Horner syndrome (miosis, ptosis, and facial anhidrosis due to invasion of the cervical sympathetic nerves), hoarseness (due to invasion of the recurrent laryngeal nerve), and edema of the arm with facial swelling (due to compression of the superior vena cava). Treatment is varied and involves a combination of radiation, chemotherapy, and surgery.\n\nWu et al. studied the role of the tyrosine-kinase inhibitor, osimertinib, in the treatment of completed resected epidermal growth factor receptor (EGFR)-mutated non-small cell lung cancer (NCSLC). The authors find that 89% of patients in the osimertinib group and 52% of the placebo groups were alive and disease free at 24 months. The authors recommend the use of osimertinib in patients with stage IB to IIIA EGFR-mutated NSCLC.\n\nFigure/Illustration A demonstrates a right apical lung mass found on computed tomography (CT) scan (green arrow).\n\nIncorrect Answers:\nAnswer B: Mesothelioma is a neoplasm of the pleura found in people with occupational exposures to asbestos, such as shipyard workers. While this neoplasm can present with hemoptysis and dyspnea, chest imaging of mesothelioma reveals pleural plaques and thickening, not an isolated apical lung lesion.\n\nAnswer C: Rotator cuff tendinopathy presents with pain with abduction and external rotation of the arm. This patient has no pain with active shoulder rotation.\n\nAnswer D: Syringomyelia is a cyst or cavity in the spinal column that can cause Horner syndrome and fluctuating radicular pain. It is unlikely to cause hemoptysis, weight loss, and the chest imaging findings in this patient.\n\nAnswer E: Tuberculosis is a possible diagnosis in this patient with hemoptysis, weight loss, and extensive travel history. Tuberculosis often presents with a fever and does not usually cause shoulder pain.\n\nBullet Summary:\nPancoast tumor presents with shoulder pain, cervical radiculopathy, Horner syndrome, and hoarseness and should be suspected in a patient with a smoking history and an apical lung mass on imaging.", "link": "https://bit.ly/3OFDpHw"} {"question": "A 36-year-old woman, G1P1001, presents to her gynecologist for an annual visit. She has a medical history of hypertension for which she takes hydrochlorothiazide. The patient\u2019s mother had breast cancer at age 68, and her sister has endometriosis. She states that she feels well and has no complaints. Her temperature is 98.6\u00b0F (37.0\u00b0C), blood pressure is 138/74 mmHg, pulse is 80/min, and respirations are 13/min. Her BMI is 32.4 kg/m^2. A pelvic exam reveals a non-tender, 12-week-size uterus with an irregular contour. A transvaginal ultrasound is performed and demonstrates two intramural leiomyomas. Which of the following is a likely complication of this patient\u2019s condition?", "choicesA": "Amenorrhea", "choicesB": "Endometrial cancer", "choicesC": "Iron deficiency anemia", "choicesD": "Uterine prolapse", "choicesE": "Uterine sarcoma", "answer_idx": "C", "answer": "Iron deficiency anemia", "explanation": "This patient presents with an irregularly enlarged uterus on examination consistent with the intramural leiomyomas (fibroids) seen on ultrasound. Leiomyomas increase the risk of iron deficiency anemia due to heavy menstrual bleeding.\n\nLeiomyoma are benign overgrowths of myometrial tissue. The most common presentation is heavy or prolonged menstrual bleeding or symptoms related to mass effect such as pelvic pressure or pain.. Affected women often develop these symptoms in their 30s or 40s, though they may occur at any age, and many are asymptomatic. Risk factors include early menarche, obesity, family history, and African American race. Submucosal fibroids that disrupt the endometrium may cause heavy menstrual bleeding. Intramural fibroids also commonly cause heavy bleeding despite the fact that they do not disrupt the endometrium due to alterations in the uterine vasculature. Patients may be managed medically with a wide range of options including oral contraceptive pills, tranexamic acid, and GnRH agonists. Definitive management is with surgical intervention. Myomectomy is preferred for patients that desire child bearing in the future. For those that do not have plans for pregnancy in the future, hysterectomy is recommended.\n\nYang et. al review the diagnosis and management of uterine fibroids. They note the range of clinical presentations to include heavy menstrual bleeding. They discuss management to include surgical intervention with myomectomy or hysterectomy.\n\nIncorrect Answers:\nAnswer A: Amenorrhea, or the absence of menstrual bleeding, is often due to anovulation or uterine adhesions. Uterine fibroids are not associated with amenorrhea.\n\nAnswer B: Endometrial cancer is a known cause of heavy menstrual bleeding. However, this patient\u2019s ultrasound notes the presence of intramural leiomyomas. Her age, history, and ultrasound findings are not suggestive of endometrial cancer.\n\nAnswer D: Uterine prolapse is associated with multiparity, chronically increased intra-abdominal pressure (from chronic obstructive pulmonary disease, obesity, etc.), and connective tissue diseases. While fibroids rarely may prolapse, they typically do not cause the entire uterus to prolapse.\n\nAnswer E: Uterine sarcoma may very rarely result from uterine fibroids, as both arise from the uterine smooth muscle. However, this patient\u2019s history and ultrasound findings are more suggestive of uterine leiomyoma.\n\nBullet Summary:\nPatients with uterine leiomyomata (fibroids) are at risk for iron deficiency anemia due to heavy bleeding.", "link": "https://step2.medbullets.com/testview?qid=109024"} {"question": "A 77-year-old man presents to the emergency department acutely obtunded. The patient lives alone and was found unresponsive by his son. Generally, the patient manages his own finances, medications, and works part-time. He has not been responding to phone calls for the past 3 days. The patient is unable to offer a history. He has a past medical history of hypothyroidism, depression, and diabetes. His temperature is 88.0\u00b0F (31.1\u00b0C), blood pressure is 92/62 mmHg, pulse is 35/min, respirations are 9/min, and oxygen saturation is 92% on room air. The patient is cold to the touch and moves all extremities to painful stimuli. His pupils are reactive and sluggish, and he does not follow commands. There are no signs of trauma or skin infections. The patient is started on IV fluids and hydrocortisone, is externally warmed, and is started on a norepinephrine drip. An ECG is performed as seen in Figure A. Which of the following is the most appropriate next step in management?", "choicesA": "Free T4 level", "choicesB": "Levothyroxine administration", "choicesC": "Thyroid stimulating hormone and free T4 level", "choicesD": "Thyroid stimulating hormone level", "choicesE": "Triiodothyronine administration", "answer_idx": "B", "answer": "Levothyroxine administration", "explanation": "This elderly patient with a past medical history of hypothyroidism is presenting with altered mental status, hypotension, hypothermia, and bradycardia, which are concerning for myxedema coma. Empiric and immediate treatment with levothyroxine is the appropriate management of this condition prior to confirming the diagnosis with lab studies.\n\nMyxedema coma is a life-threatening complication of hypothyroidism. It presents with stupor/obtundation, bradycardia, hemodynamic instability, and hypothermia. With a high clinical suspicion, appropriate history, physical exam, and vitals, it is appropriate to first treat the patient with IV hydrocortisone (or another highly potent steroid) and levothyroxine prior to confirming the diagnosis given the high morbidity and mortality associated with this condition. Triiodothyronine is given in some circumstances as well (in more ill patients) but is less dire than first giving levothyroxine. Subsequently, the diagnosis can be confirmed with a serum thyroid stimulating hormone (TSH) and free T4 level. Patients are critically ill and typically require further care in an ICU.\n\nWall discusses the management of myxedema coma. He notes this is a potentially lethal condition. He recommends immediate administration of IV levothyroxine.\n\nFigure A is an ECG with sinus bradycardia.\n\nIncorrect Answers:\nAnswers 1, 3, & 4: Thyroid stimulating hormone and free T4 levels should certainly be obtained; however, the mortality for myxedema coma is so high that empiric treatment with levothyroxine should be started with clinical suspicion alone prior to confirming the diagnosis with laboratory studies, as a delay in therapy could increase morbidity and mortality. After administration of levothyroxine and initial supportive therapy, the diagnosis can be supported with laboratory studies.\n\nAnswer E: Triiodothyronine or T3 could be appropriate in patients with myxedema coma in severe scenarios where patients do not respond to levothyroxine. In some cases, it may also be given empirically but does not take priority over T4 administration.\n\nBullet Summary:\nIn myxedema coma, empiric treatment with levothyroxine is appropriate with a high clinical suspicion for the diagnosis prior to laboratory confirmation.", "link": "https://step2.medbullets.com/testview?qid=216587"} {"question": "A 39-year-old man presents to his doctor for a wellness checkup. He is concerned about a rash that does not seem to be improving. He was recently exposed to his grandfather who has vesicular lesions on his skin and is being treated. He has a family history of skin cancer, colon cancer, and ovarian cancer. The patient has a medical history of asthma and seasonal allergies. His temperature is 98.6\u00b0F (37.0\u00b0C), blood pressure is 137/98 mmHg, pulse is 90/min, respirations are 14/min, and oxygen saturation is 98% on room air. Physical exam reveals the finding in Figure A. Which of the following describes this patient's most likely diagnosis?", "choicesA": "Benign capillary proliferation", "choicesB": "Edema of the epidermis", "choicesC": "Healthy dermatologic development", "choicesD": "Malignant blood vessel proliferation", "choicesE": "Viral infection", "answer_idx": "A", "answer": "Benign capillary proliferation", "explanation": "This patient who presents with red raised papules is presenting with a cherry angioma. These lesions are composed of benign capillary proliferations.\n\nCherry angiomas present with cherry red macules/papules that occur secondary to benign capillary and venule proliferation. The lesions are entirely benign and are cosmetic. These lesions are common in middle-aged adults. As the patient ages, more lesions will appear. In the absence of symptoms, observation and reassurance can be used for conservative management. They can be removed surgically if the patient desires.\n\nNazer et al. study the risk factors associated with the development of cherry angioma. They found that the use of clopidogrel and tamsulosin is associated with these lesions. They recommend ruling out underlying conditions.\n\nFigure/Illustration A demonstrates cherry red macules/papules classically seen in patients with cherry angioma.\n\nIncorrect Answers:\nAnswer B: Edema of the epidermis describes eczema or atopic dermatitis. This disease would present with pruritic, erythematous, and vesicular lesions. Topical creams and hygiene can be used for symptomatic management of these lesions.\n\nAnswer C: Healthy dermatologic development misses the diagnosis of a cherry angioma, which although benign, is a more accurate diagnosis for this case.\n\nAnswer D: Malignant blood vessel proliferation does not describe a cherry angioma which is benign and purely cosmetic. An angiosarcoma usually occurs in deep tissues and requires surgical excision.\n\nAnswer E: Viral infection describes herpes simplex virus which would present with vesicular lesions and neurogenic pain. These patients can be treated with valacyclovir as well as gabapentin for neuropathic pain.\n\nBullet Summary:\nCherry angiomas are benign capillary proliferations that appear as cherry red macules/papules.", "link": "https://step2.medbullets.com/testview?qid=210076"} {"question": "A newborn boy is evaluated 30 minutes after birth. He was born at 39 weeks gestation to a 27-year-old primigravid mother via cesarean section for cervical insufficiency. The pregnancy was complicated by gestational diabetes and the amniotic fluid was clear. Upon delivery, the patient had strong respiratory effort and a strong cry. His Apgar scores at 1 and 5 minutes were 7 and 8, respectively. The patient now is exhibiting increased work of breathing and is progressively more tachypneic. His birth weight is 3,568 g (7 lb 14 oz). His temperature is 99.0\u00b0F (37.2\u00b0C), blood pressure is 60/44 mmHg, pulse is 146/min, and respirations are 72/min. On physical exam, the patient is grunting with nasal flaring and subcostal retractions. Breath sounds are decreased at the bases bilaterally. The patient has central cyanosis. His chest radiograph can be seen in Figure A. Which of the following is the most likely etiology of this patient\u2019s presentation?", "choicesA": "Meconium aspiration syndrome", "choicesB": "Neonatal respiratory distress syndrome", "choicesC": "Persistent pulmonary hypertension", "choicesD": "Transient tachypnea of the newborn", "choicesE": "Viral pneumonia", "answer_idx": "D", "answer": "Transient tachypnea of the newborn", "explanation": "This newborn is presenting with increased work of breathing and progressive tachypnea in the 1st hour of life, which is consistent with a diagnosis of transient tachypnea of the newborn.\n\nTransient tachypnea of the newborn presents with respiratory distress and marked tachypnea within 2 hours of delivery. Transient tachypnea of the newborn is caused by delayed resorption and clearance of alveolar fluid, which results in mild pulmonary edema. Chest radiography typically demonstrates bilateral perihilar streaking and hyperinflation of the lungs. Transient tachypnea of the newborn is benign and usually resolves by day 2 of life. The management of this condition is supportive.\n\nAlhassen et al. review the evidence regarding the diagnosis and treatment of TTN. They discuss how this condition sometimes requires extended hospitalization and is associated with wheezing in childhood. They recommend using ultrasound to help make this diagnosis.\n\nFigure/Illustration A demonstrates prominent vascular markings around the hilum (yellow arrows), and increased lung volumes on chest radiograph, which is consistent with transient tachypnea of the newborn.\n\nIncorrect Answers:\nAnswer A: Meconium aspiration syndrome can cause respiratory distress in the immediate newborn period, but it would be unlikely in a patient with a history of clear amniotic fluid, and a more mild clinical picture. Meconium aspiration syndrome would be seen on chest radiograph as coarse, irregular infiltrates and hyper expansion. Treatment is with antibiotics and airway clearance.\n\nAnswer B: Neonatal respiratory distress syndrome is a disease of prematurity and does not often occur in infants born at term to healthy mothers. The chest radiograph in neonatal respiratory distress syndrome is also more likely to demonstrate low lung volumes. Treatment is with surfactant administration.\n\nAnswer C: Persistent pulmonary hypertension is a cause of respiratory distress in the newborn period, but a chest radiograph typically shows clear lungs. Treatment is with respiratory support as well as inhaled nitric oxide.\n\nAnswer E: Viral pneumonia is uncommon in the immediate newborn period. Bacterial pneumonia is more common but usually presents with signs of infection in the mother, which are not present in this case. Treatment is with broad-spectrum antibiotics.\n\nBullet Summary:\nTransient tachypnea of the newborn presents with worsening tachypnea within 2 hours of delivery and is caused by pulmonary edema resulting from inadequate clearance of alveolar fluid.", "link": "https://step2.medbullets.com/testview?qid=109292"} {"question": "A 55-year-old man presents to the emergency department after fainting while moving furniture. He states that he has been working in a hot, humid attic all day and forgot his water bottle. He felt dizzy when standing up and then fainted and was unconscious for roughly 1 minute. He awoke at his neurologic baseline afterwards. The patient has a history of hypertension being managed by his primary care doctor. His temperature is 98.7\u00b0F (37.1\u00b0C), blood pressure is 149/82 mmHg, pulse is 86/min, respirations are 15/min, and oxygen saturation is 99% on room air. Physical exam reveals a diaphoretic man. He demonstrates an unremarkable cardiopulmonary exam. The patient walks with a steady gait and his neurologic exam is non-focal. An ECG is performed as seen in Figure A. The patient is given 2 liters of oral fluids and feels back to his baseline. Which of the following conditions is most likely to occur in this patient based on his current history and ECG?", "choicesA": "Atrial fibrillation", "choicesB": "Complete heart block", "choicesC": "Myocardial infarction", "choicesD": "Stroke", "choicesE": "Torsades des pointes", "answer_idx": "B", "answer": "Complete heart block", "explanation": "This patient is presenting with dizziness and syncope in the setting of likely dehydration (working in a hot/humid environment without water) and feels better after rehydration, suggesting a diagnosis of orthostatic hypotension. An ECG is performed and incidentally demonstrates a bifascicular block, which is at high risk of progressing to a complete heart block.\n\nBifascular block is a conduction abnormality in 2 of the 3 fascicles that conduct electricity from atria to ventricles. This leaves conduction to the ventricles via the single remaining fascicle. There are different possibilities that may present on ECG. One possibility is a right bundle branch block with a left anterior fascicular block; the other is a right bundle branch block with a left posterior fascicular block. Note that the pathophysiology of complete heart block is an inability to conduct electricity from the atria to the ventricles. If there is only 1 fascicle left to conduct electricity, the patient would be in complete heart block if this fascicle were to no longer work, which is a common outcome in patients with bifascicular block. Treatment of complete heart block involves the placement of a pacemaker.\n\nWiberg et al. discuss bifascicular block. They note the possible outcome of a complete heart block. It is recommended to look for this finding on ECG and appropriately workup patients to prevent progression of disease.\n\nFigure/Illustration A is an ECG demonstrating bifascicular block. Note the left axis deviation as suggested by the net upward deflection of the QRS in lead I and net downward deflection in lead II (blue arrows). Then, note the appearance of a right bundle branch block with the RSR pattern in aVR (black arrow) and upward R\u2019 in lead V1 (green arrow). This is a classic pattern for bifascicular block. Contrast this to the other pattern of bifascicular block which would present with a right bundle branch block and right axis deviation (upward QRS in lead II and downward in lead I), which was not seen in this case.\n\nIncorrect Answers:\nAnswer A: Atrial fibrillation may occur secondary to aging, hypertension, or ischemia and presents with an irregularly irregular rate and rhythm with an absence of P waves. It may cause complications such as stroke. Management typically involves rate control agents such as beta-blockers or calcium channel blockers.\n\nAnswer C: Myocardial infarction is a likely outcome in individuals with risk factors such as obesity, hypertension, smoking, and dyslipidemia, in particular, without proper preventive measures and medical therapy including statins, beta blockers, and ACE inhibitors. This patient may be at risk of a myocardial infarction with his hypertension, and his underlying bifascicular block could suggest undiagnosed cardiac ischemia; however, it is more likely this patient progresses to complete heart block in the setting of his bifascicular block.\n\nAnswer D: Stroke is a possible complication of atrial fibrillation. For this reason, most patients with atrial fibrillation are on blood thinners to prevent this complication. It occurs secondary to stasis of blood in the fibrillating left atrium. Patients are often risk stratified prior to starting blood thinners with tools such as CHADS2VASC and HASBLED. Isolated syncope is a rare manifestation of stroke.\n\nAnswer E: Torsades des pointes is a progression from QT prolongation which may occur secondary to genetic conditions, medications, and/or infection. It presents on ECG with QRS complexes that twist in amplitude around the electrical baseline. Treatment involves the administration of magnesium sulfate and, if needed, electrical cardioversion.\n\nBullet Summary:\nA bifascicular block can progress to complete heart block.", "link": "https://step2.medbullets.com/testview?qid=216637"} {"question": "A 68-year-old man presents to the emergency department with right ear pain that he developed 4 days prior, and it has progressively worsened. The pain is worse at night. He has associated fever and chills. This morning, he awoke with purulent, foul-smelling discharge on his pillow. He has a history of type II diabetes mellitus and hypertension. He takes metformin, insulin, and lisinopril. His temperature is 102.2\u00b0F (39.0\u00b0C), blood pressure is 130/87 mmHg, pulse is 110/min, and respirations are 16/min. The patient is toxic in appearance and in mild distress due to pain. Examination of the external ear reveals findings shown in Figure A. Granulation tissue and purulent drainage are noted in the external auditory canal. Physical exam is otherwise unremarkable. Which of the following is the most appropriate therapy for this patient?", "choicesA": "Acyclovir", "choicesB": "Amoxicillin", "choicesC": "Ceftriaxone", "choicesD": "Ofloxacin ear drops", "choicesE": "Vancomycin and piperacillin-tazobactam", "answer_idx": "E", "answer": "Vancomycin and piperacillin-tazobactam", "explanation": "The patient in this vignette most likely has malignant otitis externa (also referred to as necrotizing otitis externa), which is a life-threatening infection requiring therapy with intravenous anti-pseudomonal and broad-spectrum antibiotics. Piperacillin-tazobactam and vancomycin is an appropriate initial regimen.\n\nMalignant otitis externa is an invasive infection of the external auditory canal and skull base, and is seen almost exclusively in immunocompromised patients. Patients with poorly controlled type II diabetes mellitus are at particularly increased risk. The infection spreads from the floor of the ear canal to the nearby tissues including the bones at the base of the skull and the temporomandibular joint. The resulting skull base osteomyelitis distinguishes the \"necrotizing\" or \"malignant\" presentation of otitis externa. Patients often present with severe otalgia and purulent drainage from the external auditory canal. Granulation tissue in the external auditory canal is a characteristic exam finding. Pseudomonas aeruginosa is the causative organism in most cases of necrotizing otitis externa. Therefore, intravenous antibiotics with sufficient coverage of Pseudomonas are the therapy of choice. As infection progresses, involvement of cranial nerves with resultant palsy may be observed.\n\nAaraj et. al review the clinical manifestations, diagnosis and management of malignant otitis externa. They note the risk for this disease among patients with diabetes or other etiologies of an immunocompromised state. They recommend management with intravenous ciprofloxacin.\n\nFigure A shows the external ear of a patient with malignant otitis externa. Note the classic granulation tissue at the cartilage-bone junction and crusted, purulent discharge.\n\nIncorrect Answers:\nAnswer A: Intravenous acyclovir would be appropriate for suspected herpes zoster, which may present with ear pain and a vesicular rash in the external auditory canal. This patient's granulation tissue in the external auditory canal and purulent drainage make malignant otitis externa more likely.\n\nAnswer B: Oral amoxicillin may be appropriate for management of acute otitis media which presents with ear pain and a distended tympanic membrane that is immobile.\n\nAnswer C: Ceftriaxone is appropriate coverage for cellulitis which presents with warmth, erythema, and tenderness of the skin. It would be too narrow of spectrum to treat malignant otitis externa.\n\nAnswer E: Topical ofloxacin may be appropriate for some cases of simple otitis externa (\"swimmer's ear\") which presents with ear pain and drainage in young, healthy patients who swim regularly. Patients are well-appearing.\n\nBullet Summary:\nMalignant otitis externa is an invasive infection of the external auditory canal and skull base that requires management with broad-spectrum intravenous antibiotics that cover Pseudomonas aeruginosa.", "link": "https://bit.ly/3QjJGL8"} {"question": "A 54-year-old man is brought to the emergency department following a skiing accident. The patient lost control of his skis and collided with a tree. He lost consciousness for about 30 seconds. When he woke up, he was mildly confused but able to ski down the rest of the mountain. The patient initially denied medical treatment but his wife convinced him to be taken by ambulance to the emergency department for evaluation. The paramedics report that at the scene the patient had a Glasgow Coma Score (GCS) of 15. During the ride, he became gradually more somnolent. His medical history is significant for hypertension, hyperlipidemia, coronary artery disease, and a prior transient ischemic attack. His medications include atorvastatin and lisinopril. In the emergency department, his temperature is 97.8\u00b0F (36.6\u00b0C), blood pressure is 141/84 mmHg, pulse is 71/min, and respirations are 16/min. He is difficult to arouse with a GCS of 7. A head CT is performed and can be seen in Figure A. Which of the following is the most likely cause of this patient\u2019s presentation?", "choicesA": "Damage to the cerebral bridging veins", "choicesB": "Damage to the sphenoid bone", "choicesC": "Occlusion of cerebral small penetrating arteries", "choicesD": "Ruptured cerebral aneurysm", "choicesE": "Secondary hemorrhage into prior area of infarction", "answer_idx": "B", "answer": "Damage to the sphenoid bone", "explanation": "This patient presents with a lucid interval followed by worsening somnolence and a biconvex lesion on head CT, which suggests a diagnosis of acute epidural hematoma. Acute epidural hematomas are most commonly caused by damage to the middle meningeal artery from trauma to the sphenoid bone.\n\nPatients with an epidural hematoma typically have an initial loss of consciousness followed by a lucid interval, during which their mental status returns to baseline or near baseline. As the hematoma expands, patients display increasing somnolence from the increase in intracranial pressure. For that reason, epidural hematomas are sometimes referred to as the \u201ctalk and die\u201d syndrome. If not relieved, this increase in intracranial pressure can lead to uncal herniation or death. Patients with neurologic findings or those with expanding bleeds require urgent surgical decompression.\n\nDadashi et al. present cases of patients with epidural hematoma from acute trauma. They discuss how patients often sustain these injuries from extreme sports. They recommend being aware of this syndrome so that acute interventions can be taken.\n\nFigure/Illustration A demonstrates a biconvex hyperdensity that is limited by suture lines (red circle). This is the typical appearance of an epidural hematoma on head CT.\n\nIncorrect Answers:\nAnswer A: Damage to the cerebral bridging veins results in a subdural hematoma (SDH). An acute SDH more commonly presents with a gradual increase in headache and confusion rather than the \u201ctalk and die\u201d syndrome presented in this vignette. However, a minority of patients with acute SDH may have a lucid interval followed by progressive neurological decline. SDH would have a crescent-shaped appearance on head CT rather than the biconvex appearance of an epidural hematoma.\n\nAnswer C: Occlusion of cerebral small penetrating arteries describes a lacunar stroke. Although this patient has 2 risk factors for lacunar stroke (hypertension and smoking), this patient\u2019s head CT demonstrates an epidural hematoma.\n\nAnswer D: A ruptured cerebral aneurysm would most likely lead to a subarachnoid hemorrhage. A subarachnoid hemorrhage presents as a sudden, severe headache (\u201cthunderclap headache\u201d), confusion or somnolence, nausea and vomiting, and meningismus. Surgical clipping or embolization of the bleeding vessel may be required.\n\nAnswer E: Secondary hemorrhage into a prior area of infarction usually occurs in the setting of a recent embolic stroke. Although this patient has a history of TIA and other risk factors for stroke, his head CT demonstrates an epidural hematoma. Treatment is supportive with the control of underlying risk factors such as hypertension.\n\nBullet Summary:\nEpidural hematomas are caused by damage to the middle meningeal artery as it passes through the foramen spinosum of the sphenoid bone.", "link": "https://bit.ly/3LeIdlx"} {"question": "A 2-day-old boy has an episode of vomiting in the hospital nursery. The vomitus was described as \u201cbright green\u201d without any traces of blood. The patient has urinated several times since he was born but has not passed any stool. He was born at 37 weeks of gestation to a 38-year-old G3P3 woman. The pregnancy was uncomplicated and the patient\u2019s mother refused all prenatal testing. The patient\u2019s 2 older siblings are both healthy. His temperature is 98.6\u00b0F (37\u00b0C), blood pressure is 67/43 mmHg, pulse is 135/min, and respirations are 34/min. On physical exam, the patient has upslanting palpebral fissures, epicanthal folds, and a single transverse palmar crease. His abdomen is non-tender, firm, and distended. Bowel sounds are hypoactive. Digital rectal exam evacuates a small amount of stool and flatulence. A nasogastric tube is placed to decompress the stomach. The patient\u2019s abdominal radiograph can be seen in Figure A. Which of the following is the most likely diagnosis?", "choicesA": "Duodenal atresia", "choicesB": "Hirschsprung disease", "choicesC": "Intestinal malrotation", "choicesD": "Meconium ileus", "choicesE": "Pyloric stenosis", "answer_idx": "B", "answer": "Hirschsprung disease", "explanation": "This pediatric patient presents with features of Down syndrome (trisomy 21), bilious vomiting, and colonic distention on an abdominal radiograph, which suggests a diagnosis of Hirschsprung disease.\n\nHirschsprung disease is associated with Down syndrome and presents with bilious vomiting, failure to pass meconium in the first 48 hours of life, and abdominal distension. A digital rectal exam may temporarily relieve the obstruction and lead to the expulsion of gas and stool (the \u201csquirt sign\u201d). Abdominal radiography typically demonstrates signs of distal intestinal obstruction such as proximal dilation and an absence of air in the rectum. Contrast enema may reveal a transition zone between the dilated proximal megacolon and the narrow rectosigmoid colon. A biopsy can confirm the diagnosis. Treatment is with surgical removal of the defective segment of the bowel.\n\nKessman reviews the evidence regarding the diagnosis and treatment of Hirschsprung disease. She discusses how the diagnosis can be made using a rectal suction biopsy. She recommends monitoring patients closely for enterocolitis for years after surgical treatment.\n\nFigure/Illustration A is an abdominal radiograph that demonstrates colonic distension (red circle). This finding is consistent with a diagnosis of Hirschsprung disease.\n\nIncorrect Answers:\nAnswer A: Duodenal atresia is associated with Down syndrome (trisomy 21) and may also present in the first 48 hours of life with bilious vomiting. The abdominal radiograph will demonstrate the \u201cdouble bubble\u201d sign, which reflects air trapping in the stomach and first portion of the duodenum. Treatment is with the surgical opening of the atretic segment.\n\nAnswer C: Intestinal malrotation may preset with bilious vomiting in a neonate due to the development of a midgut volvulus. An abdominal radiograph may show the extension of the nasogastric tube into an abnormally positioned duodenum and would not show diffuse colonic distension. Treatment is with surgical derotation of the volvulus.\n\nAnswer D: Meconium ileus is associated with cystic fibrosis and also presents with delayed (> 48 hours) passing of meconium. The distal obstruction in these patients will not be temporarily relieved by a digital rectal exam. Contrast enema will demonstrate microcolon involving the entire large bowel. Treatment is with irrigation and removal of the meconium.\n\nAnswer E: Pyloric stenosis usually presents in the first 3-6 weeks of life with post-prandial, nonbilious, projectile vomiting and a palpable olive-shaped mass in the epigastrium. This patient is presenting at just 2 days of life and has bilious vomiting. Treatment is with surgical release of the stenosis.\n\nBullet Summary:\nHirschsprung disease is associated with Down syndrome (trisomy 21) and presents with retention of stool that can be relieved by digital disimpaction, bilious vomiting, and colonic distention on abdominal radiography.", "link": "https://bit.ly/43wFOdc"} {"question": "A 15-year-old girl is brought to the clinic because her mother is worried the patient has not yet had her period. The patient\u2019s older sister had her first period at age 14. The mother had her first period at age 13. The patient reports she is doing well in school and is on the varsity basketball team. Her medical history is significant for asthma and atopic dermatitis. Her medications include albuterol and topical triamcinolone. Her temperature is 98\u00b0F (36.7\u00b0C), blood pressure is 111/72 mmHg, pulse is 65/min, and respirations are 14/min with an oxygen saturation of 99% on room air. Her body mass index (BMI) is 19 kg/m^2. Physical exam shows absent breast development and external genitalia at Tanner stage 1. Serum follicle-stimulating hormone (FSH) level is measured to be 38 mIU/mL. Which of the following is the most appropriate next diagnostic step?", "choicesA": "CYP17 gene work-up", "choicesB": "Estrogen levels", "choicesC": "Gonadotrophin-releasing hormone stimulation test", "choicesD": "Karotype", "choicesE": "Luteinizing hormone levels", "answer_idx": "D", "answer": "Karotype", "explanation": "The patient is presenting with primary amenorrhea and an increased follicle-stimulating hormone (FSH) level, suggesting a gonadal abnormality and making karyotyping the most appropriate next step in management.\n\nPrimary amenorrhea can be diagnosed if a patient has normal secondary sex characteristics but no menarche by age 16, or if she has no secondary sex characteristics and no menarche as early as age 14. Primary amenorrhea is caused by either hypothalamic/pituitary (central) or gonadal (peripheral) abnormalities. Increased FSH indicates a peripheral cause or hypergonadotropic hypogonadism. Karyotyping would be the most appropriate next diagnostic step because these disorders can include Turner syndrome and androgen insensitivity syndrome.\n\nGravholt et al. review the evidence regarding the diagnosis and treatment of Turner syndrome. They discuss how this disease presents with hypergonadotropic hypogonadism. They recommend growth hormone therapy and hormone replacement therapy in these patients.\n\nIncorrect Answers:\nAnswer A: CYP17 gene work-up should be done in patients with primary amenorrhea, an absence of secondary sex characteristics, elevated FSH, and hypertension. Alterations in the CYP17 gene can cause congenital adrenal hyperplasia due to 17-hydroxylase deficiency. The resulting excess of deoxycorticosterone is associated with hypertension and hypokalemia. Treatment is with hormone repletion therapy.\n\nAnswer B: Estrogen levels would be low in this patient as evidenced by her absence of breast development. Therefore, measuring estrogen levels would provide no additional diagnostic information. Estrogen levels may be useful in cases of peripheral gonadotropin resistance but these are rare.\n\nAnswer C: Gonadotropin-releasing hormone (GnRH) stimulation test would be the next diagnostic step if FSH were decreased. A decrease in FSH indicates a central cause or hypogonadotropic hypogonadism. A GnRH stimulation test is used to assess whether the central abnormality is due to the hypothalamus or the pituitary.\n\nAnswer E: Luteinizing hormone (LH) levels with a repeat FSH level may be helpful if FSH is low or normal. Low or normal FSH levels indicate hypogonadotropic hypogonadism as caused by hypothalamic-pituitary axis abnormalities. Cases of high FSH represent a different diagnostic entity as adequate gonadotropins are produced.\n\nBullet Summary:\nIn a patient with primary amenorrhea and an increased FSH, the next step in management is karyotyping.", "link": "https://bit.ly/47X4aiT"} {"question": "A 36-year-old G3P2002 woman presents to her obstetrician\u2019s office for her 1st prenatal visit at 10 weeks and 2 days gestation. She has felt nauseous the last several mornings and has been tired for a few weeks. The patient has had 2 uncomplicated spontaneous vaginal deliveries at full term with her last child born 6 years ago. She is concerned about the risk of Down syndrome in this fetus, as her sister gave birth to an affected child at age 43. She has a history of generalized anxiety disorder, atopic dermatitis, and she is currently on escitalopram. Her temperature is 98.6\u00b0F (37.0\u00b0C), pulse is 70/min, blood pressure is 121/67 mmHg, and respirations are 13/min. The patient appears anxious, but overall comfortable, and cardiopulmonary and abdominal exams are unremarkable. Pelvic exam reveals normal external genitalia, a closed and soft cervix, a 10-week-sized uterus, and no adnexal masses. Which of the following is the most appropriate next step for definitively determining whether this patient\u2019s fetus has Down syndrome?", "choicesA": "Amniocentesis", "choicesB": "Anatomy ultrasound", "choicesC": "Chorionic villus sampling", "choicesD": "Genetic testing of patient\u2019s sister", "choicesE": "Nuchal translucency test", "answer_idx": "C", "answer": "Chorionic villus sampling", "explanation": "This patient is over 35 years of age and concerned about the risk of Down syndrome (trisomy 21) in her fetus at just over 10 weeks gestation. At this gestational age, the most definitive method for determining whether such an aneuploidy is present is chorionic villus sampling (CVS).\n\nCVS, or sampling of the placental tissue, can be performed as early as 10 weeks gestation. This is in contrast to amniocentesis, which should be avoided prior to 14 weeks. There are other screening tests for a patient concerned about aneuploidy; however, only CVS and amniocentesis offer definitive diagnoses. Complications of this procedure include pregnancy loss (at a rate of 1-2%), transverse limb abnormality, rupture of membranes, and chorioamnionitis. Patients should also have counseling if genetic abnormalities are detected using this procedure.\n\nFarina reviews the evidence regarding the complications associated with CVS. He discusses how this procedure is becoming more common as the definitive method of providing an early diagnosis. He recommends being aware of complications such as possible pregnancy loss.\n\nIncorrect Answers:\nAnswer A: Amniocentesis is a useful tool for the definitive diagnosis of aneuploidy and other genetic conditions. Due to the higher rate of fetal loss at younger gestational ages, it is reserved for those above 14 weeks.\n\nAnswer B: The anatomy ultrasound can be helpful in identifying morphologic features that are concerning for Down syndrome, such as a flat nasal bridge or an echogenic cardiac focus. This screening tool is most accurate after 18 weeks gestation.\n\nAnswer D: Genetic testing of the patient\u2019s sister may be able to help identify if there is a parental translocation or aneuploidy that led to the child\u2019s Down syndrome. This is much less likely than meiotic nondisjunction due to advanced maternal age, and this would not help diagnose Down syndrome in the patient\u2019s fetus.\n\nAnswer E: The nuchal translucency test examines the thickness of the nuchal fold. When the thickness exceeds normal measurements, there is an increased risk of aneuploidy such as Down syndrome, but this is a screening result that would not provide a definitive diagnosis.\n\nBullet Summary:\nChorionic villus sampling is a definitive method of identifying aneuploidy in fetuses starting at 10 weeks of gestational age.", "link": "https://bit.ly/46Ngn9O"} {"question": "A 9-hour-old newborn girl is found in the newborn nursery with diffuse swelling of the scalp not present at birth. The child was born at 38 weeks of gestation to a 28-year-old G3P3 mother. The mother went into spontaneous labor but the delivery was complicated by a prolonged 2nd stage of labor. A vacuum-assisted vaginal delivery was eventually performed. The child\u2019s Apgar scores were 8 and 9 at 1 and 5 minutes, respectively. The pregnancy was complicated by preeclampsia in the mother which was well-controlled throughout the pregnancy. Her temperature is 98.6\u00b0F (37\u00b0C), blood pressure is 67/43 mmHg, pulse is 135/min, and respirations are 34/min. On physical exam, she appears to be in mild distress and has a 4x5 cm ecchymotic area of swelling over the bilateral parietal bones. Serial assessments of the child\u2019s head circumference over the next 12 hours show no change in the size of the swelling. This patient\u2019s condition most likely affects which of the following spaces or potential spaces?", "choicesA": "Between dura and arachnoid mater", "choicesB": "Between periosteum and galea aponeurosis", "choicesC": "Between periosteum and skull", "choicesD": "Between scalp and galea aponeurosis", "choicesE": "Into the lateral ventricles", "answer_idx": "D", "answer": "Between scalp and galea aponeurosis", "explanation": "This patient presents following a vacuum-assisted vaginal delivery with an ecchymotic swelling of the scalp that crosses the midline, which suggests a diagnosis of caput succedaneum. Caput succedaneum is caused by bleeding between the skin and the galea aponeurosis.\n\nCaput succedaneum is an uncommon extracranial injury that occurs during a traumatic birth, particularly in deliveries in which a vacuum-assist device is used. It involves bleeding between the skin of the scalp and the galea aponeurosis. Unlike a cephalohematoma, caput succedaneum lesions may cross the suture lines of the cranial bones. Caput succedaneum is typically described as fluctuant and ecchymotic, and the swelling usually self-resolves within a few days. Treatment is supportive though patients may occasionally require phototherapy if they develop high levels of bilirubin. Transfusion is indicated if there is significant anemia present (though this is uncommon).\n\nJacob and Hoerter present evidence regarding the diagnosis and treatment of patients with caput succedaneum. They discuss how this lesion commonly crosses cranial suture lines as well as the midline. They recommend differentiating this disease from more malignant etiologies such as intracranial hemorrhage.\n\nIncorrect Answers:\nAnswer A: Bleeding between the dura and arachnoid mater describes a subdural hematoma. Subdural hematomas are the most common type of intracranial hemorrhage found in neonates, but they usually present with more serious signs of respiratory depression, apnea, or seizures. Patients with a subdural hematoma should be evaluated for non-accidental trauma. Treatment of expanding lesions may require surgical decompression.\n\nAnswer B: Bleeding between the periosteum and galea aponeurosis describes a subgaleal hemorrhage, an uncommon but serious complication of traumatic birth. Like caput succedaneum, subgaleal hemorrhages occur most commonly in newborns delivered via vacuum-assisted delivery, but the swelling in a subgaleal hemorrhage involves the entire scalp. Patients may also present with signs of extensive blood loss and neurological disturbances. Treatment involves resuscitation with fluids or blood and possibly neurosurgical intervention.\n\nAnswer C: Bleeding between the periosteum and skull describes a cephalohematoma. Cephalohematoma occurs in newborns and is caused by the rupture of the blood vessels crossing the periosteum. They are usually secondary to a prolonged second stage of labor and, unlike caput succedaneum, do not cross suture lines. The treatment for these lesions is supportive as most of these lesions resolve spontaneously.\n\nAnswer E: Bleeding into the lateral ventricles describes an intraventricular hemorrhage. Intraventricular hemorrhage is associated with premature delivery and usually presents with neurological symptoms. Neurologic findings include seizures, apnea, respiratory depression, or asymmetric reflexes. Treatment may involve urgent surgical decompression to prevent herniation.\n\nBullet Summary:\nCaput succedaneum is an extracranial injury caused by bleeding between the skin of the scalp and the galea aponeurotica in neonates.", "link": "https://bit.ly/3oZgxd5"} {"question": "At a local community hospital staffed only by attending physicians, it is noted that the time to tissue plasminogen activator administration in suspected stroke patients is roughly 10 minutes from presentation. At a teaching hospital with residents, the time to tissue plasminogen activator administration is roughly 2 hours. This has led to many adverse outcomes and increased morbidity at the teaching hospital. Which of the following is the most appropriate next step in further elucidating the problem?", "choicesA": "Automatically consult neurology for patients with concerning neurologic symptoms", "choicesB": "Empirically scan patients with concerning neurologic symptoms", "choicesC": "Improve resident education", "choicesD": "Perform a root cause analysis of the process", "choicesE": "Perform a systems-based approach implementation", "answer_idx": "D", "answer": "Perform a root cause analysis of the process", "explanation": "This case outlines a delay in treating strokes at a teaching hospital. To determine the underlying cause, a root cause analysis to determine the underlying issue is the most appropriate way to begin to address this issue.\n\nWhen a problem is found in healthcare, it is important to elucidate the underlying cause. A root cause analysis is the current preferred method of determining the underlying cause of a problem. A root cause analysis outlines all the people, materials, equipment, and processes in place and outlines them. It then locates the problem and attempts to find the breakdown or issue in all these inputs that lead to the problem. Once the problem is identified, then a strategy or intervention can be implemented to fix the underlying problem.\n\nPercarpio et al. review root cause analysis. They note the ubiquity of root cause analysis. They call into question its cost and efficacy despite it being such a widespread practice and recommend a careful assessment of the costs and benefits.\n\nIncorrect Answers:\nAnswer A: Automatically consulting neurology for patients with concerning neurologic symptoms may be appropriate if the issue was lack of access to timely neurologist assessment in the workup of stroke. While may hospitals have this as a default for patients with stroke-like symptoms, it would be premature to implement this as a solution without a root cause analysis to first determine what the problem is.\n\nAnswer B: Empirically scanning patients with concerning neurologic symptoms would be wasteful and not plausible in a healthcare system. There are many concerning neurologic chief complaints including those that originate in the brain, the spine, or the peripheral nerves; thus, merely CT scanning everyone with a neurological chief complaint is an inefficient use of resources.\n\nAnswer C: Improving resident education is assuming that the problem is a lack of knowledge based on the residents. Before proposing a solution, it is important to first find the underlying cause and address said cause as the delayed intervention may not be the fault of the residents.\n\nAnswer E: A systems-based approach is the best solution when fixing a medical error. However, a root cause analysis is first needed to determine which solution is needed. If an error in the process is noted, a systems-based approach would be the best method to address it.\n\nBullet Summary:\nWhen an error is noted in patient care, the most appropriate initial step in fixing the issue is conducting a root cause analysis.", "link": "https://step2.medbullets.com/testview?qid=216599"} {"question": "A 55-year-old man presents to the emergency department with worsening dyspnea over the past 48 hours. He recently had a cold that kept him home from work for the past week. He has a past medical history of diabetes, obesity, and hypertension. He had his Achilles tendon repaired 4 weeks ago and he has been less mobile. His temperature is 99.2\u00b0F (37.3\u00b0C), blood pressure is 150/85 mmHg, pulse is 82/min, respirations are 16/min, and oxygen saturation is 100% on room air. Physical exam is notable for dyspnea provoked by walking short distances. Jugular venous distension is noted on exam. The patient\u2019s blood pressure is 130/70 mmHg during inspiration. A bedside echocardiogram demonstrates impaired diastolic filling with normal ventricular contractility. An ECG is performed as seen in Figure A. Which of the following is the most likely diagnosis?", "choicesA": "Cardiac tamponade", "choicesB": "Constrictive pericarditis", "choicesC": "Myocardial infarction", "choicesD": "Pulmonary embolism", "choicesE": "Restrictive cardiomyopathy", "answer_idx": "B", "answer": "Constrictive pericarditis", "explanation": "This patient is presenting with a preceding viral illness, dyspnea, jugular venous distension, pulsus paradoxus (blood pressure drop from 150/85 to 130/70 mmHg during inspiration), an echocardiogram with impaired diastolic filling, and diffuse ST elevation with PR depression on ECG. This constellation of symptoms is concerning for constrictive pericarditis.\n\nConstrictive pericarditis occurs when the pericardium becomes thickened and fibrotic. In contrast to other types of pericarditis, in constrictive pericarditis, there is impaired diastolic filling and thus impaired cardiac function. There are many possible etiologies of constrictive pericarditis including infection, inflammatory conditions, or idiopathic. Patients will present with dyspnea, hypotension, tachycardia, a pericardial friction rub, and positional chest pain (usually worsened by laying back and relieved by leaning forward). Patients may also demonstrate jugular venous distension, lower extremity edema, worsening dyspnea, and congestive hepatopathy. ECG may demonstrate diffuse ST elevation with PR depression, and an echocardiogram will demonstrate impaired diastolic filling and may demonstrate a thickened pericardium. The diagnosis can be supported with pulsus paradoxus (a drop of blood pressure by > 10 mmHg during inspiration).\n\nWelch reviews constrictive pericarditis. He discusses constrictive pericarditis and how the inelastic pericardium inhibits cardiac filling possibly causing unexplained heart failure. Welch notes making the diagnosis is very challenging and requires meticulous echocardiograph assessment.\n\nFigure/Illustration A demonstrates diffuse ST elevation (blue arrows) with PR depression (black arrows). This is consistent with pericarditis.\n\nIncorrect Answers:\nAnswer A: Cardiac tamponade can occur after trauma, in the setting of infection, or in the setting of autoimmune disease and is more likely to occur with a rapid accumulation of pericardial fluid. While pulsus paradoxus would be present in this condition, muffled heart sounds and electrical alternans would be more likely to be seen on ECG. Echocardiography would demonstrate an echogenic rim of fluid around the heart.\n\nAnswer C: Myocardial infarction presents with chest pain, dyspnea, hypotension, tachycardia, and ST elevation in a vascular distribution. Note that this patient\u2019s ST elevation is diffuse and there is no reciprocal ST depression. Moreover, the pattern of chest pain in myocardial infarction is typically exertional and not positional.\n\nAnswer D: Pulmonary embolism may present with pleuritic chest pain, shortness of breath, tachycardia, hypoxia, hemoptysis, a history of hypercoagulability, and recent stasis or surgery (such as this patient\u2019s recent Achilles tendon surgery). It can cause obstructive shock leading to hypotension and tachycardia. Note that right heart strain and sinus tachycardia would be seen on ECG.\n\nAnswer E: Restrictive cardiomyopathy presents very similarly to restrictive pericarditis. Note that in restrictive cardiomyopathy, patients will also have impaired diastolic filling; however, it is due to a myocardium that will not expand rather than the pericardium impairing filling. It can occur secondary to conditions such as amyloidosis or hemochromatosis as well as other chronic inflammatory states. The ECG findings are nonspecific but may include low voltage QRS complexes, ST segment changes, T wave changes, bundle branch blocks, and pathologic Q waves (abnormally deep and wide Q waves). The diagnosis can be supported with an ultrasound demonstrating impaired diastolic filling and possible myocardial speckling if a protein (amyloid) is present and deposited in the myocardial wall. The diagnosis can only definitively be confirmed with a biopsy which is highly invasive.\n\nBullet Summary:\nRestrictive pericarditis presents with chest pain that is positional, pulsus paradoxus, a friction rub, and diffuse ST elevation with PR depression on ECG.", "link": "https://step2.medbullets.com/testview?qid=216508"} {"question": "A 4-day-old neonate is brought to the pediatrician with vaginal discharge for the last 2 days. Her daughter has been feeding and voiding well. The neonate was delivered at 39 weeks gestation via an uncomplicated vaginal delivery and was discharged home after 2 days. The prenatal course was complicated by chlamydia in the mother during the first trimester, for which she and the partner were both treated with a confirmatory test of cure. The biological father is no longer involved in the patient's life, but her mother\u2019s boyfriend has been caring for the baby whenever the mother rests. The neonate\u2019s temperature is 98.5\u00b0F (36.9\u00b0C), blood pressure is 56/35 mmHg, pulse is 138/min, and respirations are 51/min. She appears comfortable, and cardiopulmonary and abdominal exams are unremarkable. There are no bruises or marks on her skin. Examination of the genitals reveals no vulvar irritation or skin changes but there is scant pink mucoid discharge at the introitus. Which of the following is the most appropriate next step in management?", "choicesA": "Reassurance", "choicesB": "Report to child protective services", "choicesC": "Vaginal culture", "choicesD": "Vaginal exam under anesthesia", "choicesE": "Warm water irrigation of the vagina", "answer_idx": "A", "answer": "Reassurance", "explanation": "This neonate presents with bloody vaginal discharge soon after birth without vulvar irritation, which is most consistent with physiologic neonatal vaginal discharge. Reassurance is the most appropriate next step in management.\n\nVaginal discharge in a neonate that is bloody and/or mucoid is most commonly due to the rapid withdrawal of maternal hormones that cross the placenta. This results in a pink discharge that may be observed as \u201cmenses\u201d in the perinatal period. This most commonly presents several days to 1 week after birth and will self-resolve. Without concerning red flags such as genital trauma, mucopurulent discharge, or surrounding skin changes, there is no need for work-up or treatment. Additional concerning findings should prompt a work-up for child abuse.\n\nGuritzky and Rudnitsky review the evidence regarding the work-up of a bloody neonatal diaper. They discuss how using an Apt test can be helpful in distinguishing between maternal and fetal blood. They recommend being aware of neonatal conditions that can cause bleeding to avoid missing pathologies.\n\nIncorrect Answers:\nAnswer B: Reporting the case to child protective services would be necessary if child abuse were suspected. Although having an unrelated adult as a caregiver is traditionally a risk factor, there is no evidence that this neonate is being abused. Risk factors include vaginal trauma and bruising.\n\nAnswer C: Vaginal culture may be indicated if an infectious etiology such as chlamydia or group A streptococcus is suspected; however, this would more commonly present with bloody and mucopurulent discharge with or without surrounding skin inflammation. Furthermore, the patient\u2019s mother was successfully treated for chlamydia during pregnancy, making chlamydial infection unlikely.\n\nAnswer D: Vaginal exam under anesthesia would be appropriate if a foreign body or trauma is suspected because prepubescent children should not undergo exams while awake. In a neonate with physiologic discharge, an exam is unnecessary. Foreign bodies would present with bloody discharge, pain, and potential purulence.\n\nAnswer E: Warm water irrigation of the vagina may be helpful in locating a foreign body, which is the most common cause of vaginal bleeding and discharge in a prepubertal girl. This is unlikely in a neonate and there is no evidence of trauma or irritation in the perineum.\n\nBullet Summary:\nVaginal discharge in a neonate is most commonly due to the withdrawal of maternal hormones and will self-resolve.", "link": "https://bit.ly/44zO6BC"} {"question": "A 47-year-old man is brought in after a motor vehicle accident. He was driving under the influence of alcohol when he sustained a crash against a pole. His medical history is significant for hypertension but he does not take any medications. His temperature is 98.6\u00b0F (37.0\u00b0C), blood pressure is 110/80 mmHg, pulse is 60/min, and respirations are 12/min on a ventilator. His Glasgow coma score (GCS) on presentation is 6. Initial urine toxicology is negative. The patient is admitted to the intensive care unit and appropriate care is initiated. One week later, the patient\u2019s GCS score is now 3. Laboratory studies do not show significant electrolyte, acid-base, or endocrine disturbances. Sedative and paralytic agents are withdrawn. Which of the following findings on physical exam would be compatible with brain death?", "choicesA": "Blinking after saline is applied to the cornea", "choicesB": "Conjugate eye movement to the left with rotation of the head to the right", "choicesC": "Flexion of the thigh, leg, and foot upon noxious stimulation of the foot", "choicesD": "Grimacing after noxious stimulus to the arm", "choicesE": "Spontaneous respiration 1 minute after disconnection from a ventilator", "answer_idx": "C", "answer": "Flexion of the thigh, leg, and foot upon noxious stimulation of the foot", "explanation": "This patient with a Glasgow coma score (GCS) of 3 and a known mechanism of injury due to motor vehicle accident trauma is normothermic, normotensive, not on sedatives, and without laboratory abnormalities so they are appropriate for examination of brain death. Cortical and brain stem function must be absent in brain death; however, spinal cord reflexes such as triple flexion (flexion of the thigh, leg, and foot upon noxious stimulation of the foot) may remain intact in patients with brain death.\n\nThe clinical brain death exam may be performed in patients with evidence of irreversible central nervous system catastrophe after a known mechanism of trauma or medical injury. The patient must not have other causes for obtundation, and should be euthermic, not on sedative or paralytic agents, not be intoxicated due to drugs or poisoning, and not have other severe electrolyte, acid-base, or endocrine disturbances. In brain death, there must be no cerebral motor response such as moaning or grimacing to pain in all extremities and the face. Brain stem reflexes such as the pupillary light reflex, corneal reflex, oculocephalic reflex, vestibulo-ocular reflex, gag reflex, and apnea test should also be absent in brain death. Spinal cord and deep tendon reflexes may still be present in brain death. Complex movements mediated by the spinal cord such as triple flexion and the Lazarus sign (raising the arms and dropping them crossed on the chest after noxious stimulus) do not preclude a diagnosis of brain death.\n\nGreer et al. review neurologic criteria for the determination of brain death. They discuss how to clinically differentiate spinal motor responses from brain-mediated motor responses. They recommend understanding these distinctions in order to properly examine patients with brain death.\n\nIncorrect Answers:\nAnswer A: Blinking after saline is applied to the cornea indicates a positive corneal reflex. In the corneal reflex, the ophthalmic branch of the trigeminal (V1) nerve senses stimulus to the cornea, and bilateral temporal branches of the facial nerve (VII) activate the orbicularis oculi muscles to blink. Brainstem reflexes must be absent in brain death, so the presence of a corneal reflex is not compatible with a diagnosis of brain death.\n\nAnswer B: Conjugate eye movement to the left with rotation of the head to the right describes the oculocephalic reflex. In this reflex, movement of the head stimulates endolymph in the horizontal semicircular canal to rotate, causing the firing of the ipsilateral vestibulocochlear (VIII) nerve. This leads to activation or inhibition of the contralateral abducens (VI) and oculomotor (III) nerves. Brainstem reflexes must be absent in brain death, so the presence of an oculocephalic reflex is not compatible with a diagnosis of brain death.\n\nAnswer D: Grimacing after the noxious stimulus of the arm indicates some residual cortical function. Grimacing is not a spinal cord reflex and requires some cortical processing of sensory information. All cortical function must be absent in brain death, so grimacing to pain would not be compatible with brain death.\n\nAnswer E: Spontaneous respiration 1 minute after disconnection from a ventilator in a patient without diabetes insipidus and with normal blood gas values and systolic blood pressure above 90 mmHg indicates a negative apnea test. The apnea test assesses respiratory control system reflexes in the brainstem. Apnea testing is an important component of the brain death exam, and a negative apnea test is incompatible with brain death.\n\nBullet Summary:\nAlthough brain death criteria universally include the absence of cortical and brainstem function, spinal cord and deep tendon reflexes may be preserved in patients with brain death.", "link": "https://bit.ly/3FgV041"} {"question": "A 55-year-old man presents to the emergency department with epistaxis for the past hour. He has never had epistaxis before and has been unable to control it with pressure, head elevation, and ice. The patient has a past medical history of atrial fibrillation and is currently taking metoprolol and warfarin. His temperature is 98.5\u00b0F (36.9\u00b0C), blood pressure is 123/81 mmHg, pulse is 90/min, respirations are 13/min, and oxygen saturation is 97% on room air. Physical exam reveals a comfortable man in no distress. Inspection reveals active bleeding in the anterior nares. There is not a clear, single bleeding vessel. Which of the following is the most appropriate initial step in management?", "choicesA": "Cauterization with silver nitrate", "choicesB": "Fresh frozen plasma and intravenous phytonadione", "choicesC": "Intravenous tranexamic acid", "choicesD": "Placement of posterior nasal packing", "choicesE": "Topical oxymetazoline", "answer_idx": "E", "answer": "Topical oxymetazoline", "explanation": "This patient on warfarin is presenting with epistaxis from the anterior nares (and is currently hemodynamically stable) suggesting anterior epistaxis. Prior to more invasive measures, topical therapy with oxymetazoline should be tried as it is very successful in reducing or resolving epistaxis.\n\nEpistaxis can be a benign or life-threatening presentation. For anterior epistaxis, the management is centered on compression of the anterior nares as most cases are caused by bleeding from Kiesselbach plexus. If this fails, the next step in management is having the patient blow their nose to remove any clots, followed by vigorous application of topical oxymetazoline which vasoconstricts the vessels in the nares and is highly effective in treating epistaxis. If this fails, other topical agents include topical tranexamic acid, and thrombogenic foams. If pressure and topical therapy fail to resolve epistaxis, then anterior nasal packing (such as nasal tampons) may be used.\n\nKucik and Clenney review treatments for epistaxis. They note that the first-line treatment of anterior epistaxis is compression of the anterior nares. If this fails, they recommend other measures such as topical vasoconstrictors, cautery, and packing.\n\nIncorrect Answers:\nAnswer A: Cauterization with silver nitrate is appropriate for anterior epistaxis, but generally should be reserved for patients who have failed initial medical therapy with topical agents. It may also be appropriate first-line therapy in patients with recurrent epistaxis from a clear, single bleeding vessel (which is not present in this patient).\n\nAnswer B: Fresh frozen plasma and intravenous phytonadione is the appropriate reversal agent for warfarin. While this may stop the patient's epistaxis, he is hemodynamically stable and the bleeding is not life-threatening; thus, the risk of thrombus from atrial fibrillation is likely greater than the risk of bleeding in this case of epistaxis. Less invasive medical therapy should be started/tried first.\n\nAnswer C: Intravenous tranexamic acid would be appropriate in an acutely hemorrhaging patient to reduce bleeding in the setting of hemorrhagic shock. Topical (not intravenous) tranexamic acid may be an appropriate first-line agent for anterior epistaxis.\n\nAnswer D: Placement of posterior nasal packing would be indicated if there was persistent bleeding in the setting of posterior epistaxis (often from bleeding from the sphenopalatine artery), which presents with more profuse bleeding that may threaten the airway. This patient's bleeding is clearly anterior as it can easily be visualized in the anterior nares; thus, this intervention is not needed.\n\nBullet Summary:\nTopical oxymetazoline and other topical agents such as tranexamic acid are appropriate first-line agents in the management of epistaxis if pressure alone fails to resolve symptoms.", "link": "https://step2.medbullets.com/testview?qid=216397"} {"question": "A 66-year-old man presents to the emergency department with shortness of breath. He typically has poor exercise tolerance and is only able to walk from his house to his mailbox before becoming dyspneic. He now has shortness of breath even at rest. He had a cold recently but has no new medical problems. His past medical history is notable for a myocardial infarction 7 years ago, COPD, heart failure with a left ventricular ejection fraction of 22%, obesity, diabetes, and peripheral vascular disease. His temperature is 97.7\u00b0F (36.5\u00b0C), blood pressure is 124/82 mmHg, pulse is 140/min, respirations are 32/min, and oxygen saturation is 78% on room air. Physical exam reveals an uncomfortable man with increased work of breathing. He demonstrates bilateral crackles and wheezing on pulmonary exam. The patient is started on BiPAP, and his oxygen saturation improves to 94%. An ECG is performed, as seen in Figure A, and a chest radiograph is performed, as seen in Figure B. Which of the following is the most appropriate next step in management?", "choicesA": "Albuterol and prednisone", "choicesB": "Digoxin", "choicesC": "Diltiazem", "choicesD": "Intubation", "choicesE": "Metoprolol", "answer_idx": "B", "answer": "Digoxin", "explanation": "This patient with a history of heart failure with reduced ejection fraction is presenting with respiratory distress (increased work of breathing, hypoxia, wheezes, and crackles) in the setting of atrial fibrillation (ECG with irregularly irregular rhythm without P waves) with rapid ventricular response and pulmonary edema on chest radiograph. Digoxin is an appropriate medication that can both control the heart rate in atrial fibrillation and increase cardiac contractility and potentially improve respiratory symptoms in this decompensated patient with heart failure.\n\nAtrial fibrillation presents on ECG with an irregularly irregular rhythm without discernible P waves. The initial management of atrial fibrillation with rapid ventricular response is rate control. If the patient is stable, possible first-line agents include beta-blockers (e.g., metoprolol) or calcium channel blockers (e.g., diltiazem). These agents may reduce cardiac contractility and worsen complications such as pulmonary edema in the setting of heart failure with reduced ejection fraction when the patient is in a decompensated state. If the patient has a reduced ejection fraction and requires rate control, then digoxin is a highly effective medication that blocks the AV node and increases cardiac contractility. Any hemodynamically unstable patient with atrial fibrillation with rapid ventricular response should immediately be cardioverted.\n\nMoskowitz et. al review the management of atrial fibrillation. They note that in the management of atrial fibrillation with rapid ventricular response, metoprolol is commonly used in the intensive care setting, is more successful than amiodarone, and had better rate control when compared to diltiazem.\n\nEhle et. al note that digoxin is approved to control ventricular response in patients with chronic atrial fibrillation and is the only oral agent with positive inotropic effects approved for the management of mild to moderate heart failure.\n\nFigure/Illustration A is an ECG demonstrating an irregularly irregular rate/rhythm (note the spacing of the blue arrows indicating the QRS complexes) without P waves, which is suggestive of atrial fibrillation. Figure B is a chest radiograph demonstrating bilateral, hazy interstitial infiltrates, which are concerning for pulmonary edema.\n\nIncorrect Answers:\nAnswer A: Albuterol and prednisone would be appropriate management of a COPD flare, which presents in a patient with a history of smoking with dyspnea, wheezing, increased work of breathing, and a barrel chest. While this patient\u2019s wheezing may be secondary to his COPD, it is much more likely to be a cardiac wheeze given his chest radiograph findings including pulmonary edema. Treating his atrial fibrillation is more likely to alleviate his pulmonary symptoms. Other treatments in COPD include ipratropium and BiPAP.\n\nAnswer C: Diltiazem would be an appropriate rate control agent for atrial fibrillation with rapid ventricular response, particularly in the setting of a COPD flare as it will not impair the mechanism of albuterol (which should be given as a nebulizer in COPD flares). In this patient, further impairing cardiac contractility with a calcium channel blocker may worsen his underlying pulmonary edema (given his reduced ejection fraction).\n\nAnswer D: Intubation would be indicated if this patient\u2019s respiratory status were worsening, including failure to oxygenate, failure to ventilate, or impending respiratory failure. This patient\u2019s respiratory status has improved on BiPAP; thus intubation is not needed.\n\nAnswer E: Metoprolol, like diltiazem, would reduce cardiac contractility and may worsen pulmonary edema, in particular, in a patient with a reduced ejection fraction. It also may cause COPD symptoms to emerge, as beta-2 blockade may cause bronchoconstriction (even though metoprolol is beta-1 selective). It would be indicated as outpatient rate control for a patient with atrial fibrillation who can tolerate a beta-blocker, and is preferred in patients with a history of acute coronary syndrome, as it reduces mortality.\n\nBullet Summary:\nPatients with atrial fibrillation with rapid ventricular response with a reduced ejection fraction may be optimally managed with digoxin, which blocks the AV node and increases cardiac contractility.", "link": "https://step2.medbullets.com/testview?qid=216458"} {"question": "A 65-year-old man presents to the emergency department with low back pain. The pain started 6 hours ago when he was gardening and carrying a heavy bag. The pain is rated as 10/10 in severity and radiates down the posterior aspect of the right thigh. He reports no fevers or chills. His only medical problem is hypertension for which he takes amlodipine. The patient is a non-smoker, uses alcohol infrequently, and has never used illicit drugs. His temperature is 99.6\u00b0F (37.6\u00b0C), blood pressure is 140/70 mmHg, pulse is 90/min, and respirations are 20/min. Physical examination reveals 3/5 strength to hip extension, knee flexion and extension, and plantar flexion bilaterally. Sensation to pinprick is diminished over the posterolateral legs and lateral aspects of both feet. Ankle and knee reflexes are absent bilaterally. The patient\u2019s underwear is wet and a bladder scan reveals 800 mL of urine. Which of the following is the most likely underlying diagnosis?", "choicesA": "Acute inflammatory demyelinating polyneuropathy", "choicesB": "Cauda equina syndrome", "choicesC": "L5/S1 disc herniation", "choicesD": "Overflow incontinence", "choicesE": "Spinal stenosis", "answer_idx": "B", "answer": "Cauda equina syndrome", "explanation": "This patient presenting with acute, severe, radiating low back pain with sensorimotor deficits (3/5 strength in hip extension, knee flexion/extension, plantar flexion, reduced sensation to pinprick) in multiple nerve root distributions (L3-S5), lower motor neuron signs, urinary incontinence (wet underwear), and acute urinary retention (bladder scan showing 800 mL of urine) most likely has cauda equina syndrome.\n\nCauda equina syndrome is characterized by deficits in 2 or more nerve roots that comprise the cauda equina of the spinal cord. It has multiple etiologies, including disc herniation, epidural abscess, tumor, or lumbar spine spondylosis. Cauda equina syndrome presents with severe low back pain radiating into the legs, flaccid paralysis or lower extremity weakness which can be symmetric or asymmetric (often asymmetric), bladder and/or rectal sphincter paralysis, impotence/erectile dysfunction, sensory loss in affected dermatomes, and saddle anesthesia (sensory loss in the perineum). Urinary retention occurs due to bladder paralysis. Diagnosis is aided by emergency magnetic resonance imaging. Cauda equina syndrome is a surgical emergency; the goal of surgical treatment is to decompress the cauda equina to prevent permanent neurological injury, such as with a lumbar laminectomy. This can be differentiated from conus medullaris syndrome, which has perianal numbness without numbness through other dermatomes (down the leg in the S1 or S2 patterns), less severe pain (cauda equina presents with more severe pain), and hyperreflexia in the knees (cauda equina presents with lower motor neuron signs in the legs which lead to decreased reflexes - though conus medullaris may present with absent ankle reflexes, the knees are typically hyperreflexic).\n\nGardner et al. review the symptoms, diagnosis, and medico-legal landscape of cauda equina syndrome. They discuss how the most frequent etiology is a large central lumbar disc herniation. They recommend being vigilant for urinary retention in patients who may have compression as many patients have urinary retention on presentation.\n\nIncorrect Answers:\nAnswer A: Acute inflammatory demyelinating polyneuropathy, also known as Guillain-Barre syndrome, occurs after respiratory or gastrointestinal infections and presents with ascending paralysis and diminished reflexes. While bladder dysfunction can occur, back pain is not typical. Guillain-Barre syndrome can be treated with intravenous immunoglobulin.\n\nAnswer C: L5/S1 disc herniation affects the traversing S1 nerve root. S1 radiculopathy presents with posterolateral leg and dorsolateral foot sensory loss and absence of the ankle reflex. Typically, bowel/bladder symptoms are not present. Treatment includes decompression of the affected nerve root.\n\nAnswer D: Overflow incontinence is caused by either bladder outlet obstruction or poor bladder tone. While this could explain this patient\u2019s urinary retention, it does not account for his back pain or sensorimotor deficits. Isolated overflow incontinence can be treated with catheterization.\n\nAnswer E: Spinal stenosis presents with lower extremity claudication, characterized by pain with standing or ambulation. The pain of neurogenic claudication is characteristically relieved by spinal flexion (\u201cshopping cart sign\u201d). Sensory loss and motor weakness can be present; however, bladder dysfunction is atypical. Treatment includes surgical decompression of the area of stenosis.\n\nBullet Summary:\nCauda equina syndrome is characterized by impingement of 2 or more nerve roots of the cauda equina, resulting in radicular pain, bilateral lower extremity weakness, lower motor neuron signs, sensory loss in affected dermatomes, bowel/bladder dysfunction, and saddle anesthesia.", "link": "https://bit.ly/3LSMEDm"} {"question": "A 69-year-old man presents to his primary care physician for trouble sleeping. He recently retired from working the day shift at a cemetery. When the patient retired, his goal was to finally be able to go out with his wife; however, he finds that he is unable to stay awake past 6 pm in the evening. His wife is disappointed that they cannot do any activities in the evening together. The patient has tried drinking caffeine but finds that it does not help. The patient\u2019s wife claims that the patient seems to sleep peacefully, and the patient states he feels rested when he awakes. The patient has a medical history of irritable bowel syndrome which is managed with fiber supplements. His temperature is 98.6\u00b0F (37\u00b0C), blood pressure is 125/83 mmHg, pulse is 87/min, and respirations are 11/min. The patient\u2019s neurological exam is within normal limits. Which of the following is the most likely diagnosis?", "choicesA": "Advanced sleep phase disorder", "choicesB": "Chronic insomnia", "choicesC": "Delayed sleep phase disorder", "choicesD": "Normal aging", "choicesE": "Obstructive sleep apnea", "answer_idx": "A", "answer": "Advanced sleep phase disorder", "explanation": "This patient is presenting with difficulty staying awake in the early evening suggesting a diagnosis of advanced sleep phase disorder (also known as advanced sleep phase syndrome).\n\nAdvanced sleep phase disorder can present in patients of all ages. Patients will experience difficulty staying awake in the early evening (5 to 8 pm); however, they will typically sleep a normal quantity of hours. These patients generally feel well rested in the morning and are able to function normally during the day. This condition can impair social activities that occur in the evening, which can cause distress to the patient. The diagnosis of advanced sleep phase disorder can be made when these symptoms are present in addition to an absence of another organic cause (such as obstructive sleep apnea). Treatment is with bright light visual stimulation therapy and reconditioning of circadian rhythms.\n\nYaremchuk reviews the diagnosis and treatment of patients with sleep phase disorder. She discusses how sleep disorders such as advanced sleep phase disorder should be distinguished from normal aging. She recommends intervening when sleep quality is compromised as it reduces quality of life in the elderly.\n\nIncorrect Answers:\nAnswer B: Chronic insomnia will present with trouble falling asleep or staying asleep, often alternating with other sleep pathologies. This patient does not claim to have any trouble falling or staying asleep. He also feels well rested in the morning so his sleep quality is adequate. Treatment of insomnia may include melatonin and sleep hygiene.\n\nAnswer C: Delayed sleep phase disorder typically presents in younger patients with difficulty falling asleep early. These patients will often fall asleep very late and struggle to wake up early for obligations such as school or work. Treatment is with melatonin and sleep hygiene interventions.\n\nAnswer D: Normal aging presents with patients claiming that their sleep is less restful in the absence of an organic cause (such as obstructive sleep apnea). As patients age, it is natural for all phases of their sleep to shorten. However, it is not normal for this patient to have such difficulty staying awake in the early evening, so advanced sleep phase disorder is a more likely diagnosis.\n\nAnswer E: Obstructive sleep apnea presents with fatigue, morning headaches, and non-restful sleep. Patients will typically snore loudly and typically are overweight. This patient does not have any of these risk factors or symptoms in his history. Treatment is with positive airway pressure machines and weight loss.\n\nBullet Summary:\nAdvanced sleep phase disorder presents with difficulty staying awake in the early evening that impairs social functioning.", "link": "https://bit.ly/3CxfeFC"} {"question": "A 25-year-old G1P0 woman presents to her physician for a prenatal visit. She is at 12 weeks gestational age and has not received prior prenatal care. She has mild morning sickness but no other symptoms. She has no known medical problems and takes only a prenatal multivitamin. She has unprotected sexual intercourse with 3 male partners. Her temperature is 98.6\u00b0F (37.0\u00b0C), blood pressure is 118/70 mmHg, pulse is 80/min, and respirations are 18/min. Physical exam reveals a well-appearing woman in no acute distress. There are no lesions appreciated on skin exam. Cardiopulmonary exam is unremarkable. An antibody test for human immunodeficiency virus (HIV)-1/2 is positive. The patient is started on an appropriate treatment regimen. Assuming that her HIV viral load is undetectable by the time of delivery, which of the following maternal regimens is she most likely receiving and what neonatal prophylaxis is indicated (Figure A)?", "choicesA": "A", "choicesB": "B", "choicesC": "C", "choicesD": "D", "choicesE": "E", "answer_idx": "C", "answer": "C", "explanation": "This pregnant patient presents with a positive HIV antibody test in the setting of unprotected sexual intercourse. An appropriate maternal HIV treatment regimen is abacavir, lamivudine, and raltegravir; prophylaxis in the infant consists of zidovudine alone if maternal HIV viral load is suppressed to < 50 copies/mL.\n\nAll pregnant women should be screened for HIV as early as possible during pregnancy. Maternal treatment for HIV should be initiated as early as possible (before pregnancy if possible) with highly-active anti-retroviral therapy (HAART) consisting of 2 nucleoside reverse transcriptase inhibitors (NRTIs) such as abacavir/lamivudine, plus an integrase inhibitor (e.g., raltegravir) or a protease inhibitor (e.g., ritonavir/atazanavir). Regimens for prophylaxis in infants of HIV-positive mothers depends on the HIV viral load at the time of delivery. If the viral load is < 50 copies/mL, infant prophylaxis with zidovudine alone for 4 weeks is sufficient. If the viral load is \u2265 50 copies/mL or the mother did not receive HAART during pregnancy, infant prophylaxis should consist of the 3-drug regimen of: zidovudine, lamivudine, and either nevirapine or raltegravir for 6 weeks.\n\nHarris et al. reviewed the treatment of HIV infection in pregnancy. They found that although anti-retroviral therapy during pregnancy significantly reduces mother-to-child transmission of HIV, it is still associated with certain perinatal complications such as low birthweight and preterm delivery. Despite these risks, they recommend anti-retroviral therapy during pregnancy and during breastfeeding to prevent transmission.\n\nFigure A shows the answer choices.\n\nIncorrect Answers:\nAnswer A: Maternal treatment with abacavir/lamivudine/raltegravir and no neonatal treatment is inappropriate because all infants of HIV-positive mothers should receive zidovudine (if maternal viral load < 50 copies/mL at delivery). Lamivudine and raltegravir would be added if maternal viral load is \u2265 50 copies/mL at delivery.\n\nAnswer B: Maternal treatment with abacavir/lamivudine/raltegravir and neonatal prophylaxis with zidovudine/lamivudine/raltegravir would be appropriate if the maternal viral load were \u2265 50 copies/mL at delivery. The HIV viral load in this patient was assumed to be undetectable at delivery.\n\nAnswer D: Maternal treatment with raltegravir/ritonavir/atazanavir and neonatal prophylaxis with zidovudine/lamivudine/raltegravir is inappropriate because raltegravir is an integrase inhibitor and ritonavir/atazanavir are protease inhibitors. HAART must include 2 NRTIs. Atazanavir is often co-administered with ritonavir to increase the former\u2019s bioavailability. Additionally, zidovudine/lamivudine/raltegravir is the neonatal regimen for infants whose mothers have a viral load \u2265 50 copies/mL at delivery.\n\nAnswer E: Maternal treatment with raltegravir/ritonavir/atazanavir and neonatal prophylaxis with zidovudine only is inappropriate because HAART for the mother must include 2 NRTIs.\n\nBullet Summary:\nMaternal HIV infection should consist of highly-active anti-retroviral therapy consisting of 2 nucleoside reverse transcriptase inhibitors plus either an integrase inhibitor or protease inhibito, while neonatal prophylaxis consists of zidovudine only for infants of mothers with viral load < 50 copies/mL at delivery.", "link": "https://bit.ly/43fo59y"} {"question": "A 66-year-old woman presents to the emergency room after experiencing a tonic-clonic seizure. She has been having worsening headaches and intermittent nausea over the past 5 months. The headaches are constant and dull, and they typically worsen when she sneezes or laughs. Recently, she has become increasingly nauseous and has vomited twice in the past 10 days. Her medical history is significant for type 2 diabetes mellitus and hypertension for which she takes metformin and losartan. Her temperature is 98.6\u00b0F (37.0\u00b0C), blood pressure is 115/70 mmHg, pulse is 70/min, and respirations are 12/min. She is noted to have papilledema on fundoscopy, but physical exam is otherwise normal. Her CT scan findings are presented in Figure A. Which of the following is the most likely diagnosis?", "choicesA": "Cerebral toxoplasmosis", "choicesB": "Glioblastoma multiforme", "choicesC": "Hemorrhagic stroke", "choicesD": "Metastatic brain tumor", "choicesE": "Neurosarcoidosis", "answer_idx": "B", "answer": "Glioblastoma multiforme", "explanation": "This patient is presenting with symptoms of increased intracranial pressure (i.e., positional headaches, nausea, vomiting, and papilledema). Combined with her CT finding of a space-occupying butterfly-shaped lesion, the most likely diagnosis is glioblastoma multiforme.\n\nGlioblastoma multiforme is the most common malignant primary nervous system cancer, most commonly occurring in adults above the age of 65. The tumor arises from the cerebral hemispheres and classically crosses the corpus callosum, producing a butterfly appearance on CT/MRI. Pseudopalasading necrosis is seen on biopsy, and tissue biopsy is usually glial fibrillary acidic protein (GFAP) positive. It has an unfavorable prognosis with a life expectancy of 6 months to 1 year. Management of glioblastoma includes surgical resection and palliative radiotherapy/chemotherapy.\n\nStupp et al. studied the role of electrical field therapy (TTFields) in conjunction with temozolomide in the treatment of glioblastoma multiforme. The authors found that combination therapy improved median progression-free survival (6.7 vs. 4.0 months, p < 0.001). The authors recommend that clinicians consider the use of TTFields to treat selected patients with glioblastoma multiforme.\n\nFigure/Illustration A demonstrates a non-contrast axial head CT showing the classic butterfly lesion characteristic of glioblastoma multiforme (red arrow).\n\nIncorrect Answers:\nAnswer A: Cerebral toxoplasmosis presents on imaging with multiple ring-enhancing lesions and is most common in patients with acquired immunodeficiency syndrome (AIDS), especially with CD4 T-cell counts of less than 100 cells/mm^3. This patient has no signs of infection or ring-enhancing lesions.\n\nAnswer C: Hemorrhagic stroke presents acutely with hemiparesis, altered mental status, slurred speech, or sensorimotor symptoms. CT imaging would reveal a hemorrhage.\n\nAnswer D: Metastatic brain tumor often presents with imaging findings of multiple well-circumscribed lesions with surrounding edema. These lesions are generally found at the highly vascularized gray-white junction. In a patient with no smoking history, a single brain lesion, and no other extra-cranial symptoms, this is a less likely diagnosis.\n\nAnswer E: Neurosarcoidosis can be caused by infiltrative sarcoidosis and often presents with bitemporal hemianopsia due to infiltration of the pituitary gland. Patients with sarcoidosis typically also present with pulmonary symptoms (e.g., wheezing and coughing) which are absent in this patient.\n\nBullet Summary:\nGlioblastoma multiforme should be suspected in patients presenting with symptoms of increasing intracranial pressure and classic CT/MRI findings of a butterfly shaped lesion with central necrosis.", "link": "https://bit.ly/3Pv88aE"} {"question": "A 55-year-old man presents to the emergency department after feeling lightheaded then fainting while he was carrying boxes upstairs. The patient works on a farm and does not see a doctor often. He has a history of hypertension but does not take medications routinely. He has not had any chest pain, dyspnea, or weakness during this time frame and currently feels at his baseline. He smokes cigarettes occasionally. His temperature is 98.1\u00b0F (36.7\u00b0C), blood pressure is 152/93 mmHg, pulse is 86/min, respirations are 15/min, and oxygen saturation is 98% on room air. Physical exam is unremarkable and the patient walks with a stable gait and no focal weakness. An ECG is performed as seen in Figure A. Laboratory studies are ordered as seen below.\n\nHemoglobin: 12 g/dL\nHematocrit: 36%\nLeukocyte count: 6,500/mm^3 with normal differential\nPlatelet count: 197,000/mm^3\n\nSerum:\nNa+: 140 mEq/L\nCl-: 102 mEq/L\nK+: 4.3 mEq/L\nHCO3-: 24 mEq/L\nBUN: 20 mg/dL\nGlucose: 139 mg/dL\nCreatinine: 1.1 mg/dL\nTroponin: < 0.01 ng/mL\n\nWhich of the following is the most likely diagnosis?", "choicesA": "Left ventricular hypertrophy", "choicesB": "Non-ST elevation myocardial infarction", "choicesC": "ST elevation myocardial infarction", "choicesD": "Torsades des pointes", "choicesE": "Wolff Parkinson White syndrome", "answer_idx": "A", "answer": "Left ventricular hypertrophy", "explanation": "This patient with chronic untreated hypertension is presenting with syncope and an ECG demonstrating large voltages in leads V1-V6 and a left ventricular strain pattern (ST depressions in V5/V6 and aVL), which are concerning for left ventricular hypertrophy.\n\nLeft ventricular hypertrophy occurs secondary to chronic hypertension or conditions that increase afterload such as aortic stenosis. This leads to an increase in the mass of the left ventricle. There are many possible criteria on ECG for left ventricular hypertrophy including left axis deviation, large R waves, a large S wave in aVR, or ST segment depression and T wave inversion in the left heart leads (referred to as left ventricular strain). A more simple method of recalling left ventricular hypertrophy is very large voltages in the ECG leads that serve the left ventricle. Left ventricular hypertrophy should be considered in presentations such as syncope. Note that while ECG findings suggest left ventricular hypertrophy, the diagnosis must be supported with echocardiography. Management involves treating the underlying cause (most commonly, this is hypertension).\n\nArtham et al. discuss left ventricular hypertrophy. They note that it is a risk factor for future cardiac events and increased mortality. They recommend early diagnosis and management to improve outcomes.\n\nFigure/Illustration A is an ECG with very large voltages (red arrows) and ST segment depressions/T wave inversions (blue arrows, a more classic left ventricular strain pattern), concerning for left ventricular hypertrophy.\n\nIncorrect Answers:\nAnswer B: Non-ST elevation myocardial infarction would present with nonspecific ST segment and T wave changes with an elevated troponin. While this patient does demonstrate ST depressions in the lateral leads, the troponin level is normal.\n\nAnswer C: ST elevation myocardial infarction would present with ST elevation in a vascular distribution on ECG in the setting of chest pain, diaphoresis, and an elevated troponin level. Note that this patient\u2019s lack of symptoms (pain) and normal troponin in the setting of the ST elevation in leads V1-V3 make this unlikely to be ST elevation myocardial infarction and more likely to be left ventricular hypertrophy.\n\nAnswer D: Torsades des pointes occurs in patients with a prolonged QT interval that evolves into twisting of the QRS complexes around an isoelectric baseline. It may cause syncope or cardiac arrest. Note there is no QT prolongation on this patient\u2019s ECG that would predispose him to this condition.\n\nAnswer E: Wolff Parkinson White syndrome presents with a delta wave on ECG (a slurred upstroke into the QRS complex) and may undulate with supraventricular or ventricular tachycardia. There is no delta wave in this patient\u2019s ECG supporting this diagnosis. Ablation is the most effective long-term treatment.\n\nBullet Summary:\nLeft ventricular hypertrophy can be associated with syncope and presents with large voltages in the leads that serve the left ventricle and a left ventricular strain pattern.", "link": "https://step2.medbullets.com/testview?qid=216626"} {"question": "A 28-year-old woman presents to her primary care provider with a headache. Every few weeks she has an episode of a right-sided, throbbing headache. The episodes began several years ago and are accompanied by nausea and bright spots in her vision. The headache usually subsides if she lies still in a dark, quiet room for several hours. The patient denies any weakness, numbness, or tingling during these episodes. Her medical history is significant for acne, hypothyroidism, obesity, and endometriosis. Her home medications include levothyroxine, oral contraceptive pills, and topical trans-retinoin. Her temperature is 98.6\u00b0F (37\u00b0C), blood pressure is 125/81 mmHg, pulse is 64/min, and respirations are 11/min. She has 2 glasses of wine with dinner several nights a week and has never smoked. She works as a receptionist at a marketing company. On physical exam, the patient has no focal neurologic deficits. A CT of the head is performed and shows no acute abnormalities. Which of the following is the most appropriate treatment for this patient during these episodes?", "choicesA": "Acetazolamide", "choicesB": "High-flow oxygen", "choicesC": "Sumatriptan", "choicesD": "Topiramate", "choicesE": "Verapamil", "answer_idx": "C", "answer": "Sumatriptan", "explanation": "This patient presents with episodic, unilateral throbbing headaches with photophobia, which suggests a diagnosis of migraine. The most appropriate treatment during these episodes is an abortive medication such as sumatriptan.\n\nMigraines present with unilateral, \"throbbing\" pain and are associated with nausea and vomiting, photophobia, and phonophobia. Migraines can occur after exposure to specific triggers or they can occur idiopathically. Patients may also endorse visual auras such as bright lines or shapes in the visual field and sensory auras such as paresthesias. Medications like NSAIDs or sumatriptan are usually used as abortive treatment during acute episodes of migraine. Patients with recurrent migraines can be managed with prophylactic medications like topiramate, beta-blockers, or amitriptyline.\n\nMayans and Walling present evidence regarding the treatment of migraines. They discuss how acetaminophen, nonsteroidal anti-inflammatory drugs, triptans, antiemetics, ergot alkaloids, and combination analgesics are effective against this disease. They recommend providing these treatments in order to mitigate symptoms.\n\nIncorrect Answers:\nAnswer A: Acetazolamide is used to treat idiopathic intracranial hypertension (IIH). IIH is most commonly seen in obese women of childbearing age and can be caused by medications such as oral isotretinoin (not topical tretinoin, as in this case) and tetracyclines. The initial symptom is usually a pulsatile headache that worsens with lying down and improves upon standing.\n\nAnswer B: High-flow oxygen is an abortive treatment for cluster headaches. Cluster headaches typically present in male patients with unilateral pain around the eye that is worse at night. Associated symptoms include spasms of the unilateral face as well as tearing and visual abnormalities in the eye.\n\nAnswer D: Topiramate can be used as prophylaxis for migraines but is not commonly used as an abortive medication. Chronic prophylactic medications generally function by decreasing spasms of the cranial blood supply in order to prevent a migraine from developing. They have minimal effectiveness after a migraine has already developed.\n\nAnswer E: Verapamil is a prophylactic medication used for cluster headaches. In addition to presenting in the peri-orbital region, cluster headaches are typically accompanied by lacrimation, rhinorrhea, and focal findings such as ptosis. It would not be effective in terminating a migraine.\n\nBullet Summary:\nTriptans are used as an abortive treatment for migraines, which present as unilateral, throbbing headaches that may be associated with nausea and/or vomiting, photophobia, phonophobia, and aura.", "link": "https://bit.ly/3LoaHJF"} {"question": "A 27-year-old man presents to the emergency department after he developed pain in his right leg after landing off a ski jump. The patient is otherwise healthy and does not take any medications. He states his pain is 10/10. He is requesting medications and is crying out in pain. His temperature is 98.7\u00b0F (37.1\u00b0C), blood pressure is 149/85 mmHg, pulse is 103/min, respirations are 15/min, and oxygen saturation is 99% on room air. Physical exam reveals swelling over his right leg and knee. The patient cries out in pain with passive motion of the foot at the ankle. A radiograph is performed as seen in Figure A. Which of the following is the most likely diagnosis?", "choicesA": "Anterior cruciate ligament tear", "choicesB": "Apophysitis of the tibial tubercle", "choicesC": "Patellar fracture", "choicesD": "Posterior dislocation of the knee", "choicesE": "Tibial plateau fracture", "answer_idx": "E", "answer": "Tibial plateau fracture", "explanation": "This patient is presenting after an axial load on his lower extremity (landing off a ski jump) with severe pain, a fracture on radiography, and severe pain with passive range of motion. This suggests compartment syndrome in the setting of a tibial plateau fracture.\n\nTibial plateau fractures occur as a result of an axial load, where the femur forcefully impacts the tibia, leading to intense pain in the proximal tibia region. One potential complication of this type of fracture is compartment syndrome, which presents with exquisite pain out of proportion to exam with passive motion of the lower extremity. While radiography can aid in diagnosing tibial plateau fractures, a negative radiograph should prompt further investigation with a CT scan in patients with a high suspicion of this type of fracture. Treatment involves performing a fasciotomy if compartment syndrome is present. Surgical repair is warranted if there is evidence of neurovascular compromise or a significantly displaced fracture. Otherwise, immobilizing the knee and providing outpatient orthopedic follow-up care is deemed appropriate for certain less severe fractures.\n\nSchneiderman et al. studied compartment syndrome in high-energy tibial plateau fractures. The authors found that younger patients aged 12-29 years old are more likely to develop compartment syndrome, likely due to increased muscle mass and thicker fascial tissue. The risk of deep surgical site infection remains high at 20-25% of cases. The authors recommend that clinicians should be aware of factors associated with increased risk for compartment syndrome, including young age, male gender, and a high-energy mechanism of injury.\n\nFigure/Illustration A is a radiograph demonstrating a tibial plateau fracture with the red arrows demonstrating a fracture of the tibial plateau.\n\nIncorrect Answers:\nAnswer A: Anterior cruciate ligament tear would present with a sudden popping sensation of the knee with swelling, knee instability, and laxity to anterior traction when force is applied to the tibia causing anterior displacement of the tibia relative to the femur. Management involves assessment with MRI and surgical repair in complete tears or immobilization in partial tears.\n\nAnswer B: Apophysitis of the tibial tubercle (Osgood-Schlatter disease) is common in pediatric patients and presents after exertion (such as basketball or jumping) with pain and tenderness over the tibial tubercle. It is caused by repeated microtrauma or microavulsion, separation of the proximal patellar tendon from the tibial tubercle, and subsequent callous deposition at the tibial tubercle. Management involves rest, ice, and nonsteroidal anti-inflammatory drugs.\n\nAnswer C: Patellar fracture would present with tenderness over the patella with a fracture line on the patella. Treatment of non-displaced fractures includes rest and immobilization in a straight-leg knee immobilizer.\n\nAnswer D: Posterior dislocation of the knee presents with sudden trauma to the knee with pain, swelling, and a loss of pulses in the extremity when the popliteal artery is compromised. Radiography is diagnostic and will show a knee dislocation. Treatment involves immediate reduction to avoid permanent injury to the popliteal artery.\n\nBullet Summary:\nTibial plateau fractures present after trauma with severe lower extremity pain and are highly associated with compartment syndrome.", "link": "https://bit.ly/45s1lW2"} {"question": "A 4-day-old girl presents with her parents to the pediatrician for a routine visit. She was feeding well until this morning when she had several episodes of vomiting. The parents describe the vomitus as \u201cbright green\u201d and the patient has shown little interest in feeding since then. She was born at 36 weeks gestation to a 37-year-old G2P2 mother. The pregnancy was complicated by gestational diabetes, and all prenatal testing was unremarkable. The patient has not yet regained her birth weight and is in the 46th percentile for height and the 36th percentile for weight. The patient\u2019s temperature is 99.2\u00b0F (37.3\u00b0C), blood pressure is 68/46 mmHg, pulse is 132/min, and respirations are 32/min. On physical exam, the patient is in mild distress. She has no dysmorphic features. Her abdomen is distended and firm without guarding. Bowel sounds are hypoactive. The patient\u2019s abdominal radiograph can be seen in Figure A and the patient\u2019s upper gastrointestinal series can be seen in Figure B. Which of the following is the most likely etiology of this patient\u2019s condition?", "choicesA": "Duodenal atresia", "choicesB": "Hirschsprung disease", "choicesC": "Intestinal malrotation", "choicesD": "Jejunal atresia", "choicesE": "Meconium ileus", "answer_idx": "C", "answer": "Intestinal malrotation", "explanation": "This patient presents with bilious vomiting and an upper gastrointestinal series demonstrating the \u201ccorkscrew\u201d appearance of the distal duodenum and proximal jejunum, which is consistent with a diagnosis of intestinal malrotation.\n\nIntestinal malrotation most commonly presents in newborns with bilious emesis. This condition increases the risk of midgut volvulus where the intestines create a closed loop. Abdominal radiographs are rarely diagnostic but are useful in excluding bowel perforation. An upper gastrointestinal series is the diagnostic test of choice and demonstrates an abnormally placed duodenum with a ligament of Treitz on the right side of the abdomen and a \u201ccorkscrew\u201d appearance of the distal duodenum and proximal jejunum. Treatment is with surgical derotation of the bowel or exploratory laparotomy if perforation has occurred.\n\nAdams and Stanton review the evidence regarding the diagnosis and treatment of neonatal bowel obstruction. They discuss how malrotation, jejunoileal atresia, duodenal atresia, and colonic atresia are common causes of this finding. They recommend surgical management of these conditions.\n\nFigure/Illustration A is an abdominal radiograph showing a paucity of air in the abdomen (red circle). These findings are consistent with nonspecific findings of intestinal malrotation.\n\nFigure/Illustration B is an upper gastrointestinal series showing a right-sided ligament of Treitz and a \u201ccorkscrew\u201d appearance of the distal duodenum (red circle). These are classic findings of intestinal malrotation.\n\nIncorrect Answers:\nAnswer A: Although duodenal atresia presents with bilious vomiting, the \u201cdouble bubble\u201d sign would be seen on an abdominal radiograph, which reflects air trapping in both the stomach and the 1st portion of the duodenum. Treatment is with surgical repair of the obstructed segment.\n\nAnswer B: Hirschsprung disease may also cause bilious vomiting, but an abdominal radiograph would demonstrate proximal colonic distension with little air in the rectosigmoid colon. Hirschsprung disease is also well visualized on contrast enema, on which a transition point can be seen between the proximal dilated colon and the narrow sigmoid. Treatment is with excision of the defective segment.\n\nAnswer D: Jejunal atresia can also lead to bilious vomiting, although a \u201ctriple bubble\u201d sign would be seen on an abdominal radiograph with air additionally filling the proximal jejunum. Treatment is with excision of the atretic segment and anastomosis of the bowel lumen.\n\nAnswer E: Meconium ileus may present with bilious emesis, but multiple dilated loops of bowel would be seen on an abdominal radiograph. Meconium ileus is better characterized by contrast enema, on which a microcolon can be visualized. Treatment is with the removal of the meconium and treatment of the underlying cystic fibrosis, if present.\n\nBullet Summary:\nIntestinal malrotation presents in newborns with bilious emesis, an upper gastrointestinal series demonstrating a misplaced duodenum, and a \u201ccorkscrew\u201d appearance of the distal duodenum and proximal jejunum.", "link": "https://bit.ly/3ZgT30W"} {"question": "A 72-year-old man presents to his primary care physician for a general checkup. He states that he has been a bit more fatigued lately but believes it is secondary to poor sleep at his new house. The patient is otherwise healthy and takes no medications. His temperature is 98.0\u00b0F (36.7\u00b0C), blood pressure is 141/90 mmHg, pulse is 82/min, respirations are 16/min, and oxygen saturation is 98% on room air. Physical exam is notable only for minor pallor but is otherwise unremarkable. Basic laboratory studies are ordered as seen below.\n\nHemoglobin: 10 g/dL\nHematocrit: 30%\nLeukocyte count: 67,500/mm^3\nPlatelet count: 119,000/mm^3\n\nSerum:\nNa: 141 mEq/L\nCl: 103 mEq/L\nK: 4.0 Eq/L\nHCO3-: 24 mEq/L\nBUN: 22 mg/dL\nGlucose: 109 mg/dL\nCreatinine: 1.2 mg/dL\nCa: 10.0 mg/dL\n\nWhich of the following is the most likely diagnosis?", "choicesA": "Acute lymphoblastic leukemia", "choicesB": "Acute myelogenous leukemia", "choicesC": "Chronic lymphocytic leukemia", "choicesD": "Chronic myelogenous leukemia", "choicesE": "Hairy cell leukemia", "answer_idx": "C", "answer": "Chronic lymphocytic leukemia", "explanation": "This elderly patient is presenting with minor fatigue, anemia, thrombocytopenia, and a leukocyte count >50,000/mm^3, which is most likely caused by chronic lymphocytic leukemia (CLL).\n\nCLL is a monoclonal proliferation of incompetent mature B-cells. It is common in elderly men and is the most common form of leukemia in the United States. Patients are often asymptomatic when the diagnosis is made or may present with vague constitutional symptoms including fatigue, malaise, and weight loss. It is important to note that symptoms may be subtle or absent. Other findings include non-tender lymphadenopathy and splenomegaly. The diagnosis is supported when the white blood cell count is >50,000/mm^3 and can be supported with a peripheral smear which will show smudge cells. The cells in CLL may be CD5, CD20, and CD23 positive. The most common cause of death in this population is infection given the lack of competent B-cells.\n\nHallek reviews CLL. They discuss updated guidelines on the diagnosis and management of CLL. They recommend the use of targeted agents in initial therapy and acknowledge the need for further clinical trials to guide therapy.\n\nIncorrect Answers:\nAnswer A: Acute lymphoblastic leukemia (ALL) is a common malignancy seen in children and presents with pancytopenia, anemia, fatigue, leukopenia (leading to recurrent infections), and thrombocytopenia (leading to bleeding). Other findings may include fever, bone pain, lymphadenopathy, splenomegaly, and hepatomegaly. Translocations in ALL include: t(12:21) which is the most common and denotes a favorable prognosis and t(9:22) (the Philadelphia chromosome) which is less common and denotes a poor prognosis\n\nAnswer B: Acute myelogenous leukemia occurs secondary to the proliferation of myeloblasts and presents in older patients. Patients will present with fatigue, malaise, weight loss, anorexia, fever, leukocytosis, anemia, thrombocytopenia, and neutropenia. Auer rods are frequently present on a blood smear.\n\nAnswer D: Chronic myelogenous leukemia is a neoplastic proliferation of myeloid stem cells and presents with a leukocyte count often >50,000/mm3. The increased myeloid lines include red blood cells, granulocytes, monocytes, and platelets. Many patients present in the chronic phase with minimal symptoms; however, the blast phase can lead to a hematologic emergency termed a blast crisis which may cause a hyperviscosity syndrome. This condition is more symptomatic and less common than CLL.\n\nAnswer E: Hairy cell leukemia occurs when there is a neoplastic proliferation of mature B cells with abundant cytoplasm with \u201chairy\u201d cytoplasmic processes. The cells are tartrate-resistant acid phosphatase positive. Patients will present with splenomegaly, pancytopenia, weakness, and fatigue. This is a more rare diagnosis when compared to CLL.\n\nBullet Summary:\nChronic lymphocytic leukemia (CLL) presents in elderly patients with malaise, fatigue, and other nonspecific signs (but can also be asymptomatic), with anemia, thrombocytopenia, and a white blood cell count often >50,000/mm^3.", "link": "https://step2.medbullets.com/testview?qid=216611"} {"question": "A 27-year-old G1P0000 woman presents to her obstetrician\u2019s office at 24 weeks gestation complaining of new hair growth on her upper lip. She now needs to shave her face every day or else thick hair appears. She also has worsening acne that started several weeks ago. Her pregnancy thus far has been complicated by gestational diabetes, which is managed by diet and exercise. She has a male fetus based on her 20 week anatomy ultrasound. The patient has a medical history of polycystic ovary syndrome. The patient\u2019s temperature is 98.5\u00b0F (36.9\u00b0C), blood pressure is 121/76 mmHg, pulse is 70/min, and respirations are 13/min. The cardiopulmonary exam is normal, and the patient\u2019s abdomen has a fundal height of 31 cm. A pelvic exam reveals an unremarkable closed cervix and a right-sided adnexal mass without tenderness. Transvaginal ultrasound is performed and shown in Figure A. Which of the following is the most appropriate next step in management?", "choicesA": "Laparoscopy to prevent detrimental effects on the fetus", "choicesB": "Laparoscopy to remove a malignancy", "choicesC": "Percutaneous umbilical blood sampling to determine prognosis", "choicesD": "Reassurance", "choicesE": "Spironolactone to treat hirsutism and acne", "answer_idx": "D", "answer": "Reassurance", "explanation": "This patient presents with hirsutism and acne, symptoms of hyperandrogenism, as well as a solid pelvic mass on ultrasound, most consistent with a luteoma. The most appropriate next step in management is reassurance as the mass tends to self-resolve after pregnancy.\n\nLuteomas are ovarian tumors that arise during pregnancy and produce androgens, causing maternal and sometimes fetal virilization. They appear as solid masses on ultrasound and are not malignant. The likelihood of virilizing the fetus increases if the luteoma arises in the 1st trimester and affected female fetuses present with clitoromegaly or labioscrotal fusion. Male fetuses are not affected. Treatment is supportive as the masses tend to resolve after pregnancy.\n\nWang et al. review the evidence regarding the diagnosis and treatment of luteomas. They discuss how this is a rare tumor that primarily affects women who have had multiple pregnancies. They recommend early diagnosis of this condition in order to inform treatment options.\n\nFigure/Illustration A shows a transvaginal ultrasound with a solid mass on the right ovary (red circle). This appearance is consistent with a luteoma.\n\nIncorrect Answers:\nAnswer A: Laparoscopy to prevent detrimental effects on the fetus is not considered a treatment option for luteomas. In this patient, the exposure to high androgen levels is late in pregnancy and the fetus is a male, making it especially unnecessary to perform surgery.\n\nAnswer B: Laparoscopy to remove a malignancy is not indicated for luteomas, which are benign tumors and self-regress after delivery. Malignancies that can cause virilization in or outside of pregnancy include Sertoli-Leydig cell tumors and Krukenberg tumors.\n\nAnswer C: Percutaneous umbilical blood sampling to determine prognosis may be performed in women who have early virilization and female fetuses. High androgen levels in the umbilical cord are required for the virilization of the fetus, whereas high androgen levels in the mother\u2019s peripheral blood may not predict this outcome.\n\nAnswer E: Spironolactone to treat hirsutism and acne is used outside of pregnancy in women with polycystic ovary syndrome. However, because it is an anti-androgen, spironolactone should not be used in pregnancy as it can cause feminization of male fetuses.\n\nBullet Summary:\nLuteomas generally do not require treatment as they regress after pregnancy.", "link": "https://bit.ly/3YmWmmI"} {"question": "A 37-year-old machinist presents to his primary care physician with eye problems. He has felt a mass in his eye that has persisted for the past month. The patient has a medical history of blepharitis treated with eye cleansing and squamous cell carcinoma of the skin treated with Mohs surgery. His temperature is 99.5\u00b0F (37.5\u00b0C), blood pressure is 157/102 mmHg, pulse is 90/min, respirations are 17/min, and oxygen saturation is 98% on room air. Physical exam is notable for a firm and rubbery nodule palpable inside the patient's left eyelid. Physical exam does not elicit any pain. Which of the following is the most likely diagnosis?", "choicesA": "Chalazion", "choicesB": "Foreign body", "choicesC": "Hordeolum", "choicesD": "Ingrown eyelash follicle", "choicesE": "Meibomian cell carcinoma", "answer_idx": "A", "answer": "Chalazion", "explanation": "This patient presenting with a painless firm and rubbery nodule has symptoms that are suggestive of a chalazion.\n\nChalazion presents with a hard and painless eyelid nodule. This pathology occurs secondary to granulomatous inflammation of the Meibomian gland. The lesion can be treated with simple excision; however, recurrent lesions are concerning for Meibomian cell carcinoma. The most important differential diagnosis to consider in these patients is a hordeolum which presents with a firm and painful lid nodule (in contrast to painless). Treatment is with warm compresses followed by incision and curettage in refractory cases.\n\nLi et al. review the evidence regarding the diagnosis and treatment of chalazion. They discuss how gland loss is a possible sequelae from this disorder. They recommend using warm compresses for symptomatic relief.\n\nIncorrect Answers:\nAnswer B: Foreign body is a possible diagnosis given this patient's occupation as a machinist; however, this would typically present with pain over the patient's cornea with a sudden onset of symptoms. Treatment is with removal of the foreign body.\n\nAnswer C: Hordeolum presents with a hard and painful lid nodule in contrast to a hard and painless lid nodule. Treatment is with warm compresses and possible debridement in refractory cases.\n\nAnswer D: Ingrown eyelash follicle could present with a firm lid nodule; however, it would be located near the margin of the eyelid and would likely be inflamed and tender. Treatment is with warm compresses and possible debridement in refractory cases.\n\nAnswer E: Meibomian cell carcinoma is a possible diagnosis but is epidemiologically less common than a chalazion. This diagnosis would be more likely if the lesion was recurrent despite excision. Treatment is with wide excision of the lesion.\n\nBullet Summary:\nChalazion presents with a firm and painless eyelid nodule.", "link": "https://step2.medbullets.com/testview?qid=109961"} {"question": "A 59-year-old woman comes to the clinic due to a 3-month history of \u201calways feeling the urge to urinate\u201d as well as vaginal pruritis. She states that she has had involuntary loss of urine about 4 times each day over the past 3 months. It is not associated with coughing or sneezing. Menopause occurred at age 52. She has a history of chlamydial infection in her 20's and has had 3 urinary tract infections over the past year. The patient has not been sexually active with her husband due to pain with intercourse. She has no other significant medical history and does not take any medications. Her temperature is 98.6\u00b0F (37.0\u00b0C), blood pressure is 115/70 mmHg, pulse is 70/min, and respirations are 12/min. On physical examination, the vulvar skin is pale and atrophic with reduced elasticity and multiple areas of petechiae. The vaginal mucosa also has a loss of rugae and several areas of friable irritation. There is minimal clear vaginal discharge. Urinalysis is normal and fecal occult blood test is negative. Pelvic ultrasound reveals a small, retroverted uterus. Which of the following is the most appropriate next step in management?", "choicesA": "Metronidazole", "choicesB": "Nitrofurantoin", "choicesC": "Topical clobetasol", "choicesD": "Topical estrogen", "choicesE": "Vaginal moisturizers", "answer_idx": "E", "answer": "Vaginal moisturizers", "explanation": "This postmenopausal patient is presenting with vulvovaginal dryness and irritation, dyspareunia, urinary incontinence, and recurrent urinary tract infections, likely due to genitourinary syndrome of menopause (GSM). The most appropriate initial treatment is symptomatic relief with vaginal moisturizers and lubricants.\n\nReduced estrogen levels after menopause lead to vulvovaginal atrophy by causing reduced blood flow and collagen content in vulvovaginal tissues. Patients typically have thin vulvar skin and loss of vaginal tissue pliability. The bladder and urethral epithelium may also atrophy leading to dysuria and urinary frequency. Atrophic tissue becomes thin, dry, and susceptible to injury. The first line of treatment is vaginal moisturizers and lubricants. If this does not improve symptoms, the next step in treatment is vaginal estrogen to help restore blood flow in vulvovaginal tissues. Finally, hormone replacement therapy is indicated if symptoms are refractory to all other modalities.\n\nBhupathiraju et al. studied the association between vaginal estrogen and chronic disease risk. The authors found no association between low-dose vaginal estrogen and cardiovascular disease, cancer, and hip fractures over an 18-year period. The authors recommended that low-dose vaginal estrogen can be used indefinitely without concomitant progestin therapy.\n\nIncorrect Answers:\nAnswer A: Metronidazole is the treatment for trichomoniasis or bacterial vaginosis. While patients with trichomoniasis can present with cervical petechiae and punctate hemorrhages, they typically also have frothy green discharge (absent in this patient). While patients with bacterial vaginosis can also have vaginal discharge, the discharge is typically malodorous and symptoms of pruritis are generally absent.\n\nAnswer B: Nitrofurantoin is the treatment of choice for urinary tract infections. While this patient has increased urinary frequency and a history of recurrent urinary tract infections, her normal urinalysis makes the diagnosis of urinary tract infection unlikely.\n\nAnswer C: Topical clobetasol cream is the treatment for lichen sclerosis. While lichen sclerosis can present with pain with intercourse and vulvar pruritis, patients often have plaques that disrupt the normal vulvar architecture. Lichen sclerosis does not affect the vaginal tissues.\n\nAnswer D: Topical estrogen is the treatment of choice for cases of vulvovaginal atrophy resistant to treatment with vaginal moisturizers and lubricants. This patient should first be trialed on vaginal moisturizers and lubricants.\n\nBullet Summary:\nVulvovaginal atrophy is initially treated with vaginal moisturizers and lubricants.", "link": "https://step2.medbullets.com/testview?qid=215032"} {"question": "A 16-year-old boy presents to his primary care physician for a routine follow-up visit. He feels disappointed because his voice has not changed. He is concerned that he is not as tall as his classmates. He denies any headache, vision changes, nausea, or vomiting. He recently started playing for his high school basketball team and generally eats healthy food. He has no significant medical history and takes no medications. His temperature is 98.6\u00b0F (37.0\u00b0C), blood pressure is 115/70 mmHg, pulse is 70/min, and respirations are 12/min. On physical examination, the patient does not have facial hair or acne. His height is 60 inches and his arm span is 50 inches. His visual fields are full and he has no cranial nerve abnormalities. His strength is 5/5 in bilateral upper and lower extremities. Chest palpation and abdominal examination are unremarkable. He has no pubic hair and his testicles are symmetric with a volume of 3 mL. Which of the following is the most appropriate next step in management?", "choicesA": "Karyotyping", "choicesB": "Magnetic resonance imaging (MRI) of the brain with gadolinium", "choicesC": "Radiograph of the hand and wrist", "choicesD": "Reassurance with close follow-up", "choicesE": "Testicular ultrasound", "answer_idx": "C", "answer": "Radiograph of the hand and wrist", "explanation": "This patient is greater than 14 years of age without the development of secondary sex characteristics (e.g., testicular volume < 4 mL), which is concerning for delayed puberty. The patient should have a radiograph of the hand and wrist to evaluate for discrepancies between bone age and chronological age.\n\nDelayed puberty in boys is defined as the absence or incomplete development of secondary sexual characteristics (increased testicular volume and pubic hair) at 14 years or older. It has a variety of etiologies including chromosomal abnormalities, gonadotropin-releasing hormone (GnRH) deficiencies, and constitutional delay of puberty. In addition to obtaining a history and physical examination, initial diagnostic studies used to work up delayed puberty includes a radiograph of the hand and wrist to determine bone age, and serum studies (complete blood count, complete metabolic panel, follicular-stimulating hormone [FSH], luteinizing hormone [LH], and testosterone). Radiography can compare the patient's bone age to the chronological age (as constitutional delay presents with delayed bone age), and serum endocrine studies can help distinguish between primary causes of hypogonadism (high FSH and LH) and secondary causes of hypogonadism (low to normal FSH and LH). Treatment in cases with a specific cause of delayed puberty is aimed at the underlying cause. For male patients with presumed constitutional delay of puberty, therapy can include hormone replacement with testosterone.\n\nSaengkaew et al. discuss the genetics of pubertal delay. The authors find that pathologic mechanisms that may underlie constitutional delay of puberty are largely related to GnRH neuronal development and biology. The authors recommend further study of GnRH neuronal biology and network functionality to provide the opportunity for improved therapies for patients with disorders of puberty.\n\nIncorrect Answers:\nAnswer A: Karyotyping is a useful diagnostic study in patients with Klinefelter syndrome, which would demonstrate 47, XXY. Patients would present with a eunuchoid habitus (arm span at least 5 cm longer than height), gynecomastia, and testicular atrophy.\n\nAnswer B: Magnetic resonance imaging (MRI) of the brain with gadolinium is a useful diagnostic study in patients with neurological symptoms in addition to hypogonadism. These symptoms would suggest that the patient has a prolactinoma causing impingement of the optic chiasm leading to bitemporal hemianopsia and sometimes a headache. Laboratory studies consistent with a prolactinoma include elevated serum prolactin with low to normal FSH and TSH.\n\nAnswer C: Reassurance with close follow-up would be appropriate in some patients with constitutional delay of puberty, which is the most common cause of delayed puberty. It is believed to be caused by a transient defect in the production of GnRH. Patients typically have a family history of \"late bloomers,\" characterized as family members having a late growth spurt or late puberty.\n\nAnswer E: Testicular ultrasound would be appropriate in patients with asymmetric testicles since gonadal tumors can occur in a number of intersex disorders (e.g., androgen insensitivity syndrome) and present with impaired sexual maturation.\n\nBuller Summary:\nDelayed puberty in boys is defined as a lack of testicular enlargement by age 14 and initial diagnostic evaluation should include radiography of the hand and wrist as well as serum testing (FSH, LH, and testosterone).", "link": "https://bit.ly/3MdLLon"} {"question": "A 27-year-old woman presents to her primary care physician with pain in her hands, shoulders, and knees. The pain has lasted for several months but seems to have worsened recently. Any activity such as opening jars, walking, or brushing her teeth is painful. The patient has a medical history of a suicide attempt in college, constipation, anxiety, depression, and a sunburn associated with surfing which was treated with aloe vera gel. Her temperature is 99.5\u00b0F (37.5\u00b0C), blood pressure is 137/78 mmHg, pulse is 92/min, respirations are 14/min, and oxygen saturation is 98% on room air. Laboratory values are obtained and shown below. Hemoglobin: 9 g/dL Hematocrit: 33% Leukocyte count: 2,500/mm^3 with normal differential Platelet count: 107,000/mm^3 Serum: Na+: 139 mEq/L Cl-: 102 mEq/L K+: 4.4 mEq/L HCO3-: 24 mEq/L BUN: 21 mg/dL Glucose: 90 mg/dL Creatinine: 1.0 mg/dL Ca2+: 10.2 mg/dL AST: 12 U/L ALT: 10 U/L Which of the following findings is the most likely to be positive in this patient?", "choicesA": "Anti-cyclic citrullinated peptide antibodies", "choicesB": "Anti-dsDNA antibodies", "choicesC": "Anti-histone antibodies", "choicesD": "Degenerated cartilage in weight bearing joints", "choicesE": "IgM against parvovirus B19", "answer_idx": "B", "answer": "Anti-dsDNA antibodies", "explanation": "This patient is presenting with arthralgias, pancytopenia, possible malar rash, and a history of depression/abnormal behavior. Epidemiologically, the most likely diagnosis is systemic lupus erythematosus (SLE) which is associated with anti-dsDNA antibodies.\n\nSLE is an autoimmune disorder that most commonly occurs in young adult women. It presents with at least 4 criteria from SOAP BRAIN MD - Serositis, Oropharyngeal ulcers, Anti-nuclear antibodies, Photosensitivity, Blood disorders (pancytopenia and hemolytic anemia), Renal abnormalities (proteinuria), Arthralgias, Immune disease (hemolytic anemia and anti-dsDNA), Neurologic (psychosis, stroke, and seizure), Malar rash, and Discoid rash. The most common presenting symptoms for SLE can often be just arthralgias/arthritis with vague other symptoms suggestive of a rheumatologic disorder. Anti-nuclear antibodies are very sensitive for this pathology and anti-dsDNA antibodies are very specific. Treatment may include hydroxychloroquine as well as other immunomodulators and management of end-organ dysfunction.\n\nWang and Xia review the evidence regarding double stranded DNA in patients with SLE. They discuss how these antibodies are a very specific finding in this disease. They recommend measuring these levels.\n\nIncorrect Answers:\nAnswer A: Anti-cyclic citrullinated peptide is associated with rheumatoid arthritis which is a possible diagnosis in this patient, but it is less likely epidemiologically given her age and other symptoms of pancytopenia and a history of sunburn (which is likely a malar rash or photosensitivity). Treatment is with methotrexate.\n\nAnswer C: Anti-histone antibodies would be found in drug-induced lupus. This patient's medication history is not known; however, this is a less likely diagnosis compared to SLE. Treatment is with discontinuation of the offending medication.\n\nAnswer D: Degenerated cartilage in weight bearing joints describes osteoarthritis which tends to affect the knees and hips in older patients. Treatment is with conservative management or with total joint replacement.\n\nAnswer E: IgM against parvovirus B19 describes a parvovirus B19 infection which commonly presents with arthralgias; however, her other symptoms point towards a diagnosis of SLE. Treatment is supportive.\n\nBullet Summary:\nArthritis/arthralgias are often the most common presenting symptom for SLE.", "link": "https://bit.ly/3Qj2QR9"} {"question": "A 31-year-old woman presents to her obstetrician for her first prenatal visit after having a positive home pregnancy test 1 week ago. Her last menstrual period was 8 weeks ago. The patient has a medical history of type 1 diabetes mellitus since childhood and her home medications include insulin. Her hemoglobin A1c 2 weeks ago was 13.7%. At that time, she was also found to have microalbuminuria on routine urinalysis. Her primary care provider prescribed lisinopril but the patient has not yet started taking it. The patient\u2019s brother is autistic but her family history is otherwise unremarkable. Her temperature is 98.6\u00b0F (37.0\u00b0C), blood pressure is 124/81 mmHg, pulse is 75/min, and respirations are 14/min. A physical exam is unremarkable. This fetus is most likely at increased risk for which of the following complications?", "choicesA": "Aneuploidy", "choicesB": "Neonatal hyperglycemia", "choicesC": "Neural tube defect", "choicesD": "Oligohydramnios", "choicesE": "Post-term delivery", "answer_idx": "C", "answer": "Neural tube defect", "explanation": "This patient has type 1 diabetes mellitus and presents in early pregnancy with a markedly elevated hemoglobin A1c, suggesting poor glycemic control. Poor glycemic control increases the risk of a neural tube defect in the fetus.\n\nInfants of diabetic mothers (IDM) are at increased risk of many congenital malformations as well as postnatal complications. These risks are more likely if there is poor maternal glycemic control, especially in the periconceptional period. Poorly controlled hemoglobin A1c and microalbuminuria on urinalysis indicate long-standing poor glycemic control and indicate a poor prognosis. The most common anomalies in IDM are cardiovascular defects such as septal defects and transposition of the great arteries as well as neural tube defects such as anencephaly and spina bifida. Sacral agenesis, or a poorly developed caudal spine and spinal cord, as well as small left colon syndrome due to a meconium plug, are other common malformations seen in IDM. General complications in IDM include prematurity, intrauterine growth restriction or macrosomia, and miscarriage. Risk is minimized by tight glycemic control throughout pregnancy.\n\nDean et al. summarize the evidence surrounding micronutrients and obesity in pregnancy. They found that patients with diabetes had a higher risk of having infants with a neural tube or a congenital heart defect. They recommend using folic acid supplementation in all patients to reduce the risk of neural tube defects.\n\nIncorrect Answers:\nAnswer A: Aneuoploidy risk is not increased in IDM. This complication is associated with advanced maternal age. This patient is at baseline risk since she is below the \u201cadvanced maternal age\u201d cutoff of 35 years. Some of the values used in aneuploidy screening (maternal serum alpha\u00adfetoprotein, estriol, and inhibin A) are generally lower in women with diabetes. Guidelines specific to this patient population should be used for the aneuploidy screen so as to avoid false positive results.\n\nAnswer B: Neonatal hyperglycemia is not seen in IDM. Patients may experience hypoglycemia. This is due to the crossing of glucose from the mother to the fetus via the placenta and subsequent hyperinsulinemia and \u00df-cell hyperplasia from the fetus. After delivery, the high insulin levels cause hypoglycemia because there is no longer an overabundant source of glucose. Hyperglycemia in infants is much less common and can be due to glucose infusion or sepsis.\n\nAnswer D: Oligohydramnios is not specifically associated with maternal hyperglycemia. Instead, patients may experience polyhydramnios. The mechanism for increased amniotic fluid is unclear, though some speculate that fetal hyperglycemia results in polyuria and thus polyhydramnios. Severe oligohydramnios can cause pulmonary hypoplasia and clubfoot as well as cord compression, while polyhydramnios is associated with cord prolapse, placental abruption, and prematurity.\n\nAnswer E: Post-term delivery occurs after 42 weeks gestation. Maternal diabetes typically causes prematurity. Post-term delivery is associated with fetal malnutrition, hypoxemia, and oligohydramnios due to suboptimal functioning of the placenta. The risk of meconium aspiration increases as gestational age increases. Induction of labor is generally performed at 42 weeks to avoid these complications.\n\nBullet Summary:\nMaternal hyperglycemia from diabetes mellitus is associated with congenital heart defects, neural tube defects, and small left colon syndrome.", "link": "https://step2.medbullets.com/testview?qid=108737"} {"question": "A 32-year-old G1P0 woman delivers a boy at 40 weeks and 3 days of gestation. The delivery is uncomplicated and the neonate has Apgar scores of 9 and 9 at 1 and 5 minutes respectively. After 15 minutes, he attempts to nurse for the 1st time. His mother notices that his face, chest, and extremities turn dusky blue during nursing. She also notes that he seems to be snoring loudly and breathing very quickly. When she removes him from her breast, he begins to cry and his color improves. The neonate\u2019s prenatal course was unremarkable, and his mother has a medical history of asthma with occasional albuterol inhaler use. As the infant is resting, vital signs are taken. His temperature is 98.2\u00b0F (36.7\u00b0C), pulse is 130/min, respirations are 45/min, and oxygen saturation is 97% on room air. A physical exam demonstrates well-perfused skin but is notable for the facial features shown in Figure A. Which of the following is most likely to be observed in this neonate?", "choicesA": "Areflexia in all extremities", "choicesB": "Digital clubbing", "choicesC": "Inability to pass a nasogastric tube", "choicesD": "Markedly decreased lung volume", "choicesE": "Patent foramen ovale", "answer_idx": "C", "answer": "Inability to pass a nasogastric tube", "explanation": "This neonate presents with central cyanosis upon feeding, resolution with crying, and midface hypoplasia on exam, most consistent with choanal atresia. A nasogastric (NG) tube would not pass through the nasal canal.\n\nChoanal atresia is caused by failed recanalization of the nasal fossae during fetal development. This disease typically presents with noisy breathing, cyanosis when feeding, and improvement with crying and the use of the mouth for breathing. Cyanosis in this condition affects the entire body as opposed to just the hands and feet. Bilateral atresia often presents early in life. Clinical suspicion of choanal atresia should prompt an attempt to pass an NG tube, which will meet resistance at the posterior part of the nasal canal. Treatment is with the surgical creation of the nasal passages.\n\nLesciotto et al. review the diagnosis and pathophysiology of patients with choanal atresia. They discuss how this condition is often comorbid with craniosynostosis. They recommend a better understanding of how these patients develop decreased nasal airway volume.\n\nFigure/Illustration A is a clinical photograph showing midface hypoplasia with a poorly developed nasal bone (red circle). These findings are consistent with a diagnosis of choanal atresia.\n\nIncorrect Answers:\nAnswer A: Areflexia in all extremities may be seen in spinal muscular atrophy, a genetic condition that causes loss of motor neurons. This condition may cause difficulty feeding due to weakness of the oropharyngeal and glossopharyngeal muscles, but it typically does not cause cyanosis. Treatment is with steroid use and nusinersen.\n\nAnswer B: Digital clubbing can result from chronic hypoxemia, as seen in conditions such as cystic fibrosis. It would be unexpected for a neonate to have clubbing as the characteristic finger appearance depends on the long-term deposition of platelet-derived growth factor and would not develop within minutes. This patient also does not have cyanosis at baseline. Treatment of cystic fibrosis is with enzyme repletion and respiratory therapy.\n\nAnswer D: Markedly decreased lung volume can occur in congenital diaphragmatic hernia, in which the abdominal contents herniate into the chest cavity and cause pulmonary hypoplasia. Infants may present with cyanosis due to respiratory failure, but there would be no association with feeding or improvement with crying. Vital signs would also be abnormal. Treatment is with surgical repair of the diaphragm.\n\nAnswer E: A patent foramen ovale occurs when the connection between the left and right atria does not close soon after birth. It does not result in cyanosis and is typically asymptomatic, though it may cause paradoxical embolism and stroke later in life. Treatment of a symptomatic patent foramen ovale is surgical closure of the defect.\n\nBullet Summary:\nChoanal atresia presents with cyanosis and respiratory distress that worsens with feeding and improves with crying associated with failure to pass a nasogastric tube.", "link": "https://bit.ly/43OL5NI"} {"question": "A 24-year-old woman presents to clinic for a routine annual exam. She has generally been feeling well, but notes feeling intermittent palpitations over the past few months. Her past medical history is unremarkable and she is not currently taking any medications. She denies drinking alcohol, smoking cigarettes, or using recreational drugs. She attributes her palpitations to recently drinking more caffeine, but would like to obtain an electrocardiogram since her symptoms are worrisome and affecting her ability to concentrate on her career as a fitness instructor. Her temperature is 36.9\u00b0C (98.4\u00b0F), blood pressure is 116/76 mmHg, pulse is 55/min, respirations are 12/min, and oxygen saturation is 98% on room air. Her electrocardiogram is shown in Figure A. Which of the following is the most likely diagnosis?", "choicesA": "First degree atrioventricular block", "choicesB": "Second degree atrioventricular block, Mobitz type I", "choicesC": "Second degree atrioventricular block, Mobitz type II", "choicesD": "Sinus bradycardia", "choicesE": "Third degree atrioventricular block", "answer_idx": "B", "answer": "Second degree atrioventricular block, Mobitz type I", "explanation": "This patient's progressively increasing PR intervals until a P wave is not followed by a QRS complex on ECG is consistent with second degree atrioventricular (AV) block, Mobitz type I, also referred to as Wenckebach phenomenon.\n\nSecond degree AV block, Mobitz type I can occur in a variety of settings, such as in the context of digoxin, beta blockers, calcium channel blockers, or increased vagal tone. On ECG, there is progressive lengthening of the PR intervals until a P wave is not followed by a QRS complex (a dropped beat); then, the PR interval resets. Patients are typically asymptomatic, but should undergo evaluation for an underlying etiology of the dysrhythmia, such as electrolyte imbalances, medication side effect, or structural heart disease, as well as a follow-up ECG to evaluate for any progression. If the dysrhythmia is caused by medication, the offending agent should be discontinued, but otherwise there are no specific treatments necessary for asymptomatic patients. Patients who are symptomatic and hemodynamically stable can undergo telemetry monitoring and receive atropine with limited efficacy. Patients who are hemodynamically unstable and not responsive to atropine may need cardiac pacing. Generally, there is a low risk of progression to third degree AV block, and a pacemaker is rarely required (in contrast to second degree heart block type II).\n\nBarold and Hayes review second-degree heart block. They note the changes in AV node conduction time on ECG in this condition. They recommend using the correct definition of the different heart blocks for the correct underlying treatment.\n\nFigure/Illustration A is an ECG showing PR intervals that gradually prolong until a P wave is seen not followed by a QRS complex (note the blue bars that show the progressive lengthening of the PR intervals until the dropped beat).\n\nIncorrect Answers:\nAnswer A: First degree AV block is seen on ECG with prolonged PR intervals greater than 0.2 sec, but a consistent 1:1 ratio of P waves to QRS complexes. This finding is typically found in younger patients with increased vagal tone or in athletes, and does not require treatment.\n\nAnswer C: Second degree AV block, Mobitz type II is seen with fixed PR intervals and occasional dropped QRS complexes. Patients with Mobitz II sometimes present with syncope and frequently progress to third degree AV block. This finding may be associated with fibrotic changes to the conduction system or from acute, subacute, or prior myocardial infarction, and should be treated with placement of a pacemaker in most cases.\n\nAnswer D: Sinus bradycardia findings on ECG include normal sinus rhythm with a ventricular rate of less than 60 beats per minute. Patients with sinus bradycardia may be asymptomatic, but may also present with syncope, lightheadedness, chest pain, or hypotension. No treatment is required if the patient is asymptomatic, but atropine may be given to increase the heart rate in symptomatic cases. Pacemaker placement is required for patients with chronic symptoms.\n\nAnswer E: Third-degree AV block occurs when the atria and ventricles depolarize independently on ECG. There is no relationship between P waves and QRS complexes. Patients may present with syncope, dizziness, acute heart failure, or hypotension. The treatment is pacemaker placement.\n\nBullet Summary:\nSecond degree atrioventricular block, Mobitz type I, is characterized by progressively lengthening PR intervals until a P wave is not followed by a QRS complex.", "link": "https://step2.medbullets.com/testview?qid=216259"} {"question": "A 59-year-old man presents accompanied by his wife with nausea and dizziness. He is unsure when his symptoms started, but they have been affecting him for \u201ca while.\u201d It began as episodes of \u201cunsteadiness\u201d that progressed to a feeling of \u201cspinning.\u201d He cannot tell if his symptoms change with position but reports that if he does not lie down he will become nauseous. He also has worsening hearing loss worse on his right side. The patient\u2019s medical history is significant for hypertension, alcohol use disorder, and chronic obstructive pulmonary disease. His medications include aspirin, amlodipine, and fluticasone-salmeterol. He drinks a glass of red wine every night with dinner and smokes a cigar on the weekends. His temperature is 98.6\u00b0F (37\u00b0C), blood pressure is 135/91 mmHg, pulse is 72/min, and respirations are 12/min. Examination shows delayed horizontal nystagmus. Which of the following is the most appropriate treatment for this patient's condition?", "choicesA": "CN VIII ablation", "choicesB": "Epley maneuver", "choicesC": "Low-salt diet", "choicesD": "Meclizine", "choicesE": "Thiamine", "answer_idx": "C", "answer": "Low-salt diet", "explanation": "This patient is presenting with chronic vertigo, hearing loss, and horizontal nystagmus, which is suggestive of Meniere disease. The most appropriate initial step in the management of this disease is a low-salt diet.\n\nMeniere disease is caused by an abnormal accumulation of endolymph within the inner ear. Symptoms will include chronic relapsing and remitting episodes of vertigo, sensorineural hearing loss, tinnitus, and nausea. Physical exam, audiometry, and vestibular testing can be performed to confirm the diagnosis. First-line treatment is a thiazide diuretic as well as lifestyle modifications including a low-salt diet as well as avoidance of alcohol, nicotine, and caffeine. If symptoms persist, additional therapy may involve vestibular nerve ablation or a labyrinthectomy.\n\nTassinari et al. review the evidence regarding the treatment of Meniere disease. They discuss how vertigo, low-pitched tinnitus, and hearing loss are characteristic of this disease. They recommend creating a database to allow for better research into this disease.\n\nIncorrect Answers:\nAnswer A: CN VIII ablation can be used to treat Meniere disease that is uncontrolled by diet modifications or thiazide diuretics. Patients usually respond to conservative care so a complete course of these treatments should be trialed first.\n\nAnswer B: The Epley maneuver can be used to treat benign paroxysmal positional vertigo (BPPV). Patients will present with vertigo that changes with the patient's position without hearing loss or tinnitus. This maneuver functions by dislodging otoliths from the semicircular canals.\n\nAnswer D: Meclizine can be used to treat other types of vertigo such as BPPV or viral labyrinthitis. Labyrinthitis can present with similar symptoms to the symptoms of Meniere disease, but they will be acute and self-limited. There will also be inflammation of the cochlear portion of the inner ear.\n\nAnswer E: Thiamine can be used for Wernicke-Korsakoff syndrome. A history of chronic heavy alcohol use, confusion with confabulation, ataxia, memory loss, and nystagmus are all concerning signs that may indicate thiamine deficiency.\n\nBullet Summary:\nMeniere disease should be treated initially with a low-salt diet, avoidance of triggers, and thiazide diuretics.", "link": "https://bit.ly/3PrMDbI"} {"question": "A 66-year-old man presents to the clinic for evaluation of a skin lesion on his hand. The lesion has grown rapidly over the previous 2 weeks. He endorses trauma to the area when he accidentally scraped his hand against a window and notes that there was some bleeding. His medical history is significant for chronic renal failure for which he received a renal transplant 5 years prior, as well as multiple surgeries for skin cancer. His current medications include aspirin, atorvastatin, prednisone, tacrolimus, and mycophenolate mofetil. His temperature is 98.6\u00b0F (37\u00b0C), blood pressure is 136/91 mmHg, pulse is 82/min, and respirations are 11/min. Physical exam is notable for a 2 x 2.5 cm nodular lesion of the dorsal hand as seen in Figure A. Which of the following is the most likely diagnosis?", "choicesA": "Basal cell carcinoma", "choicesB": "Keratoacanthoma", "choicesC": "Molluscum contagiosum", "choicesD": "Pyogenic granuloma", "choicesE": "Verruca vulgaris", "answer_idx": "B", "answer": "Keratoacanthoma", "explanation": "This patient presents with a rapidly enlarging, well-circumscribed, volcano-like papule with a central keratotic core consistent with the diagnosis of keratoacanthoma.\n\nKeratoacanthomas represent a well-differentiated variant of cutaneous squamous cell carcinoma. They present as a rapidly enlarging skin lesion with a characteristic hyperkeratotic core. Most lesions occur on the sun-exposed areas of the body, especially the head, neck, and extremities. Risk factors for developing a keratoacanthoma include immunosuppression, arsenic exposure, old scars or burns, xeroderma pigmentosum, or exposure to high levels of ionizing radiation. Most keratoacanthomas tend to involute within several months after initial onset; however, surgical excision can be performed because of the histologic similarity to squamous cell carcinoma. Biopsy of lesions with malignant transformation will show atypical keratinocytes and malignant cells, invasion into dermis, and keratin \u201cpearls\u201d on histology.\n\nKwiek and Schwartz present a review of keratoacanthoma diagnosis and treatment. They discuss how these lesions characteristically exhibit spontaneous regression after rapid growth. They recommend surgical excision as first-line therapy for the lesion.\n\nFigure/Illustration A is a clinical photograph showing a well-circumscribed, dome-shaped papule with a hyperkeratotic core (red circle) consistent with keratoacanthoma.\n\nIncorrect Answers:\nAnswer A: Basal cell carcinoma is the most common skin cancer in humans. It most commonly presents as a pearly pink papule or ulcer with telangiectasias and a rolled border. Compared to keratoacanthoma, basal cell carcinomas are slow-growing and are only locally invasive. Surgical excision is the treatment of choice for most basal cell carcinomas.\n\nAnswer C: Molluscum contagiosum is a common viral infection caused by a poxvirus that is most frequently seen in children. It presents with solitary or multiple umbilicated papules and is spread by person-to-person contact. Immunosuppressed individuals, such as those with HIV/AIDS, may have disseminated molluscum contagiosum, which is highly recalcitrant to treatment.\n\nAnswer D: Pyogenic granulomas are vascular proliferations that present as rapidly growing, pink to red, friable papules that are prone to bleeding with minor trauma. They are most commonly seen in children and pregnant women. The oral mucosa is a common area of presentation, although pyogenic granulomas may present in any skin location. Treatment is through mechanical removal via surgical excision, laser ablation, or electrical curettage.\n\nAnswer E: Verruca vulgaris is a common viral infection of the skin caused by several subtypes of human papilloma virus (HPV). They present as rough-textured or filiform papules. Close examination may reveal pinpoint areas of red/black, which represent thrombosed blood vessels within the wart. Treatment options for common warts include salicylic acid or cryotherapy; however, recurrence after treatment is common.\n\nBullet Summary:\nKeratoacanthomas presents as rapidly enlarging, well-circumscribed, volcano-like papules with a central hyperkeratotic core.", "link": "https://step2.medbullets.com/testview?qid=108605"} {"question": "A 72-year-old man with a history of type 2 diabetes mellitus, hypertension, and hyperlipidemia presents to his primary care provider for concerns about his cognitive decline. His wife has noticed that he has a worsening ability to organize, plan, and exhibit impulse control over the last month. The patient states that he is able to complete his activities of daily living without assistance, but has some weakness of his left upper and lower extremities that began 3 months prior. He needs reminders from his wife about daily tasks to complete. He enjoys spending time with his spouse, playing cards with his friends, and taking daily walks around his neighborhood. His temperature is 98.2\u00b0F (36.8\u00b0C), blood pressure is 149/87 mmHg, pulse is 87/min, and respirations are 12/min. Physical exam reveals an elderly man who is oriented to person, place, and time. He has 4/5 left-sided weakness of the upper and lower extremities and associated pronator drift of the left upper extremity. A mini-mental status exam reveals he is able to remember 2 out of 3 words after 4 minutes. He is able to appropriately draw a clock. A magnetic resonance imaging study of the brain is shown in Figure A. Which of the following is the most likely diagnosis?", "choicesA": "Alzheimer disease", "choicesB": "Mild cognitive impairment", "choicesC": "Normal aging", "choicesD": "Pseudodementia", "choicesE": "Vascular dementia", "answer_idx": "E", "answer": "Vascular dementia", "explanation": "This patient with cardiovascular risk factors (hypertension, type 2 diabetes mellitus, hyperlipidemia) who presents with sudden cognitive decline of executive function and neurologic symptoms consistent with vasculopathy and possible prior stroke given his neurologic exam likely has vascular dementia, which can be corroborated with magnetic resonance imaging revealing white matter focal involvement consistent with vascular infarcts.\n\nVascular dementia is a form of dementia that results from compromise of blood supply to the brain, such as minor strokes or microinfarcts to the white matter. These insults lead to worsening cognitive abilities and presents classically as a stepwise decline (sudden/sharp declines in cognitive function that occur at distinct periods of time, not gradually) in cognitive function (thinking, planning, self-control, flexibility in thought). This is often associated with neurologic deficits as byproducts of these strokes. Unlike other forms of dementia, this presentation is step-wise. Treatment involves optimizing medical management to treat underlying medical comorbidities (managing cardiovascular risk factors such as hypertension, type 2 diabetes mellitus, hyperlipidemia), physical therapy/rehab for stroke symptoms, and caregiver support at home. Magnetic resonance imaging reveals focal white matter changes consistent with small vascular infarcts.\n\nKhan et al. review the diagnosis and management of vascular dementia while including commentary on recent updates in the literature. They found that risk factors for vascular diseases such as stroke, high blood pressure, high cholesterol, and smoking also raise the risk of vascular dementia. They thus recommend medical management of underlying medical comorbidities such as hypertension, diabetes mellitus, or hyperlipidemia as treatments for vascular dementia.\n\nFigure/Illustration A shows magnetic resonance imaging with white matter small vessel ischemic changes (red arrows) characteristic of vascular dementia within a background of diffuse generalized atrophy.\n\nIncorrect Answers:\nAnswer A: Alzheimer disease is a form of dementia that presents with progressive forgetfulness as the first and most jarring clinical symptom, followed by worsening visual/spatial recognition, and then worsening behavioral function. It is most common in patients over the age of 75. It is not associated with focal deficits or weakness on neurological exam.\n\nAnswer B: Mild cognitive impairment is a stage between the expected cognitive decline of normal aging and forms of dementia. Patients and family members may notice problems with memory, thinking, or judgment, but they are not significant enough to cause interference with activities of daily living.\n\nAnswer C: Normal aging does not involve a noticeable decline in cognition and does not involve neurologic deficits. Declines are present, but are infrequent and do not impede activities of daily living.\n\nAnswer D: Pseudodementia is the result of untreated depression, which can present as forgetfulness in the elderly. This etiology can be distinguished by screening for signs or symptoms of depression, which might include sleep disturbance, anhedonia, lack of energy, increased or decreased appetite, fatigue, and/or loss of concentration. These symptoms are absent in this case.\n\nBullet Summary:\nVascular dementia presents with a step-wise decline in executive cognitive function and neurologic deficits in a patient with cardiovascular risk factors and is associate with MRI findings including white matter changes consistent with vascular infarcts.", "link": "https://step2.medbullets.com/testview?qid=217241"} {"question": "A 59-year-old man presents to his primary care physician for fatigue. In general, he has been in good health; however, he recently has experienced weight loss, abdominal pain, and general fatigue. He has a medical history of anxiety, diabetes, a fracture of his foot sustained when he tripped, and a recent cold that caused him to miss work for 1 week. His current medications include metformin, insulin, buspirone, vitamin D, calcium, and sodium docusate. His temperature is 99.5\u00b0F (37.5\u00b0C), blood pressure is 150/100 mmHg, pulse is 90/min, respirations are 18/min, and oxygen saturation is 98% on room air. Physical exam reveals a calm gentleman. A mild systolic murmur is heard in the left upper sternal region. The rest of the physical exam is within normal limits. Laboratory values are ordered as seen below. Hemoglobin: 12 g/dL Hematocrit: 36% Leukocyte count: 66,500/mm^3 with normal differential Platelet count: 177,000/mm^3 Leukocyte alkaline phosphatase: elevated Serum: Na+: 139 mEq/L Cl-: 100 mEq/L K+: 4.3 mEq/L BUN: 20 mg/dL Glucose: 120 mg/dL Creatinine: 1.1 mg/dL Ca2+: 10.9 mEq/L AST: 12 U/L ALT: 10 U/L Which of the following is the most likely diagnosis?", "choicesA": "Acute lymphoblastic lymphoma", "choicesB": "Chronic lymphocytic leukemia", "choicesC": "Chronic myeloid leukemia", "choicesD": "Leukemoid reaction", "choicesE": "Multiple myeloma", "answer_idx": "D", "answer": "Leukemoid reaction", "explanation": "This patient is presenting with leukocytosis and an elevated leukocyte alkaline phosphatase suggestive of a leukemoid reaction.\n\nA leukemoid reaction typically presents with a drastically elevated leukocyte count (typically > 50,000/mm^3) in the setting of an elevated leukocyte alkaline phosphatase score (LAP). This reaction typically occurs after a major infection and is an immunologic response to the pathogen that caused the illness. In contrast, chronic myeloid leukemia presents with a similar elevation in leukocytes but a low LAP score in addition to symptoms of general malaise, fatigue, and weight loss. The LAP score is the critical differentiating factor when comparing these 2 pathologies. Treatment for a leukemoid reaction is by addressing the underlying pathology.\n\nPortich and Faulhaber review the evidence regarding the diagnosis of leukemoid reaction. They discuss how the most common cause is infection. They recommend considering paraneoplastic syndromes in patients without evidence of infection.\n\nIncorrect Answers:\nAnswer A: Acute lymphoblastic lymphoma presents in a pediatric patient with pallor, fatigue, easy bruising, fever, bone pain, hepatomegaly, and splenomegaly. Treatment is with combination chemotherapy.\n\nAnswer B: Chronic lymphocytic leukemia can present similarly to a leukemoid reaction with non-specific systemic symptoms and lymphadenopathy but is a less likely diagnosis in this patient given his age and LAP score. Treatment is with ibrutinib and rituximab as well as other agents.\n\nAnswer C: Chronic myeloid leukemia presents with systemic symptoms, fatigue, malaise, weight loss, and abdominal pain with an elevated leukocyte count (typically > 50,000/mm^3) but a depressed LAP score. Treatment is with tyrosine kinase inhibitors.\n\nAnswer E: Multiple myeloma presents with systemic symptoms, weight loss, pathologic fractures, and hypercalcemia. It is not associated with an elevated leukocyte score and an elevated LAP score. Treatment is with steroids and chemotherapy.\n\nBullet Summary:\nBoth a leukemoid reaction and chronic myeloid leukemia present with an elevated leukocyte count (> 50,000/mm^3), but an elevated leukocyte alkaline phosphatase score is suggestive of a leukemoid reaction.", "link": "https://bit.ly/46ZosaU"} {"question": "A 65-year-old man presents to the emergency department with anxiety and intermittent palpitations. He began feeling the palpitations 3 days ago while eating dinner. He denies chest pain, shortness of breath, or loss of consciousness. He has a history of hypertension, major depressive disorder, Raynaud disease, and chronic obstructive pulmonary disease (COPD) on 2 liters of oxygen at home. Current medications include lisinopril, inhaled umeclidinium-vilanterol, and as-needed albuterol. He drinks 4 beers a day and has smoked 1 pack of cigarettes a day for 40 years. His temperature is 98.9\u00b0 F (37.2\u00b0 C), blood pressure is 130/85 mmHg, pulse is 125/min, and respirations are 16/min. Physical exam is notable for an irregular pulse and scattered end-expiratory wheezes. An echocardiogram performed 1 month ago showed a left ventricular ejection fraction of 60-65%. The patient requires 3 liters of oxygen today. An ECG is performed as seen in Figure A. Which of the following is the most appropriate treatment for the patient\u2019s tachycardia?", "choicesA": "Amiodarone", "choicesB": "Clopidogrel", "choicesC": "Digoxin", "choicesD": "Metoprolol", "choicesE": "Verapamil", "answer_idx": "E", "answer": "Verapamil", "explanation": "This patient\u2019s new-onset palpitations, irregularly irregular pulse, and absent p-waves on ECG are consistent with atrial fibrillation. In hemodynamically stable patients with a relative contraindication to \u03b2-blockers (COPD on home oxygen with increasing oxygen need), rate control with a non-dihydropyridine calcium channel blocker such as verapamil is preferred.\n\nAtrial fibrillation can present with palpitations, lightheadedness, or shortness of breath. ECG findings will be an irregularly irregular rhythm and absent P waves with an erratic baseline as shown in Illustration A. Treatment is centered on rate control. First-line agents for rate control include \u03b21 selective blockers (metoprolol and esmolol) and non-dihydropyridine calcium channel blockers (verapamil and diltiazem). In patients with COPD, non-selective \u03b2-blockers (propranolol, timolol, and sotalol) can worsen bronchoconstriction by activating \u03b22-adrenoreceptors. Other relative contraindications to \u03b2-blockers include Raynaud disease and depression (increased risk of fatigue and sexual dysfunction). In patients with relative contraindications to \u03b2-blockers, non-dihydropyridine calcium channel blockers are the preferred initial treatment option.\n\nVan Gelder et al. studied lenient (resting heart rate < 110 beats per minute) versus strict rate control (resting heart rate < 80 beats per minute) in patients with atrial fibrillation in a randomized control trial. The authors found that there was no difference in the primary composite outcome of death from cardiovascular causes, hospitalization for heart failure, stroke, bleeding, and life-threatening arrhythmias between the 2 groups. The authors recommend a lenient rate control strategy in patients with atrial fibrillation.\n\nFigure/Illustration A is an ECG lacking P waves (red arrows) and irregularly irregular RR intervals (red/blue lines), which are classic findings in atrial fibrillation.\n\nIncorrect Answers:\nAnswer A: Amiodarone is an antiarrhythmic that can be used in patients to convert atrial fibrillation to sinus rhythm. It is often suitable to use in patients with depressed left ventricular function. Amiodarone has multiple multi-system adverse effects including pulmonary toxicity, and it is not used as a first-line agent.\n\nAnswer B: Clopidogrel is an antiplatelet agent that can be used to reduce the risk of thrombosis. Although patients with atrial fibrillation are at increased risk of stroke, the preferred anticoagulation agents are warfarin or direct oral anticoagulants (apixaban).\n\nAnswer C: Digoxin can be used as a rate control agent in atrial fibrillation as it directly suppresses atrioventricular nodal conduction. Due to its narrow therapeutic window and risk of toxicity, it is used as second-line therapy when first-line agents fail.\n\nAnswer D: Metoprolol is a \u03b2-blocker that is a first-line agent for rate control in atrial fibrillation. In patients with contraindications to \u03b2-blockers, non-dihydropyridine calcium channel blockers are the treatment of choice. This patient has COPD with increasing oxygen needs making calcium channel blockers possibly a better choice.\n\nBullet Summary:\nIn hemodynamically stable patients with atrial fibrillation and relative contraindications to \u03b2-blockers, non-dihydropyridine calcium channel blockers are the treatment of choice.", "link": "https://step2.medbullets.com/testview?qid=215054"} {"question": "A 65-year-old man presents to the emergency department with sudden-onset altered mental status. He was last seen normal 1 hour ago. His wife noticed that he was confused, slurring his speech, and had notable weakness that caused him to drop his cup of coffee. The patient is confused and not able to answer any questions. Facial drooping of the left lower face is noted, and the patient cannot follow any commands. He has a past medical history of diabetes and atrial fibrillation and is currently taking warfarin. His other medications are not known. His temperature is 98.0\u00b0F (36.7\u00b0C), blood pressure is 150/90 mmHg, pulse is 115/min, and respirations are 17/min. The patient is not able to follow any commands for further neurologic evaluation but does not appear to be moving his right upper extremity. In addition to further physical exam, which of the following is the most appropriate next step in management?", "choicesA": "Alteplase", "choicesB": "CT head", "choicesC": "Fingerstick blood glucose", "choicesD": "Fresh frozen plasma and vitamin K", "choicesE": "MRI brain", "answer_idx": "C", "answer": "Fingerstick blood glucose", "explanation": "This patient is presenting with sudden onset confusion, slurred speech, asymmetric smile, and weakness in the setting of risk factors for stroke (atrial fibrillation, diabetes, and hypertension). The most important initial step in management in a patient with altered mental status and neurological deficits is to obtain a fingerstick blood glucose.\n\nThe workup of stroke is first centered on a thorough history and physical and immediately calculating an NIH stroke scale (11-domain assessment of stroke severity involving patient consciousness level, orientation, ability to follow commands, degree of gaze palsy, degree of vision loss, ataxia, motor drift, sensation, inattention, language/speech) as well as ascertaining when the patient was last known to be neurologically normal. While performing the exam and NIH stroke scale, a fingerstick blood glucose should be performed in all patients as hypoglycemia may present identically to a stroke. Severe hypoglycemia may present with lethargy, confusion, focal or global weakness, nausea, vomiting, agitation, and even seizures. If the patient is hypoglycemic, dextrose can be given which will rapidly correct the patient's underlying deficits.\n\nOhshita et al. review other possible causes of stroke-like symptoms. They note that hypoglycemia may mimic stroke and cause focal neurological signs with common findings being unilateral motor weakness with mild or moderate alteration of consciousness. They noted that all patients improved within 1 hour of glucose injection in this study, thus the recommendation to check a fingerstick blood glucose in all stroke patients.\n\nIncorrect Answers:\nAnswer A: Alteplase would be indicated if a patient presented with disabling neurologic deficits from an ischemic stroke within 4.5 hours of symptom onset only after a CT scan of the head has ruled out an intracranial bleed. The patient must have no other contraindications to thrombolytics including but not limited to recent trauma, known intracranial hemorrhage, rapidly improving neurologic deficits (spontaneously), and no other clear reversible cause (among many other contraindications).\n\nAnswer B: CT head is the most appropriate initial step in the management of an acute stroke; however, a rapid fingerstick blood glucose takes precedence as hypoglycemia can mimic a stroke and be immediately addressed thus removing the need for further workup if there is a complete resolution of symptoms with normoglycemia. While an exam is ongoing, a fingerstick blood glucose is being obtained, and neuroimaging is also often ordered to be performed immediately after.\n\nAnswer D: Fresh frozen plasma and vitamin K would be indicated in this patient if he had a hemorrhagic stroke (a CT of the head would need to confirm this) to reverse his anticoagulation with warfarin. Factor concentrate can also be used to rapidly reverse anticoagulation with warfarin.\n\nAnswer E: MRI brain would be indicated in the workup of a stroke or transient ischemic attack after all emergency interventions and labs including CT head, serum chemistries, CBC, and ECG. It delineates the extent of the stroke and gives useful prognostic and diagnostic information. It also may elucidate strokes missed on other forms of neuroimaging such as CT perfusion.\n\nBullet Summary:\nA fingerstick blood glucose should be obtained immediately in all patients with stroke-like symptoms or altered mental status.", "link": "https://step2.medbullets.com/testview?qid=216359"} {"question": "A 35-year-old man presents to his primary care physician with pain along the bottom of his foot. The patient is a long-time runner but states that the pain has been getting worse recently. When running and at rest he has a burning and aching pain along the bottom of his foot that sometimes turns to numbness. Taking time off from training does not improve his symptoms. The patient has a medical history of surgical repair of his Achilles tendon, ACL, and medial meniscus. He is currently not taking any medications. The patient lives with his wife and they both practice a vegan lifestyle. His temperature is 98.6\u00b0F (37\u00b0C), blood pressure is 114/72 mmHg, pulse is 81/min, and respirations are 12/min. On physical exam, the patient states that he is currently not experiencing any pain in his foot but rather is experiencing numbness/tingling along the plantar surface of his foot. Strength is 5/5 and reflexes are 2+ in the lower extremities. Which of the following is the most likely diagnosis?", "choicesA": "Common fibular nerve compression", "choicesB": "Herniated disc", "choicesC": "Plantar fasciitis", "choicesD": "Tarsal tunnel syndrome", "choicesE": "Vitamin B12 deficiency", "answer_idx": "D", "answer": "Tarsal tunnel syndrome", "explanation": "This patient is presenting with pain, tingling, and numbness on the plantar surface of his foot that is not associated with exertion or rest suggesting a diagnosis of tarsal tunnel syndrome.\n\nTarsal tunnel syndrome occurs due to compression of the tibial nerve as it travels through the tarsal tunnel. Compression of the nerve leads to symptoms along the plantar surface of the foot which can include pain, tingling, or numbness. These symptoms can be exacerbated/triggered by tapping on the tunnel, which lies posterior to the medial malleolus. In addition, the symptoms can occur at rest or during activity and are not improved with rest. Treatment is rest, NSAIDs, properly fitted shoes/orthotics, and in refractory cases, surgical release of the tarsal tunnel.\n\nGould reviews the evidence regarding the diagnosis and treatment of tarsal tunnel syndrome. He discusses how this syndrome represents a collection of different entities including space-filling lesions and local inflammation. He recommends definitive diagnosis prior to surgical treatment given that the syndrome is often misdiagnosed.\n\nIncorrect Answers:\nAnswer A: Common fibular nerve compression typically occurs when there is trauma/compression just inferior to the head of the fibula. This can occur when an individual wears work boots and after fractures of the fibular neck. Symptoms of common fibular nerve compression include foot drop and numbness on the dorsum of the foot. Treatment is with activity modification and a foot orthosis to prevent an equinus contracture.\n\nAnswer B: Herniated disc presents with symptoms of radiculopathy including pain that travels down the leg through the toes. Symptoms are worsened with flexion of the leg such as during the straight leg raise test. Treatment is with NSAIDs and corticosteroid injections with surgical decompression of the disc in refractory cases.\n\nAnswer C: Plantar fasciitis can present very similarly to tarsal tunnel syndrome with pain along the plantar surface of the foot. The pain is typically exacerbated by activity and improves with rest in contrast to this patient. In addition, there is an absence of burning, pain, and numbness that occur in tarsal tunnel syndrome. Tenderness will be at the medial insertion of the plantar fascia into the calcaneal tuberosity. Treatment is conservative with NSAIDs and activity modification.\n\nAnswer E: Vitamin B12 deficiency could present with symptoms of peripheral neuropathy. This is possible given this patient's vegan diet. This patient has pain only in one of his feet and he does not have symptoms of anemia such as fatigue or pallor. Treatment is with the repletion of vitamin B12.\n\nBullet Summary:\nTarsal tunnel syndrome presents with pain, numbness, tingling, and aching along the plantar surface of the foot that is not associated with activity.", "link": "https://step2.medbullets.com/testview?qid=108977"} {"question": "A 25-year-old woman presents to her primary care physician complaining of recent hair growth along her jawline, now requiring her to shave every 2 days. She has not shaved in about a week. She has also gained about 10 pounds in the last several months, and her periods have become irregular over the last year. Her last menstrual period was 3 months ago. Her temperature is 98.6\u00b0F (37.0\u00b0C), pulse is 72/min, blood pressure is 136/86 mmHg, and respirations are 13/min. Her BMI is 26 kg/m^2. Her skin exam reveals hirsutism along the jawline and acanthosis nigricans in the axillary folds. Cardiopulmonary and abdominal exams are unremarkable. A pelvic exam reveals normal external genitalia, a mobile and non-tender 6-week-sized uterus, and no adnexal masses or tenderness. Transvaginal ultrasound is performed and shown in Figure A. This patient is at increased future risk of which of the following?", "choicesA": "Endometrial carcinoma", "choicesB": "Ovarian cancer", "choicesC": "Ovarian cyst rupture", "choicesD": "Type 1 diabetes mellitus", "choicesE": "Virilization", "answer_idx": "A", "answer": "Endometrial carcinoma", "explanation": "This patient presents with hirsutism, oligomenorrhea, and multiple follicles on pelvic ultrasound, which is most consistent with polycystic ovarian syndrome (PCOS). PCOS is associated with an increased risk of endometrial carcinoma.\n\nPCOS is an acquired endocrinopathy characterized by ovarian dysfunction and high rates of amenorrhea. Patients with PCOS are at risk for endometrial carcinoma due to unopposed estrogen exposure. Anovulation results in decreased progesterone, which prevents excessive endometrial build-up and thus increases the risk of endometrial hyperplasia and subsequent carcinoma. Increased adiposity in PCOS also increases estrogen, which is aromatized from testosterone in adipose cells, and further raises the risk for endometrial carcinoma. Treatment is with weight loss, metformin, and hormonal contraceptives.\n\nKhan et al. review the genetic basis underlying PCOS. They discuss how this disease is the most common endocrinopathy in reproductive-age women resulting in anovulatory infertility. They recommend using multimodal methods to treat this disease.\n\nFigure/Illustration A shows a transvaginal ultrasound of the ovary with many peripheral follicles (red circle). These findings are characteristic of PCOS.\n\nIncorrect Answers:\nAnswer B: Ovarian cancer does not occur with increased incidence in patients with PCOS. They are at baseline risk and do not require additional screening. The rates of ovarian cancer increase with hereditary cancer syndromes such as BRCA mutation.\n\nAnswer C: Ovarian cyst rupture is not a complication of PCOS, as PCOS does not actually cause cysts (despite the name). The cystic spaces seen on ultrasound are immature follicles rather than cysts. Recurrent infections can lead to cysts and rupture.\n\nAnswer D: Type 1 diabetes mellitus is not associated with PCOS, but type 2 diabetes mellitus is. Type 2 diabetes can cause acanthosis nigricans via insulin resistance. Patients with recurrent bouts of pancreatitis are predisposed to diabetes mellitus.\n\nAnswer E: Virilization is not seen in PCOS, as the testosterone levels are elevated but not to the degree necessary to produce frank masculinization. If there is evidence of virilization, such as voice deepening or clitoromegaly the differential should be broadened to other diagnoses.\n\nBullet Summary:\nPatients with PCOS are at increased risk of endometrial carcinoma as a result of unopposed estrogen from anovulation as well as aromatization in adipose tissue.", "link": "https://bit.ly/44nTuaV"} {"question": "A 35-year-old man is brought into the emergency department by ambulance for a stab wound to the left leg near the inguinal ligament. The patient has no significant medical history, and takes no other medication. He was not stabbed or otherwise injured elsewhere. His temperature is 98.6\u00b0F (37.0\u00b0C), pulse is 130, blood pressure is 85/50, and respirations are 22. Primary and secondary survey and FAST exam are unremarkable for other associated injuries. Pressure is removed from the wound, and pulsatile bleeding from the wound is noted. Which of the following is the most appropriate next step in management?", "choicesA": "Angiography of the left lower extremity", "choicesB": "CT scan of the left lower extremity", "choicesC": "Emergent surgical intervention", "choicesD": "Observation", "choicesE": "Wound closure", "answer_idx": "C", "answer": "Emergent surgical intervention", "explanation": "This patient with pulsatile bleeding from a wound to the left thigh and hemodynamic instability has hard signs of vascular injury. Patients with penetrating wounds and hard signs of vascular injury should be managed with emergent surgical intervention.\n\nVascular injury commonly occurs as a result of penetrating trauma, such as a stab wound as in this patient. Evaluation begins with a primary and secondary trauma survey to assess for other injuries. Hard signs of vascular injury include pulsatile bleeding, expanding hematoma, and absent pulses distal to the wound. For patients with suspected vascular injury that display either hard signs or hemodynamic instability, emergent surgical intervention is the most appropriate next step in management. For those with soft signs of vascular injury such as non-pulsatile bleeding or associated bony injury, the most appropriate next step in management is CT angiography of the affected limb.\n\nBall et al. discuss penetrating trauma to the extremities. They state that extremity trauma is the most common source of vascular injury (51%), with femoral artery as the most frequently injured vessel (35%). They also state that though obtaining pulses is important, distal pulses can continue to be present in many limbs with arterial injuries and therefore may require confirmatory evaluation via doppler, CT scan, or even angiography.\n\nIncorrect Answers:\nAnswer A: Angiography of the left lower extremity may be utilized if this patient did not present with hard signs of vascular injury. However, this patient's presentation warrants emergent surgical intervention.\n\nAnswer B: CT scan of the left lower extremity may be utilized if this patient did not present with hard signs of vascular injury. This patient's presentation warrants emergent surgical intervention.\n\nAnswer D: Observation may be appropriate if this patient was hemodynamically stable, and showed no hard or soft signs of vascular injury.\n\nAnswer E: Wound closure may eventually be appropriate if this patient was hemodynamically stable, and showed no hard or soft signs of vascular injury.\n\nBullet Summary:\nFor patients with suspected vascular injury with hard signs of vascular injury the first step in management is emergent surgical intervention.", "link": "https://bit.ly/3QOZdTR"} {"question": "A 67-year-old man presents to the emergency room with difficulty speaking for the past 1 hour. He was having dinner at home with his wife when he suddenly noticed he had difficulty eating and speaking. He also noticed a new-onset weakness on his left side. His medical history is significant for type 2 diabetes mellitus, hypertension, and hyperlipidemia. His current medications include metformin, atorvastatin, and lisinopril. His temperature is 98.6\u00b0F (37\u00b0C), blood pressure is 130/80 mmHg, pulse is 70/min, and respirations are 15/min. On examination, strength is 5/5 in the right upper and lower extremities and 3/5 in the left upper and lower extremities. On cranial nerve examination, his tongue deviates to the right side. There is also decreased sensation to light touch and vibration on the left side of his body. Which of the following arteries is most likely involved in this patient\u2019s presentation?", "choicesA": "Anterior inferior cerebellar artery", "choicesB": "Anterior spinal artery", "choicesC": "Lenticulostriate branches of the middle cerebral artery", "choicesD": "Pontine perforators of the basilar artery", "choicesE": "Posterior inferior cerebellar artery", "answer_idx": "B", "answer": "Anterior spinal artery", "explanation": "This patient with hypertension, diabetes, hyperlipidemia, deviation of the tongue to the right, left-sided hemiparesis, and decreased sensation to light touch and vibration on the left side of his body most likely has a stroke of the right medial medulla. The medial medulla is supplied by paramedian branches of the vertebral arteries and the anterior spinal artery.\n\nMedial medullary syndrome (Dejerine syndrome) is usually caused by atherosclerotic occlusion of paramedian branches of the anterior spinal artery, vertebral arteries, or basilar artery which supply the medial medulla. Medial medullary structures include the lateral corticospinal tract, the medial lemniscus, and the hypoglossal nerve. Lesions to the lateral corticospinal tract lead to contralateral paresis of the upper and lower extremities. Lesions to the medial lemniscus lead to decreased vibration, proprioception, and light touch on the contralateral body. Ipsilateral hypoglossal nerve dysfunction leads to ipsilateral loss of function of the intrinsic and extrinsic muscles of the tongue including the genioglossus muscle, which protrudes the tongue forward from the root and is unopposed on the contralateral side. This pushes the tongue towards the side of the lesion, leading to ipsilateral tongue deviation. Treatment includes reperfusion via thrombolytics if the patient presents within 4.5 hours of the time last known well.\n\nKim and Han present clinical findings, radiographic findings, angiographic data, and long-term outcomes from a case series of 86 patients with medial medullary infarction. The authors found that old age and severe motor dysfunction at admission were predictive factors portending poor prognosis. The authors recommend close follow-up of these patients for the development of central post-stroke pain.\n\nIncorrect Answers:\nAnswer A: Anterior inferior cerebellar artery infarction leads to a lesion of the lateral pons, with damage to the facial nucleus, vestibular nuclei, spinothalamic tract, spinal trigeminal nucleus, sympathetic fibers, labyrinthine artery, and middle and inferior cerebellar peduncles. This presents with complete facial hemiparesis, vomiting, decreased pain and temperature sensation on the contralateral body and ipsilateral face, ipsilateral Horner syndrome, ipsilateral ataxia and dysmetria, and ipsilateral sensorineural deafness and vertigo.\n\nAnswer C: Lenticulostriate branches of the middle cerebral artery supply the internal capsule. Strokes of the posterior limb of the internal capsule lead to contralateral hemiparesis. The findings of ipsilateral tongue deviation and sensory changes are not consistent with a lesion to the internal capsule.\n\nAnswer D: Pontine perforators of the basilar artery supply the medial pons. Strokes of the medial pons lead to quadriplegia and loss of facial, mouth, and tongue movements with preserved sensorium. This patient\u2019s findings of hemiparesis, ipsilateral tongue deviation, and changes in sensorium are more suggestive of a lesion to the medial medulla, not the medial pons.\n\nAnswer E: Posterior inferior cerebellar artery infarction leads to a stroke of the lateral medulla (Wallenberg syndrome). This leads to lesions to the nucleus ambiguus, vestibular nuclei, lateral spinothalamic tract, spinal trigeminal nucleus, sympathetic fibers, and inferior cerebellar peduncle. Clinically this presents with dysphagia, hoarseness, decreased gag reflex, vomiting, vertigo, decreased pain and temperature sensation from the contralateral body and ipsilateral face, ipsilateral Horner syndrome, and ipsilateral ataxia.\n\nBullet Summary:\nOcclusion of the anterior spinal artery can lead to medial medullary syndrome, which presents with ipsilateral deviation of the tongue, contralateral paresis of the upper and lower extremities, and contralateral loss of light touch and vibration sensation.", "link": "https://bit.ly/3u27ZnQ"} {"question": "A 26-year-old man presents to his primary care physician with a 6-month history of increasing lower back pain. He first felt the pain while lifting boxes at work but thought that he had just strained a muscle. The pain appears to be worse in the mornings and after rest. Exercise and physical activity appear to temporarily make the pain better. He has taken acetaminophen and ibuprofen for the pain. He has no significant medical history and takes no medications. His temperature is 98.6\u00b0F (37.0\u00b0C), blood pressure is 115/70 mmHg, pulse is 70/min, and respirations are 12/min. On physical exam, he is found to have limited flexion and extension of his spine as well as tenderness to palpation over the insertion of his patellar tendons bilaterally. Results from laboratory tests are shown below:\n\nSerum:\nErythrocyte sedimentation rate (ESR): 61 mm/hr\nC-reactive protein (CRP): 36 mg/L (normal <10 mg/L)\n\nWhich of the following is the most accurate test for this patient's condition?", "choicesA": "Bone scan", "choicesB": "Computed tomography", "choicesC": "Magnetic resonance imaging", "choicesD": "Radiograph", "choicesE": "Ultrasound", "answer_idx": "C", "answer": "Magnetic resonance imaging", "explanation": "This young man presents with low back pain that is relieved by physical activity, decreased spinal mobility, enthesitis of his patellar tendons, and elevated ESR/CRP values most likely has ankylosing spondylitis (AS). The most accurate diagnostic test for early ankylosing spondylitis is MRI.\n\nMultiple imaging tests can diagnose ankylosing spondylitis. Radiographs of the spine will show squaring of vertebrae with vertical or marginal syndesmophytes resulting in a \"bamboo spine\" appearance. The earliest radiographic sign is the erosion of the iliac side of the sacroiliac joint, but this finding is not sensitive in the early stages of the disease. MRI is sensitive in detecting sacroiliac inflammation, which makes this the best modality for early detection of ankylosing spondylitis in young patients. Initial therapy for ankylosing spondylitis includes lifestyle modifications (exercise, smoking cessation, physical therapy) and non-steroidal anti-inflammatory drugs (NSAIDs). Therapy for patients with inadequate response to NSAIDs includes tumor necrosis factor inhibitors (etanercept, infliximab, adalimumab) and interleukin 17 inhibitors (secukinumab, ixekizumab).\n\nJung et al. studied whether signs of inflammation on MRI are correlated with bone quality in patients with ankylosing spondylitis. The authors find that bone marrow edema is negatively correlated with the trabecular bone score. The authors recommend control of active bone inflammation to prevent osteoporosis in patients with ankylosing spondylitis.\n\nIncorrect Answers:\nAnswer A: Bone scan will show inflammation in the sacroiliac joints, but this study lacks specificity for this disease compared with other inflammatory forms of arthritis.\n\nAnswer B: CT will show bony changes but not active inflammation, so it is most commonly used to diagnose cervical fractures in patients with ankylosing spondylitis. While it can support the diagnosis, it is less accurate than an MRI.\n\nAnswer D: Radiographs will show erosion of the iliac side of the sacroiliac joint, but this finding is not sensitive in the early stages of the disease when bony ankylosis has not yet occurred. Radiographs may be an appropriate initial test.\n\nAnswer E: Ultrasound is useful in evaluating muscle strains or tendon tears; however, it is not able to detect bony abnormalities and is therefore of limited use in ankylosing spondylitis.\n\nBullet Summary:\nMRI is the most sensitive and accurate test for the early detection of ankylosing spondylitis.", "link": "https://bit.ly/3PkQ1FJ"} {"question": "A 3-day-old boy is evaluated in the neonatal intensive care unit for failure to pass meconium. The patient is otherwise doing well and feeding every 2 hours. He is urinating 8-10 times per day. The patient has had 2 episodes of vomiting that were described as green in color. He was born at 39 weeks gestation to a 38-year-old G3P3 mother. The pregnancy was uncomplicated, and the patient\u2019s mother received routine prenatal care. She declined prenatal testing. One of the patient\u2019s older siblings has Down syndrome. The patient's temperature is 98.2\u00b0F (36.8\u00b0C), blood pressure is 48/32 mmHg, pulse is 164/min, and respirations are 48/min. On physical exam, the patient is in no acute distress. His abdomen is firm, non-tender, and distended with hypoactive bowel sounds. A digital rectal exam fails to relieve the obstruction. An abdominal radiograph is performed and the result can be seen in Figure A. Which of the following is the most appropriate next step in management?", "choicesA": "Abdominal CT", "choicesB": "Contrast enema", "choicesC": "Rectal suction biopsy", "choicesD": "Sweat testing", "choicesE": "Upper gastrointestinal series", "answer_idx": "B", "answer": "Contrast enema", "explanation": "This patient presents with delayed passage of meconium, bilious emesis, unsuccessful disimpaction on digital rectal exam, and dilated loops of bowel on abdominal radiograph which suggests a diagnosis of meconium ileus. The most appropriate next step in management is an enema with water-soluble contrast which may be both diagnostic and therapeutic.\n\nMeconium ileus presents with bilious emesis and failure to pass meconium in the first 48 hours of life. Abdominal radiography typically shows dilated loops of the small intestine proximal to the obstruction. Enema with water-soluble contrast (Gastrografin) is both diagnostic and therapeutic, as the contrast not only visualizes a microcolon but also relieves the obstruction via the osmotic pull of water into the lumen of the colon. Surgery may be necessary if the contrast enema fails. Patients should also be evaluated for underlying conditions such as cystic fibrosis.\n\nAdams and Stanton review the evidence regarding the diagnosis and treatment of neonatal bowel obstruction. They discuss how malrotation, jejunoileal atresia, duodenal atresia, and colonic atresia are common causes of this finding. They recommend surgical management of these conditions.\n\nFigure/Illustration A is an abdominal radiograph demonstrating dilated loops of bowel and a \u201csoap bubble\u201d appearance of meconium mixed with bowel gas in the right lower quadrant (red circle). These findings are classically seen in patients with meconium ileus.\n\nIncorrect Answers:\nAnswer A: An abdominal radiograph would be faster but less accurate than an abdominal CT. The use of CT should also be restricted in the pediatric population due to the radiation exposure involved.\n\nAnswer C: Rectal suction biopsy is the most accurate test to diagnose Hirschsprung disease, which may present like meconium ileus with bilious vomiting, delayed passage of meconium, and dilated loops of bowel on an abdominal radiograph. A digital rectal exam typically temporarily relieves the intestinal obstruction in Hirschsprung disease and produces an expulsion of gas and stool called the \u201csquirt sign.\u201d\n\nAnswer D: Sweat testing is used to diagnose cystic fibrosis, which is strongly associated with meconium ileus. An enema with water-soluble contrast would be the best next step to both confirm the diagnosis and relieve the obstruction. The patient should subsequently undergo sweat testing.\n\nAnswer E: Upper gastrointestinal series is the diagnostic test of choice for intestinal malrotation which may also present with bilious vomiting. Malrotation is less likely to present with delayed passage of meconium, and an abdominal radiograph would more likely demonstrate a midgut volvulus. Treatment is with surgical derotation.\n\nBullet Summary:\nPatients with meconium ileus should undergo an enema with water-soluble contrast (Gastrografin) to both confirm the diagnosis and relieve the obstruction.", "link": "https://bit.ly/44eJlxH"} {"question": "A 50-year-old man presents to the emergency department with chest pain. He states that the pain is dull in quality, started 30 minutes ago, has been gradually worsening, and is worse with exertion. He also endorses some shortness of breath. He also believes the pain is worse when leaning back and improved when leaning forward. His wife noticed he fainted after the pain started but regained consciousness shortly after. He has a medical history of diabetes and hypertension for which he is on metformin and losartan. His temperature is 99.2\u00b0F (37.3\u00b0C), blood pressure is 130/87 mmHg, pulse is 99/min, respirations are 22/min, and oxygen saturation is 100% on room air. Physical exam reveals an overweight man. A normal S1 and S2 are auscultated with clear breath sounds. An ECG is performed as seen in Figure A. Which of the following is the most likely diagnosis?", "choicesA": "Hyperkalemia", "choicesB": "Hypertrophic obstructive cardiomyopathy", "choicesC": "Hypokalemia", "choicesD": "Myocardial infarction", "choicesE": "Pericarditis", "answer_idx": "D", "answer": "Myocardial infarction", "explanation": "This patient is presenting with chest pain and an ECG demonstrating hyperacute T waves (leads V2-V6) which is an early finding concerning for a myocardial infarction.\n\nA myocardial infarction occurs when an atherosclerotic plaque ruptures causing occlusion of a coronary artery. Patients classically present with chest pain, dyspnea, and diaphoresis. Atypical manifestations of an acute myocardial infarction may include abdominal pain, nausea, reflux symptoms, and dizziness. The most important initial step in management is to administer aspirin as it lowers mortality and carries minimal risk. An ECG should then be performed. Classically, an ST-elevation myocardial infarction (STEMI) will demonstrate ST elevations in a vascular distribution. However, the progression to a STEMI is usually stepwise and may present with ST depressions, hyperacute T waves, pseudonormalization of the ST segment (a normal ST segment after a depression as it elevates), ST elevations, and finally tombstoning (a concave down wave that looks like a tombstone). If a STEMI is suspected after an ECG is performed, the patient should be started on a second antiplatelet agent (e.g., ticagrelor), heparin, and sent immediately to the cardiac catheterization laboratory.\n\nNable et al. review the typical electrophysiologic changes found in STEMI. The authors note that T-wave inversions and ST-segment depression can before, during, or after the STEMI event. The authors recommend obtaining serial ECGs in patients with suspected STEMI.\n\nFigure/Illustration A demonstrates hyperacute T waves (blue arrows) that are only occurring in a vascular distribution (V2-V6).\n\nIncorrect Answers:\nAnswer A: Hyperkalemia can present with peaked T waves; however, peaked T waves would be demonstrated in all leads. Other findings of hyperkalemia can include QRS widening. Immediate management of hyperkalemia with ECG changes should include calcium administration followed by drugs that shift potassium intracellularly such as insulin, albuterol, or sodium bicarbonate. Further management may include diuresis and/or dialysis.\n\nAnswer B: Hypertrophic obstructive cardiomyopathy may demonstrate increased voltages throughout the ECG, in particular, in leads V2-V6 with deep Q waves in the septal leads. It may present with syncope upon exertion in a young patient (and in rare cases, sudden cardiac death). Ultrasound would show hypertrophy and left ventricular outflow tract obstruction. Management involves staying hydrated, avoiding strenuous activities, and beta blockers.\n\nAnswer C: Hypokalemia can present initially on ECG with U waves and may progress to cardiac dysrhythmias. Repletion with potassium is the preferred treatment, and the magnesium level should be checked as low magnesium can lead to potassium wasting in the kidneys.\n\nAnswer E: Pericarditis presents with chest pain worse by laying back and relieved by leaning forward. ECG may demonstrate diffuse ST elevations with PR depressions, and echocardiography may demonstrate a pericardial effusion. Management involves the administration of ibuprofen or colchicine and steroids in refractory cases. However, these therapies should not be used after myocardial infarction.\n\nBullet Summary:\nHyperacute T waves are an early finding that is concerning for myocardial infarction.", "link": "https://step2.medbullets.com/testview?qid=215170"} {"question": "A 31-year-old man presents to clinic for evaluation of a 3-year history of infertility. He has no significant medical history and takes no medications. He exercises twice a day, often with heavy weightlifting, and eats a high-protein diet. He reports occasional alcohol use and remote marijuana use. His temperature is 99.5\u00b0F (37.5\u00b0C), blood pressure is 148/90 mmHg, pulse is 84/min, and respirations are 15/min. He is sexually active with his wife and has no history of sexually transmitted infections. Physical exam reveals small testes and a receding hairline. He also has palpable tissue underneath his nipples bilaterally. His muscle tone is normal in his upper and lower extremities bilaterally. His laboratory values are below:\n\nSerum:\nCreatinine: 1.3 mg/dL\nGlucose: 110 mg/dL\nTotal bilirubin: 0.8 mg/dL\nAspartate aminotransferase (AST, GOT): 410 U/L\nAlanine aminotransferase (ALT, GPT): 275 U/L\n\nHemoglobin: 12 g/dL\nLeukocyte count: 5,300/mm^3\nPlatelet count: 250,000/mm^3\n\nWhich of the following is the most likely cause of this patient\u2019s infertility?", "choicesA": "Anabolic steroid use", "choicesB": "Cryptorchidism", "choicesC": "Excess growth hormone use", "choicesD": "Klinefelter syndrome", "choicesE": "Myotonic dystrophy", "answer_idx": "A", "answer": "Anabolic steroid use", "explanation": "This patient with infertility is presenting with decreased testicular size, receding hairline, gynecomastia (palpable breast tissue), and transaminitis. This is most likely due to exogenous testosterone from anabolic (androgenic) steroid use.\n\nYoung people (especially athletes and heavy weightlifters) looking to improve their physical performance may rely on the use of anabolic steroids (testosterone, boldenone, and stanozolol). These steroids adversely impair endogenous testicular function, causing infertility secondary to decreased testicular size and sperm count. Furthermore, hepatic dysfunction and cardiac dysfunction can occur at high doses. Excess testosterone is converted to estradiol, leading to breast enlargement and signs of gynecomastia. Aggression and mood disturbances may also occur in patients with excessive anabolic steroid use. Finally, skin findings of anabolic steroid use include accelerated male pattern baldness and acne. Patients should be counseled on avoiding these agents.\n\nVan Wagoner et al. study the chemical composition of products marketed as selective androgen receptor modulators (SARMs). The authors find that less than half of the analyzed products contained compounds in the amount listed on the label. The authors recommend that clinicians counsel athletes and bodybuilders that these products are often mislabeled, are not regulated, and have not been shown to be safe or effective.\n\nIncorrect Answers:\nAnswer B: Cryptorchidism is the failure of testicular descent into the scrotum and typically manifests in infancy. While uncorrected cryptorchidism can be a cause of infertility, this is an unlikely diagnosis in a man with palpable testes. Furthermore, cryptorchidism would not explain this patient\u2019s receding hairline or gynecomastia.\n\nAnswer C: Excess growth hormone use should be suspected because growth hormone can be taken exogenously to improve athletic performance and can present with hypertension. This typically also presents with hyperglycemia and enlarged hands and feet.\n\nAnswer D: Klinefelter syndrome is an inherited disorder that also presents with infertility, gynecomastia, and decreased testicular size. This disorder also causes mental impairment and would not explain the patient\u2019s elevated aminotransferases.\n\nAnswer E: Myotonic dystrophy is a multi-system disorder that can also present with testicular atrophy, reduced fertility, and hair loss. Patients with myotonic dystrophy also have widespread muscular atrophy and weakness, as well as clinical features of myotonia (e.g., difficulty releasing a handshake).\n\nBullet Summary:\nAnabolic steroid use in men can cause reduced fertility by suppressing endogenous testosterone activity, and presents with gynecomastia, testicular atrophy, accelerated male balding patterns, and hepatotoxicity.", "link": "https://step2.medbullets.com/testview?qid=215034"} {"question": "A 4-day-old girl is brought in by her mother for bilateral purulent eye discharge. Pregnancy and delivery were uncomplicated. The newborn had been sleeping regularly and feeding well. Then 1 day ago, the patient developed eyelid edema and eye redness. This morning, the newborn developed a thick yellow-white eye discharge. The mother denies that anyone in the house smokes. The family has a cat at home. The patient\u2019s rectal temperature is 99\u00b0F (37.2\u00b0C), blood pressure is 80/50 mmHg, pulse is 134/min, and respirations are 34/min with an oxygen saturation of 98% O2 on room air. Figure A shows a photograph of one of the patient's eyes. The patient's physical exam is otherwise unrevealing. Which of the following most appropriately would have prevented this infant's condition?", "choicesA": "Artificial tears", "choicesB": "Intramuscular ceftriaxone", "choicesC": "Oral erythromycin", "choicesD": "Topical bacitracin", "choicesE": "Topical erythromycin", "answer_idx": "E", "answer": "Topical erythromycin", "explanation": "This patient is presenting with bilateral purulent eye discharge 4 days post-birth, suggesting the diagnosis of gonococcal conjunctivitis. Standard prophylaxis for this condition is topical erythromycin ointment.\n\nNeonatal gonococcal conjunctivitis is caused by Neisseria gonorrhea and presents in newborns that are 0-5 days old. Symptoms include purulent discharge and eye swelling. Standard prophylaxis for gonococcal conjunctivitis is erythromycin ointment. Silver nitrate drops can also be used as prophylaxis, but this treatment has been abandoned in most practices as it can cause chemical conjunctivitis. Treatment for gonococcal conjunctivitis includes intravenous or intramuscular ceftriaxone. All neonates with gonococcal conjunctivitis should also be treated for chlamydia.\n\nFranco and Hammerschlag prevent the current methods of providing prophylaxis against gonococcal conjunctivitis. They discuss how the development of macrolide resistance may make erythromycin ointment ineffective. They recommend screening and treatment of pregnant women to prevent the development of this disease.\n\nFigure/Illustration A shows a newborn's eye that is exhibiting eyelid swelling, redness, and purulent discharge (red circle). These findings are consistent with gonococcal conjunctivitis.\n\nIncorrect Answers:\nAnswer A: Artificial tears can be used in the management of chemical conjunctivitis. The most common cause of neonatal chemical conjunctivitis is post-delivery use of silver nitrate drops. Symptoms usually present within the first 24 hours following birth/exposure as mild conjunctival injection and tearing without purulent discharge.\n\nAnswer B: Intramuscular ceftriaxone is used as treatment, not prophylaxis, of neonatal gonococcal conjunctivitis. This disease would present with purulent discharge and eye swelling. The goal of prophylaxis is to prevent the development of this condition.\n\nAnswer C: Oral erythromycin is used to treat neonatal chlamydial conjunctivitis. Symptoms of chlamydial conjunctivitis usually present 5-14 days post-birth with mild hyperemia and scant mucoid or watery discharge. Patients who are diagnosed with gonorrhea should also be treated empirically for chlamydia.\n\nAnswer D: Topical bacitracin can be used as a treatment for gram-positive neonatal bacterial conjunctivitis. It would not be used as a prophylaxis. These patients would present with rapidly progressive infection associated with surrounding soft tissue cellulitis or abscess formation.\n\nBullet Summary:\nNeonatal gonococcal conjunctivitis is treated with an intramuscular shot of ceftriaxone and standard prophylaxis to prevent this condition is with erythromycin ointment.", "link": "https://bit.ly/3LBleST"} {"question": "A 69-year-old man presents to his primary care physician with hip and back pain. He states this weekend he had a barbecue. When he was lifting a heavy object, he suddenly felt pain in his lower back. He describes the pain as in his buttocks but states that at times it travels down his leg. He states that it feels electrical and burning in nature. The patient has a medical history of obesity, diabetes, depression, anxiety, diverticulosis, constipation, and a surgical repair of his anterior cruciate ligament. His current medications include metformin, insulin, lisinopril, fluoxetine, and sodium docusate. His temperature is 98.6\u00b0F (37\u00b0C), blood pressure is 131/82 mmHg, pulse is 82/min, and respirations are 14/min. On physical exam, he is found to have an intact motor and sensory function in his lower extremities bilaterally. No clonus or Babinski sign is appreciated. An exam of the cranial nerves is intact bilaterally. Which of the following is most likely to help confirm the diagnosis?", "choicesA": "Flexion, abduction, and external rotation of the thigh", "choicesB": "Internal rotation of the extended hip", "choicesC": "Palpating the paraspinal muscles", "choicesD": "Radiography", "choicesE": "Straight leg raise", "answer_idx": "E", "answer": "Straight leg raise", "explanation": "This patient is presenting with lower back pain that is electrical and travels down his legs after lifting an object suggesting a diagnosis of a herniated disc. The straight leg test can help confirm the diagnosis.\n\nA herniated disc classically presents in a middle-aged or older individual who was lifting a heavy object with improper form. They will typically experience lower back pain that can be described as radicular. The pain will typically travel down the legs or \"shoot down\" the legs and is typically described as feeling electrical. The straight leg test on physical exam supports the diagnosis without an MRI. When the individual raises their leg, this causes compression of the spinal nerve roots leading to an exacerbation of symptoms. The most appropriate initial step in management for these patients is limiting strenuous activity, physical therapy, and NSAIDs. Patients who have chronic symptoms that have failed conservative management can undergo surgical laminectomy.\n\nCamino and Piuzzi present the data regarding the diagnostic value of the straight leg raise test. They discuss how this test can exacerbate patients with radicular symptoms. They recommend all practitioners learn how to perform and interpret this test.\n\nIncorrect Answers:\nAnswer A: Flexion, abduction, and external rotation of the thigh is referred to as the FABER test and is used to evaluate for sacroiliitis which can present with lower back pain and pain in the buttocks. This exam maneuver will cause intense pain in patients who have inflammation of the joint. Treatment is with physical therapy and NSAIDs.\n\nAnswer B: Internal rotation of the extended hip is the Freiberg test which evaluates for piriformis syndrome. Piriformis syndrome presents with lower back pain similar to a herniated disc; however, is less likely to have occurred after a single lifting episode and does not typically present with electrical pain that originates in the buttocks.\n\nAnswer C: Palpating the paraspinal muscles is a non-specific test that could further support a diagnosis of a muscle strain. It will likely not provide much information in the diagnosis of a herniated disc. Patients with muscle strain can be treated conservatively with relative rest and NSAIDs.\n\nAnswer D: Radiography is unlikely to offer information regarding this patient's herniated disc. While it can detect bony abnormalities, the detail of this imaging modality will not depict a herniated disc. An MRI is required to offer the detail necessary to make the diagnosis and is the most accurate test for a herniated disc.\n\nBullet Summary:\nA herniated disc typically presents with pain in the lower back that radiates down the legs and is exacerbated by the straight leg raise test.", "link": "https://step2.medbullets.com/testview?qid=109003"} {"question": "A 5-year-old boy presents to his primary care physician for recurrent colds, fatigue, and fussiness. Over the past several weeks, the child has been more fatigued and his parents state that \u201che always seems to be sick.\u201d They state that sometimes he complains about his bones hurting and they note that he is less playful. He is up to date on his vaccinations. His temperature is 102\u00b0F (38.9\u00b0C), blood pressure is 92/60 mmHg, pulse is 115/min, respirations are 23/min, and oxygen saturation is 99% on room air. Physical exam is notable for diffuse non-tender lymphadenopathy. Abdominal exam reveals hepatosplenomegaly. Laboratory studies are ordered as seen below.\n\nHemoglobin: 8.0 g/dL\nHematocrit: 23%\nLeukocyte count: 27,500/mm^3 with 35% lymphoblasts\nPlatelet count: 49,000/mm^3\n\nSerum:\nNa+: 139 mEq/L\nCl-: 101 mEq/L\nK+: 4.0 Eq/L\nHCO3-: 24 mEq/L\nBUN: 16 mg/dL\nGlucose: 100 mg/dL\nCreatinine: 0.7 mg/dL\n\nWhich of the following is the most likely diagnosis?", "choicesA": "Acute lymphoblastic leukemia", "choicesB": "Acute myelogenous leukemia", "choicesC": "Chronic myelogenous leukemia", "choicesD": "Leukemoid reaction", "choicesE": "Parvovirus B19 infection", "answer_idx": "A", "answer": "Acute lymphoblastic leukemia", "explanation": "This pediatric patient is presenting with recurrent colds, fatigue, non-tender lymphadenopathy, hepatosplenomegaly, and leukocytosis with increased lymphoblasts, which are concerning for acute lymphoblastic leukemia (ALL).\n\nALL is the most common hematologic cancer in pediatric patients. It initially presents with a vague constellation of symptoms including fatigue, fevers, easy bleeding/bruising, bone pain, lymphadenopathy, hepatosplenomegaly, and possibly testicular enlargement. The diagnosis is supported when the white blood cell count (which can be low, normal, or high) demonstrates increased lymphoblasts. Further confirmatory testing may include testing for the 12:21 translocation, which is more common with a better prognosis, or the 9:22 translocation (the Philadelphia chromosome), which is less common and denotes a poor prognosis.\n\nOnciu reviews ALL. He notes that ALL has a high cure rate with intensive chemotherapeutic regimens and that new research is improving outcomes in this very common condition. He recommends early treatment with these regimens for an improved prognosis.\n\nIncorrect Answers:\nAnswer B: Acute myelogenous leukemia is a malignancy of myeloblasts and is more common in the elderly. Patients will present with systemic symptoms, weight loss, anorexia, leukocytosis, anemia (causing fatigue), thrombocytopenia (causing bleeding including gingival bleeding and bleeding from the gastrointestinal tract), and neutropenia (predisposing patients to life-threatening infection). Myeloblasts produce increased amounts of myeloperoxidase which can lead to the formation of Auer rods that can be seen on peripheral smear.\n\nAnswer C: Chronic myelogenous leukemia is a neoplastic proliferation of myeloid stem cells and presents with a leukocyte count > 50,000/mm3. There are different presentations for this condition. Most patients present in the chronic phase with minimal symptoms (and a low blast count); however, they can also present with an accelerated phase and a blast phase which can lead to a hematologic emergency termed a blast crisis which may cause a hyperviscosity syndrome.\n\nAnswer D: Leukemoid reaction is an overly robust immune reaction (often to infection) that resembles chronic myelogenous leukemia. However, the leukocyte alkaline phosphatase level is elevated. This elevated level indicates it is a robust immune response rather than increased cell levels from malignancy.\n\nAnswer E: Parvovirus B19 infection presents with erythema infectiosum (fifth disease) which may include \"slapped cheek\" rash, fever, malaise, and possibly respiratory symptoms. In healthy patients, symptoms are usually minimal and self-limited. However, in patients with hemoglobinopathies (such as sickle cell disease), the transient halt in erythropoiesis that occurs in addition to hemolysis may lead to significant anemia requiring transfusion.\n\nBullet Summary:\nAcute lymphoblastic leukemia presents in pediatric patients with pancytopenia, fatigue, repeat infections, easy bleeding, bone pain, lymphadenopathy, hepatosplenomegaly, and increased blast cells on peripheral smear.", "link": "https://step2.medbullets.com/testview?qid=216615"} {"question": "A 15-year-old boy presents to his pediatrician after an appendectomy 1 week ago. The patient denies any abdominal pain, fevers, chills, nausea, vomiting, diarrhea, or constipation. He eats solids and drinks liquids without difficulty. He is back to playing basketball for his school team without any difficulty. His urine appears more amber than usual but he suspects dehydration. His temperature is 98.6\u00b0F (37\u00b0C), blood pressure is 110/70 mmHg, pulse is 76/min, and respirations are 15/min. His physical exam is unremarkable. The laparoscopic incision sites are all clean without erythema. The pediatrician orders a urinalysis, which is notable for the following. Urine: Epithelial cells: Scant Glucose: Negative Protein: 3+ WBC: 3/hpf Bacteria: None Leukocyte esterase: Negative Nitrites: Negative The patient is told to return in 3 days for a follow-up appointment; however, his urinalysis at that time is similar. Which of the following is the most appropriate next step in management?", "choicesA": "CT scan abdomen and pelvis with contrast", "choicesB": "Renal biopsy", "choicesC": "Repeat urinalysis", "choicesD": "Urine dipstick in the morning and in the afternoon", "choicesE": "Urine electrolytes and creatinine", "answer_idx": "D", "answer": "Urine dipstick in the morning and in the afternoon", "explanation": "This patient is found to have proteinuria on urinalysis in the absence of clinical symptoms or signs of nephrotic syndrome. Given the likely diagnosis of transient proteinuria, the most appropriate next step is to repeat the urine dipstick in the morning and in the afternoon.\n\nTransient proteinuria can be due to febrile illnesses, seizures, strenuous exercise, emotional stress, serious illnesses like heart failure, and abdominal surgery. A detailed workup is not needed if the clinical story is clear, but an orthostatic test can be completed. One way to do this is to assess the urine dipstick in the morning and in the afternoon. Another way requires the patient to collect urine at multiple times throughout the day. A diagnosis is confirmed if the morning sample is protein-free but the evening sample contains protein. In general, pediatric patients with orthostatic proteinuria should have under 1 gram of protein in a full 24 hours. Treatment is with reassurance because this condition usually resolves by itself.\n\nIngold and Bhatt review the evidence regarding the diagnosis and treatment of orthostatic proteinuria in patients. They discuss how this condition is benign and typically occurs in adolescents. They recommend reassurance and supportive treatment.\n\nIncorrect Answers:\nAnswer A: A CT scan of the abdomen and pelvis with contrast could assess for any anatomic obstruction and a possible post-renal cause of kidney dysfunction. It would be more prudent to obtain basic labs and a timed urinalysis before exposing a pediatric patient to radiation.\n\nAnswer B: Renal biopsy is indicated if the patient's clinical presentation is concerning for a glomerular or tubulointerstitial etiology for proteinuria. Patients will have edema, hematuria, or dysuria.\n\nAnswer C: Repeat urinalysis will not change the presence of proteinuria. Generally, a patient who has proteinuria should have a repeat urinalysis performed to rule out transient proteinuria. If the repeat urinalysis demonstrates proteinuria (as in this patient), then the diagnosis is unlikely to be transient proteinuria, and further workup is needed.\n\nAnswer E: Urine electrolytes and creatinine will assess the kidney's ability to concentrate urine and assess the etiology of acute kidney injury. Patients will present with oliguria or anuria.\n\nBullet Summary:\nConsistent proteinuria in a child should by evaluated by performing a urinalysis in the morning and in the afternoon to see if proteinuria comes on toward the end of the day.", "link": "https://step2.medbullets.com/testview?qid=210475"} {"question": "A 24-year-old woman with no significant medical history presents to the emergency department for progressive shortness of breath. She endorses decreased exercise tolerance as well as intermittent palpitations. She returned from a recent missionary trip to South America 12 months prior where she endorsed eating a local delicacy as well as a period of fevers and eyelid swelling during her stay. Her temperature is 99.1\u00b0F (37.3\u00b0C), pulse is 95/min, blood pressure is 130/85 mmHg, respirations are 15/min, and oxygen saturation is 97% on room air. Physical exam is notable for a laterally displaced point of maximal impulse as well as a 2/6 diastolic murmur radiating to the axilla. A blood smear is shown in Figure A. Her laboratory results are:\n\nLeukocyte count and differential:\nLeukocyte count: 10,000/mm^3\nNeutrophils: 57%\nLymphocytes: 30%\nMonocytes: 3%\nEosinophils: 9%\nBasophils: 1%\nHemoglobin: 13.1 g/dL\nPlatelet count: 420,000/mm^3\n\nWhich of the following is the most appropriate treatment?", "choicesA": "Amphotericin B", "choicesB": "Benznidazole", "choicesC": "Erythromycin", "choicesD": "Ganciclovir", "choicesE": "Praziquantel", "answer_idx": "B", "answer": "Benznidazole", "explanation": "This patient presents with symptoms of heart failure, eosinophilia, trypanosomes on peripheral smear, and a history of recent travel to South America consistent with the diagnosis of Chagas disease. Treatment options for Chagas disease include benznidazole and nifurtimox.\n\nChagas disease is caused by the parasite Trypanosoma cruzi and is most commonly transmitted by the reduvvid (kissing) bug. The disease is endemic to South America and presents in an acute and chronic phase. The acute phase is characterized by fever, headaches, myalgias, and unilateral eyelid swelling (Roma\u00f1a\u2019 sign) where the infectious agent has been rubbed into the eye. The chronic phase of the disease is characterized by dilated apical cardiomyopathy leading to conduction abnormalities and valvular dysfunction, megacolon, and megaesophagus. Laboratory testing reveals a peripheral eosinophilia and a peripheral blood smear may reveal the infectious trypanosomes. Treatment is with benznidazole and/or nifurtimox.\n\nPerez-Molina and Molina present the data surrounding the diagnosis and treatment of Chagas disease. They discuss how many cases progress to a chronic state characterized by cardiomyopathy, arrhythmias, and organomegaly. They recommend prompt treatment of the disease to avoid end organ damage.\n\nFigure/Illustration A shows a peripheral blood smear with the trypomastigote form of Trypanosoma cruzi (red circle). Note the anterior location of the dense basophilic nucleus.\n\nIncorrect Answers:\nAnswer A: Amphotericin B is a potent antifungal that has application in the treatment of multiple fungal infections. It is used as a first-line agent for mucormycosis and cryptococcal meningitis (in addition to flucytosine). Amphotericin B with sodium stibogluconate is used to treat visceral leishmaniasis, a protozoal infection most commonly caused by Leishmania donovani. Side effects can include nausea, vomiting, rigors, fever, hypertension or hypotension, renal dysfunction, and hypoxia.\n\nAnswer C: Erythromycin is a macrolide antibiotic that is used in the treatment of bacterial infections. It has a pro-motility effect and can also be used as a gastric pro-motility agent. Potential side effects include gastrointestinal upset, prolonged QT interval, inhibition of the P450 system, and pyloric stenosis in infants.\n\nAnswer D: Ganciclovir is a synthetic nucleoside analogue that is mainly used in the treatment of cytomegalovirus (CMV) infections. The most common side effects are bone marrow suppression (granulocytopenia, thrombocytopenia, and anemia) and gastrointestinal upset.\n\nAnswer E: Praziquantel is an anti-helminthic used in the treatment of tapeworms and flukes including Echinococcus, Taenia solium (cysticerosis), Schistosoma sp., Clonorchis sinensis (liver fluke), and Paragonimus westermani (lung fluke). Side effects of praziquantel develop due to release of contents from dead parasites and can include gastrointestinal upset, elevated liver enzymes, and dizziness.\n\nBullet Summary:\nTreatment of Chagas disease is with benznidazole or nifurtimox.", "link": "https://step2.medbullets.com/testview?qid=108604"} {"question": "A 27-year-old man presents for an appointment to establish care. He recently was released from prison. He has felt very fatigued and has had a cough. He has lost roughly 15 pounds over the past 3 weeks. He attributes this to intravenous drug use in prison. His temperature is 99.5\u00b0F (37.5\u00b0C), blood pressure is 127/68 mmHg, pulse is 100/min, respirations are 18/min, and oxygen saturation is 98% on room air. QuantiFERON gold testing is positive. The patient is started on appropriate treatment. Which of the following is the most likely indication to discontinue this patient's treatment?", "choicesA": "Elevated liver enzymes", "choicesB": "Hyperuricemia", "choicesC": "Optic neuritis", "choicesD": "Peripheral neuropathy", "choicesE": "Red body excretions", "answer_idx": "A", "answer": "Elevated liver enzymes", "explanation": "This patient is presenting with weight loss and cough with a positive quantiFERON gold test suggestive of tuberculosis and has been started on treatment. The most likely indication to discontinue tuberculosis therapy is elevation of liver enzymes.\n\nTuberculosis typically presents with weight loss, cough, and night sweats usually in high-risk patients (prisoners or the immunosuppressed). It can be diagnosed using sputum samples as these organisms will be positive on an acid-fast test. These bacteria are slow to grow so cultures need to be held for an extended period of time. QuantiFERON gold testing can be used to make the diagnosis as well. Treatment for tuberculosis is RIPE therapy: Rifampin, Isoniazid, Pyrazinamide, and Ethambutol. The most common indication to alter or discontinue therapy is an elevation of liver enzymes 5 times greater than baseline.\n\nTweed et al. review the evidence regarding liver injury in patients with RIPE therapy. They discuss how patients on HIV therapy are at higher risk. They recommend measuring liver enzymes to detect this potential side effect.\n\nIncorrect Answers:\nAnswer B: Hyperuricemia is a side-effect of pyrazinamide; however, this is not the most common indication for discontinuing RIPE therapy. Patients can be treated with supportive therapy or with uric acid lowering agents in those with arthralgias.\n\nAnswer C: Optic neuritis is a side-effect of ethambutol; however, it is not the most common indication for discontinuing RIPE therapy. Treatment is with discontinuation of ethambutol.\n\nAnswer D: Peripheral neuropathy is a side-effect of isoniazid; however, it is typically not an indication to discontinue RIPE therapy. Treatment is with supplementation of vitamin B6.\n\nAnswer E: Red body excretions are a side-effect of rifampin; however, this is a cosmetic concern rather than an indication to discontinue RIPE therapy. Patients should be counseled regarding this side effect prior to initiation of treatment.\n\nBullet Summary:\nElevation of liver enzymes 5 times above normal is the most common indication to alter or discontinue RIPE therapy when treating tuberculosis.", "link": "https://bit.ly/48TRMkR"} {"question": "A 37-year-old man presents to the emergency department for a persistent fever. The patient has felt unwell for the past week and has felt subjectively febrile. The patient has a medical history of a suicide attempt and alcohol abuse. He is not currently taking any medications. The patient admits to using heroin and cocaine and drinking 5-8 alcoholic drinks per day. His temperature is 103\u00b0F (39.4\u00b0C), blood pressure is 92/59 mmHg, pulse is 110/min, respirations are 20/min, and oxygen saturation is 96% on room air. Cardiopulmonary exam is notable for a systolic murmur heard best along the left sternal border. Dermatologic exam reveals scarring in the antecubital fossa. IV fluids are administered and the blood pressure is subsequently 120/70 mmHg. Which of the following is the most appropriate next step in management for the most likely underlying diagnosis?", "choicesA": "Blood cultures", "choicesB": "Chest radiograph", "choicesC": "CT scan", "choicesD": "Ultrasound", "choicesE": "Vancomycin and gentamicin", "answer_idx": "A", "answer": "Blood cultures", "explanation": "This patient is presenting with fever and a systolic murmur suggestive of bacterial endocarditis. The most appropriate initial step in management is to draw blood cultures.\n\nBacterial endocarditis presents with fever, systemic symptoms (aches, myalgias, and fatigue), and a new murmur typically in a patient with intravenous drug use. These patients introduce bacteria intravenously which then flows to the right side of the heart and can cause right-sided endocarditis. The most appropriate initial step in management is to draw blood cultures. Once blood cultures have been drawn, broad-spectrum antibiotics can be started (vancomycin and gentamicin), and an ultrasound can be performed to further support the diagnosis (or rule out vegetations) and plan operative interventions if necessary.\n\nWang et al. review the evidence regarding the diagnosis and treatment of infectious endocarditis. They discuss how echocardiogram and blood cultures are the mainstay of diagnosis. They recommend broad spectrum antibiotics.\n\nIncorrect Answers:\nAnswer B: A chest radiograph may be performed in the workup for bacterial endocarditis; however, it is not the most appropriate initial step in management and is neither sensitive nor specific.\n\nAnswer C: A CT scan would not be a very helpful confirmatory test in the diagnosis of bacterial endocarditis. A better confirmatory test would be an ultrasound. It may workup alternative causes of fever such as pneumonia.\n\nAnswer D: Ultrasound (echocardiography) would be the imaging test of choice for bacterial endocarditis. It would be performed after blood cultures have been drawn and antibiotics had been started.\n\nAnswer E: Vancomycin and gentamicin are the broad-spectrum antibiotics of choice for bacterial endocarditis. They would be started after blood cultures have been drawn and reduce mortality when given early in sepsis.\n\nBullet Summary:\nThe most appropriate initial step in the management of bacterial endocarditis after initial stabilization is to draw blood cultures.", "link": "https://bit.ly/3tU2wPT"} {"question": "A 32-year-old woman presents to her primary care provider with pelvic pain. She reports that for the last several years, she has had chronic pain that is worse just before her menstrual period. Over the past 2 months, she has also had worsening pain during intercourse. She denies dysuria, vaginal discharge, or vaginal pruritus. The patient has never been pregnant and previously used a copper intrauterine device (IUD) for contraception, but she had the IUD removed 1 year ago because it worsened her menorrhagia. She has now been using combined oral contraceptive pills (OCPs) for nearly 1 year. The patient reports improvement in her menorrhagia on the OCPs but denies any improvement in her pain. Her medical history is otherwise unremarkable. Her temperature is 98.0\u00b0F (36.7\u00b0C), blood pressure is 124/73 mmHg, pulse is 68/min, and respirations are 12/min. The patient has pain with lateral displacement of the cervix on pelvic exam. A pelvic ultrasound shows no abnormalities, and a urine pregnancy test is negative. Which of the following is most likely to confirm the diagnosis?", "choicesA": "Abdominal ultrasound", "choicesB": "Hysterosalpingogram", "choicesC": "Hysteroscopy", "choicesD": "Laparoscopy", "choicesE": "Pelvic MRI", "answer_idx": "D", "answer": "Laparoscopy", "explanation": "This patient presents with chronic pelvic pain that is worse before her menstrual period, vaginal tenderness on physical exam, and lateral displacement of the cervix, which is consistent with a diagnosis of endometriosis. Since this patient has failed therapy with oral contraceptive pills (OCPs), she should undergo laparoscopy.\n\nEndometriosis classically presents with chronic pelvic pain and the \u201c3 D symptoms\u201d of dysmenorrhea, dyspareunia, and dyschezia. Since laparoscopy with biopsy is required for definitive diagnosis, patients with suspected endometriosis may be empirically managed with OCPs and NSAIDs. Most patients achieve symptom control on this regimen. If patients fail empiric therapy, have contraindications to hormonal therapy, or have severe symptoms they should undergo laparoscopy for both definitive diagnosis and treatment of the disease.\n\nVercellini et al. review the evidence regarding the diagnosis and treatment of endometriosis. They discuss how the diagnosis of this disorder is carried out by direct visualization and histologic examination of lesions. They recommend medical treatment for symptomatic relief and surgical excision in refractory patients.\n\nIncorrect Answers:\nAnswer A: Abdominal ultrasound is less accurate than a transvaginal ultrasound which has already been performed. Since there were no findings on transvaginal ultrasound, it is unlikely that abdominal ultrasound would reveal pathology. This modality would be effective for intra-abdominal disorders such as cholelithiasis or appendicitis (in pediatric and pregnant patients to avoid radiation exposure).\n\nAnswer B: Hysterosalpingogram is more useful in the workup of infertility, particularly for patients with a history of pelvic inflammatory disease. It is not routinely used to diagnose endometriosis. Patients with pelvic inflammatory disease present with fever, purulent discharge, and cervical motion tenderness. Treatment is with antibiotics.\n\nAnswer C: Hysteroscopy is used in the workup of abnormal uterine bleeding, fibroids, and endometrial thickening or polyps. Since endometriosis is defined as endometrial glands and stroma outside of the uterine cavity, a hysteroscopy is not particularly helpful in making the diagnosis. Treatment for polyps or fibroids is with surgical excision.\n\nAnswer E: Pelvic MRI can be used to better characterize an abnormality seen on pelvic ultrasound or CT or to diagnose adenomyosis. It can aid in making the diagnosis of endometriosis but is not the most accurate/confirmatory test. A laparoscopy would both be diagnostic and therapeutic for this disorder.\n\nBullet Summary:\nPatients with suspected endometriosis may be managed empirically with oral contraceptive pills and NSAIDs, but laparoscopy with biopsy is necessary for definitive diagnosis and management.", "link": "https://bit.ly/42HsiTA"} {"question": "A 27-year-old man presents to his primary care physician for a checkup. He has no health concerns and has not seen a physician in years. The patient has a medical history of depression treated with fluoxetine and lithium. His temperature is 99.5\u00b0F (37.5\u00b0C), blood pressure is 122/78 mmHg, pulse is 90/min, respirations are 13/min, and oxygen saturation is 98% on room air. Physical exam is notable for the finding in Figure A. Which of the following represents the most likely risk factor for this patient's presenting condition?", "choicesA": "Alcohol consumption", "choicesB": "Antibiotic use", "choicesC": "Intravenous drug use", "choicesD": "Poor dental hygiene", "choicesE": "Sexual intercourse", "answer_idx": "A", "answer": "Alcohol consumption", "explanation": "This patient is presenting with adherent white plaques on the inside of the cheek consistent with oral leukoplakia. Alcohol consumption is a risk factor for this disease.\n\nOral leukoplakia presents with a white patch/plaque that occurs over the oral mucosa that is adherent and non-scrapable. Risk factors for this condition include the risk factors for squamous cell carcinoma such as consumption of alcohol and smoking. Though the cause of this lesion is largely idiopathic, any irritating substance can induce it. Other potential risk factors include candidiasis, vitamin deficiency, endocrine disturbances, and viral infections. A diagnosis can be made using a biopsy showing the degree of dysplasia. Treatment is with smoking and alcohol cessation as well as surgical removal for suspicious lesions.\n\nMohammed and Fairozekhan review the evidence regarding the diagnosis and risk factors for leukoplakia. They discuss how this disease is associated with smoking and alcohol use. They recommend screening patients for these modifiable risk factors.\n\nFigure/Illustration A is a clinical photograph demonstrating adherent white plaques on the inside of the cheek (red circle). These findings are consistent with oral leukoplakia.\n\nIncorrect Answers:\nAnswer B: Antibiotic use is a risk factor for candidiasis; however, this patient is not presenting with symptoms of candidiasis which is itself a risk factor for oral leukoplakia. Candidiasis presents with opaque white lesions that can be scraped off.\n\nAnswer C: Intravenous drug use does make this patient riskier overall for many conditions; however, alcohol is a more direct risk factor for oral leukoplakia. Immunodeficiency conditions can cause widespread fungal and viral infections on the mucosal surfaces.\n\nAnswer D: Poor dental hygiene may be found more often in patients with oral leukoplakia; however, it is not itself a direct cause.\n\nAnswer E: Sexual intercourse would predispose this patient to infections and viruses (such as Epstein-Barr virus); however, it is a less likely risk factor than is alcohol consumption or smoking. These infections tend to present with ulcers more commonly than leukoplakia.\n\nBullet Summary:\nRisk factors for oral leukoplakia and squamous cell carcinoma include alcohol consumption and smoking.", "link": "https://bit.ly/3FB3X8u"} {"question": "A 32-year-old man presents to the emergency department for evaluation of headache. His symptoms started last night and have persisted. He occasionally has migraine headaches, but he has not had a headache like this in the past. He reports associated nausea, vomiting, fevers, and neck pain. He has a medical history of asthma. His temperature is 100.4\u00b0F (38.0\u00b0C), blood pressure is 110/60 mmHg, pulse is 95/min, respirations are 17/min, and oxygen saturation is 98% on room air. On physical exam, he appears uncomfortable. Neurologic exam reveals no focal deficits. There is pain with passive neck flexion. No edema or rashes are noted. Lungs are clear to auscultation bilaterally. Lumbar puncture is performed and CSF results are obtained as shown below. Cerebrospinal fluid (CSF): Cell count: 175/mm^3 RBCs: 0/mm^3 Cl-: 119 mEq/L Glucose: 49 mg/dL Pressure: 150 mmH2O Total protein: 55 mg/dL Based on these results, which of the following is the most appropriate treatment regimen for this patient?", "choicesA": "Acyclovir", "choicesB": "Amphotericin and 5-flucytosine", "choicesC": "Ceftriaxone and vancomycin", "choicesD": "Ceftriaxone, vancomycin and steroids", "choicesE": "Supportive care and monitoring", "answer_idx": "E", "answer": "Supportive care and monitoring", "explanation": "This patient with headache, fever, and neck stiffness has a presentation and CSF studies consistent with a diagnosis of viral or aseptic meningitis. Supportive care is the most appropriate management strategy for patients with viral meningitis.\n\nAseptic (viral) meningitis presents similarly to bacterial meningitis with neck stiffness, photophobia, fever, malaise, and flu-like symptoms. Patients should be worked up and treated as if they have bacterial meningitis until proven otherwise. CSF findings in aseptic meningitis show an elevated cell count (though typically < 300/mm^3 in contrast to bacterial meningitis), a normal or elevated protein, and a normal glucose. CSF should be sent for Gram stain and culture to confirm the diagnosis and rule out bacterial meningitis. Once the diagnosis has been confirmed, management of viral meningitis consists of supportive care and monitoring for complications such as seizures, cerebral edema, or SIADH.\n\nKohil et. al review viral meningitis. They discuss the pathophysiology, clinical manifestation and diagnosis. They recommend that most patients with viral meningitis be treated with supportive care.\n\nIncorrect Answers:\nAnswer A: Acyclovir would be appropriate for suspected HSV or varicella encephalitis or meningitis. This patient with no RBCs in the CSF and no changes in mental status is not likely to have infection with HSV or varicella.\n\nAnswer B: Amphotericin and 5-flucytosine followed by long-term fluconazole is the appropriate management of cryptococcal meningitis. Cryptococcal meningitis presents similarly to bacterial and viral meningitis, almost exclusively in immunocompromised patients.\n\nAnswer C: Ceftriaxone and vancomycin would be appropriate if bacterial meningitis was suspected. Steroids are generally added if a bacterial source is likely. However, this patient has a presentation that is less concerning for bacterial meningitis, and CSF studies that are more consistent with aseptic meningitis.\n\nAnswer D: Ceftriaxone, vancomycin and steroids are an appropriate initial regimen for bacterial meningitis. Dexamethasone reduces inflammatory injury while patients are being treated with antibiotics.\n\nBullet Summary:\nViral or aseptic meningitis is managed with supportive care and monitoring.", "link": "https://bit.ly/49HJCwj"} {"question": "A 23-year-old woman presents with a 7-day history of abdominal pain, and now bloody diarrhea that brings her to her primary care physician. Review of systems is notable for a 12-pound unintentional weight loss and intermittent loose stools. She has a family history notable for a father with coronary artery disease and a mother with primary sclerosing cholangitis. Her temperature is 37.2\u00b0C (98.9\u00b0F), blood pressure is 100/72 mmHg, pulse is 72/min, respirations are 12/min, and oxygen saturation is 100% on room air. Her abdomen is soft, mildly diffusely tender to deep palpation, and non-distended. She is found to have the finding on colonoscopy in Figure A. Serum perinuclear antineutrophil cytoplasmic antibodies (p-ANCA) are positive. Which of the following is most likely to be present in this patient?", "choicesA": "Cobblestoning and skip lesions", "choicesB": "Continuous lesion beginning in the rectum", "choicesC": "Fistulae and stricture formation", "choicesD": "Perianal disease", "choicesE": "Worse disease severity near the ileocecal valve", "answer_idx": "B", "answer": "Continuous lesion beginning in the rectum", "explanation": "This patient presents with features suggestive of inflammatory bowel disease (IBD), and her colonoscopic and serologic studies suggest ulcerative colitis (UC). UC typically begins in the rectum and progresses proximally towards the ileocecal valve in a continuous fashion.\n\nThe diagnosis of IBD should always prompt further investigation to better delineate between UC, Crohn disease, or indeterminate colitis. Further delineation guides treatment and informs prognosis. UC has a peak incidence in the second and third decades of life. It is characterized by disease extending from the rectum proximally towards the ileocecal valve in a continuous fashion. UC presents as mucosal and submucosal inflammation, with characteristic crypt distortion and presence of crypt abscesses. P-ANCA is positive in most individuals with UC. Repeated mucosal injury can induce pseudopolyp formation, and pancolitis or disease in the descending colon warrants screening colonoscopy due to increased risk of colon cancer. On imaging, the colon can be dilated by edema, losing its haustral markings and giving it a \"lead pipe\" appearance.\n\nGajendran et al. review the pathophysiology, diagnosis, and treatment of UC. They found that colonoscopy with biopsy is the best way to make a definitive diagnosis of UC. They recommend the Mayo scoring system as a commonly used method of assessing disease severity, guiding treatment options, and monitoring patients during therapy.\n\nFigure A is a colonoscopy image showing continuous colonic mucosal inflammation typical of ulcerative colitis.\n\nIncorrect Answer:\nAnswer A: Cobblestoning and skip lesions refer to the patchy areas of inflammation noted in Crohn disease. In contrast, the inflammation seen in UC involves continuous stretches of colon mucosa.\n\nAnswer C: Fistulae and stricture formation are features of Crohn disease, not UC. Fistulizing and stricture formation can affect the entire gastrointestinal tract in Crohn disease.\n\nAnswer D: Perianal disease is a feature of Crohn disease. Although UC commonly affects the rectum, it tends to spare the anus.\n\nAnswer E: Worse disease severity near the ileocecal valve is a feature of Crohn disease and not UC. UC tends to be characterized by more uniform disease severity spanning in continuous fashion from rectum towards the cecum.\n\nBullet Summary:\nUlcerative colitis is characterized by disease extending from the rectum proximally towards the ileocecal valve in a continuous fashion.", "link": "https://bit.ly/460v5JM"} {"question": "A 71-year-old woman arrives to clinic with sudden vision loss. She was in her usual state of health until waking up this morning unable to see out of her right eye. She denies pain, but reports her right eye \u201cfeels funny.\u201d The patient\u2019s medical history is significant for a previous myocardial infarction, hypertension, and osteoporosis. Her current medications include aspirin, metoprolol, rosuvastatin, lisinopril, and alendronate. Her temperature is 98\u00b0F (36.7\u00b0C), blood pressure is 145/86 mmHg, pulse is 62/min, and respirations are 12/min with an oxygen saturation of 98% on room air. The patient's pupils are symmetric in size and equally reactive to light with accommodation. A fundoscopic exam of the right eye is shown in Figure A. The left optic fundus is unremarkable. Which of the following is the most likely diagnosis?", "choicesA": "Central retinal artery occlusion", "choicesB": "Closed angle glaucoma", "choicesC": "Papilledema", "choicesD": "Retinal detachment", "choicesE": "Retinal vein occlusion", "answer_idx": "E", "answer": "Retinal vein occlusion", "explanation": "This patient is presenting with sudden, painless, unilateral vision loss with a fundoscopic exam positive for venous dilation and retinal hemorrhage. These findings are consistent with the diagnosis of retinal vein occlusion.\n\nRetinal vein occlusion can occur in either central or branch retinal veins and is often caused by compression from nearby arterial atherosclerosis. It presents as acute onset, painless vision loss, normally in one eye. On fundoscopic exam, classic findings of this disease include venous engorgement, retinal hemorrhage, cotton wool exudates, and edema/swelling of the optic disk can be seen. There is no definitive treatment for this disorder but symptomatic treatment includes intravitreal glucocorticoid injections and retinal laser photocoagulation. Retinal vein occlusion may resolve spontaneously or progress to permanent vision loss.\n\nRobinson and Halpern present evidence regarding the treatment of retinal vein occlusion. They discuss how the treatments for the acute phase of this disease have been disappointing. They recommend managing long-term complications such as neovascularization with phototherapy.\n\nFigure/Illustration A is a fundoscopic exam showing engorged retinal veins, optic disk swelling, and widespread retinal hemorrhages (red circle). These findings are characteristic of central retinal vein occlusion.\n\nIncorrect Answers:\nAnswer A: Central retinal artery occlusion can present as sudden onset, unilateral vision loss. Fundoscopic exam will show a \u201ccherry-red\u201d spot at the fovea and a \u201cfuzzy\u201d retina with attenuated vessels. Embolic sources may include carotid artery atherosclerosis or cardiac vegetation. Treatment is with risk factor control.\n\nAnswer B: Closed-angle glaucoma presents as sudden, painful, vision loss caused by increased intraocular pressure. The eye will appear red and often feel \u201chard.\u201d The optic disc will show characteristic cupping. Intraocular pressure will be levated and treatment is with acetazolamide, steroids, timolol, bromidine, and pilocarpine. Irodotomy is reserved for refractory cases.\n\nAnswer C: Papilledema may present as an enlargement in the blind spot and is usually bilateral, as it is caused by increased intracranial pressure. On fundoscopic exam, there will be bilateral optic disk swelling, seen as an elevated optic disk and blurred margins. Patients should undergo a neurologic evaluation to determine the etiology of the papilledema.\n\nAnswer D: Retinal detachment presents as monocular loss of vision, often described as a \u201ccurtain pulled down,\u201d and preceded by \u201cflashers\u201d and \u201cfloaters.\u201d On fundoscopic exam, crinkling of the retinal tissue and abrupt changes in vessel direction can be seen. Surgical reattachment of the retina is required to prevent loss of vision.\n\nBullet Summary:\nRetinal vein occlusion presents with a sudden, monocular, painless loss of vision with optic disk swelling, dilated veins, cotton wool spots, and retinal hemorrhages.", "link": "https://step2.medbullets.com/testview?qid=108987"} {"question": "A 23-year-old woman presents to her psychiatrist concerned about her mood. She has felt tired and unwilling to engage in any activities lately. She states that her limbs feel heavy all the time and that completing any activity takes tremendous effort. She no longer finds any happiness in activities that she previously enjoyed. She struggles to sleep and at times can't sleep for several days. The patient is started on appropriate first-line therapy and sent home. She returns 1 week later stating that her symptoms have not improved. She is requesting help as her performance at work and school is suffering. Her temperature is 99.5\u00b0F (37.5\u00b0C), blood pressure is 115/72 mmHg, pulse is 60/min, respirations are 13/min, and oxygen saturation is 98% on room air. Which of the following is the most appropriate next step in management?", "choicesA": "Add lithium to treatment regimen", "choicesB": "Change treatment to duloxetine", "choicesC": "Change treatment to lithium", "choicesD": "Continue current therapy", "choicesE": "Electroconvulsive therapy", "answer_idx": "D", "answer": "Continue current therapy", "explanation": "This patient is presenting with depression and should be treated with a selective serotonin reuptake inhibitor (SSRI) for at least 4 weeks. Patients who have tried the therapy for a shorter duration of time may not yet have experienced the benefit of the medication.\n\nDepression presents with SIGE CAPS (Sleeplessness, Interest loss, Guilt, Energy decreased, Concentration abnormalities, Appetite changes, Psychomotor retardation, and Suicidal ideation). The most appropriate initial step in management is to treat the patient with an SSRI for at least 4 weeks since SSRI's take several weeks to reach their potential. If the treatment has failed after 4 weeks of use, altering the patient's medications could be appropriate. Treatment and side effects should be monitored by a physician.\n\nKato et al. review the evidence regarding the treatment of depression with SSRI medications. They discuss how medications should be used for at least 4 weeks to determine treatment response. They recommend not allowing patients to switch between drug classes too early.\n\nIncorrect Answers:\nAnswer A: Adding lithium to the treatment regimen could be an appropriate treatment if this patient had failed to respond to therapy after 4 weeks of using the drug. Lithium is an augmenting agent that could potentiate the effect of an SSRI.\n\nAnswer B: Changing the treatment to duloxetine would be inappropriate as this patient has not taken the first-line SSRI for at least 4 weeks. Patients should try a full treatment course in order to determine if they may derive some benefit.\n\nAnswer C: Changing the treatment to lithium would be appropriate if this patient's diagnosis was bipolar disorder, which would present with episodes of depression and mania. Altered sleep alone does not indicate mania.\n\nAnswer E: Electroconvulsive therapy is the most effective treatment for depression; however, it is typically a last-line treatment. This therapy can be useful in refractory cases.\n\nBullet Summary:\nSSRI medications are the most appropriate first-line agent for depression and should be tried for at least 4 weeks before changing therapy.", "link": "https://bit.ly/47eVBPi"} {"question": "A 72-year-old man presents to the clinic for his annual exam. For the past few months, he has pain in his chest with physical activity. The pain goes away after he takes a break. He reports no chest pain while he is resting, and the pain is not worsening. His past medical history is notable for hypertension, type 2 diabetes mellitus, and hyperlipidemia. His medications include amlodipine, atorvastatin, and metformin. His temperature is 36.5\u00b0C (97.7\u00b0F), blood pressure is 132/80 mmHg, pulse is 74/min, respirations are 14/min, and oxygen saturation is 98% on room air. He has a regular rate and rhythm, normal S1 and S2, and no murmurs, rubs, or gallops. His anterior, lateral, and posterior chest are non-tender to palpation. His electrocardiogram is shown in Figure A. Troponin I level is < 0.017 ng/mL. Which of the following is the most likely diagnosis?", "choicesA": "Costochondritis", "choicesB": "Non-ST elevation myocardial infarction", "choicesC": "ST elevation myocardial infarction", "choicesD": "Stable angina", "choicesE": "Unstable angina", "answer_idx": "D", "answer": "Stable angina", "explanation": "This patient who presents with exertional chest pain that is relieved with rest without increasing frequency or intensity of chest pain, a normal electrocardiogram (ECG), and normal troponin level most likely has stable angina.\n\nAngina pectoris is chest pain secondary to myocardial ischemia and is most commonly caused by atherosclerosis, though it can be caused by any etiology leading to narrowing of the coronary arteries. The pain tends to be substernal, dull, squeezing, or pressure-like and can also radiate to the neck or arm. It may also be associated with shortness of breath, dizziness, lightheadedness, nausea/vomiting, or diaphoresis. In stable angina, the chest pain is precipitated by exertion or stress and is consistently relieved with rest or nitroglycerin. ECG tends to be normal, and troponin I levels are not elevated. Treatment of stable angina includes beta blockers and nitrates (for episodes of pain), as well as therapy to prevent disease progression disease progression which may include antiplatelet therapy (aspirin), lipid-lowering medications, and glycemic control. Lifestyle measures are also the mainstay of therapy and may include smoking cessation, exercise, and dietary modification.\n\nJoshi and de Lemos review stable angina. They note the optimal management of this condition. They recommend lifestyle interventions, lipid-lowering therapy, and antiplatelets to reduce morbidity and mortality.\n\nFigure A shows an electrocardiogram with regular rate and rhythm. Every P wave is followed by a narrow QRS complex, which is consistent with normal sinus rhythm.\n\nIncorrect Answers:\nAnswer A: Costochondritis presents with pain of the chest wall that can be sharp, aching, or pressure-like and typically worsens with cough or deep breathing. It is typically caused by injury to the chest, physical strain, or can be associated with inflammatory conditions such as arthritis. On exam, the pain can be reproduced with palpation of the chest wall, in particular, the costochondral junction. This patient's chest pain is associated with exertion and relieved with rest, and his pain is not reproduced upon palpation of the chest wall. Thus, he is unlikely to have costochondritis.\n\nAnswer B: Non-ST elevation myocardial infarction presents similarly to unstable angina with substernal chest pain that is new-onset, accelerating, or occurring at rest, without ST elevations on ECG. However, ST depressions or T wave inversions may be present on ECG, and troponin I and creatine kinase-MB isoenzyme (CK-MB) would be elevated, indicating myocardial injury.\n\nAnswer C: ST-elevation myocardial infarction presents with acute onset substernal chest pain that can radiate to the neck, arm, shoulders, or jaw and is sometimes associated with diaphoresis, nausea/vomiting, shortness of breath, lightheadedness, or anxiety. ECG shows ST elevations in a vascular distribution, and troponin I and CK-MB levels are elevated. This patient's ECG does not show ST elevations, and his symptoms are more consistent with stable angina.\n\nAnswer E: Unstable angina is characterized by substernal chest pain occurring at rest or occurring with more severity, longer duration, or increasing frequency. ECG may be normal or show ST depression or T wave inversion, and the troponin level is not elevated. It is caused by incomplete coronary artery occlusion by a thrombus or atherosclerosis, commonly due to a ruptured plaque with subsequent clot formation. This patient's chest pain is exertional, does not occur at rest, and is consistently relieved with rest, and thus would be consistent with stable angina.\n\nBullet Summary:\nStable angina is characterized by exertional chest pain that is not changing in quality or quantity and is relieved with rest.", "link": "https://step2.medbullets.com/testview?qid=216268"} {"question": "A 49-year-old man presents with 3 months of generalized weakness. He has had a chronic cough and a 20 pound unintentional weight loss over the past 6 months. He has a medical history significant for high blood pressure, hyperlipidemia, and seasonal allergies with no prior surgical history. He has no history of tobacco use. He moved from India 7 years prior to presentation. He currently lives in Oregon and has not left the west coast since moving. His temperature is 99\u00b0F (37.2\u00b0C), blood pressure is 100/70 mmHg, pulse is 76/min, and respirations are 16/min. His physical exam is significant for hyperpigmentation of the oral mucosa and temporal muscle wasting. Radiography of the abdomen demonstrates bilateral adrenal calcifications. Laboratory values are as follows:\n\nSerum:\nNa+: 130 mEq/L\nCl-: 96 mEq/L\nK+: 5.2 mEq/L\nHCO3-: 24 mEq/L\nMorning cortisol: 2 ug/dL\n30-minute cortisol during cosyntropin test: 8 ug/dL (normal > 18 ug/dL)\nMorning adrenocorticotropic hormone (ACTH) concentration (serum): 80 pg/mL (normal 10-60 pg/mL)\n\nWhich of the following is the most appropriate next step in management?", "choicesA": "Bilateral adrenalectomy, prednisone, and fludrocortisone", "choicesB": "Dexamethasone", "choicesC": "Hydrocortisone and immunoglobulin therapy", "choicesD": "Prednisone taper", "choicesE": "Rifamycin, isoniazid, pyrazinamide, ethambutol, and hydrocortisone therapy", "answer_idx": "E", "answer": "Rifamycin, isoniazid, pyrazinamide, ethambutol, and hydrocortisone therapy", "explanation": "This man from India presents with generalized weakness, chronic cough, unintentional weight loss, skin hyperpigmentation, hyponatremia, hyperkalemia, hypocortisolism, cosyntropin stimulation test failure, elevated ACTH, and bilateral adrenal calcifications on radiography. These symptoms and signs in this patient are most consistent with adrenal tuberculosis requiring RIPE therapy (rifamycin, isoniazid, pyrazinamide, ethambutol) as well as glucocorticoid and mineralocorticoid therapy.\n\nAdrenal insufficiency occurs when the adrenal glands do not produce enough cortisol and aldosterone. Primary adrenal insufficiency is due to dysfunction at the level of the adrenal glands themselves, as opposed to upstream factors such as the pituitary gland. Primary adrenal insufficiency in developed nations is most often caused by autoimmune disease. In developing nations, adrenal tuberculosis is a significant cause of primary adrenal insufficiency. Adrenal tuberculosis often presents with a history of tuberculosis or tuberculosis symptoms, as well as symptoms of adrenal insufficiency such as fatigue, unintentional weight loss, and hyperpigmentation. Patients often have hyponatremia, hyperkalemia, low morning cortisol, a failed cosyntropin test, and high ACTH concentrations. Radiography may demonstrate bilateral adrenal calcifications, and/or adrenal enlargement. Tuberculosis can infect all cortices of the adrenal glands bilaterally, and therefore treatment requires eradicating the infection with multi-drug therapy (RIPE therapy) as well as replacement of glucocorticoids and mineralocorticoids, which may be achieved by high-dose hydrocortisone or other combinations of steroids that together provide glucocorticoid and mineralocorticoid replacement.\n\nVinnard and Blumberg outline the signs and symptoms, histopathology, lab results, imaging, differential diagnosis, and treatments associated with adrenal tuberculosis. They recommend early recognition and treatment.\n\nIncorrect Answers:\nAnswer A: Bilateral adrenalectomy, prednisone therapy, and fludrocortisone therapy are incorrect because medical therapy should first be attempted to eradicate the infection before undergoing surgery. Bilateral adrenalectomy may be considered if there is suspicion of lung cancer with metastasis to the bilateral adrenal glands. Patients will require lifelong replacement therapy.\n\nAnswer B: Dexamethasone alone is incorrect because this patient likely has tuberculosis and should be treated for it. Additionally, this patient has signs of mineralocorticoid deficiency (hyponatremia, hyperkalemia), and patients often require mineralocorticoid therapy or a high dose of a glucocorticoid with mineralocorticoid activity (e.g., hydrocortisone). Dexamethasone has minimal to absent mineralocorticoid activity.\n\nAnswer C: Hydrocortisone therapy and immunoglobulin therapy are incorrect because this patient has adrenal tuberculosis. Although autoimmune adrenalitis can cause primary adrenal insufficiency, immunoglobulins are not used to treat this condition.\n\nAnswer D: Prednisone taper would be appropriate for acute conditions such as an asthma or COPD exacerbation. While prednisone will treat the symptoms of primary adrenal insufficiency, it will not treat active tuberculosis infection. Primary adrenal insufficiency in developed nations is most often caused by autoimmune adrenalitis (Addison disease) and presents with fatigue, unintentional weight loss, abdominal pain, hyperpigmentation, and muscle weakness. In this condition, long-term steroid therapy is indicated.\n\nBullet Summary:\nPatients with adrenal tuberculosis should be treated with rifamycin, isoniazid, pyrazinamide, ethambutol, as well as glucocorticoid and mineralocorticoid replacement.", "link": "https://step2.medbullets.com/testview?qid=216385"} {"question": "A 23-year-old man presents to the emergency department for altered mental status after a finishing a marathon. He has a medical history of obesity and anxiety and is not currently taking any medications. His temperature is 104\u00b0F (40\u00b0C), blood pressure is 147/88 mmHg, pulse is 200/min, respirations are 33/min, and oxygen saturation is 99% on room air. Physical exam reveals dry mucous membranes, hot flushed skin, and inappropriate responses to the physician's questions. Laboratory values are ordered as seen below. Hemoglobin: 15 g/dL Hematocrit: 44% Leukocyte count: 8,500/mm^3 with normal differential Platelet count: 199,000/mm^3 Serum: Na+: 165 mEq/L Cl-: 110 mEq/L K+: 4.0 mEq/L HCO3-: 20 mEq/L BUN: 30 mg/dL Glucose: 133 mg/dL Creatinine: 1.5 mg/dL Ca2+: 10.2 mg/dL AST: 12 U/L ALT: 10 U/L Which of the following is the most appropriate next step in management?", "choicesA": "50% normal saline 50% dextrose", "choicesB": "Dextrose solution", "choicesC": "Hypotonic saline", "choicesD": "Lactated ringer", "choicesE": "Normal saline", "answer_idx": "D", "answer": "Lactated ringer", "explanation": "This patient is presenting with hypernatremia and heat stroke for which the most appropriate next step in management is the administration of lactated ringer solution.\n\nHeat stroke presents with hot/flushed skin, fever, altered mental status, and hypernatremia secondary to loss of free water. Patients should immediately be cooled with external measures and started on fluids. Minor cases where the patient is cognitively intact can be treated with oral replacement with an electrolyte balanced solution (though this would be heat exhaustion and not heat stroke). Patients with altered mental status should be started on normal saline or lactated ringer, and sodium levels should be corrected slowly. Rapid correction of serum sodium or administration of hypotonic fluids can cause cerebral edema and seizures; however, this is more common in hypernatremia that has persisted for days as it has led to cerebral adaptations.\n\nTinawi reviewed the evidence regarding the use of intravenous fluids in the hospital setting. He discusses how a large volume of normal saline can result in hypernatremia and hyperchloremia. He recommends considering the risks and benefits of colloid versus crystalloid usage.\n\nIncorrect Answers:\nAnswers 1-3: 50% normal saline 50% dextrose, dextrose solution, and hypotonic saline are hypotonic solutions and would treat the hypernatremia too rapidly predisposing the patient to seizures.\n\nAnswer E: Normal saline would be a reasonable option for this patient; however, it is less desirable given that this patient is suffering from hypernatremia, hyperchloremia, and a metabolic acidosis which could be worsened by normal saline given the large sodium and chloride load of this solution.\n\nBullet Summary:\nHypernatremia and dehydration should be treated with lactated ringer and sodium levels should be corrected no more rapidly than 1 mEq/L/hour.", "link": "https://bit.ly/3Qni72e"} {"question": "A 4-year-old boy is brought in by his grandmother for worsening seizure activity. She reports that occasionally he \u201csuddenly drops\u201d when running around the house. She says that his first seizure occurred 1 year ago. Since then, he has been on valproic acid but the seizure activity remains unchanged. He was recently enrolled in daycare because she could no longer care for him full-time. Since then, the seizure activity has increased and the daycare facility also noted concerns over his lack of response to voice commands. Family history is significant for a cousin who died suddenly at a young age. His temperature is 98.6\u00b0F (37\u00b0C), blood pressure is 105/52 mmHg, pulse is 110/min, and respirations are 25/min. A physical exam is significant for lack of response to auditory stimulation. An EEG is performed, which is unrevealing. An ECG is shown in Figure A. The patient is then startled when a door is closed quickly and he suddenly loses consciousness. An ECG is quickly performed and is shown in Figure B. An electrolyte panel is drawn with the following findings:\n\nSerum:\nNa+: 142 mEq/L\nCl-: 88 mEq/dL\nK+: 3.4 mEq/L\nHCO3-: 24 mEq/L\nBUN: 11 mg/dL\nGlucose: 67 mg/dL\nCreatinine: 0.6 mg/dL\nMg2+: 1.7 mg/dL\n\nWhich of the following is the most appropriate next step in management?", "choicesA": "Administer intravenous lorazepam", "choicesB": "Implant a pacemaker", "choicesC": "Replete potassium and magnesium and start a beta-blocker", "choicesD": "Replete potassium and magnesium and start procainamide", "choicesE": "Start ethosuximide", "answer_idx": "C", "answer": "Replete potassium and magnesium and start a beta-blocker", "explanation": "The patient who presents with deafness and long QT syndrome that progressed to torsades de pointes most likely has Jervell and Lange-Nielsen syndrome. The most appropriate next step in management is to replete potassium and magnesium and then start a beta-blocker.\n\nJervell and Lange-Nielsen syndrome is an autosomal recessive disorder caused by defective potassium ion channels. This causes abnormal function of the inner ear resulting in congenital sensorineural deafness as well as abnormal myocardial conduction resulting in long QT syndrome. A prolonged QT interval increases the risk of ventricular dysrhythmias, especially torsades de pointes, which may manifest as syncope, or sudden death. These episodes are usually triggered by exertion or by being startled. Management of symptomatic congenital long QT syndromes or torsades de pointes should involve the repletion of potassium and magnesium as well as the initiation of a beta-blocker medication. If symptoms are refractory to beta-blockers, a pacemaker can be implanted to avoid cardiac arrest.\n\nAdam et al. review the evidence regarding the diagnosis and treatment of Jervell and Lange-Nielsen syndrome. They discuss how a beta blocker medication should be used for medical control of this syndrome. They recommend pacemaker placement in refractory cases.\n\nFigure/Illustration A is an ECG showing a prolonged QT interval (blue lines). This interval can be prolonged due to certain medications such as ondansetron or it can be due to long QT syndrome, which may be acquired or congenital.\n\nFigure/Illustration B is an EKG showing a polymorphic ventricular tachycardia with varying QRS complex amplitudes over time (blue curve). This feature where the QRS complexes appear to \u201ctwist\u201d around the isoelectric line is seen in torsades de pointes.\n\nIncorrect Answers:\nAnswer A: IV lorazepam is the medical treatment for pediatric status epilepticus. Status epilepticus is a seizure that lasts for greater than 5 minutes or recurrent seizures with no return to baseline. This patient has an ECG showing torsades de pointes, which explains his sudden loss of consciousness as well as a normal EEG.\n\nAnswer B: A pacemaker can be used to treat refractory dysrhythmias caused by Jervell and Lange-Nielsen syndrome, but would not be the initial step in management for this patient. Beta-blocker medications should be used prior to pacemaker placement.\n\nAnswer D: While potassium and magnesium should be repleted, procainamide is a class 1A anti-arrhythmic medication that prolongs the QT interval. This class of medication is contraindicated in patients with long QT syndrome or torsades de pointes.\n\nAnswer E: Ethosuximide is used to treat absence seizures. The loss of consciousness associated with Jervell and Lange-Nielsen syndrome can be mistaken for absence seizures, but the patient's prolonged QT interval on ECG makes Jervell and Lange-Nielsen syndrome the most likely diagnosis. Of note, valproic acid is also a medication for absence seizures.\n\nBullet Summary:\nJervell and Lange-Nielsen syndrome should be treated with repletion of potassium and magnesium as well as initiation of beta-blocker medication with pacemaker placement reserved for refractory cases.", "link": "https://bit.ly/3O2B9LC"} {"question": "A 67-year-old man presents to the emergency room with a 2 day history of progressively worsening fatigue and shortness of breath. His past medical history is significant for previous myocardial infarction, a 30-pack-year smoking history, gastroesophageal reflux disease, and poorly controlled hypertension. The patient\u2019s only home medication is omeprazole. His temperature is 98.6\u00b0F (37\u00b0C), blood pressure is 140/90 mmHg, pulse is 90/min, respirations are 30/min, and oxygen saturation is 88% on room air. Physical exam is significant for a jugular venous pressure of 15 cm, an S3 heart sound, bibasilar crackles, and 2+ pitting edema to the knees bilaterally. His abdomen is soft and non-tender. His neurological exam is nonfocal and he walks with a steady gait. An arterial blood gas shows the following:\n\npH: 7.56 (normal 7.35-7.45)\npCO2: 25 mmHg (normal 35-45 mmHg)\nHCO3: 29 mEq/L (normal 22-26 mEq/L)\npO2: 62 mmHg (normal 80-100 mmHg)\nSaO2: 87% (normal 95-100%)\n\nWhich of the following is the most appropriate treatment?", "choicesA": "Furosemide", "choicesB": "Ivabradine", "choicesC": "Metoprolol", "choicesD": "Sacubitril and valsartan", "choicesE": "Spironolactone", "answer_idx": "A", "answer": "Furosemide", "explanation": "This patient with multiple risk factors for heart failure (previous myocardial infarction, smoking history, hypertension), worsening fatigue, shortness of breath, respiratory alkalosis, and signs of fluid overload (S3 heart sound, elevated jugular venous pressure (JVP), bibasilar crackles, peripheral edema) most likely has acute decompensated heart failure (ADHF). The most appropriate initial management for patients in ADHF is a loop diuretic such as furosemide.\n\nHeart failure (HF) is a condition in which the heart does not pump effectively and can be due to either difficulty with relaxation in diastole or contraction in systole or a combination of both. Risk factors for this condition include myocardial infarction, diabetes mellitus, valvular disease, cigarette smoking, hypertension, and obesity. Left-sided HF leads to decreased forward flow into the systemic circulation and increased congestion in the pulmonary circulation which clinically manifests as shortness of breath, pulmonary edema, and crackles on lung exam. Early pulmonary edema leads to a ventilation/perfusion (V/Q) mismatch which causes hypoxia. Hypoxia leads to a compensatory increase in the respiratory rate, which leads to hypocapnia and respiratory alkalosis. The most appropriate initial treatment for patients in ADHF is a loop diuretic such as furosemide, which helps alleviate fluid overload. Other treatments for acute exacerbations may include nitroglycerin (in hypertensive patients) and BIPAP.\n\nYancy et al. discuss the most recent ACC/AHA guidelines for the management of heart failure. They review current testing and treatment regimens. Acute episodes may be managed with loop diuretics to reduce volume overload and improve symptoms.\n\nIncorrect Answers:\nAnswer B: Ivabradine inhibits the funny current, an electrical current that leads to spontaneous depolarization in pacemaker cells, leading to decreased heart rate. It reduces the risk of hospitalization in patients with chronic heart failure with reduced ejection fraction (HFrEF). However, it should not be started in ADHF due to lack of mortality benefit and should be held in ADHF patients with severe decompensation due to decreased heart rate leading to decreased cardiac output.\n\nAnswer C: Metoprolol is a beta-blocker that reduces mortality when used in patients with chronic HFrEF. However, beta-blockers can worsen decompensated HFrEF due to negative inotropy and should not be started in ADHF.\n\nAnswer D: Sacubitril and valsartan is a combination angiotensin receptor-neprilysin inhibitor (ARNI) that improves mortality in patients with chronic HFrEF. ARNIs, angiotensin-converting enzyme (ACE) inhibitors and angiotensin II receptor blockers (ARBs) should not be started in ADHF due to concern for hypotension and worsening renal function. However, either an ARNI, ACE inhibitor or ARB should be started prior to discharge.\n\nAnswer E: Spironolactone is a mineralocorticoid receptor antagonist that reduces mortality in chronic HFrEF. However, high-dose spironolactone does not improve outcomes as first-line therapy in ADHF (Anstrom et al. JAMA Cardiology 2017).\n\nBullet Summary:\nPatients with acute decompensated heart failure present with fatigue, dyspnea, bibasilar crackles, respiratory alkalosis on arterial blood gas, edema, and other signs of fluid overload.", "link": "https://step2.medbullets.com/testview?qid=216576"} {"question": "A 71-year-old man arrives to the emergency room appearing cyanotic and having weak, shallow respirations. He is brought in by his home care nurse, who reports that the patient has a history of myasthenia gravis and frequent urinary tract infections. The patient was in his normal state of health until 5 days ago when he developed a urinary tract infection. He was receiving gentamicin infusions for his infections. This morning, he experienced poor grip strength and progressive difficulty breathing. The patient\u2019s medications include pyridostigmine and aspirin, both of which he takes as prescribed. His temperature is 99.0\u00b0F (37.2\u00b0C), blood pressure is 128/78 mmHg, pulse is 92/min, and respirations are 28/min with an oxygen saturation of 86% O2 on room air. The patient has gray-blue skin, hypophonia, weak upper extremities, and normal leg strength. An arterial blood gas is drawn with results as shown below:\n\nPO2: 55 mmHg\nPCO2: 60 mmHg\npH: 7.30\n\nThe patient is intubated. Which of the following is the most appropriate next step in management?", "choicesA": "Atropine", "choicesB": "Edrophonium", "choicesC": "Neostigmine", "choicesD": "Plasmapheresis", "choicesE": "Thymectomy", "answer_idx": "D", "answer": "Plasmapheresis", "explanation": "This patient is presenting with a history of myasthenia gravis, recent exposure to an aminoglycoside, and respiratory failure, suggesting a diagnosis of myasthenic crisis. The most appropriate initial step in management is intubation and plasmapheresis.\n\nMyasthenic crisis is an exacerbation of myasthenia gravis caused by under-dosing myasthenia gravis treatments or precipitated by infection, surgery, and certain medications such as aminoglycosides and beta-blockers. Symptoms include bulbar muscle weakness and type 2 respiratory failure (inadequate alveolar ventilation). The most appropriate initial step in management is to maintain adequate respiratory function by intubation. Plasmapheresis or IVIG is used to remove acetylcholine receptor antibodies from circulation, and steroids are started for long-term therapy. Avoiding risk factors and pre-medication before procedures with acetylcholinesterase inhibitors is important for preventing repeat occurrences.\n\nRodrigues et al. review the evidence regarding the occurrence of myasthenic crisis in patients with COVID. They discuss how treatment with IVIG and plasma exchange were important in the care of these patients. They recommend also considering whether rituximab and tocilizumab may be effective in these cases.\n\nIncorrect Answers:\nAnswer A: Atropine can be used in a cholinergic crisis. A cholinergic crisis is an acute exacerbation of muscle weakness due to over-medication with cholinergic anti-cholinesterases. This disease would present with diarrhea, sweating, miosis, bronchoconstriction, and bradycardia. It rarely occurs with proper dosing of pyridostigmine.\n\nAnswer B: Edrophonium can be used in the diagnosis of myasthenia gravis. It is a cholinesterase inhibitor. In a positive test, a patient will exhibit temporary resolution of muscle weakness. In this patient\u2019s case, the diagnosis of myasthenia gravis has already been made and he also had a recent exposure to a pharmaceutical known to precipitate myasthenic crisis.\n\nAnswer C: Neostigmine is an acetylcholinesterase inhibitor used in the management of myasthenia gravis, but it is contraindicated in the initial treatment of myasthenic crisis. Acetylcholinesterase inhibitors may increase respiratory secretions, which can worsen respiratory failure.\n\nAnswer E: Thymectomy is indicated in refractory cases of myasthenia gravis (and for increased risk of thymoma). In an unstable condition such as myasthenic crisis, thymectomy should be deferred. For patients over the age of 60, thymectomy is controversial.\n\nBullet Summary:\nMyasthenic crisis can be precipitated by infection, surgery, and medications (aminoglycosides and beta-blockers) and should be managed with intubation followed by plasmapheresis or IVIG.", "link": "https://bit.ly/3N81HdG"} {"question": "A 2-year-old girl presents to the pediatrician with her mother for a routine well-child visit. Her mother is concerned that the patient is a picky eater and refuses to eat vegetables. She drinks milk with meals and has juice sparingly. She goes to sleep easily at night and usually sleeps for 11-12 hours. The patient has trouble falling asleep for naps but does nap for 1-2 hours a few times per week. She is doing well in daycare and enjoys parallel play with the other children. Her mother reports that she can walk downstairs with both feet on each step. She has a vocabulary of 10-25 words that she uses in the form of 1 word commands. She is in the 42nd percentile for height and 48th percentile for weight, which is consistent with her growth curves. Her temperature is 98.6\u00b0F (37\u00b0C), blood pressure is 92/56 mmHg, pulse is 106/min, and respirations are 23/min. On physical exam, she appears well nourished. She can copy a line and throw a ball. She can follow the command to \u201cgive me the ball and then close the door.\u201d Which of the following is abnormal in this patient?", "choicesA": "Expressive language skills", "choicesB": "Fine motor skills", "choicesC": "Gross motor skills", "choicesD": "Social and receptive language skills", "choicesE": "This child is developmentally normal", "answer_idx": "A", "answer": "Expressive language skills", "explanation": "This child is meeting her milestones in every category but expressive language. By 2 years of age, a child should have a vocabulary of over 50 words and be using 2-word phrases.\n\nAssessing whether a child is meeting developmental milestones is important for identifying delays and enabling early intervention. In the gross motor category, a 2-year-old child should be able to jump and walk both up and down stairs with both feet on each step. In the fine motor category, a child should be able to copy a line and build a tower of 6 cubes. In the expressive language category, a child should have a vocabulary of over 50 words and be using 2-word phrases. In the social category, a child should participate in parallel play and be able to follow 2-step commands. Detecting a persistent developmental delay can be important as the first step in identifying a correctable condition such as hearing loss.\n\nScharf et al. review the evidence regarding the use of developmental milestones in assessing children. They discuss how early identification of delays can allow for referral to required services. They recommend paying special attention to sensory function to avoid missing a correctable hearing deficit.\n\nIncorrect Answers:\nAnswer B: This child's fine motor skills are appropriate, as she can copy a line. She may also be able to copy a circle and make a tower of 6 cubes at this age.\n\nAnswer C: This child's gross motor skills are appropriate because she can walk downstairs with both feet on each step and throw a ball. She should also be able to jump and run at this age.\n\nAnswer D: This child's social and receptive language skills are developing appropriately since she can follow 2-step commands and engages in parallel play. She should also be able to point to pictures, body parts, and pieces of clothing when named.\n\nAnswer E: This child is not developmentally normal due to her deficits in expressive language. She should be using 2-word sentences and have a vocabulary of over 50 words. Her language should also be intelligible to strangers 50% of the time.\n\nBullet Summary:\nBy 2 years of age, a child should be able to walk up and down stairs, copy a line, follow a 2-step command, have a vocabulary of at least 50 words, and use 2-word phrases.", "link": "https://bit.ly/3Pmj3mJ"} {"question": "A 52-year-old man presents to the clinic with a left foot ulcer that he noticed 6 days ago. He has a history of obesity, hypertension, type 2 diabetes mellitus, and depression. His current medications include metformin, aspirin, rosuvastatin, lisinopril, and fluoxetine. He has a 25-pack-year smoking history and drinks 1 glass of wine a day. His temperature is 98.6\u00b0F (37.0\u00b0C), blood pressure is 115/70 mmHg, pulse is 70/min, and respirations are 12/min. On physical exam, he has a 1.2 x 1.7 cm ulcer on the plantar surface of his left metatarsal head. Which of the following tests will most appropriately assess this patient\u2019s future risk of foot ulcers?", "choicesA": "Ankle-brachial index", "choicesB": "Capillary refill time", "choicesC": "Contrast-enhanced foot magnetic resonance imaging", "choicesD": "Knee reflex testing", "choicesE": "Monofilament testing", "answer_idx": "E", "answer": "Monofilament testing", "explanation": "This patient with a history of diabetes, hypertension, and obesity now presenting with a foot ulcer most likely has a diabetic foot ulcer. He should undergo monofilament testing to assess the subsequent risk of developing foot ulcers.\n\nDiabetic neuropathy is the most common underlying cause of diabetic foot ulcers. The pathophysiology of diabetic foot ulcers is thought to be the loss of pain and pressure sensation, which leads to muscular imbalance and foot deformities. This subsequently decreases microcirculation and impairs the integrity of the skin, leading to the formation of foot ulcers. Diabetic foot ulcers, like other neuropathic ulcers, occur commonly under bony prominences in the foot. During monofilament testing, a 10-g monofilament is placed on the plantar surface of the foot at a right angle, and pressure is applied until the patient can no longer tolerate it or the filament buckles. Patients with diabetic neuropathy have a higher-pressure threshold due to loss of sensation, and this is a useful predictor of the severity of diabetic neuropathy and future foot ulcer formation. The management of diabetic neuropathy consists of tight glycemic control, weight loss, blood pressure control, exercise, preventing complications, and pain management with pharmacotherapy (serotonin-norepinephrine reuptake inhibitors, tricyclic antidepressants, or gabapentinoid antiseizure medications).\n\nTesfaye et al. study the efficacy of various first-line medications for diabetic nephropathy including amitriptyline, duloxetine, pregabalin, or gabapentin. The authors found that there were no differences between any of the medications as monotherapy. The authors recommended adding combination pharmacotherapy for patients with painful diabetic neuropathy that do not respond to initial monotherapy.\n\nIncorrect Answers:\nAnswer A: Ankle-brachial index is a calculation made in the assessment of peripheral arterial disease (PAD). While PAD can present with arterial ulcers, these are usually located at the tip of the toes and not the plantar surface. Furthermore, the ankle-brachial index does not accurately assess small vessel disease, which is associated with diabetic foot ulcers.\n\nAnswer B: Capillary refill time, when delayed, may suggest decreased limb perfusion. This is a nonspecific test, as decreased limb perfusion can be caused by peripheral arterial disease, volume depletion, or hypotension.\n\nAnswer C: Contrast-enhanced foot MRI is the most sensitive and specific test to identify soft tissue complications of acute osteomyelitis. This patient has no clinical features of osteomyelitis (fever, chills, or malaise, ulcer that probes to bone) and does not require imaging currently.\n\nAnswer D: Knee reflex testing is not useful in assessing diabetic neuropathy. While diabetic neuropathy can affect large nerve fibers in the lower extremities, it typically impairs ankle reflexes while sparing knee reflexes.\n\nBullet Summary:\nPreventative measures for diabetic foot ulcers should focus on regular foot exams with monofilament testing to assess for diabetic neuropathy.", "link": "https://step2.medbullets.com/testview?qid=215040"} {"question": "A 59-year-old man presents to the emergency department after a sudden onset of slurred speech and right upper extremity weakness. His symptoms lasted 20 minutes but by the time he arrived at the emergency department he was symptom-free. The patient denies any preceding symptoms. He has a history of diabetes, obesity, and hypertension but does not see a physician for his underlying conditions. His temperature is 99.2\u00b0F (37.3\u00b0C), blood pressure is 140/85 mmHg, pulse is 82/min, respirations are 16/min, and oxygen saturation is 100% on room air. Physical exam reveals normal vision, strength, and sensation with a stable gait. Finger-nose testing and tandem gait are unremarkable. A CT scan of the head is performed and is unremarkable. Which of the following is the most appropriate next step in management?", "choicesA": "Aspirin", "choicesB": "Atorvastatin", "choicesC": "CTA head/neck", "choicesD": "Labetalol", "choicesE": "MRI brain", "answer_idx": "A", "answer": "Aspirin", "explanation": "This patient is presenting with a transient episode of neurologic dysfunction (slurred speech and right upper extremity weakness) that has completely resolved, which is concerning for a transient ischemic attack. In the setting of his normal head CT, aspirin should be administered next.\n\nA transient ischemic attack (TIA) is a brief episode of focal neurologic dysfunction that completely resolves without any residual deficits. Patients with a TIA are at high risk for future TIAs and stroke. For this reason, management is immediately centered on a thorough neurological exam followed by a CT scan of the head to rule out a hemorrhage. Subsequently, aspirin should immediately be administered which reduces the incidence of future stroke. Patients will subsequently be discharged on aspirin in addition to other morbidity-lowering agents including a statin and tight blood pressure and glycemic control as well as a second antiplatelet agent in appropriate patients. Further workup for a TIA may include a CTA head/neck, an MRI brain, and an echocardiogram. Modification of risk factors is also critical in these patients including weight loss, smoking cessation, exercise, glycemic control, and blood pressure control.\n\nClissold et al. review transient ischemic attacks. They note that transient ischemic attacks (TIA), if untreated, carry a high risk of early subsequent stroke. They recommend, after appropriate diagnostic workup, early administration of an antiplatelet agent.\n\nIncorrect Answers:\nAnswer B: Atorvastatin is an important initial intervention in the management of patients who have TIA/stroke. However, this intervention takes weeks of consistent therapy in order to have a significant effect on morbidity, thus it would not be the most important next step in management when compared to aspirin, which is more important in reducing acute stroke risk. Further, a lipid panel is generally performed first to determine if a statin is needed.\n\nAnswer C: CTA head/neck is a part of the workup of a TIA to rule out carotid stenosis/dissection/atherosclerosis as the cause of the patient's symptoms. It is more important to give aspirin early in TIA/stroke prior to obtaining further imaging (though this study would still be performed).\n\nAnswer D: Labetalol and other blood pressure medications are important in risk factor modification to prevent future strokes/TIAs. Patients with diabetic nephropathy should be on an ACE inhibitor or angiotensin-receptor blocker which are nephroprotective. Of note, his blood pressure currently is not very elevated nor is it causing end-organ dysfunction.\n\nAnswer E: MRI brain should be performed to further characterize whether there was any ischemic injury to the brain; however, this usually will occur after a CTA head/neck and certainly after the patient receives an aspirin. It can more accurately characterize if a stroke occurred, and the extent of the injury.\n\nBullet Summary:\nThe workup of a transient ischemic attack should be centered on a thorough neurological exam, a non-contrast CT scan of the head, and administration of aspirin assuming there is no intracranial bleed.", "link": "https://step2.medbullets.com/testview?qid=216403"} {"question": "A 2-month-old girl is brought to the pediatrician by her mother for a follow-up visit. The patient was born at 29 weeks gestation to a G1P1 mother. Her postnatal course was complicated by a prolonged neonatal intensive care unit stay for neonatal respiratory distress syndrome (NRDS). Since discharge from the hospital, her mother reports that the patient has been doing well without health problems. She has been gaining weight appropriately. Her temperature is 98.6\u00b0F (37.0\u00b0C), blood pressure is 80/50 mmHg, pulse is 120/min, and respirations are 25/min. The patient's mother asks the pediatrician about potentially delaying the 2-month vaccine doses due to the baby's medical history. Which of the following is the correct vaccination plan for this infant in terms of the vaccinations below?\n- Diphtheria, Tetanus, and acellular Pertussis - DTaP\n- Hepatitis A - HAV\n- Hepatitis B - HBV\n- Haemophilus influenzae type B conjugate - HiB\n- Pneumococcal conjugate - PCV 13", "choicesA": "Administer DTaP, HAV, HBV, HiB, PCV 13, poliovirus, and rotavirus vaccines", "choicesB": "Administer DTaP, HBV, HiB, PCV 13, and poliovirus vaccines", "choicesC": "Administer DTaP, HBV, HiB, PCV 13, poliovirus, and rotavirus vaccines", "choicesD": "Delay all vaccines for 2 months due to the infant's history of NRDS", "choicesE": "Delay all vaccines for 2 months because the infant was born 2 months premature", "answer_idx": "C", "answer": "Administer DTaP, HBV, HiB, PCV 13, poliovirus, and rotavirus vaccines", "explanation": "This 2-month-old girl has a history of prematurity (born at 29 weeks gestation) complicated by a NICU stay for NRDS but is healthy at this time. It is recommended that she receive all vaccines (DTaP, HBV, HiB, PCV 13, poliovirus, and rotavirus vaccines) as scheduled during this visit.\n\nPremature infants can safely be vaccinated on the regular schedule against serious infections. A history of both prematurity and respiratory disease makes a patient more at risk for these serious infections, increasing the importance of vaccinations. The regular vaccine schedule recommends the following vaccines at 2 months of age: DTaP (inactivated), HBV (subunit), HiB (inactivated), PCV 13 (inactivated), poliovirus (inactivated), and rotavirus (live-attenuated). The first dose of the HBV (subunit) vaccine is still given during the first 24 hours of life unless the child is born with a birth weight of < 2000 grams (4 pounds 6 ounces).\n\nSaari et al. discuss the recommendations of the American Academy of Pediatrics Committee on Infectious Diseases for the immunization of preterm infants. The authors find that preterm infants are less likely to receive recommended vaccines on time. The authors recommend also administering the influenza vaccine beginning at 6 months of age during the influenza season.\n\nIncorrect Answers:\nAnswer A: Administering DTaP, HAV, HBV, HiB, PCV 13, poliovirus, and rotavirus vaccines is incorrect as this answer choice includes the vaccine for the hepatitis A virus. The first dose of this vaccine is given at 12-24 months of age.\n\nAnswer B: Administering DTaP, HBV, HiB, PCV 13, and poliovirus vaccines is incorrect as this answer choice is missing the rotavirus vaccine. Despite being premature and having a history of NRDS, live vaccines are safe to administer on schedule to this infant.\n\nAnswer D: Delaying all vaccines for 2 months due to the infant's history of NRDS is incorrect as while this child has a history of NRDS, she is currently healthy. It is safe to administer all vaccines on the regular schedule to this patient.\n\nAnswer E: Delaying all vaccines for 2 months because the infant was born 2 months premature is incorrect as it is not recommended that previously premature infants, with or without a perinatal complication such as NRDS, delay vaccines to account for the discrepancy between gestational and chronologic age.\n\nBullet Summary:\nChildren born with a history of prematurity, with or without health complications in their early life secondary to their prematurity, should receive all recommended vaccines as scheduled.", "link": "https://step2.medbullets.com/testview?qid=215043"} {"question": "A 29-year-old woman is recovering on the obstetrics floor after the vaginal delivery of 8-pound twin boys born at 42 weeks gestation. The patient is fatigued but states that she is doing well. Currently, she is complaining that her vagina hurts. The next morning, the patient experiences chills and a light red voluminous discharge from her vagina. She states that she feels pain and cramps in her abdomen. The patient's medical history is notable for diabetes which was managed during her pregnancy with insulin. Her temperature is 99.5\u00b0F (37.5\u00b0C), blood pressure is 107/68 mmHg, pulse is 97/min, respirations are 16/min, and oxygen saturation is 98% on room air. Laboratory values are obtained and shown below. Hemoglobin: 12 g/dL Hematocrit: 36% Leukocyte count: 9,750/mm^3 with normal differential Platelet count: 197,000/mm^3 Serum: Na+: 139 mEq/L Cl-: 101 mEq/L K+: 4.2 mEq/L HCO3-: 23 mEq/L BUN: 20 mg/dL Glucose: 111 mg/dL Creatinine: 1.1 mg/dL Ca2+: 10.2 mg/dL AST: 12 U/L ALT: 10 U/L Which of the following is the most appropriate next step in treatment for this patient?", "choicesA": "Cefoxitin and doxycycline", "choicesB": "Clindamycin and gentamicin", "choicesC": "Supportive therapy only", "choicesD": "Vancomycin and clindamycin", "choicesE": "Vancomycin and gentamicin", "answer_idx": "C", "answer": "Supportive therapy only", "explanation": "This patient is presenting with a normal postpartum period which is managed with supportive therapy only.\n\nA normal postpartum period is associated with breast tenderness, chills, edema, and abdominal cramps from a contracting uterus. Vaginal discharge (lochia) is normal in this phase and progresses from a red color (lochia rubra) to a clear/fluid appearance (lochia serosa) to a white color (lochia alba). No management other than supportive therapy is needed during this period. Symptoms that indicate additional intervention may be necessary include fever, foul-smelling drainage, or hypotension.\n\nFletcher et al. review the evidence regarding normal lochia after pregnancy. They discuss how this discharge can occur for up to 24-36 days after delivery. They recommend understanding normal discharge patterns to allow for the identification of pathologic conditions.\n\nIncorrect Answers:\nAnswer A: Cefoxitin and doxycycline can be used to treat postpartum endometritis in which the suspected organism is Chlamydia trachomatis or Neisseria gonorrhoea. Suspect these organisms in a patient with risky sexual behavior.\n\nAnswer B: Clindamycin and gentamicin offer broad-spectrum coverage for endometritis, which presents with fever, abdominal tenderness, and a foul-smelling vaginal discharge. This patient has no evidence of infection at this time.\n\nAnswer D: Vancomycin and clindamycin is not a typical broad-spectrum antibiotic pair that is used routinely. Both of these agents are often effective against MRSA but are typically not used concurrently.\n\nAnswer E: Vancomycin and gentamicin offer broad-spectrum antibiotic coverage and can be used for bacterial endocarditis which presents with fever and a new murmur in a high-risk individual (such as those who use IV drugs).\n\nBullet Summary:\nA normal postpartum period presents with chills, abdominal cramps, and lochia (rubra, serosa, or alba) and is managed with supportive care.", "link": "https://bit.ly/44HVD16"} {"question": "A 76-year-old woman is brought to a primary care physician by her daughter who is concerned about her mother's growing inability to take care of herself. Initially, she attributed her mother's forgetfulness and word-finding difficulties to normal aging, but over the past few years, her mother's memory has worsened, her mood has grown more irritable, and her mother has been found wandering the neighborhood unsure of how to get home. Her medical history includes type 2 diabetes mellitus which is well-controlled with metformin. On exam, her temperature is 98.2\u00b0F (36.8\u00b0C), blood pressure is 115/82 mmHg, pulse is 73/min, respirations are 12/min, and oxygen saturation is 99% on room air. Cardiopulmonary exam reveals normal S1 and S2, no murmurs, and clear lungs bilaterally. She scores 16/30 on the Montreal Cognitive Assessment (MoCA) test. Which of the following is the most likely diagnosis?", "choicesA": "Alzheimer dementia", "choicesB": "Creutzfeldt-Jakob disease", "choicesC": "Frontotemporal dementia", "choicesD": "Normal pressure hydrocephalus", "choicesE": "Vascular dementia", "answer_idx": "A", "answer": "Alzheimer dementia", "explanation": "This elderly patient who presents with slowly progressive memory loss, language difficulties, and progression to inability to care for herself most likely has Alzheimer dementia.\n\nAlzheimer dementia is a neurodegenerative disorder that is characterized by a gradual and progressive cognitive decline (memory loss, visuospatial and language deficits, impaired judgment), difficulties with executive function or carrying out activities of daily living, and sometimes psychiatric disturbances and behavioral changes (apathy, irritability, social isolation). It is the most common cause of dementia and its risk factors include increasing age, female gender, family history, and trisomy 21 (due to the amyloid precursor protein found on chromosome 21). This is a clinical diagnosis based on cognitive testing (such as the Montreal Cognitive Assessment or the mini-mental status exam) and ruling out other causes of dementia (vitamin B12 deficiency, depression, or hypothyroidism).\n\nWeller et al. review the current understanding of Alzheimer dementia pathophysiology. They discuss recent advances in clinical evaluation and treatment. They recommend having a high index of suspicion for the disease in elderly patients who appear to be regressing in their ability to take care of themselves.\n\nIncorrect Answers:\nAnswer B: Creutzfeldt-Jakob disease is a rapidly progressive neurodegenerative disorder caused by a prion protein that is fatal, leading to death usually within 1 year of illness onset. Dementia progresses rapidly in this disease and is sometimes associated with startle myoclonus.\n\nAnswer C: Frontotemporal dementia (FTD) is characterized primarily by disinhibition, apathy, or aphasia as a result of atrophy of the frontal and temporal lobes. It tends to occur at a younger age than does Alzheimer dementia. Behavioral changes are often the first noticeable symptom in FTD while they tend to occur later in Alzheimer disease progression. Problems with visuospatial orientation are more common in Alzheimer dementia than in FTD.\n\nAnswer D: Normal pressure hydrocephalus (NPH) typically presents with a triad of dementia, gait instability (specifically magnetic gait, marked by an inability to lift feet off the floor), and urinary incontinence. In NPH, excess cerebrospinal fluid accumulates in the brain's ventricles, and treatment involves drainage of that fluid, usually via surgical placement of a shunt.\n\nAnswer E: Vascular dementia typically presents as stepwise cognitive decline in the setting of stroke or brain injury and usually occurs in patients with the same risk factors as stroke, such as hypertension, hyperlipidemia, peripheral arterial disease, and smoking. This patient has neither hypertension nor stepwise decline.\n\nBullet Summary:\nAlzheimer dementia presents with slowly progressive dementia, typically manifesting with memory loss, visuospatial and language difficulties, impaired judgment, and inability to care for oneself.", "link": "https://bit.ly/3CrrhE1"} {"question": "A 33-year-old woman presents to the emergency department with foul-smelling vaginal discharge. She states that it started 1 day ago and has not improved with hygiene products. She is sexually active and uses condoms. She does not complain of any abdominal or pelvic pain but states she has some mild burning when urinating. Her temperature is 97.6\u00b0F (36.4\u00b0C), blood pressure is 111/74 mmHg, pulse is 81/min, respirations are 12/min, and oxygen saturation is 98% on room air. Physical exam is notable for thick, white discharge from the cervix. There is no cervical motion or adnexal tenderness or masses. Some whitish discharge is noted from the urethra. A urine pregnancy test is negative. Which of the following is the most appropriate treatment for this patient?", "choicesA": "Azithromycin", "choicesB": "Ceftriaxone", "choicesC": "Ceftriaxone and azithromycin", "choicesD": "Ceftriaxone and doxycycline", "choicesE": "Metronidazole", "answer_idx": "D", "answer": "Ceftriaxone and doxycycline", "explanation": "This nonpregnant woman is presenting with a thick, white cervical discharge, dysuria, and urethral discharge without cervical motion tenderness or adnexal tenderness which is concerning for urethritis without pelvic inflammatory disease. She should be treated with ceftriaxone and doxycycline to cover for the most common organisms.\n\nThe most common bacterial sexually transmitted infections include Neisseria gonorrhoeae and Chlamydia trachomatis. They can cause urethritis, cervicitis, and pelvic inflammatory disease. The diagnosis is made clinically with the appropriate history (vaginal discharge, high-risk sexual behavior) and physical exam (purulent cervical discharge +/- cervical motion tenderness, adnexal tenderness). The diagnosis should be confirmed with PCR; however, this should not delay treatment. Treatment should be given empirically, and the preferred regimen is ceftriaxone and doxycycline in men and nonpregnant women. In pregnant women, ceftriaxone and azithromycin may be used. Metronidazole is added empirically to cover for Trichomonas vaginalis.\n\nJennings and Krywko review pelvic inflammatory disease. They note that pelvic inflammatory disease is an inflammation of the upper genital tract due to an infection that can affect the uterus, fallopian tubes, and ovaries. They discuss the need for treatment and complications that can occur without treatment. They recommend timely diagnosis and treatment.\n\nIncorrect Answers:\nAnswer A: Azithromycin is only indicated to cover chlamydia in pregnancy since doxycycline cannot be given to pregnant patients. Coverage for gonorrhea is also mandatory given this patient\u2019s symptoms.\n\nAnswer B: Ceftriaxone is insufficient coverage for chlamydia or gonorrhea as chlamydia requires treatment with doxycycline, and gonorrhea requires double coverage from ceftriaxone in addition to doxycycline.\n\nAnswer C: Ceftriaxone and azithromycin is the preferred regimen to treat chlamydia and gonorrhea in pregnant patients since pregnant patients cannot receive doxycycline.\n\nAnswer E: Metronidazole is an appropriate treatment for bacterial vaginosis which presents with a fishy odor, white/gray vaginal discharge, and positive clue cells on wet mount.\n\nBullet Summary:\nThe treatment of choice for urethritis is ceftriaxone and doxycycline in nonpregnant patients.", "link": "https://step2.medbullets.com/testview?qid=216513"} {"question": "A 64-year-old woman presents to the emergency room with whole-body itching. She noticed her symptoms while in the bathtub at home. She has never had symptoms like this before. Over the previous several months she has had episodes of joint swelling and pain in her hands as well as redness, burning pain, and swelling of her hands and feet. Her medical history is significant for type 2 diabetes mellitus, hypertension, and osteoporosis for which she takes metformin, enalapril, and alendronate. She was found to have a deep vein thrombosis of her left leg 3 months prior to presentation. Her temperature is 98.6\u00b0F (37.0\u00b0C), pulse is 80/min, blood pressure is 135/85 mmHg, and respirations are 13/min. Physical exam is notable for a woman in discomfort with excoriations over the skin on her forearms. Laboratory tests are shown below.\n\nSerum:\nNa+: 135 mEq/L\nCl-: 100 mEq/L\nK+: 5.0 mEq/L\nHCO3-: 22 mEq/L\nBUN: 19 mg/dL\nGlucose: 130 mg/dL\nCreatinine: 1.0 mg/dL\n\nHematocrit: 64%\nLeukocyte count: 19,000 cells/mm^3 with normal differential\nPlatelet count: 900,000/mm^3\n\nWhich of the following is the most appropriate long-term treatment?", "choicesA": "Cyclophosphamide", "choicesB": "Diphenhydramine", "choicesC": "Febuxostat", "choicesD": "Hydroxyurea", "choicesE": "Prednisone", "answer_idx": "D", "answer": "Hydroxyurea", "explanation": "This patient presents with polycythemia, leukocytosis, and thrombocytosis in the setting of pruritus after bathing, as well as episodes suggestive of acute gout flares. These findings are consistent with polycythemia vera, which can be treated with hydroxyurea.\n\nPolycythemia vera is a malignancy of the bone marrow that results in the overproduction of red blood cells, platelets, and white blood cells. Classic symptoms include pruritus after hot baths as well as swelling, burning pain, and rubor of the hands and feet (erythromelalgia). Patients may also have gout due to increased cell turnover leading to hyperuricemia. Older patients (> 60 years old) and those with prior thrombosis should be treated with a myelosuppressive agent, most commonly hydroxyurea with or without aspirin. Therapeutic phlebotomy is often indicated on these patients.\n\nStuart and Viera review the evidence regarding the diagnosis and treatment of polycythemia vera. They discuss how treatment includes phlebotomy with the possible addition of myelosuppressive agents. They recommend consultation with a hematologist for any patients with this syndrome.\n\nIncorrect Answers:\nAnswer A: Cyclophosphamide is a chemotherapeutic agent that is used in the treatment of certain leukemias and lymphomas, as well as severe symptoms of autoimmune disease. A malignancy would present with vague systemic symptoms, weight loss, malaise, and abnormalities detected on CBC. Cyclophosphamide is not used in the treatment of polycythemia vera.\n\nAnswer B: Diphenhydramine is a 1st-generation antihistamine that can be used to treat pruritus, but would not be indicated for the treatment of polycythemia vera which is the underlying condition. This treatment represents a symptomatic treatment versus an agent that addresses the underlying cause.\n\nAnswer C: Febuxostat is a xanthine oxidase inhibitor that is used to reduce uric acid levels in the management of chronic gout. Gout would present with acute severe joint pain (typically the great toe) with risk factors of alcohol use, obesity, and thiazide use. It is not used in the acute management of gout or in polycythemia vera.\n\nAnswer E: Prednisone is a glucocorticoid that is used in many different clinical situations but is not used in the management of polycythemia vera. It could be used in acute inflammatory conditions (like Crohn disease or granulomatosis with polyangiitis) or to reduce inflammation during a flare of asthma or COPD.\n\nBullet Summary:\nPolycythemia vera presents with pruritus after bathing, erythromelalgia, and gout and should be treated with hydroxyurea with or without aspirin.", "link": "https://bit.ly/3qUQrsf"} {"question": "A 17-year-old girl presents to the emergency department with a headache. The patient has had headaches in the past but this is the worst headache of her life. Her symptoms started yesterday and have been getting progressively worse. The patient states that the pain is mostly on the left side of her head. There has been a recent outbreak of measles at the patient\u2019s school and the patient\u2019s mother has been trying to give her daughter medicine to prevent her from getting sick. Her mother fears that her daughter may have caught measles. Her temperature is 98.6\u00b0F (37\u00b0C), blood pressure is 123/74 mmHg, pulse is 85/min, and respirations are 13/min. On exam, the patient is an obese girl who is clutching her head with the light in the room turned off. Her neurological exam is within normal limits. Fundoscopic exam reveals mild bilateral papilledema. An MRI of the head is obtained and reveals cerebral edema. A lumbar puncture reveals an increased opening pressure with a normal glucose level. Which of the following is the most likely diagnosis?", "choicesA": "Bacterial meningitis", "choicesB": "Fat-soluble vitamin overuse", "choicesC": "Migraine headache", "choicesD": "Subarachnoid hemorrhage", "choicesE": "Viral meningitis", "answer_idx": "B", "answer": "Fat-soluble vitamin overuse", "explanation": "This patient is presenting with headache, bilateral papilledema, obesity, and increased opening pressure on lumbar puncture suggesting a diagnosis of Idiopathic intracranial hypertension (IIH or pseudotumor cerebri). This disease is associated with hypervitaminosis A (a fat-soluble vitamin).\n\nPseudotumor cerebri classically presents in an obese woman with symptoms of headache and double vision. Overuse of supplements containing vitamin A is a risk factor for developing idiopathic intracranial hypertension. Vitamin A is often used to decrease the duration and severity of a measles infection. On exam, patients will have bilateral papilledema, an increased CSF opening pressure, and cerebral edema. A lumbar puncture is the most accurate diagnostic test for this disease. Treatment is with cessation of offending agents and with carbonic anhydrase inhibitors such as acetazolamide to decrease the rate of cerebrospinal fluid production. Optic nerve sheath fenestration can also be performed in patients who fail medical management.\n\nFuerst et al. present evidence regarding patients with hypervitaminosis A. They discuss how these patients can present with pseudotumor cerebri. They recommend being vigilant about vitamin A toxicity.\n\nIncorrect Answers:\nAnswer A: Bacterial meningitis presents with a stiff neck, photophobia, fever, and a CSF finding of low glucose in contrast to this patient\u2019s normal glucose. The most common causes of bacterial meningitis in young adults are N. meningitidis or S. pneumonia. Patients should be treated empirically with ceftriaxone and vancomycin because this is a medical emergency.\n\nAnswer C: Migraine headache can present with a unilateral pulsing/pounding headache and photophobia; however, it would not present with physical exam findings of papilledema or cerebral edema. Acute migraines can be treated with NSAIDs and triptan medications. Chronic prophylaxis is with beta-blockers or calcium channel blockers.\n\nAnswer D: Subarachnoid hemorrhage presents with the classic, \u201cworst headache of my life,\u201d as this patient has; however, there would be blood visible on MRI which was not described. In addition, there are no symptoms of meningeal irritation from a subarachnoid bleed. Patients may need surgical embolization or clipping of the bleeding vessel.\n\nAnswer E: Viral meningitis would present with a stiff neck, photophobia, fever, and CSF that would only demonstrate lymphocytic pleocytosis. Though this patient\u2019s CSF glucose is normal, her other symptoms do not point toward a diagnosis of meningitis. Treatment is either supportive or with antiviral agents depending on the causative organism.\n\nBullet Summary:\nHypervitaminosis A can cause pseudotumor cerebri, which typically presents in overweight women with headache, double vision, papilledema, and cerebral edema on MRI.", "link": "https://bit.ly/3AgFkva"} {"question": "A 45-year-old morbidly obese woman with a history of asthma and type 2 diabetes mellitus presents to her primary care physician for advice on weight loss. She thinks that her husband is having sex with other women because they have not had sex over the past year. She feels that her co-workers also disrespect her for her weight and constant sweat stains around her armpits and chest. She has noticed that the sweat stains get itchy and induce a burning sensation unless she showers or changes her shirt. She has been compliant with her medications, which include albuterol, metformin, glyburide, and atorvastatin. Her temperature is 98.6\u00b0F (37\u00b0C), blood pressure is 128/85 mmHg, pulse is 91/min, and respirations are 11/min. On physical exam, the patient has a flat affect, with moist oral mucosa and nasal polyps. She denies sinus tenderness. Her neck is thick with a posterior cervical fat pad. During cardiac auscultation, the finding in Figure A is noted below her breasts. What is the most appropriate next step in management for this finding?", "choicesA": "Fluconazole", "choicesB": "Prednisone", "choicesC": "Topical clobetasol", "choicesD": "Topical imiquimod", "choicesE": "Topical nystatin", "answer_idx": "E", "answer": "Topical nystatin", "explanation": "This patient has a rash consistent with candidal intertrigo given the location and history of a damp, pruritic rash. The most appropriate next step in management is topical nystatin powder.\n\nIntertrigo is caused by the Candida species and is characterized by pruritic, painful, and erythematous superficial patches surrounded by satellite lesions. They are commonly seen in intertriginous areas such as the breasts, groin, axilla, or abdominal pannus. Obesity and diabetes are significant risk factors. Initial management involves maintaining skin dryness along with medical treatment using topical nystatin, clotrimazole, or miconazole. Persistent lesions may require oral antifungal treatment.\n\nNobles and Miller review the evidence regarding the diagnosis and treatment of intertrigo. They discuss how these lesions often become infected with Candida. They recommend the use of antifungal medications.\n\nFigure/Illustration A shows candidal intertrigo in the breast fold. Note the satellite lesions that surround the erythematous patch.\n\nIncorrect Answers:\nAnswer A: Fluconazole is indicated for moderate to severe disease or if the rash is refractory to topical antifungals. Topical antifungal agents should be used before progressing to oral medications.\n\nAnswer B: Prednisone is not indicated and would potentially worsen the rash through immunosuppression and exacerbation of the patient's diabetes mellitus.\n\nAnswer C: Topical clobetasol is a high-potency corticosteroid. This would not be used because it can promote infection and lead to skin atrophy. Low-potency corticosteroids such as triamcinolone or hydrocortisone can be used as adjuncts for symptom control only.\n\nAnswer D: Topical imiquimod is indicated for autoimmune skin diseases such as alopecia areata not for candidal intertrigo. This disease would present with segmental regions of hair loss\n\nBullet Summary:\nThe treatment of candidal intertrigo involves topical antifungals including nystatin, clotrimazole, and miconazole.", "link": "https://bit.ly/44tTwi7"} {"question": "A 6-hour-old newborn boy is noted to have a \u201clump on his head\u201d by his mother. She denies that the lump was present at birth and is concerned about an infection. The child was born at 39 weeks gestation to a 34-year-old G2P2 mother by vacuum-assisted vaginal delivery after prolonged labor. His birth weight was 3.8 kg (8.4 lb) and his length and head circumference are at the 40th and 60th percentiles, respectively. The mother was diagnosed during this pregnancy with gestational diabetes mellitus and received prenatal care throughout. All prenatal screening was normal and the 20-week anatomy ultrasound was unremarkable. His temperature is 98.6\u00b0F (37\u00b0C), blood pressure is 65/42 mmHg, pulse is 131/min, and respirations are 36/min. On physical exam, the child is in no acute distress. He has a 3x3 cm fluctuant swelling over the right parietal bone that does not cross the midline. There is no discoloration of the overlying scalp. Laboratory testing is performed and reveals the following:\n\nTotal bilirubin: 5.5 mg/dL\nDirect bilirubin: 0.7 mg/dL\n\nWhich of the following is the most appropriate next step in management?", "choicesA": "Incision and drainage", "choicesB": "Intensive phototherapy", "choicesC": "Neurosurgical decompression", "choicesD": "Observation only", "choicesE": "Red blood cell transfusion", "answer_idx": "D", "answer": "Observation only", "explanation": "This patient presents with a fluctuant swelling of the scalp that is limited by suture lines, which is the classic description of a cephalohematoma. The majority of cases of cephalohematoma self-resolve, thus the most appropriate next step in management is observation.\n\nA cephalohematoma is a relatively common traumatic birth injury that occurs after prolonged labor or instrumentation due to rupture of the periosteal bridging veins. Because bleeding through these veins is slow, they typically present several hours after birth with a fluctuant swelling of the scalp that is limited by suture lines. The majority of cases self-resolve between 2 weeks to 3 months. In rare cases, the sequestration of blood in the cephalohematoma may cause an unconjugated hyperbilirubinemia that requires phototherapy but this is a rare complication. Treatment is with reassurance and outpatient monitoring.\n\nRaines et al. review the evidence regarding the diagnosis and treatment of cephalohematoma. They discuss how this injury presents gradually and does not cross suture lines. They recommend conservative treatment.\n\nIncorrect Answers:\nAnswer A: An incision and drainage procedure is contraindicated in cephalohematoma without signs of infection, such as erythema of the skin overlying the swelling. In cases where the cephalohematoma is thought to be secondarily infected, patients should undergo incision and drainage to prevent osteomyelitis. Patients would present with increasing pain, erythema, and systemic findings such as fever.\n\nAnswer B: Intensive phototherapy may be used to manage the unconjugated hyperbilirubinemia that may occur due to cephalohematoma, but the majority of cases resolve spontaneously. The unconjugated hyperbilirubinemia found in this patient does not necessitate phototherapy. Unconjugated bilirubin is produced due to the metabolism of hemoglobin.\n\nAnswer C: Neurosurgical decompression may be necessary to manage other neonatal birth injuries, such as intracranial hemorrhages. Because a cephalohematoma involves bleeding between the periosteum and skull (i.e., outside of the skull), it does not require neurosurgical intervention. Intracranial hemorrhages would present with asymmetric reflexes or obtundation.\n\nAnswer E: Red blood cell transfusions are sometimes necessary to manage intracranial hemorrhages in neonates with significant blood loss. They are not usually required to manage cephalohematoma as the bleeding in this condition is limited by the suture lines. Patients who are tachycardic or hypotensive may require transfusion.\n\nBullet Summary:\nMost cases of cephalohematoma resolve spontaneously but can sometimes present with an unconjugated hyperbilirubinemia that requires treatment with phototherapy.", "link": "https://bit.ly/45Y8H4f"} {"question": "A 24-year-old motorcyclist is involved in a head-on collision with a motor vehicle and suffers various traumatic injuries. She has no significant medical history, is up to date on all vaccinations, and her last tetanus shot was 3 years ago. On arrival, her temperature is 36.7\u00b0C (98\u00b0F), blood pressure is 82/63 mmHg, pulse is 120/min, respirations are 20/min, and oxygen saturation is 97% on 2L oxygen nasal cannula. On physical exam, there are extensive ecchymoses and abrasions along her left posterior ribs and left flank. A CT scan of the abdomen is obtained, which is shown in Figure A. After the appropriate surgical intervention is performed for the grade V splenic injury, which of the following is the most appropriate next step in management?", "choicesA": "Insertion of a nasogastric tube", "choicesB": "Insertion of a thoracostomy tube", "choicesC": "Prophylactic intravenous antibiotics", "choicesD": "Tetanus vaccination", "choicesE": "Vaccination against Streptococcus pneumoniae, Neisseria meningitidis, and Haemophilus influenzae", "answer_idx": "E", "answer": "Vaccination against Streptococcus pneumoniae, Neisseria meningitidis, and Haemophilus influenzae", "explanation": "This patient has an extensive splenic injury and hemodynamic instability requiring splenectomy, as evidenced by trauma to the left posterior ribs and left flank, low blood pressures with reflex tachycardia, and evidence of splenic rupture and hemoperitoneum on computed tomography imaging. Subsequently, this asplenic patient will require vaccination against encapsulated bacteria such as S. pneumoniae, N. meningitidis, and H. influenzae.\n\nThe capsules of encapsulated bacteria serve as an antiphagocytic virulence factor. To clear these bacteria, opsonization and subsequent clearance by the spleen must occur. In the setting of asplenia, opsonization capability is significantly decreased and thus asplenic patients are at high risk of severe infections by encapsulated bacteria. As such, asplenic patients require vaccination against the encapsulated bacteria S. pneumoniae, N. meningitidis, and H. influenza.\n\nPatton et al. review recommendations for serogroup B meningococcal vaccination. The authors find that for patients aged 10 to 25 years old who are at increased risk for meningococcal disease (asplenic patients); 3 doses of the MenB-FHbp vaccine should be given. The authors recommend the administration of only 2 doses of the vaccine to patients that are not at elevated risk.\n\nFigure/Illustration A depicts a traumatic splenic rupture with perisplenic hemoperitoneum (red circle).\n\nIncorrect Answers:\nAnswer A: Insertion of a nasogastric tube would be appropriate in a patient with bowel obstruction or significant post-operative ileus. There is no evidence that this patient is experiencing obstruction or ileus, which typically presents with nausea, vomiting, abdominal pain, bloating, and lack of flatus or bowel movements.\n\nAnswer B: Insertion of a thoracostomy tube is not indicated in this patient as there is no evidence of pneumothorax or hemothorax. Patients with pneumothorax or hemothorax typically present with tachycardia, tachypnea, and decreased oxygen saturation. On exam, such patients may have decreased breath sounds. Ultrasonography may also reveal the absence of lung sliding.\n\nAnswer C: Prophylactic intravenous antibiotics are not indicated in the immediate post-operative period for asplenic adults. However, daily oral antibiotic prophylaxis is indicated in asplenic patients with a history of another concurrent immunocompromising condition or a history of severe infection due to an encapsulated organism.\n\nAnswer D: Tetanus vaccination is indicated every 10 years in patients who have previously received 3 doses of the tetanus vaccine and have clean or minor wounds. In patients with more extensive wounds who have previously received 3 doses of tetanus vaccine, re-vaccination is only necessary if their last tetanus vaccine was 5 or more years ago. This patient has extensive abrasions but is up to date on all vaccines and last received a tetanus vaccine 3 years ago, so a re-vaccination during this time is not indicated.\n\nBullet Summary:\nAsplenic patients are at increased risk of severe infection by encapsulated bacteria; thus patients that undergo splenic removal require vaccination against Streptococcus pneumoniae, Neisseria meningitidis, and Haemophilus influenzae within 14 days of the procedure.", "link": "https://step2.medbullets.com/testview?qid=215049"} {"question": "A 17-year-old high school student was in shop class when he accidentally sawed off a portion of his right index finger. The teacher applied dressings and pressure to the patient's injured digit and immediately transported the patient to the emergency department. He arrived within 20 minutes of the accident. The patient has a medical history of asthma and his only medication is albuterol. His temperature is 98.0\u00b0F (36.6\u00b0C), blood pressure is 120/70 mmHg, pulse is 105/min, and respirations are 17/min. Exam was significant for pulsatile bleeding from a clean-cut wound on his right second finger. Radiography of the hand revealed a complete amputation of the right finger from the distal interphalangeal joint. The wound was cleaned, compression applied, analgesics administered, and the hand surgeons were notified. The teacher states that he left the amputated finger in the classroom, but the principal would be transporting it to the hospital. Which of the following is the correct method for transporting the amputated finger?", "choicesA": "Rinse the finger in normal saline, wrap the finger in moist gauze, put in a plastic bag, and place the bag in ice water", "choicesB": "Rinse the finger with hydrogen peroxide, wrap the finger in moist gauze, and place on ice", "choicesC": "Submerge the finger in ice water", "choicesD": "Wrap the finger in moist gauze and place in a plastic bag", "choicesE": "Wrap the finger in moist gauze and place on ice", "answer_idx": "A", "answer": "Rinse the finger in normal saline, wrap the finger in moist gauze, put in a plastic bag, and place the bag in ice water", "explanation": "This patient is presenting with a fingertip amputation with loss of pulp, nail, and bone (with a possibility for surgical reattachment and repair, efforts should be made to recover the severed tissue). The correct way to transport an amputated digit is to rinse the finger in normal saline to remove gross contaminants, wrap the appendage in gauze moistened by normal saline, place it in a plastic bag, and then place the bag in ice water.\n\nAmputation injuries are common with finger and thumb amputations being the most common. With rapid and proper transport of the patient and appendage, successful reattachment could be possible pending a surgical evaluation. It is important to know how to appropriately package the severed digit so as to optimize tissue viability. Cooling with ice water can slow the metabolic rate of the tissue and result in less tissue damage; however, there is a delicate balance. There must be proper barriers between the digit and ice water to avoid ice burns which could compromise the tissue. Patients should be given IV antibiotics to prevent infection as well.\n\nZhang et al. note that once a finger amputation has occurred, ischemic tolerance times are 12 hours if warm and up to 24 hours if cold. For more proximal amputations, these times are halved due to the presence of muscle tissue, which can undergo irreversible changes after 6 hours of ischemia.\n\nIncorrect Answers:\nAnswer B: Rinsing the amputated finger with hydrogen peroxide could damage the tissue decreasing the chances of salvageability. Avoiding the use of caustic solutions with amputated appendages preserves tissue.\n\nAnswer C: Submerging the finger in ice water could lead to irreversible tissue damage and swelling making reattachment impossible.\n\nAnswer D: Wrapping the finger in moist gauze and then placing in a plastic bag is not enough to optimize a severed finger for transport. Cooling further with ice water is necessary in order to slow down tissue metabolism and prevent further ischemic damage.\n\nAnswer E: Wrapping the finger in moist gauze and placing it directly on ice could damage the finger by direct contact with ice rendering the amputated part non-viable.\n\nBullet Summary:\nThe correct handling of an amputated appendage is to first rinse it off with normal saline to remove any gross debris, wrap it in normal saline soaked gauze, put it in a plastic bag, and then place that bag in ice water.", "link": "https://bit.ly/3NOcdHz"} {"question": "A 77-year-old man presents to the emergency department with a complaint of sudden onset weakness in his right upper extremity. At home, the patient thought he was simply dehydrated, but he rapidly began to have trouble speaking. When his wife noted this she brought him into the hospital. On arrival to the emergency department, the patient is not responding to verbal stimuli and only withdraws his left upper extremity and lower extremity to pain. His past medical history is notable for hypertension and atrial fibrillation treated with metoprolol, apixaban, and lisinopril. His temperature is 99.0\u00b0F (37.2\u00b0C), blood pressure is 170/100 mmHg, pulse is 95/min, and respirations are 16/min. The patient has a seizure and subsequently demonstrates agonal breathing. He no longer responds to painful stimuli. A fingerstick blood glucose is 122 mg/dL. Which of the following is the most likely etiology of this patient's symptoms?", "choicesA": "Epidural hematoma", "choicesB": "Intracerebral hemorrhage", "choicesC": "Ischemic stroke", "choicesD": "Subarachnoid hemorrhage", "choicesE": "Subdural hematoma", "answer_idx": "B", "answer": "Intracerebral hemorrhage", "explanation": "This patient with a medical history of hypertension on blood thinners (apixaban) is presenting with rapidly progressive neurologic symptoms. Initially, his symptoms are in his right upper extremity, which then progresses to affect his speech and cause altered mental status, obtundation, and seizures, which is suggestive of a left-sided intraparenchymal hemorrhage.\n\nIntraparenchymal (IPH) cerebral hemorrhages often present with sudden-onset focal neurologic deficits. A unique feature is that IPHs tend to present with progressive/worsening symptoms in contrast to ischemic strokes which often demonstrate stable or improving symptoms. It is not uncommon for a patient to present with focal neurologic deficits, which then progress to seizures and obtundation. Risk factors include older age, hypertension, and blood thinners. The prognosis is poor in intraparenchymal hemorrhages, and outcomes are dependent on the size and location of the bleed. Initial workup should be centered on obtaining a fingerstick blood glucose, securing the airway, and obtaining CNS imaging (often with a non-contrast CT scan). Further management is centered on elevating the head of the bed, reversing the patient's anticoagulation status, and possible neurosurgical intervention (though prognosis is very poor in general).\n\nGross et al. review IPH. They note that IPH has very high morbidity and mortality even when treated. There is ongoing research into minimally invasive approaches for evacuation of primary IPH to optimize and improve outcomes, although immediate medical therapy is needed to reduce mortality which is recommended.\n\nIncorrect Answers:\nAnswer A: Epidural hematomas present after trauma with the \"talk and die\" syndrome beginning with a period of unconsciousness, a lucid interval, and then progressive obtundation. A CT of the head would demonstrate a lens-shaped lesion, and a burr hole to drain the collection of blood is the treatment of choice. Acute epidural hematomas do not typically begin spontaneously.\n\nAnswer C: Ischemic strokes present with sudden onset, focal neurologic deficits. Generally, after the initial ischemic event, patients will be stable or even note an improvement in their symptoms. It is less likely that this patient's progressive symptoms and seizures are caused by an ischemic stroke when compared to a hemorrhagic stroke or intraparenchymal hemorrhage. This diagnosis is certainly possible, and further workup with CT imaging and possibly MRI is needed to clarify the diagnosis and direct further care.\n\nAnswer D: Subarachnoid hemorrhages generally present with a \"thunderclap\" headache with meningeal signs. It is possible that patients may present with obtundation and confusion with severe subarachnoid hemorrhages. Seizures are uncommon in subarachnoid hemorrhages and generally do not present with such focal neurologic deficits. CT will show blood in the subarachnoid space. The diagnosis can be supported with CTA or a lumbar puncture. Further management may involve nimodipine, neurosurgical intervention, and observation if the bleed is small and has stopped on its own.\n\nAnswer E: Subdural hematomas are more common in the elderly patients and patients with alcohol use disorder and present after trauma with a gradual progression of altered mental status to obtundation. Note that large subdural hematomas may present with a rapid progression of symptoms. CT of the head will demonstrate a crescent-shaped lesion that crosses suture lines. Management may involve neurosurgical evacuation with large subdural hematomas causing mass effect.\n\nBullet Summary:\nIntraparenchymal hemorrhages may occur spontaneously in the setting of hypertension and blood thinners and present with progressive neurologic deficits that get worse over time with possible seizures.", "link": "https://step2.medbullets.com/testview?qid=216357"} {"question": "A 44-year-old woman presents to the emergency department with fluctuating right upper quadrant abdominal pain. The pain was initially a 4/10 in severity but has increased recently to a 6/10 prompting her to come in. The patient has a medical history of type 2 diabetes mellitus, depression, anxiety, and irritable bowel syndrome. Her current medications include metformin, glyburide, escitalopram, and psyllium husks. Her temperature is 99.2\u00b0F (37.3\u00b0C), pulse of 95/min, blood pressure of 135/90 mmHg, respirations of 15/min with 98% oxygen saturation on room air. On exam, the patient is an obese woman with pain upon palpation of the right upper quadrant. Initial labs are are below:\n\nNa+: 140 mEq/L\nK+: 4.0 mEq/L\nCl-: 100 mEq/L\nHCO3-: 24 mEq/L\nAST: 100 U/L\nALT: 110 U/L\nAmylase: 30 U/L\nAlkaline phosphatase: 125 U/L\n\nBilirubin\nTotal: 2.5 mg/dL\nDirect: 1.8 mg/dL\n\nThe patient is sent for a right upper quadrant ultrasound demonstrating an absence of stones, no pericholecystic fluid, a normal gallbladder contour and no abnormalities noted in the common bile duct. MRCP with secretin infusion is performed demonstrating patent biliary and pancreatic ductal systems. Her lab values and clinical presentation remain unchanged 24 hours later. Which of the following is the most appropriate next step in management?", "choicesA": "Analgesics and await resolution of symptoms", "choicesB": "Elective cholecystectomy", "choicesC": "ERCP with manometry", "choicesD": "Laparoscopy", "choicesE": "MRI of the abdomen", "answer_idx": "C", "answer": "ERCP with manometry", "explanation": "This patient is presenting with classic symptoms of acute cholecystitis or choledocholithiasis such as right upper quadrant pain and direct hyperbilirubinemia with ultrasound findings demonstrating the absence of gallstones. In this setting, sphincter of Oddi dysfunction should be suspected, and ERCP with manometry should be performed to confirm the diagnosis.\n\nCholecystitis is inflammation of the gallbladder caused by irritation due to an intraluminal stone and possible bacterial translocation. The classic risk factors for this disease can be remembered as \"fat, female, fertile and forty\". Symptoms of acute cholecystitis are right upper quadrant pain that relapses and remits over time. Lab values will show elevated AST, ALT, alkaline phosphatase, and bilirubin which are also seen in patients where a stone is impacted in the common bile duct. A right upper quadrant ultrasound and MRCP can be used to confirm the diagnosis. Dysfunction of the Sphincter of Oddi should be suspected in patients with classic findings but negative imaging studies. In these cases, the most appropriate test is ERCP with manometry of the sphincter of Oddi. Ultimately, a sphincterotomy will be needed once the diagnosis is confirmed.\n\nCoucke et al. present the current evidence for the diagnosis and treatment of biliary obstruction. They discuss how the most common etiology of biliary obstruction is choledocholithiasis or gallstones. They recommend having a high index of suspicion for cholangitis as this condition can be rapidly fatal if untreated.\n\nIncorrect Answers:\nAnswer A: Analgesics and await resolution of symptoms (in addition to NPO and IV fluids) would be appropriate management for pancreatitis. The diagnosis in this patient has not yet been confirmed (given the absence of findings on ultrasound) thus prompting another diagnostic test.\n\nAnswer B: An elective cholecystectomy would be the most appropriate next step in management if the patient had cholecystitis.\n\nAnswer D: Laparoscopy is an inappropriate and invasive method of diagnosing and treating disease of the biliary tree as there are better alternatives such as MRCP or ERCP. Laparoscopy would be effective in the treatment of intestinal tract disorders such as appendicitis.\n\nAnswer E: An MRI of the abdomen would not confirm the diagnosis of sphincter of Oddi dysfunction. Though a CT scan could be performed as a more cost-effective alternative, given the absence of stones on ultrasound it is likely that a CT scan will not give more information.\n\nBullet Summary:\nSphincter of Oddi dysfunction can be diagnosed with ERCP and manometry.", "link": "https://step2.medbullets.com/testview?qid=108516"} {"question": "A 27-year-old man presents to the emergency department with altered mental status. He has become gradually more confused over the past several days. His wife also notes he has had diarrhea, nausea and vomiting, and abdominal pain for the past week. The patient has a history of depression and multiple suicide attempts. He takes fluoxetine as well as over-the-counter pain medications for a recent muscle strain. He works in a large industrial compound that manufactures semiconductors. He exercises regularly and recently has started making his own beer at home. His temperature is 98.3\u00b0F (36.8\u00b0C), blood pressure is 107/75 mmHg, pulse is 110/min, respirations are 22/min, and oxygen saturation is 99% on room air. Physical exam reveals a confused man with a garlic odor on his breath and the finding in Figure A. Cardiopulmonary exam reveals a rapid heart rate with no murmurs and clear breath sounds. Which of the following is the most likely etiology of this patient's symptoms?", "choicesA": "Acetaminophen", "choicesB": "Arsenic", "choicesC": "Cyanide", "choicesD": "Iron", "choicesE": "Lead", "answer_idx": "B", "answer": "Arsenic", "explanation": "This patient who works in the semiconductor industry and presents with confusion, hypotension, tachycardia, a garlic odor, and Mees lines (Figure A) most likely has arsenic poisoning.\n\nArsenic poisoning can occur due to exposure to contaminated drinking water, industrial jobs, insecticides, and certain preservatives. Arsenic inhibits enzymes requiring lipoic acid as a cofactor; thus, it disrupts ATP production and causes oxidative stress. Signs and symptoms of arsenic poisoning include Mees lines (white horizontal/parallel lines on the nail beds), nausea, vomiting, abdominal pain, diarrhea, confusion, hypotension, tachycardia, and a garlic odor on the breath. Severe complications are more common with acute exposures and can include shock, pulmonary edema, rhabdomyolysis, and death. Findings consistent with chronic poisoning include anemia, neuropathy, ataxia, and somnolence. A urine arsenic level can support the diagnosis. The treatment of arsenic poisoning is chelating agents such as dimercaprol and supportive therapy. Dimercaprol is a nephrotoxic chelating agent with a narrow therapeutic index and should be used with care.\n\nMoon et al. study the association between low and moderate arsenic exposure and cardiovascular disease. The authors find that even low-dose exposure to arsenic increased the risk of cardiovascular disease, coronary artery disease, and stroke mortality. The authors recommend population-level interventions to decrease exposure to arsenic.\n\nFigure/Illustration A is the physical exam finding of Mees lines with multiple parallel white lines in the nail bed (blue arrows).\n\nIncorrect Answers:\nAnswer A: Acetaminophen overdose has a spectrum of presentations, from non-specific symptoms which can include no symptoms at all to a toxic appearance with nausea, vomiting, jaundice, and altered mental status. Patients should have a 4-hour acetaminophen level drawn. If it falls above the treatment line on the Rummack Matthew nomogram, treatment with N-acetylcysteine should be started. Acetaminophen can cause fulminant liver failure and markedly elevated liver enzymes.\n\nAnswer C: Cyanide inhibits complex IV in the electron transport chain. It can present after exposure to industrial fires. Patients may present with altered mental status and a profound lactic acidosis that does not respond to oxygen administration with rapid progression to death. The treatment involves the administration of hydroxocobalamin which combines with cyanide to form the non-toxic compound cyanocobalamin.\n\nAnswer D: Iron overdose presents with nausea, vomiting, diarrhea (bloody), hemorrhagic gastritis, and lactic acidosis. A serum iron level should be ordered to aid in diagnosis. Treatment of acute overdose includes chelation therapy with deferoxamine.\n\nAnswer E: Lead poisoning can present with constipation, irritability, lethargy, lead lines on radiography, and microcytic anemia with basophilic stippling. Treatment depends on the severity and can involve removal from exposure or chelating therapy with agents such as succimer.\n\nBullet Summary:\nArsenic poisoning presents with Mees lines, nausea, vomiting, abdominal pain, confusion, unstable vitals, and a classic \u201cgarlic odor.\u201d", "link": "https://bit.ly/3LX4YvI"} {"question": "A 6-week-old boy is referred to a pediatric hematologist-oncologist for further evaluation following an abnormal newborn hemoglobinopathy screening. The father\u2019s brother died of sickle cell anemia at an early age. Parental studies reveal that both mother and father are carriers of the sickle cell gene. The patient's temperature is 98.6\u00b0F (37.0\u00b0C), blood pressure is 80/45 mmHg, pulse is 130/min, and respirations are 25/min. The physician decides to repeat testing for the patient and his 3-year-old sister. The results are shown in Figure A. The physician begins counseling the family. Which of the following is the most likely complication that the patient may have in the future?", "choicesA": "Diabetes", "choicesB": "Hematuria", "choicesC": "Infection", "choicesD": "Splenectomy", "choicesE": "Stroke", "answer_idx": "B", "answer": "Hematuria", "explanation": "Based on the presence of both hemoglobin (Hb) A and S on gel electrophoresis, this patient has sickle cell trait. Episodes of painless hematuria are commonly seen in these patients.\n\nSickle cell trait is a condition that refers to carrying 1 normal and 1 abnormal beta-globin allele. On electrophoresis, this will present as 3 different bands consisting of about 50-60% HbA, 35-45% HbS, and < 2% HbF (fetal hemoglobin). HbF decreases significantly after the first 6 weeks of life. Individuals with sickle cell trait tend to be asymptomatic and do not have a decreased lifespan compared to the general population. However, they are at higher risk of some conditions. Patients are at increased risk of rhabdomyolysis during strenuous physical activity and may have episodic painless hematuria due to renal papillary necrosis. The hyperosmolar, acidotic, and hypoxic environment of the renal medulla is thought to trigger red cell sickling which results in ischemia and subsequent hematuria. Patients with sickle cell trait are managed similarly to the general population. These patients should receive reproductive counseling and use appropriate preventive measures to prevent dehydration during intensive exercise.\n\nHulsizer et al. studied the association between sickle cell trait and common medical conditions using insurance claims data. The authors found that sickle cell trait is associated with increased risks of diabetes, hypertension, heart disease, chronic kidney disease, and retinopathy. The authors recommend further evaluation of these associations using prospective studies derived from clinical data.\n\nFigure/Illustration A shows gel electrophoresis with an example of a normal (95-98% HbA and < 2% HbF), sickle cell trait (50-60% HbA, 35-45% HbS, and < 2% HbF), and sickle cell anemia (85-95% HbS and 5-15% HbF) patient from left to right.\n\nIncorrect Answers:\nAnswer A: Diabetes has not been associated with sickle cell trait in the majority of studies.\n\nAnswer C: Infection is a complication of sickle cell anemia. Mechanisms include functional asplenism, reduced tissue perfusion during a sickle crisis, and chronic transfusions with an indwelling catheter. Common infections include bacteremia, meningitis, and pneumonia due to Streptococcus pneumoniae, Haemophilus influenzae, and Neisseria meningitis. Sickle cell trait is not clearly associated with an increased risk of infection and actually confers some protection against malaria.\n\nAnswer D: Splenectomy from auto-infarction is often seen in sickle cell anemia, but it is rare with sickle cell trait. In sickle cell anemia, splenic sequestration of sickled red blood cells causes infarction, leading to progressive atrophy and functional hyposplenism. Most individuals with sickle cell disease are functionally asplenic by age 2 to 4. The spleen subsequently cannot phagocytose encapsulated organisms properly, which predisposes sickle cell patients to infection.\n\nAnswer E: Stroke is a complication of sickle cell anemia but not sickle cell trait. This is due to the increased viscosity of the blood and reduced deformability of red cells in patients with sickle cell anemia.\n\nBullet Summary:\nSickle cell trait is associated with episodes of painless hematuria.", "link": "https://bit.ly/3KxXaiO"} {"question": "A 69-year-old man presents to his primary care physician for a general checkup. Overall he is doing well. Since he retired, he has been working on projects at home and taking time to exercise every day. He eats a balanced diet and has been spending time with his wife every evening. Despite this, the patient claims that he feels less well-rested when he wakes up in the morning. The patient states that he used to sleep 9 hours a night in his youth and felt excellent. Now he sleeps 7 hours a night and doesn\u2019t feel as well rested as he used to. The patient\u2019s wife states that he seems to sleep peacefully. His medical history is significant for hypertension and diabetes for which he takes lisinopril and metformin. His temperature is 98.6\u00b0F (37\u00b0C), blood pressure is 131/85 mmHg, pulse is 71/min, and respirations are 12/min. His neurological exam is within normal limits. The patient is muscular and has a healthy weight with a pleasant demeanor. He denies feeling fatigued or tired currently. Which of the following physiological changes would most likely be seen in this patient?", "choicesA": "Decreased melatonin", "choicesB": "Decreased orexin", "choicesC": "Increased acetylcholine", "choicesD": "Increased dopamine", "choicesE": "Increased norepinephrine", "answer_idx": "A", "answer": "Decreased melatonin", "explanation": "This elderly patient is presenting with sleep that is sufficient but less refreshing than sleep was for him in his past suggesting a diagnosis of normal aging. Melatonin levels naturally decrease as patients age.\n\nNormal aging shortens most phases of sleep. It is normal for melatonin levels to decline during normal aging; however, this change is not responsible for all the symptoms associated with declining quality of sleep. Other changes that occur include decreased REM sleep, decreased stage N3 sleep, and a compensatory increase in stage N2 sleep. Patients will often complain of sleep that is less refreshing than it was in their youth, yet they are still functional and not excessively tired. Treatment is with sleep hygiene though some patients will also benefit from treatment with melatonin supplementation.\n\nCardinali reviews the role of melatonin in aging. He discusses how natural levels of melatonin decrease in aging animals, which may have implications on the circadian clock. He recommends studies on whether melatonin supplementation may be beneficial to mitigate the effects of aging.\n\nIncorrect Answers:\nAnswer B: Decreased orexin levels would be seen in patients with narcolepsy, which presents with sudden episodes of falling asleep. This tends to occur in younger patients and is also associated with cataplexy or sudden loss in muscle tone. Treatment of this disorder is with stimulant medications such as modafinil.\n\nAnswer C: Increased acetylcholine levels would not be seen in patients as they age and would not explain the changes this patient is experiencing. Rather, acetylcholine levels tend to decline as we age. Drinking caffeinated drinks is associated with increased acetylcholine levels in the brain.\n\nAnswer D: Increased dopamine would not be a principle change seen in age-related sleep changes. Higher dopamine levels would increase feelings of being awake as well as alertness. In contrast, decreased dopamine levels can be seen in patients with Parkinson disease, which would present with bradykinesia and a resting tremor.\n\nAnswer E: Increased norepinephrine would result in increased wakefulness and potentially insomnia; however, it would not explain the changes seen in normal aging. This chemical is naturally produced during the fight or flight response and can be artificially increased by stimulant drugs such as cocaine.\n\nBullet Summary:\nNormal aging results in a shortening of most phases of sleep as well as decreased melatonin levels and presents with sleep that is subjectively less restful.", "link": "https://bit.ly/4634T1E"} {"question": "A 33-year-old man presents to the emergency department with dizziness. He states he has a constant sensation that the room is spinning. He is now having trouble walking and has been vomiting intermittently. He has no past medical history and takes no medications. His temperature is 98.0\u00b0F (36.7\u00b0C), blood pressure is 122/84 mmHg, pulse is 80/min, respirations are 16/min, and oxygen saturation is 98% on room air. Physical exam reveals a young man who is vomiting. His gait is ataxic and he exhibits rightward nystagmus. His dizziness is constant and unchanged with performance of the Dix-Hallpike maneuver. The head impulse test reveals a corrective saccade and there is no skew deviation. Which of the following is the most appropriate next step in management?", "choicesA": "CT head", "choicesB": "Epley maneuver", "choicesC": "MRI brain", "choicesD": "Prednisone", "choicesE": "Tissue plasminogen activator", "answer_idx": "D", "answer": "Prednisone", "explanation": "This young, healthy patient is presenting with constant vertigo, ataxia, vomiting, and a head impulse test revealing a corrective saccade, nystagmus, and test of skew (HINTS) exam, which suggests a peripheral etiology of vertigo, most likely vestibular neuritis. As the diagnosis can be made clinically, steroids are the only treatment needed.\n\nDifferentiating peripheral (the vestibular system) versus central (the brain) vertigo is critical in management. Peripheral vertigo is commonly caused by benign paroxysmal positional vertigo, vestibular neuritis, and labyrinthitis. Vertigo that is constant is more consistent with vestibular neuritis or labyrinthitis but could also be a cerebellar stroke. To differentiate, age, risk factors, and onset of symptoms are critical. Moreover, the HINTS exam can point toward 1 cause of vertigo versus another. HINTS is a mnemonic for: Horizontal head impulse testing (Head Impulse), Direction-changing nystagmus in eccentric gaze (Nystagmus), and Vertical skew (Test of Skew). Findings that are suggestive of a peripheral cause of vertigo include a positive head impulse exam, positive nystagmus (also seen in stroke), and a negative test of skew. In the appropriate clinical context with appropriate risk factors, peripheral causes such as vestibular neuritis or labyrinthitis can be managed with steroids (which may improve outcomes) and meclizine (for symptom control).\n\nKattah et al. discuss the HINTS exam. They note how it can be used to differentiate peripheral from central vertigo. It is recommended to perform this exam when differentiating central versus peripheral vertigo.\n\nIncorrect Answers:\nAnswers 1 & 3: Head imaging such as a CT head or MRI brain is not necessary in patients with a clear peripheral cause of their vertigo/dizziness with a HINTS exam suggesting a peripheral etiology. Clinically, this patient has vestibular neuritis and can be treated accordingly. Imaging would be needed if the patient had an acute cerebellar stroke.\n\nAnswer D: The Epley maneuver is only indicated in cases of benign paroxysmal peripheral vertigo, which presents with severe vertigo (with changes in head position) that diminish with time and keeping the head stationary. Note that this patient\u2019s Dix-Hallpike maneuver is negative and that his vertigo is constant rather than intermittent or positional.\n\nAnswer E: Tissue plasminogen activator is indicated within 4.5 hours in the setting of an ischemic stroke after a head CT rules out a head bleed, other reversible causes are treated, and assuming there are no contraindications to thrombolytics. Tissue plasminogen activator can be given in posterior circulation strokes.\n\nBullet Summary:\nPeripheral causes of vertigo generally do not require head imaging, and causes such as labyrinthitis or vestibular neuritis can be treated with steroids.", "link": "https://step2.medbullets.com/testview?qid=216590"} {"question": "A 13-day-old boy is brought by his mother for eye redness and ocular discharge. The mother reports that the patient has developed a cough and nasal discharge. Pregnancy and delivery were uncomplicated but the mother had limited prenatal care during the third trimester. Immediately after delivery, the baby was given silver nitrate drops and vitamin K. His temperature is 99\u00b0F (37.2\u00b0C), blood pressure is 81/52 mmHg, pulse is 135/min, and respirations are 36/min with an oxygen saturation of 98% O2 on room air. Upon visual examination of the eyes, mucoid ocular discharge and eyelid swelling are noted. A fluorescein test is negative. On lung exam, scattered crackles are appreciated. A chest radiograph is performed that shows hyperinflation with bilateral infiltrates. Which of the following is the most appropriate pharmacotherapy?", "choicesA": "Artificial tears", "choicesB": "Intravenous acyclovir", "choicesC": "Intravenous ceftriaxone", "choicesD": "Oral erythromycin", "choicesE": "Topical erythromycin", "answer_idx": "D", "answer": "Oral erythromycin", "explanation": "This patient is presenting with mucoid ocular discharge around 2 weeks post-birth, suggesting the diagnosis of neonatal chlamydial conjunctivitis. The most appropriate treatment for this disease includes oral erythromycin.\n\nNeonatal chlamydial conjunctivitis is caused by Chlamydia trachomatis. It presents in newborns that are 5-14 days old. Symptoms include watery or mucoid discharge and eye swelling. The conjunctiva may be especially irritated. Diagnosis can be established by direct antibody testing or PCR. Treatment for chlamydial conjunctivitis includes both topical and oral erythromycin as there is likely dissemination of the infection. The most common complication of chlamydial conjunctivitis is pneumonia. Of note, all neonates with gonococcal conjunctivitis should also be treated for chlamydia.\n\nSmith-Norowitz et al. studied whether antibiotic prophylaxis was beneficial compared with silver nitrate for chlamydia conjunctivitis. They found that there was no benefit to topical antibiotic treatment compared with silver nitrate alone. They recommend testing infants for the disease if it is suspected so that oral antibiotics can be administered effectively.\n\nIncorrect Answers:\nAnswer A: Artificial tears can be used in chemical conjunctivitis. The most common cause of neonatal chemical conjunctivitis is post-delivery silver nitrate drops. It usually presents within the first 24 hours following birth/exposure with mild conjunctival injection and tearing, and will spontaneously resolve within 2-4 days.\n\nAnswer B: Intravenous acyclovir is used to treat conjunctivitis caused by herpes simplex virus. The newborn would present between 1-2 weeks post-birth with lid edema, conjunctival injection, and non-purulent serosanguineous discharge. Corneal involvement with microdendrites or ulcers may be seen with fluorescein stain.\n\nAnswer C: Intravenous ceftriaxone is used to treat gonococcal conjunctivitis. Gonococcal conjunctivitis usually presents in newborns 0-5 days old with irritation or discharge. Patients with this disease will have very thick discharge with a purulent appearance.\n\nAnswer E: Topical erythromycin would be used to treat chlamydial conjunctivitis in conjunction with oral erythromycin; however, it is insufficient on its own. Oral erythromycin is the best answer choice for this patient as the disease has likely disseminated. Topical erythromycin can also be used as prophylaxis for gonococcal conjunctivitis.\n\nBullet Summary:\nNeonatal chlamydial conjunctivitis presents with watery or mucoid discharge as well as eye swelling and is treated with topical and oral erythromycin.", "link": "https://step2.medbullets.com/testview?qid=109042"} {"question": "A 70-year-old woman is brought to the emergency department by ambulance. She was found on the floor of her apartment after her neighbor called 911. She is confused and is unable to provide any history, but complains of generalized pain. Her temperature is 99.2\u00b0F (37.3\u00b0C), blood pressure is 129/64 mmHg, pulse is 63/min, respirations are 13/min, and oxygen saturation is 99% on room air. Physical exam reveals, a confused, ill-appearing woman. Lungs are clear to auscultation bilaterally. An electrocardiogram is obtained as shown in Figure A. Dipstick urinalysis is notable for 4+ blood and dark colored urine. Which of the following is the most appropriate next step in management?", "choicesA": "Albuterol and IV fluid resuscitation", "choicesB": "Calcium gluconate", "choicesC": "Insulin, glucose, and IV fluid resuscitation", "choicesD": "IV fluid resuscitation", "choicesE": "Sodium polystyrene sulfonate", "answer_idx": "B", "answer": "Calcium gluconate", "explanation": "This patient with confusion, dark urine, and peaked T waves on electrocardiogram in the setting of a prolonged time spent down likely has developed rhabdomyolysis with associated hyperkalemia. For patients with suspected hyperkalemia and electrocardiogram changes, the most appropriate next step in management is treatment with calcium gluconate\n\nRhabdomyolysis occurs when muscle cells lyse and release their intracellular contents. This can occur secondary to intense athletics, seizures, stimulant drug use, or a prolonged period of immobility. Lysis of muscle cells leads to release of potassium, myoglobin, purines, and other intracellular contents. Release of large amounts of myoglobin can result in renal impairment and myoglobinuria. On dipstick urinalysis, myoglobin causes a positive result for blood. Renal impairment further increases the risk for development of hyperkalemia. On ECG, hyperkalemia manifests with a \"peaked\" appearance of T waves, especially in the precordial leads. As hyperkalemia worsens, electrocardiogram changes progress to loss of P waves, QRS complex widening, and eventual sine wave rhythm. For patients with suspected hyperkalemia and characteristic changes noted on the electrocardiogram, the most appropriate immediate step in management is treatment with intravenous calcium gluconate. Calcium gluconate acts to stabilize cardiac myocyte membranes and prevent development of life-threatening arrhythmias.\n\nGupta et. al review rhabdomyolysis. They discuss the pathophysiology, clinical manifestations and diagnosis. They further detail complications that may arise such as hyperkalemia, and discuss respective management strategies.\n\nFigure A demonstrates an electrocardiogram with features characteristic of hyperkalemia. Note the peaked appearance of the T waves in the precordial leads.\n\nIncorrect Answers:\nAnswer A: Albuterol and IV fluid resuscitation may eventually be appropriate. Albuterol (a beta agonist) will act to shift potassium to the intracellular compartment. This patient's electrocardiogram changes warrant immediate treatment with calcium gluconate in order to stabilize cardiac myocyte membranes.\n\nAnswer C: Insulin, glucose, and IV fluid resuscitation are indicated in this patient. Insulin acts to shift potassium to the intracellular compartment, and fluid resuscitation is warranted for the patient's likely renal impairment. Calcium gluconate is the more immediate priority to prevent development of arrhythmia.\n\nAnswer D: IV fluid resuscitation is indicated for this patient with likely renal impairment. However, calcium gluconate is the more immediate priority to address this patient's hyperkalemia and prevent development of arrhythmia.\n\nAnswer E: Sodium polystyrene is an oral potassium binder that increases GI excretion of potassium. It may be appropriate to treat mild hyperkalemia in some patients. It takes a prolonged amount of time to have any significant effect.\n\nBullet Summary:\nFor patients with suspected hyperkalemia and characteristic changes noted on ECG, the most appropriate immediate step in management is treatment with intravenous calcium gluconate.", "link": "https://step2.medbullets.com/testview?qid=210867"} {"question": "A 17-year-old girl presents to the clinic reporting 7 months without a menstrual period. Menarche was at age 12 and she had regular periods up until 2 years ago. At that time, her periods became less regular until around 7 months ago when they ceased altogether. She is otherwise healthy, participates in multiple school sports teams, and has been training rigorously for upcoming competitions. She denies alcohol use, smoking, and recreational drugs. She is not sexually active and does not take oral contraceptives. Her temperature is 36.7\u00b0C (98\u00b0F), blood pressure is 121/80 mmHg, pulse is 62/min, respirations are 11/min, oxygen saturation is 100% on room air, and BMI is 20 kg/m^2. Her pelvic exam reveals an anteverted uterus, no adnexal masses, a normal-appearing cervix with no cervical motion tenderness, and normal vaginal anatomy. Which of the following is the most likely cause of this patient's amenorrhea?", "choicesA": "Anorexia nervosa", "choicesB": "Functional hypothalamic amenorrhea", "choicesC": "Hypothyroidism", "choicesD": "Polycystic ovarian syndrome", "choicesE": "Pregnancy", "answer_idx": "B", "answer": "Functional hypothalamic amenorrhea", "explanation": "This patient presenting with secondary amenorrhea (defined as 6 or more consecutive months without menstruation in women who have passed menarche) and a history of rigorous exercise most likely has functional hypothalamic amenorrhea.\n\nFunctional hypothalamic amenorrhea is a condition in which relative caloric deficiency (whether through severe caloric restriction, increased energy expenditure, or increased stress) leads to functional disruption of the normal pulsatile release of gonadotropin-releasing hormone (GnRH). This in turn leads to decreased levels of follicle-stimulating hormone (FSH) and luteinizing hormone (LH), disruption to estrogen levels, and amenorrhea. A common associated clinical presentation is known as the \"female athlete triad\", which includes decreased calorie availability or increased energy expenditure through exercise, decreased bone mineral density due to a decrease in estrogen levels, and menstrual dysfunction. Treatment includes increasing caloric intake, behavioral therapy as needed, and possible estrogen replacement therapy.\n\nAckerman et al. reviews the role of estrogen replacement in improving bone mineral density in patients with functional hypothalamic amenorrhea. The authors find that spine and femoral neck bone mineral density z-scores significantly increased with estrogen replacement. The authors recommend the use of transdermal estradiol over 12 months in improving bone mineral density in patients with functional hypothalamic amenorrhea.\n\nIncorrect Answers:\nAnswer A: Anorexia nervosa will often manifest with secondary amenorrhea in women. This patient has a normal BMI and no history of disordered eating. The mechanism of amenorrhea in the setting of anorexia nervosa relates to the same hypothalamic-pituitary-ovarian axis at play in functional hypothalamic amenorrhea wherein the state of energy deficiency from inadequate caloric intake disrupts normal pulsatile GnRH secretion, leading to decreased FSH, LH, estrogen levels, and subsequent amenorrhea.\n\nAnswer C: Hypothyroidism is a cause of secondary amenorrhea wherein abnormal thyroid function can lead to altered levels of sex hormone-binding protein, prolactin, and gonadotropin-releasing hormone, thus causing menstrual dysfunction. This patient does not present with other signs of hypothyroidism, such as fatigue, sensitivity to cold, dry skin, muscle weakness, weight gain, constipation, joint stiffness, or thinning hair.\n\nAnswer D: Polycystic ovary syndrome is a condition often associated with obesity in which hyperinsulinemia or insulin resistance is hypothesized to alter the hypothalamus feedback response, leading to the increased luteinizing hormone to follicle-stimulating hormone ratio, increased androgen levels, and decreased rate of follicular maturation leading to unruptured follicles. Patients will often present with amenorrhea or oligomenorrhea, hirsutism, acne, and decreased fertility.\n\nAnswer E: Pregnancy is a cause of secondary amenorrhea that is less likely in this patient given her lack of sexual history and other associated symptoms such as decreased appetite, nausea, and insomnia. A beta-human chorionic gonadotropin (beta-hCG) test should always be part of the workup for secondary amenorrhea.\n\nBullet Summary:\nFunctional hypothalamic amenorrhea is a condition in which caloric restriction, increased energy expenditure through exercise, and/or increased stress leads to a disrupted pulsatile secretion of gonadotropin-releasing hormone with the downstream effect of amenorrhea.", "link": "https://step2.medbullets.com/testview?qid=214976"} {"question": "A 44-year-old homeless man presents to the emergency department after being stabbed multiple times in the abdomen. The patient is intoxicated and is unable to offer further history. His temperature is 97.5\u00b0F (36.4\u00b0C), blood pressure is 92/52 mmHg, pulse is 145/min, respirations are 33/min, and oxygen saturation is 90% on room air. He is intubated to protect his airway, given 2 units of packed red blood cells, and sent to the operating room for an exploratory laparotomy. The patient is subsequently admitted to the intensive care unit. He is extubated the next day and appears well. His vitals are within normal limits. Physical exam reveals sparse hair that is falling out throughout the patient\u2019s body. He has a rash by the corners of his mouth. The patient has multiple loose bowel movements and on day 6 the patient\u2019s surgical wound does not appear to be healing. It is held together only by the sutures with minimal underlying healing. Which of the following is most likely deficient in this patient?", "choicesA": "Riboflavin", "choicesB": "Thiamine", "choicesC": "Vitamin C", "choicesD": "Vitamin D", "choicesE": "Zinc", "answer_idx": "E", "answer": "Zinc", "explanation": "This homeless patient who likely has poor nutritional status presents with angular cheilitis, sparse hair that is falling out, diarrhea, and poor wound healing, all of which are suggestive of zinc deficiency.\n\nZinc is involved in many physiologic processes and is an important mineral to include in the diet. Zinc deficiency is common in patients with poor nutritional status such as the homeless, individuals living in developing nations without access to food, and patients with eating disorders. Many malabsorptive conditions can cause zinc deficiency including steatorrhea and chronic pancreatitis. Zinc deficiency presents with impaired wound healing and impaired immune function. Other symptoms suggestive of zinc deficiency include xerosis, alopecia, stomatitis, oral ulceration, angular cheilitis, impaired vision (night blindness), impaired smell/taste, diarrhea, and anorexia. The treatment of zinc deficiency involves zinc supplementation and workup for any other nutritional deficiencies.\n\nNagata etc al. study the incidence of zinc deficiency among younger patients with anorexia nervosa. The authors find that men and women are both equally likely to have zinc deficiency when admitted. The authors recommend screening all patients with eating disorders for zinc deficiency.\n\nIncorrect Answers:\nAnswer A: Riboflavin deficiency may present with angular cheilitis, stomatitis, and a non-specific rash. The treatment involves supplementing or consuming foods with riboflavin; however, it is rare in the developed world to be riboflavin deficient (due to fortified foods) and would not cause such significant wound healing issues as in this patient.\n\nAnswer B: Thiamine deficiency is common in alcoholics and can cause Wernicke-Korsakoff syndrome, which presents with ataxia, nystagmus, and ophthalmoplegia in addition to amnesia. The treatment involves the administration of high-dose thiamine.\n\nAnswer C: Vitamin C deficiency causes scurvy, which presents with easy fractures, bleeding gums or loose teeth, delayed wound healing, and perifollicular hemorrhages. It is common in the homeless and alcoholics. Treatment involves vitamin C replacement or a diet rich in fruits and vegetables.\n\nAnswer D: Vitamin D deficiency is common in the homeless, alcoholics, or those who suffer from steatorrhea (such as chronic pancreatitis or Crohn disease). It presents with weak, soft bones (osteomalacia/osteoporosis), and hypocalcemia which may present with paresthesias and tetany. Treatment involves replacement of vitamin D.\n\nBullet Summary:\nZinc deficiency presents with poor wound healing, hair loss, infertility, diarrhea, anorexia, and impaired immune function.", "link": "https://step2.medbullets.com/testview?qid=214943"} {"question": "A 51-year-old man presents to his primary care provider for intermittent shaking of his hands. For the last several years he has noticed this \"shaking\" when he brushes his teeth and prepares a cup of coffee in the morning. The shaking then gradually improves over the course of the day. His medical history is otherwise notable for hypertension and hyperlipidemia. His only home medication is pravastatin. The patient smokes half a pack per day of cigarettes and drinks 2-3 beers throughout the day. His family history is significant for Parkinson disease in his father. On physical exam, his temperature is 98.6\u00b0F (37.0\u00b0C), blood pressure is 159/84, pulse is 74/min, and respirations are 12/min. He has a high-frequency bilateral hand tremor elicited on finger-to-nose testing. His neurological exam is otherwise unremarkable. Which of the following medications should this patient be started on?", "choicesA": "Alprazolam", "choicesB": "Primidone", "choicesC": "Propranolol", "choicesD": "Topiramate", "choicesE": "Trihexyphenidyl", "answer_idx": "C", "answer": "Propranolol", "explanation": "This patient presents with a high frequency bilateral action tremor, which suggests a diagnosis of benign essential tremor. In light of his concomitant hypertension, the patient should be started on propranolol.\n\nBenign essential tremor usually presents as a bilateral tremor of the hands that is observed during fine motor movements. Patients present with worsening tremors with posturing and with voluntary movement and improve with alcohol and rest. First-line treatment includes propranolol and primidone, with propranolol being preferred for patients with hypertension or vascular risk factors. Refractory cases can be treated with deep brain stimulation which is often a last-line intervention.\n\nShanker reviewed the diagnosis and treatment of patients with essential tremors. She discusses how this disease presents with an action tremor that lasts for at least 3 years. She recommends treatment with propranolol or primidone.\n\nIncorrect Answers:\nAnswer A: Alprazolam is sometimes used as second-line therapy for essential tremor refractory to propranolol and primidone but would not be used as initial therapy in this patient. Benzodiazepines can be used in patients with panic disorders.\n\nAnswer B: Primidone is a barbiturate that is another first-line treatment for essential tremor. For this patient, propranolol would be preferred to also treat this patient\u2019s underlying hypertension. Primidone can be used in patients without other underlying comorbidities.\n\nAnswer D: Topiramate is an antiepileptic that may be used as a second-line treatment option for essential tremor. It would not be used as initial drug therapy. Topiramate can be used in the treatment of patients with partial seizures that present with motor symptoms.\n\nAnswer E: Trihexyphenidyl is an anticholinergic medication that is used to treat early Parkinson disease in which tremor is the predominant symptom. Despite this patient\u2019s family history of Parkinson disease, the tremor in Parkinson disease usually presents as an asymmetric, \u201cpill-rolling\u201d resting tremor rather than an action tremor.\n\nBullet Summary:\nPropranolol is the first-line treatment for benign essential tremor in patients with concomitant hypertension.", "link": "https://bit.ly/3RGVt6x"} {"question": "A 1-hour-old newborn boy is evaluated in the delivery room. He was born at 37 weeks gestation to a 39-year-old G3P3 mother. The mother initially labored at home with a midwife but was transferred to the hospital for failure to progress. The infant was eventually delivered via Caesarean section. The mother declined all prenatal screening tests during this pregnancy. His temperature is 98.6\u00b0F (37\u00b0C), blood pressure is 63/41 mmHg, pulse is 133/min, and respirations are 39/min. His Apgar scores were 7 and 8 at 1 and 5 minutes, respectively. His weight is 3.0 kg (6.6 lb), and his height and his head circumference are in the 30th and 40th percentiles, respectively. On physical exam, he is found to have a 3 cm full-thickness defect in the abdominal wall to the right of the umbilicus with evisceration of a loop of the bowel. The abdominal defect is immediately covered in sterile saline dressings and an orogastric tube and 2 peripheral intravenous lines are placed. This condition is most likely associated with which of the following additional findings?", "choicesA": "A normal cardiac exam", "choicesB": "Bicuspid aortic valve", "choicesC": "Endocardial cushion defect", "choicesD": "Tetralogy of Fallot", "choicesE": "Ventricular septal defect", "answer_idx": "A", "answer": "A normal cardiac exam", "explanation": "This patient presents with a full-thickness defect in the abdominal wall lateral to the umbilicus with evisceration of the bowel, which suggests a diagnosis of gastroschisis. Gastroschisis is not associated with any cardiac defects.\n\nGastroschisis and omphalocele are the most common abdominal wall defects found in infants and there are important differences between them. Whereas the eviscerated bowel in gastroschisis is not covered by a membrane, the exposed bowel in the omphalocele is covered by a membranous sac. The abdominal wall defect in gastroschisis is located lateral to the umbilicus, whereas in omphalocele the defect is midline. The distinction is also important for management, as omphalocele is commonly associated with other extraintestinal defects whereas fewer than 10% of cases of gastroschisis are associated with an extraintestinal abnormality. Treatment is with surgical correction of the abdominal wall defect.\n\nSkarsgard reviews the evidence regarding the diagnosis and treatment of gastroschisis. He discusses how this is one of the most common birth defects managed in the ICU setting. He recommends urgent surgical correction.\n\nIncorrect Answers:\nAnswer B: A bicuspid aortic valve occurs in many cases of Turner syndrome. This disease presents with a webbed neck, short stature, low hairline, broad (\u201cshield\u201d) chest, bicuspid aortic valve, horseshoe kidney, and streak ovaries. Treatment is supportive with surgical fixation of cardiac defects and hormone supplementation with growth hormone.\n\nAnswer C: Endocardial cushion defects are most commonly found in trisomy 21 (Down syndrome). Over a third of individuals with trisomy 21 have a complete atrioventricular septal defect, and another third have isolated ventricular septal defects. Infants with Down syndrome commonly present with the upslanting palpebral fissures, flat nasal bridge, transverse palmar crease, and hypotonia. Treatment is with supportive care of these deformities such as surgical correction of cardiac defects.\n\nAnswer D: Tetralogy of Fallot is most often associated with trisomy 21 (Down syndrome) and 22q11 deletion syndromes. Infants with Down syndrome commonly present with upslanting palpebral fissures, flat nasal bridge, transverse palmar crease, and hypotonia. Treatment is with supportive care of these deformities such as surgical correction of cardiac defects.\n\nAnswer E: Ventricular septal defects occur in several genetic syndromes, including trisomy 21 (Down syndrome) and trisomy 13 (Patau syndrome). Infants with trisomy 13 commonly present with cleft lip or palate, polydactyly, and severe intellectual disability. This disease is not compatible with survival past 1 year of age so treatment can involve termination of pregnancy or supportive care.\n\nBullet Summary:\nGastroschisis presents with an abdominal wall defect with evisceration of the bowel that is lateral to the umbilicus and is not usually associated with other birth defects.", "link": "https://step2.medbullets.com/testview?qid=109109"} {"question": "A 6-month-old boy is brought to the pediatrician by his parents for a routine visit. His parents report that he is feeding and stooling well and they have just started experimenting with solid foods. His parents are concerned because a few weeks ago he started rolling from back to front in addition to front to back, but he now struggles when placed on his back. The patient was noted to be developmentally appropriate for his age at his last visit 2 months ago. On physical exam, the patient seems well-nourished and has no dysmorphic features. His temperature is 98.6\u00b0F (37\u00b0C), blood pressure is 77/49 mmHg, pulse is 112/min, and respirations are 24/min. In the examination room, he is able to roll from front to back but cannot roll from back to front. He is not able to sit without support and makes no attempt at bouncing when supported in a standing position. The patient\u2019s abdomen is non-tender and soft without hepatosplenomegaly. His patellar reflexes are 4+ bilaterally. He is found to have the ocular finding seen in Figure A. Accumulation of which of the following would most likely be found in this patient\u2019s cells?", "choicesA": "Cerebroside sulfate", "choicesB": "Galactocerebroside", "choicesC": "Glucocerebroside", "choicesD": "GM2-ganglioside", "choicesE": "Sphingomyelin", "answer_idx": "D", "answer": "GM2-ganglioside", "explanation": "This 6-month-old boy who presents with developmental regression, a \u201ccherry-red\u201d macula, and hyperreflexia most likely has a diagnosis of Tay-Sachs disease. Tay-Sachs disease is caused by a deficiency of beta-hexosaminidase A that results in the accumulation of GM2-ganglioside.\n\nTay-Sachs disease is a genetic disorder caused by an autosomal recessive deficiency of the beta-hexosaminidase A enzyme. This disease presents in infants aged 2-6 months with progressive neurodegeneration, hypotonia, feeding difficulties, and a \u201ccherry red\u201d spot on the macula. Tay-Sachs disease shares many features with Niemann-Pick disease; however, Niemann-Pick disease presents with the additional findings of hepatosplenomegaly and hyporeflexia whereas Tay-Sachs presents with hyperreflexia without hepatosplenomegaly. Both diseases are more common in the Ashkenazi Jewish population. There is no curative treatment for this disease so treatment is generally supportive.\n\nSolovyeva et al. review the evidence regarding the diagnosis and treatment of Tay-Sachs disease. They discuss how this disease normally presents with muscle weakness, ataxia, speech, and mental disorders. They recommend considering novel treatments such as substrate reduction therapy.\n\nFigure/Illustration A is a fundoscopic exam demonstrating a \u201ccherry-red\u201d spot on the macula (blue circle). This finding is seen in both Tay-Sachs disease and Niemann-Pick disease.\n\nIncorrect Answers:\nAnswer A: Cerebroside sulfate accumulates in metachromic leukodystrophy, which is caused by an autosomal recessive deficiency in arylsufatase A. This disease presents with optic atrophy and seizures in childhood. Patients will later develop dementia, psychiatric disease, and neuropathy. There is no cure so treatment is supportive.\n\nAnswer B: Galactocerebroside accumulates in Krabbe disease, which is caused by an autosomal recessive deficiency in galactocerebrosidase. Krabbe disease presents in early infancy with developmental regression, hypotonia, and areflexia. Treatment is supportive with musculoskeletal bracing as needed.\n\nAnswer C: Glucocerebroside accumulates in Gaucher disease, which is caused by an autosomal recessive deficiency in glucocerebrosidase. Gaucher disease presents with pancytopenia and hepatosplenomegaly. \u201cGaucher cells,\u201d or lipid-laden macrophages resembling crumpled tissue paper, can be seen on histology. Treatment is with enzyme replacement therapy.\n\nAnswer E: Sphingomyelin accumulates in Niemann-Pick disease, an autosomal recessive sphingolipidosis caused by a deficiency in sphingomyelinase. Niemann-Pick disease presents with many of the same features as Tay-Sachs disease but has additional findings of hyporeflexia and hepatosplenomegaly.\n\nBullet Summary:\nTay-Sachs disease is caused by a deficiency of beta-hexosaminidase A resulting in the accumulation of GM2-ganglioside.", "link": "https://step2.medbullets.com/testview?qid=109134"} {"question": "A 25-year-old man presents to his primary care provider for a routine appointment. He feels well and has no complaints. His medical and surgical history is significant for the reconstruction of a torn anterior cruciate ligament in his right knee. His family history is significant for a mother with \u201cthyroid problems\u201d and a father with coronary artery disease. The patient is an information technology specialist at a local company. He is sexually active with men. His temperature is 98.6\u00b0F (37.0\u00b0C), blood pressure is 124/82 mmHg, pulse is 71/min, and respirations are 12/min. On physical exam, the patient is tall and slender with a BMI of 21 kg/m2. A single nodule can be palpated on the patient\u2019s thyroid. He has palpable cervical lymphadenopathy. On cardiac exam, he has a normal S1/S2 with no murmurs, rubs, or gallops, and his lungs are clear bilaterally. His abdomen is soft and non-tender without hepatosplenomegaly. A physical exam of the perioral region can be seen in Figure A. Which of the following additional findings is most likely to be found in this patient?", "choicesA": "Elevated plasma metanephrines", "choicesB": "Elevated serum calcium level", "choicesC": "Elevated serum prolactin level", "choicesD": "Fasting hypoglycemia", "choicesE": "Positive fecal occult blood test", "answer_idx": "A", "answer": "Elevated plasma metanephrines", "explanation": "This patient presents with tall stature, mucosal neuromas of the lips and tongue, an isolated thyroid nodule, and a family history of thyroid disease, which suggests a diagnosis of multiple endocrine neoplasia type 2 (MEN2B). MEN2B is also associated with pheochromocytomas, which cause elevated plasma metanephrines.\n\nMEN2B is caused by a gain-of-function mutation in the RET proto-oncogene and presents with medullary carcinoma of the thyroid, pheochromocytoma, Marfanoid habitus, and mucosal neuromas. Medullary thyroid carcinoma (MTC) occurs in almost all patients with MEN2B and is often metastatic at the time of diagnosis. Patients should be screened for pheochromocytoma using plasma fractionated metanephrines at the time of diagnosis of MTC. Intestinal neuromas may also lead to intestinal manifestations such as chronic constipation and megacolon. Treatment is with surgical excision of malignant lesions and active screening for future development of tumors.\n\nCastinetti et al. review the evidence regarding the diagnosis and treatment of MEN2B. They discuss how often the first presentation of this disease involves the diagnosis of medullary thyroid carcinoma. They recommend developing techniques for earlier diagnosis of this disease in order to allow for better survival.\n\nFigure/Illustration A is a clinical photograph demonstrating mucosal neuromas, which are painless nodules on the lips or tongue (red circles). These findings are characteristically seen in multiple endocrine neoplasia type 2B.\n\nIncorrect Answers:\nAnswer B: An elevated serum calcium level would suggest parathyroid hyperplasia, which is a feature of both type 1 and type 2A multiple endocrine neoplasia. Rather than hypercalcemia, this patient is likely to have an elevated serum calcitonin level due to his medullary thyroid carcinoma.\n\nAnswer C: An elevated serum prolactin level would suggest the presence of a prolactinoma, which is a pituitary tumor that may be seen in type 1 multiple endocrine neoplasia (MEN1). Other findings in this disorder include parathyroid and pancreatic tumors. Treatment is with surgical excision of malignant lesions.\n\nAnswer D: Fasting hypoglycemia would suggest the presence of an insulinoma, which is a less common pancreatic tumor that may be seen in MEN1. Other findings in this disorder include parathyroid and pancreatic tumors. Patients with an insulinoma require surgical excision in order to allow for proper glucose homeostasis.\n\nAnswer E: A positive fecal occult blood test would raise suspicion for colorectal cancer, which is a feature of Peutz-Jeghers syndrome (PJS). The extraintestinal manifestations of PJS include mucocutaneous pigmented macules on the lips and perioral region. Treatment is with screening for colorectal cancer and surgical excision of malignant lesions.\n\nBullet Summary:\nMultiple endocrine neoplasia type 2B presents with medullary thyroid carcinoma, pheochromocytoma, Marfanoid habitus, and mucosal neuromas.", "link": "https://bit.ly/3LbQCXB"} {"question": "A 29-year-old woman comes to the emergency department complaining of painful genital ulcers that have been present for 4 days as well as low-grade fever and malaise. She denies any recent travel, new sexual partners, or antibiotic use. Physical examination reveals multiple clustered vesicles and shallow ulcers in the vulvar region. A Tzanck smear shows multinucleated giant cells. Which of the following medications is most appropriate for treating her condition?", "choicesA": "Acyclovir", "choicesB": "Amoxicillin", "choicesC": "Fluconazole", "choicesD": "Metronidazole", "choicesE": "Trimethoprim-sulfamethoxazole", "answer_idx": "A", "answer": "Acyclovir", "explanation": "The patient's painful genital ulcers coupled with the presence of multinucleated giant cells on a Tzanck smear are indicative of a herpes simplex virus type 2 (HSV-2) infection. Acyclovir is the treatment of choice.\n\nHerpes infections can present with painful genital ulcers and low-grade fever. The Tzanck smear showing multinucleated giant cells further confirms the diagnosis though the diagnosis is typically made on clinical grounds and treated empirically. Acyclovir is the drug of choice for treating HSV-2 as it inhibits viral DNA polymerase, preventing the virus from replicating. The drug can reduce the duration and severity of symptoms but does not eradicate the virus entirely.\n\nGroves (2016) emphasizes that genital herpes, caused by the herpes simplex virus (HSV), affects over 400 million people worldwide. The infection is lifelong, with vesicular outbreaks in genital areas that can ulcerate. While HSV-1 and HSV-2 are both culprits, the latter increases the risk of HIV acquisition. The polymerase chain reaction assay is the recommended diagnostic method. Nucleoside analogues are effective treatments, and pregnant women with the condition are advised antiviral prophylaxis from the 36th week until delivery to prevent neonatal complications.\n\nIncorrect Answers:\nAnswer B: Amoxicillin is an antibiotic ineffective against viral infections like HSV-2. It is commonly used for bacterial infections such as streptococcal pharyngitis.\n\nAnswer C: Fluconazole is an antifungal agent is used for treating fungal infections like Candida which may cause esophagitis or vaginitis.\n\nAnswer D: Metronidazole is used primarily for anaerobic bacterial and protozoal infections.\n\nAnswer E: Trimethoprim-sulfamethoxazole is used for bacterial infections like urinary tract infections and also offers MRSA coverage.\n\nBullet Summary:\nAcyclovir is the drug of choice for treating herpes simplex virus infections.", "link": "https://bit.ly/45Rurxg"} {"question": "A 32-year-old G1P0 woman at 12 weeks gestation presents to her obstetrician for a follow-up appointment. She first presented 4 weeks ago with bilateral hand tremors. At that time, she was found to have a thyroid-stimulating hormone (TSH) concentration <0.01 mU/L and an elevated free T4. She was started on propranolol and propylthiouracil. The dosage of both medications has been titrated to the maximum dose. She reports mild improvement in her symptoms but continues to experience sweating and palpitations. She has a past medical history of polycystic ovary syndrome. Her family history is notable for hypothyroidism in her mother. She does not smoke cigarettes, stopped drinking alcohol prior to pregnancy, and does not use drugs. Her temperature is 97.6\u00b0F (36.4\u00b0C), blood pressure is 113/70 mmHg, pulse is 126/min, and respirations are 18/min. On physical exam, she appears anxious, mildly diaphoretic and has a non-tender prominence on her neck which is shown in Figure A. Which of the following is the most appropriate next step in management?", "choicesA": "Continue routine prenatal care only", "choicesB": "Methimazole", "choicesC": "Oral iodinated contrast", "choicesD": "Radioiodine ablation", "choicesE": "Thyroidectomy in the 2nd trimester", "answer_idx": "E", "answer": "Thyroidectomy in the 2nd trimester", "explanation": "This patient who initially presented with tremors, a suppressed TSH, and elevated T4 has Graves disease which was appropriately treated with propranolol and propylthiouracil. However, she has persistent symptoms (diaphoresis, palpitations, tachycardia, nontender goiter) despite maximum doses of these medications, which should then be treated with surgical thyroidectomy which can be performed safely in the 2nd trimester of pregnancy.\n\nGraves disease is caused by TSH receptor antibodies that activate the TSH receptor to synthesize thyroid hormone, leading to hyperthyroidism with suppressed TSH levels. The initial treatment for Graves disease is symptom control with a beta-blocker (e.g., propranolol) and inhibition of thyroid hormone synthesis with a thionamide (e.g., propylthiouracil or methimazole). In the 1st trimester of pregnancy, propylthiouracil is preferred to methimazole. In the 2nd and 3rd trimesters, methimazole is the preferred thionamide. The treatment of choice in pregnancy for Graves disease that is refractory to propranolol and a thionamide is thyroidectomy, which can safely be performed in the 2nd trimester. This is reserved only for more severe cases refractory to medical treatment. In nonpregnant patients, radioiodine ablation is the preferred treatment for refractory Graves disease but is contraindicated during pregnancy due to the risk of ablating the fetal thyroid.\n\nDavies et al. reviewed the pathogenesis, diagnosis, and treatment of Graves disease. They found that Graves disease is caused by autoantibodies to the TSH receptor, resulting in overactive thyroid hormone secretion. They recommend that Graves's disease be treated with antithyroid drugs, radioiodine ablation, or surgery depending on clinical characteristics.\n\nFigure A demonstrates goiter, a common finding in Graves disease that reflects thyroid growth in response to activation by TSH receptor antibodies.\n\nIncorrect Answers:\nAnswer A: Continuing routine prenatal care does not address the symptoms of the patient's hyperthyroidism. Poorly-controlled hyperthyroidism can lead to spontaneous abortion, premature labor, low birth weight, heart failure, and preeclampsia in pregnant individuals. Therefore, Graves disease should be treated in pregnant individuals.\n\nAnswer B: Methimazole is a thionamide that is an alternative to propylthiouracil in nonpregnant individuals. It is more potent than propylthiouracil. Methimazole is contraindicated during the 1st trimester of pregnancy due to its teratogenic effects on the fetus during organogenesis.\n\nAnswer C: Oral iodinated contrast agents inhibit the peripheral conversion of T4 to T3 and may be used in thyroid storm in nonpregnant patients outside of the United States. Patients with thyroid storm present with tachycardia, arrhythmias, heart failure, and hyperpyrexia.\n\nAnswer D: Radioiodine ablation is the preferred definitive treatment for Graves disease in nonpregnant individuals. It is contraindicated in pregnancy as fetal thyroid tissue has developed by 10-12 weeks of gestation and may be ablated alongside the maternal tissue.\n\nBullet Summary:\nInitial treatment of Graves disease in pregnancy is propranolol and propylthiouracil, whereas severe symptoms that are refractory to medical management should be treated with thyroidectomy.", "link": "https://step2.medbullets.com/testview?qid=108726"} {"question": "A 54-year-old man presents to the clinic with 4 weeks of persistently worsening back pain. The pain is localized to 1 spot in his lower back and is worse with physical activity. His medical history is pertinent for intravenous drug use. He has no past surgical history. His temperature is 36.8\u00b0C (98.2\u00b0F), blood pressure is 118/90 mmHg, pulse is 92/min, respirations are 13/min, and oxygen saturation is 99% on room air. On physical exam, he has midline back tenderness to palpation at L2-L3. Laboratory workup reveals an erythrocyte sedimentation rate of 112 mm/h and C-reactive protein of 10 mg/dL. Which of the following is the most likely diagnosis?", "choicesA": "Ankylosing spondylitis", "choicesB": "Degenerative spine disease", "choicesC": "Herniated disc", "choicesD": "Metastatic tumor", "choicesE": "Osteomyelitis", "answer_idx": "E", "answer": "Osteomyelitis", "explanation": "This patient with a history of intravenous drug use who presents with worsening focal back pain and elevated erythrocyte sedimentation rate (ESR) and C-reactive protein (CRP) most likely has vertebral osteomyelitis.\n\nVertebral osteomyelitis typically occurs via three routes: hematogenous spread from a distant site (the most common mechanism), direct inoculation from spinal surgery or trauma, or contiguous spread from adjacent soft tissue infection. Patients usually present with back or neck pain, with or without fever, and the most common clinical finding is local tenderness to percussion over the involved posterior spinous process. Risk factors include injection drug use, degenerative spine disease, prior spinal surgery, infective endocarditis, diabetes mellitus, corticosteroid therapy, and other immunocompromised states. Vital signs may be normal, and laboratory evaluation commonly reveals an elevated white blood cell count, ESR, and CRP. An MRI of the spine is the most appropriate diagnostic test. In stable patients, antimicrobial therapy should be withheld until a microbiological diagnosis is confirmed; once the diagnosis is confirmed, pathogen-directed therapy should then be administered.\n\nBeronius et al. conducted a retrospective study on vertebral osteomyelitis in pursuit of diagnostic criteria to simplify the diagnosis and classification of vertebral osteomyelitis. The authors found that elevated ESR and CRP are found in most patients with vertebral osteomyelitis. The authors recommend clinicians have a high index of suspicion for vertebral osteomyelitis in order to avoid delayed diagnosis.\n\nIncorrect Answers:\nAnswer A: Ankylosing spondylitis is an inflammatory arthritis of the spine and is often associated with one or more articular or periarticular extraspinal features, including synovitis, dactylitis, and enthesitis. Patients typically present with chronic back pain and loss of mobility before the age of 45.\n\nAnswer B: Degenerative spine disease involves osteoarthritis of the spine and is associated with increasing age, presenting commonly as sharp or chronic pain in the neck or back. It typically does not result in radiculopathy, defined as irritation or injury of a nerve root, causing pain, weakness, numbness, or tingling in specific distributions depending on the location of the affected nerve root. Though it is certainly on the differential for this patient, his elevated ESR and CRP make osteomyelitis a more likely diagnosis.\n\nAnswer C: Herniated disc can cause a variety of symptoms depending on the position and size of the herniation. If the herniated disk impinges on a nerve, patients may have radiculopathy.\n\nAnswer D: Metastatic tumor is typically suspected in patients with back pain in the setting of a known malignancy. The most common primary cancers that metastasize to bone include breast, prostate, thyroid, lung, and renal cancer.\n\nBullet Summary:\nVertebral osteomyelitis typically presents as focal back pain, with or without fever, with an elevated erythrocyte sedimentation rate and C-reactive protein.", "link": "https://bit.ly/3DkrY2C"} {"question": "A 55-year-old woman presents to the primary care clinic for her annual visit. She has no complaints and is asymptomatic. She has a history of hypertension, hyperlipidemia, and diabetes. Her medications are lisinopril, amlodipine, atorvastatin, and metformin. There is no family history of malignancy. She is a non-smoker and drinks 3-4 standard drinks per week. Her temperature is 98.2\u00b0F (36.8\u00b0C), blood pressure is 142/84 mmHg, pulse is 82/min, and respirations are 16/min. On physical exam, a solid, immobile neck mass is palpated inferior to the cricoid cartilage to the left of midline. An ultrasound is obtained and shows a 1.2 cm hypoechoic mass with microcalcifications. Serum thyroid-stimulating hormone (TSH) concentration is 7.2 \u00b5U/mL. Which of the following is the most appropriate next step in management?", "choicesA": "Fine needle aspiration", "choicesB": "Propylthiouracil", "choicesC": "Radioiodine ablation", "choicesD": "Surgical removal of the mass", "choicesE": "Thyroid scintigraphy", "answer_idx": "A", "answer": "Fine needle aspiration", "explanation": "This patient\u2019s presentation with an asymptomatic thyroid nodule with suspicious ultrasound features (\u2265 1 cm hypoechoic mass with microcalcifications) and elevated TSH is concerning for thyroid malignancy. The most appropriate next step in management is a fine needle aspiration (FNA).\n\nThe first step in the diagnostic work-up of a thyroid nodule, which can be found incidentally in asymptomatic individuals or in the presence of obstructive or vocal symptoms, includes an ultrasound and measurement of serum TSH levels. If TSH is normal or elevated, this suggests a hypofunctioning (\u201ccold\u201d) nodule, which increases the risk of cancer. These nodules should undergo FNA if suspicious sonographic findings are present. Suspicious sonographic findings include irregular margins, microcalcifications, taller-than-wide shape, and rim calcifications in nodules \u2265 1 cm in diameter. If TSH is low, then thyroid scintigraphy (radioiodine scan) should be performed to confirm whether the nodule is hyperfunctioning (\u201chot\u201d). Hyperfunctioning nodules are rarely cancerous, so these patients can often avoid FNA. Hyperfunctioning nodules such as toxic adenomas which cause overt hyperthyroid symptoms may warrant further treatment with radioiodine, surgery, or anti-thyroid medications.\n\nDurante et al. review the epidemiology and diagnostic approach to thyroid nodules. The authors find that nodules with suspicious sonographic patterns such as solid composition, hypoechogenicity, irregular margins, and microcalcifications should undergo cytological evaluation. The authors recommend molecular testing only in cases in which cytology is indeterminate.\n\nIncorrect Answers:\nAnswer B: Propylthiouracil is a thionamide that inhibits thyroid peroxidase, thus preventing thyroid hormone synthesis. It can be used for the symptomatic control of hyperthyroidism in toxic adenomas prior to definitive surgical resection. It is not indicated for the treatment of a hypofunctioning nodule.\n\nAnswer C: Radioiodine ablation can be used in the treatment of a toxic adenoma, which would take up the radioactive iodine. It is not indicated for the treatment of a hypofunctioning nodule, which would not exhibit radioiodine uptake.\n\nAnswer D: Surgical removal of the mass is a treatment option for a malignant thyroid nodule. Although a hypofunctioning nodule is more likely to be malignant, an FNA should be performed first to confirm the diagnosis.\n\nAnswer E: Thyroid scintigraphy is used for thyroid nodules with low TSH, which increases the probability of a hyperfunctioning nodule. Confirmation of a hyperfunctioning nodule can help prevent further invasive diagnostic work-up because hyperfunctioning nodules are unlikely to be malignant. Thyroid scintigraphy would not be appropriate in this patient with an elevated TSH.\n\nBullet Summary:\nThe next step in management for a thyroid nodule with suspicious sonographic features and an elevated TSH is fine needle aspiration.", "link": "https://step2.medbullets.com/testview?qid=215183"} {"question": "A 45-year-old man with a past medical history of obesity presents with a concern about his sexual performance. He has a history of depression which is well managed with fluoxetine. He states that since starting the medication, his sexual drive has increased, yet he is unable to achieve orgasm. As a result, he refuses to take the medication anymore. He has a past medical history of diabetes and notes that his home blood glucose readings have been higher lately. His temperature is 98.3\u00b0F (36.8\u00b0C), blood pressure is 127/82 mmHg, pulse is 88/min, respirations are 14/min, and oxygen saturation is 99% on room air. Physical exam reveals an obese man in no acute distress with a normal affect and non-focal neurologic exam. The physician agrees to change the patient\u2019s antidepressant. Which of the following is the most appropriate treatment for this patient?", "choicesA": "Bupropion", "choicesB": "Electroconvulsive therapy", "choicesC": "Escitalopram", "choicesD": "Mirtazapine", "choicesE": "Phenelzine", "answer_idx": "A", "answer": "Bupropion", "explanation": "This patient is presenting with anorgasmia in the setting of treatment with a selective serotonin reuptake inhibitor (SSRI). Given his obesity, worsening home blood glucose levels, and desire for improved sexual performance, bupropion is an appropriate medication.\n\nBupropion is an antidepressant that is not used first-line except in certain circumstances. Situations where bupropion is an appropriate agent for the management of depression include smoking cessation as it decreases smoking cravings. Bupropion is also weight neutral and is preferred in obese patients, patients who are gaining weight on other medications, and patients with poorly-managed diabetes. Finally, it does not have any sexual side effects in contrast to SSRIs. However, bupropion lowers the seizure threshold and should be avoided in patients at higher risk for seizures. Bupropion is sometimes given with other antidepressants for patients with persistent or refractory symptoms.\n\nPatel et al. review bupropion. They note the effectiveness of bupropion in managing depression and note that it can be given with other agents. They recommend its use, in particular, for smoking cessation or patients who need a more weight-neutral treatment.\n\nIncorrect Answers:\nAnswer B: Electroconvulsive therapy is a highly effective therapy for depression; however, it is generally used in cases of severe suicidality, malnutrition due to food refusal secondary to depressive disorder, or last line when other medications have failed. This patient is primarily experiencing sexual side-effects and has gained weight, making bupropion a more viable initial alternative.\n\nAnswer C: Escitalopram is merely a different SSRI that will lead to similar side effects. Thus, it would not be appropriate. Changing SSRIs is appropriate if the first SSRI does not work after an appropriate trial period of at least 4 weeks in the management of depression. Other changes when an SSRI fails to control symptoms include increasing the dose.\n\nAnswer D: Mirtazapine is an antidepressant that increases appetite and leads to weight gain and would further worsen this patient\u2019s obesity. It is preferred in conditions such as anorexia nervosa given weight gain is preferable in this population.\n\nAnswer E: Phenelzine is a monoamine oxidase inhibitor and is usually used third-line (or later) in the management of depression given side-effects such as hypertensive crisis and the long washout period and interactions with other medications. Bupropion is a safer initial alternative.\n\nBullet Summary:\nPatients with depression who experience sexual side effects or weight gain on SSRIs may be switched to bupropion.", "link": "https://step2.medbullets.com/testview?qid=216591"} {"question": "A 69-year-old patient presents to the emergency department with shortness of breath and substernal pain. The patient has a medical history of obesity, anxiety, and constipation. He is currently not taking any medications and has not filled his prescriptions for over 1 year. An ECG is obtained and is seen in Figure A. The patient is managed appropriately and is discharged on atorvastatin, metoprolol, aspirin, clopidogrel, and sodium docusate. The patient visits his primary care physician 4 weeks later and claims to be doing better. He has been taking his medications and resting. At this visit, his temperature is 99.5\u00b0F (37.5\u00b0C), blood pressure is 130/87 mmHg, pulse is 80/min, respirations are 11/min, and oxygen saturation is 96% on room air. The patient states he feels better since the incident. Which of the following is the most appropriate next step in management?", "choicesA": "Add hydrochlorothiazide", "choicesB": "Add lisinopril", "choicesC": "Add nifedipine", "choicesD": "Continue current medications and schedule follow up", "choicesE": "Switch beta-blocker to carvedilol", "answer_idx": "B", "answer": "Add lisinopril", "explanation": "This patient is presenting after a myocardial infarction for a follow-up appointment. The most appropriate next step in management is to start an angiotensin-converting enzyme (ACE) inhibitor such as lisinopril.\n\nWhen a patient experiences a myocardial infarction, there are certain medications that the patient should be started on that lower mortality. These medications include beta-blockers, aspirin, statins, other antiplatelet agents (clopidogrel), and ACE inhibitors. ACE inhibitors can reduce mortality after myocardial infarction by reducing afterload, allowing the damaged myocardium to maintain its ejection fraction. These medications also inhibit ventricular remodeling in this disease. ACE inhibitors most prominently reduce mortality if a patient's ejection fraction is reduced. Of note, ACE inhibitors also reduce mortality in diabetes by decreasing hyperfiltration damage to the glomeruli.\n\nJering et al. hypothesized that using an angiotensin receptor neprilysin inhibitor would improve mortality in patients who have sustained a myocardial infarction. They discuss how an ACE inhibitor is the standard of care in patients currently. They recommend performing a randomized controlled trial comparing these treatment modalities.\n\nFigure/Illustration A demonstrates an ECG with ST elevation (red circles) in leads II, III, and AVF. These ST elevations are indicative of a STEMI.\n\nIncorrect Answers:\nAnswer A: Hydrochlorothiazide can be used to manage this patient's blood pressure; however, this drug does not lower mortality like an ACE inhibitor. In the setting of patients with myocardial infarctions, inhibition of detrimental ventricular remodeling is key for lowering mortality.\n\nAnswer C: Nifedipine is a calcium channel blocker that can be used to reduce blood pressure; however, it does not reduce mortality in patients who have experienced an MI. Calcium channel blockers tend to have few side effects and can be safely started in most patients.\n\nAnswer D: Continuing current medications and scheduling a follow-up appointment misses the opportunity for the patient to be started on a mortality-lowering ACE inhibitor. In patients who are amenable to starting a new medication, an ACE inhibitor should be administered to all patients who have sustained an MI.\n\nAnswer E: Switching to carvedilol would be unnecessary and not likely to benefit this patient as much as adding an ACE inhibitor. Metoprolol is the preferred beta blocker in patients who have sustained an MI. In the absence of idiosyncratic side effects, there is no indication to use another beta blocker.\n\nBullet Summary:\nPatients who have experienced a myocardial infarction should be discharged on mortality-lowering drugs such as aspirin, beta-blockers, anti-platelet agents, and ACE inhibitors.", "link": "https://bit.ly/3LKCkh8"} {"question": "A 67-year-old man presents to the emergency department with shortness of breath. His dyspnea began 1 hour ago while he was sitting in a chair watching television. He also endorses lightheadedness and fatigue but denies chest pain. Two weeks ago, he had a mitral valve repair for mitral insufficiency secondary to myxomatous degeneration. His medical history is otherwise significant for hypertension and diabetes mellitus, for which he takes metformin and lisinopril. His temperature is 98.8\u00b0F (37.1\u00b0C), blood pressure is 85/54 mmHg, pulse is 48/min, and respirations are 22/min. On physical exam, he appears to be in moderate distress. He has normal cardiac sounds with an irregular rhythm. His lungs are clear to auscultation bilaterally, and his electrocardiogram can be seen in Figure A. The patient is given an initial dose of atropine but this has no effect. Which of the following is the most appropriate next step in management?", "choicesA": "Adenosine", "choicesB": "Atropine", "choicesC": "Cardioversion", "choicesD": "Procainamide", "choicesE": "Transvenous pacing", "answer_idx": "E", "answer": "Transvenous pacing", "explanation": "This patient who presents with lightheadedness has an ECG consistent with a Mobitz type 2 2nd-degree heart block. Because the patient is symptomatic from his dysrhythmia and it did not respond to atropine, the most appropriate next step is transvenous pacing.\n\nMobitz type 2 2nd-degree heart block can be distinguished from Mobitz type 1 (Wenckebach) by the length of the PR interval. Whereas the PR interval in Mobitz type 1 becomes progressively longer with each beat, the PR intervals in Mobitz type 2 are consistent in length and the QRS complexes drop suddenly. While Mobitz type 1 heart block is usually caused by atrioventricular (AV) node dysfunction, Mobitz type 2 is caused by dysfunction below the AV node in the His-Purkinje system. Patients with either type of heart block can present with lightheadedness or syncope. Since this pattern has a risk of progressing into 3rd-degree heart block, hemodynamic compromise, and asystole, Mobitz type 2 heart block is an indication for placement of a permanent pacemaker. Unstable patients who do not respond to atropine and/or other medical management should undergo transcutaneous or transvenous pacing.\n\nClark and Prystowsky present the pathophysiology of heart block. They discuss how dysfunction at the infranodal level leads to type 2 2nd-degree heart block. They recommend pacing in symptomatic patients.\n\nFigure/Illustration A is an ECG that is characterized by consistent PR intervals (red lines) and sudden drops in the QRS complex. This pattern is classically seen with a Mobitz type 2 2nd-degree heart block.\n\nIncorrect Answers:\nAnswer A: Adenosine can be used to terminate supraventricular tachycardia. These patients will present with tachycardia rather than bradycardia and will experience palpitations in addition to lightheadedness. The rhythm is rapid and regular with a narrow QRS complex.\n\nAnswer C: Cardioversion can be used to convert atrial flutter, atrial fibrillation, stable monomorphic ventricular tachycardia, or recalcitrant supraventricular tachycardia (non-responsive to adenosine or vagal maneuvers) to sinus rhythm, but it is not used for 2nd-degree heart block.\n\nAnswer B: Atropine can improve Mobitz type 1 2nd-degree heart block but tends not to be effective in Mobitz type 2 heart block in a stable patient. Atropine might be an appropriate treatment in symptomatic 3rd-degree heart block or Mobitz type 2 heart block; however, if it fails to improve a patient's symptoms, pacing is the most appropriate next step.\n\nAnswer D: Procainamide is a class 1A antiarrhythmic medication used to treat Wolff-Parkinson-White syndrome. This would present with an upgoing delta wave that occurs prior to the QRS complex. This medication does not play a role in the management of 2nd-degree heart block.\n\nBullet Summary:\nHemodynamically unstable patients with Mobitz type 2 2nd-degree heart block should be managed with transvenous pacing followed by a permanent pacemaker.", "link": "https://bit.ly/3nG6FEE"} {"question": "A 3-day-old boy is evaluated in the hospital nursery for the delayed passage of meconium. The patient is breastfed, has been feeding well every 2 hours, and is urinating over 8 times per day. He was born at 35 weeks gestation to a 27-year-old G4P4 mother. The patient is of Ashkenazi Jewish descent and the patient\u2019s parents refused all prenatal genetic testing. The pregnancy was uncomplicated until the patient\u2019s mother had a spontaneous rupture of membranes at 35 weeks of gestation. The patient\u2019s 3 older siblings are all healthy. The patient\u2019s temperature is 98.2\u00b0F (36.8\u00b0C), blood pressure is 48/32 mmHg, pulse is 164/min, and respirations are 48/min. On physical exam, he appears to be in moderate distress. He has no dysmorphic features, and his abdomen is distended and non-tender. Bowel sounds are absent. Which of the following is the most appropriate next step in management?", "choicesA": "Abdominal CT", "choicesB": "Abdominal radiograph", "choicesC": "Contrast enema", "choicesD": "Sweat testing", "choicesE": "Upper gastrointestinal series", "answer_idx": "B", "answer": "Abdominal radiograph", "explanation": "Delayed passage of meconium should be evaluated with an abdominal radiograph to rule out bowel perforation. Patients with a perforation require emergency surgery.\n\nAll neonates should pass meconium in the first 48 hours of life. Patients that fail do to so should be evaluated with an abdominal radiograph to exclude the possibility of bowel perforation. If free air is seen on the abdominal radiograph, patients should be taken for emergent surgery to repair the perforation. If bowel perforation is ruled out, a workup should be done to determine the etiology of the delayed passage, which would most likely include a contrast enema to differentiate between meconium ileus and Hirschsprung disease. Treatment is with meconium removal or excision of the defective segment respectively.\n\nAdams and Stanton review the evidence regarding the diagnosis and treatment of neonatal bowel obstruction. They discuss how malrotation, jejunoileal atresia, duodenal atresia, and colonic atresia are common causes of this finding. They recommend surgical management of these conditions.\n\nIncorrect Answers:\nAnswer A: An abdominal radiograph would be faster than an abdominal CT to rule out an indication for emergency surgery. The use of CT should also be restricted in the pediatric population due to the radiation exposure involved. Adult patients with unclear radiographic findings can be further assessed using a contrast CT scan.\n\nAnswer C: Contrast enema is the test of choice to differentiate meconium ileus from Hirschsprung disease, but the most important first step is to rule out bowel perforation. Patients who do not have perforation can then undergo this test to determine the etiology of failure to pass meconium.\n\nAnswer D: Sweat testing would aid in the diagnosis of cystic fibrosis, particularly in this patient who has had no prenatal screening. The first step in management should be an abdominal radiograph to rule out bowel perforation. Cystic fibrosis can be treated with pancreatic enzyme replacement and pulmonary therapy.\n\nAnswer E: Upper gastrointestinal series is used to make the diagnosis of intestinal malrotation, which more commonly presents with bilious vomiting secondary to midgut volvulus rather than the delayed passage of meconium. Treatment is with surgical derotation.\n\nBullet Summary:\nNeonates who fail to pass meconium in the first 48 hours of life should be urgently evaluated with an abdominal radiograph to rule out bowel perforation.", "link": "https://step2.medbullets.com/testview?qid=109241"} {"question": "A 25-year-old man presents to his primary care physician with anxiety and fatigue. During this past week, he has been unable to properly study and prepare for final exams because he is exhausted. He has been going to bed early but is unable to get a good night\u2019s sleep. He admits to occasional cocaine and marijuana use. Otherwise, the patient has no significant medical history and is not taking any medications. His temperature is 98.6\u00b0F (37\u00b0C), blood pressure is 115/71 mmHg, pulse is 72/min, and respirations are 12/min. On physical exam, the patient is a tired and anxious appearing young man. His neurological exam is within normal limits. The patient states that he fears he will fail his courses if he does not come up with a solution. Which of the following is the most appropriate initial step in management?", "choicesA": "Alprazolam", "choicesB": "Melatonin", "choicesC": "Polysomnography", "choicesD": "Sleep hygiene education", "choicesE": "Zolpidem", "answer_idx": "D", "answer": "Sleep hygiene education", "explanation": "This patient is presenting with trouble sleeping without a known organic cause, for which the most appropriate initial step in management is to discuss sleep hygiene with the patient.\n\nIn any patient that presents with a chief complaint of fatigue, it is always best to assess how they are sleeping. After a physical exam is performed in order to rule out signs or symptoms of a severe organic cause (neurologic deficits), the most appropriate initial step in management is to discuss sleep hygiene with the patient. Sleep hygiene includes discussing the patient\u2019s exposure to light (eliminate exposure to blue light at night), altering diet (no large meals or caffeine before bed), and behavioral modification (only using the bed for sleep). Once a trial of proper sleep hygiene has been tried, then it would be appropriate to potentially move on to other interventions including pharmacological interventions.\n\nHorne presents evidence regarding the best sleep hygiene practices. He discusses how exercise and avoidance of caffeine near bedtime is beneficial to helping with insomnia. He recommends using these interventions prior to pharmacologic treatment.\n\nIncorrect Answers:\nAnswer A: Alprazolam is a benzodiazepine that could be used to help with this patient\u2019s anxiety and could help them sleep. However, benzodiazepines are potent drugs with addictive properties and would not be used as first-line treatment for insomnia. Lifestyle modifications should be tried first to determine if these will be effective in helping the patient sleep.\n\nAnswer B: Melatonin is a supplement that is often used to improve sleep quality. Starting the patient on a supplement is plausible as an initial step in management, but it would not take precedence over educating the patient regarding sleep hygiene. These interventions often are very effective and have no side effects.\n\nAnswer C: Polysomnography could be the most appropriate initial step in management if an organic cause for this patient\u2019s sleep problems (such as obstructive sleep apnea) is suspected. There is nothing suggesting an organic cause for this patient\u2019s sleep issues and it is highly likely that a college-age student near finals period would benefit from a discussion on sleep hygiene prior to an expensive diagnostic test such as polysomnography.\n\nAnswer E: Zolpidem is a muscle relaxant and hypnotic with a similar mechanism of action to a benzodiazepine. Zolpidem has the potential to be addicting and would not be tried prior to sleep hygiene interventions. Some patients who are refractory to other interventions may benefit from the use of Zolpidem.\n\nBullet Summary:\nAny patient that presents with fatigue and trouble sleeping without a clear organic cause should first be counseled in sleep hygiene habits prior to any other interventions or diagnostic tests.", "link": "https://bit.ly/448sWLu"} {"question": "A 67-year-old man presents to the emergency room reporting sudden-onset abdominal and back pain that began 3 hours ago. He was eating dinner when he started feeling severe pain in his abdomen that made him lose his appetite. He denies any diarrhea or hematochezia and says the pain is not affected by movement. He has a history of hypertension, hyperlipidemia, and depression. He currently takes amlodipine and atorvastatin and is compliant with his medications. He has a 45-pack-year smoking history, but he does not drink alcohol or use any illicit drugs. His temperature is 98.6\u00b0F (37\u00b0C), his blood pressure is 110/64 mmHg, pulse is 97/min, and respirations are 15/min. Physical exam is notable for diffuse abdominal tenderness without rigidity or guarding. During evaluation, the patient becomes diaphoretic and pale and reports that he feels fatigued. Repeat blood pressure is 87/50 and pulse is 127. Intravenous fluid boluses are administered. Which of the following is the most likely diagnosis?", "choicesA": "Acute pancreatitis", "choicesB": "Aortic dissection", "choicesC": "Ischemic colitis", "choicesD": "Perforated viscus", "choicesE": "Ruptured abdominal aortic aneurysm", "answer_idx": "E", "answer": "Ruptured abdominal aortic aneurysm", "explanation": "This patient who presents with sudden-onset abdominal pain, hemodynamic instability (symptomatic hypotension with tachycardia), and significant history of hypertension and smoking most likely has a ruptured abdominal aortic aneurysm (AAA).\n\nAbdominal aortic aneurysms are dilations in the vessel wall that can predispose to rupture. Risk factors for AAA include advanced age (> 60 years), smoking history, male sex, and history of atherosclerosis. AAA can be asymptomatic or mildly symptomatic until it markedly expands or ruptures, leading to abdominal pain, back pain, or flank pain. The evaluation of suspected AAA depends on the hemodynamic stability of the patient and prior knowledge (or lack thereof) of the existence of an AAA. A hemodynamically unstable patient with a known history of AAA should be assumed to have a ruptured AAA and undergo surgical intervention without further workup (though bedside ultrasound can support the diagnosis). In an unstable patient without a known history, a bedside ultrasound to confirm the diagnosis should precede surgical intervention. In a hemodynamically stable patient with or without a known AAA history, an abdominal CTA is appropriate.\n\nSakalihasan et al. present a review of the presentation and treatment of abdominal aortic aneurysms. They discuss how patients are often asymptomatic until catastrophic rupture occurs. They recommend immediate surgical repair of the aneurysm during ruptures to prevent death.\n\nIncorrect Answers\nAnswer A: Acute pancreatitis also presents with abdominal pain and can present with hemodynamic instability in severe cases of hemorrhagic conversion of pancreatitis. This is a rare entity and is less likely in this patient. Treatment is immediate transcatheter arterial embolization for unstable patients with hemorrhagic pancreatitis.\n\nAnswer B: Aortic dissection has similar risk factors to AAA (history of hypertension, advanced age) and also presents with acute onset back pain. Dissection normally occurs in the setting of hypertension. This diagnosis is possible but less likely in this man with a history of smoking.\n\nAnswer C: Ischemic colitis can also present with abdominal pain in a patient with risk factors for atherosclerotic disease. It is associated with hematochezia and diarrhea and is unlikely to cause the level of hypotension in this patient. Treatment is with restoration of blood flow to the ischemic bowel segment or resection if this is not possible. Management is usually centered on medical optimization.\n\nAnswer D: Perforated viscus is a surgical emergency that can also present acutely with hemodynamic instability and severe abdominal pain. However, signs of peritoneal irritation are usually present (guarding, rigidity). Treatment is with immediate surgical exploration and repair or resection.\n\nBullet Summary:\nPatients with new-onset back and abdominal pain and a positive smoking history that progresses to hemodynamic instability should be suspected of having a ruptured abdominal aortic aneurysm.", "link": "https://step2.medbullets.com/testview?qid=216244"} {"question": "A 15-month-old boy presents with his mother for his well-child exam. His mother is concerned about his vision, as she noticed him squinting while watching television. The patient was born at 39 weeks of gestation via spontaneous vaginal delivery. He is up to date with routine vaccinations. He is able to walk alone, speak 3 words, and scribble with a crayon. His temperature is 98.6\u00b0F (37.0\u00b0C), blood pressure is 80/55 mmHg, pulse is 105/min, and respirations are 25/min. On exam, his pupils are round and reactive to light. A photograph of his eyes is shown in Figure A. When focusing on the pediatrician's penlight, the patient's left eye wanders. However, when the right eye is covered, the left eye refocuses on the light. The rest of his neurological exam, including extraocular movements, is within normal limits. Which of the following is the most appropriate next step in management for this patient?", "choicesA": "Brain MRI", "choicesB": "Head CT", "choicesC": "Patch the left eye", "choicesD": "Patch the right eye", "choicesE": "Perform a Bielschowsky head tilt test", "answer_idx": "D", "answer": "Patch the right eye", "explanation": "This patient, with a history that suggests decreased visual acuity, a positive cover test (refixation of the misaligned eye upon covering the appropriately aligned eye), and an asymmetric corneal light reflex in Figure A, is presenting with strabismus. Of the answer choices, only patching the sound eye (right eye) is an appropriate treatment.\n\nStrabismus occurs when a patient's eyes are not aligned properly. It often presents in children with one or both eyes turning inward, outward, upward, or downward. Patients will often tilt their heads or squint their eyes to correct this misalignment. Without proper treatment, there is a significant risk of developing amblyopia. Amblyopia is a developmental disorder in the visual cortex caused by abnormal visual stimuli. Untreated strabismus is the most common cause of amblyopia; it is secondary to visual loss from the deviated eye. The treatment of choice is wearing an eye patch over the appropriately tracking eye (sound eye), thereby encouraging the use of the deviated eye and preventing vision loss in this eye. Other treatments include surgical correction of strabismus, pharmacological penalization of the sound eye with atropine, and dichoptic movies and video games.\n\nGopal et al. discuss the pathophysiology, prevention, and therapy for amblyopia as it relates to strabismus. The authors find that the earlier clinically refractive error and strabismus are detected and treated, the greater the likelihood of preventing amblyopia. The authors recommend treating amblyopia with the correction of refractive errors, patching, and pharmacologic treatments.\n\nFigure/Illustration A shows displacement of the light reflex from the center of the pupil in this patient's right eye, as indicated by the black arrow. This is known as an asymmetric corneal light reflex and is seen in strabismus.\n\nIncorrect Answers:\nAnswer A: Brain MRI is part of the initial workup for retinoblastoma. Retinoblastoma typically presents in early childhood with visual loss, leukocoria, and an abnormal red reflex, which this patient does not have.\n\nAnswer B: Head CT in a pediatric patient could be used to rule out hemorrhage following blunt head trauma. This patient has not experienced head trauma and has no symptoms of traumatic brain injury (abnormal neurologic exam, loss of consciousness seizures). CT scans are not preferred in pediatric patients to avoid radiation exposure.\n\nAnswer C: Patching the left eye (the inappropriately tracking eye) would not treat this patient's strabismus and would likely increase his risk for amblyopia due to further decreasing his visual stimuli in this eye.\n\nAnswer E: The Bielschowsky head tilt test is used to help diagnose trochlear nerve (CN IV) palsy. Patients with a unilateral trochlear nerve and subsequent superior oblique palsy experience double vision when tilting their head to one side, which is improved by tilting the head towards the shoulder on the unaffected side. A trochlear nerve palsy typically presents with difficulty looking down and a head tilt away from the side of the lesion, unlike this patient who appears to have a visual acuity deficit.\n\nBullet Summary:\nPatching the appropriately aligned eye is the treatment of choice for strabismus to prevent progression to amblyopia.", "link": "https://step2.medbullets.com/testview?qid=216521"} {"question": "A 14-day-old girl is brought to the emergency department for diarrhea. Her mother reports that the patient began having \u201cexplosive\u201d diarrhea 6 hours ago. The patient has little interest in feeding since leaving the hospital after her birth and has not yet regained her birth weight. Her immediate postnatal period was otherwise significant for 2 episodes of bilious vomiting that resolved with the temporary use of a nasogastric tube and expulsion of stool during a digital rectal exam. The patient passed meconium on the 3rd day of life. Her temperature is 101.8\u00b0F (38.8\u00b0C), blood pressure is 58/37 mmHg, pulse is 148/min, and respirations are 38/min. On physical exam, the patient appears to be in moderate distress. She has no dysmorphic features and she has a normal S1/S2 without any murmurs, rubs, or gallops. Her abdomen is firm and distended. A digital rectal exam reveals a tight anal sphincter and precipitates a release of gas and liquid stool. Her abdominal radiograph can be seen in Figure A. Which of the following is the most accurate test to diagnose this patient\u2019s condition?", "choicesA": "Contrast enema", "choicesB": "Genetic testing", "choicesC": "Intraoperative cholangiogram", "choicesD": "Rectal suction biopsy", "choicesE": "Upper gastrointestinal series", "answer_idx": "D", "answer": "Rectal suction biopsy", "explanation": "This neonatal patient presents with bilious vomiting, failure to thrive, colonic distension on an abdominal radiograph, and new fever and tachycardia, which suggests a diagnosis of Hirschsprung disease complicated by enterocolitis. A rectal suction biopsy is the most accurate test to diagnose Hirschsprung disease.\n\nHirschsprung disease presents in the neonatal period with bilious vomiting, failure to pass meconium in the first 48 hours of life, failure to thrive, and temporary relief of the intestinal obstruction by digital rectal exam. The most severe neonatal complication of Hirschsprung disease is Hirschsprung-associated enterocolitis, which presents in the first few weeks to months of life with explosive diarrhea, fever, and abdominal pain/distension. The most appropriate first step to diagnose Hirschsprung disease is a contrast enema, which may visualize a \u201ctransition zone\u201d where the caliber of the intestine switches from the dilated proximal colon to the narrowed aganglionic segment. The most accurate test is a rectal suction biopsy, which will show a lack of ganglion cells. Surgical removal of the aganglionic portion of the colon is preferred management.\n\nKessman reviews the evidence regarding the diagnosis and treatment of Hirschsprung disease. She discusses how the diagnosis can be made using a rectal suction biopsy. She recommends monitoring patients closely for enterocolitis for years after surgical treatment.\n\nFigure/Illustration A is an abdominal radiograph demonstrating diffuse proximal colonic distension (red circle) with a paucity of air in the rectosigmoid. This finding suggests a diagnosis of Hirschsprung disease.\n\nIncorrect Answers:\nAnswer A: A contrast enema is both a diagnostic and therapeutic intervention for intussusception. It may also be used in the workup of bilious emesis to help differentiate between Hirschsprung disease and meconium ileus, but it is not the most accurate test.\n\nAnswer B: Genetic testing would be used to diagnose cystic fibrosis, which is associated with meconium ileus. Although meconium ileus presents with delayed passage of meconium (> 48 hours of life), meconium ileus is not relieved by a digital rectal exam. Treatment is with irrigation and removal of the meconium.\n\nAnswer C: Intraoperative cholangiogram is the gold standard for diagnosing biliary atresia. Biliary atresia presents with jaundice, pale stools or dark urine, and hepatomegaly over the first week to 2 months of life. Treatment is with urgent surgical intervention to prevent liver failure.\n\nAnswer E: Upper gastrointestinal series is used to diagnose intestinal malrotation, as the ligament of Treitz can be visualized on the right side of the abdomen. Malrotation may present with bilious vomiting if the malrotation has been complicated by midgut volvulus, but it would not be relieved by digital disimpaction. Treatment is with surgical derotation of the volvulus.\n\nBullet Summary:\nThe most accurate test to diagnose Hirschsprung disease is a rectal suction biopsy to demonstrate the absence of ganglion cells.", "link": "https://bit.ly/4395jl7"} {"question": "A 37-year-old woman presents to her physician for mood changes and recent falls over the course of 3 months. She feels more irritable with her husband and children and does not find pleasure in creating art anymore. Over the last 2 weeks, she has been sleeping approximately 5 hours a night and had a fall while walking in her home with the lights off. She denies any head trauma or loss of consciousness associated with the fall. She has a medical history of generalized anxiety disorder and hypothyroidism for which she takes escitalopram and levothyroxine. She had a Roux-en-Y gastric bypass 5 years ago. She has been in a monogamous relationship with her husband for the last 15 years and uses a copper intrauterine device. Physical examination is notable for a depressed affect. Pupils are equal, round, and reactive to light and accommodation. She has 4/5 strength in the bilateral lower extremities and brisk patellar reflexes. Which of the following is most likely the cause of this patient's symptoms?", "choicesA": "Poor iron absorption", "choicesB": "Prior Treponema pallidum exposure", "choicesC": "Reduced central nervous system dopamine and serotonin", "choicesD": "Reduced functional parietal cell count", "choicesE": "Subtherapeutic levothyroxine dose", "answer_idx": "D", "answer": "Reduced functional parietal cell count", "explanation": "This patient's mood changes and findings concerning for subacute combined degeneration (fall in the dark, impaired proprioception, weakness, and brisk patellar reflexes), and history of Roux-en-Y gastric bypass surgery are concerning for vitamin B12 deficiency. Roux-en-y gastric bypass reduces parietal cell count, leading to reduced intrinsic factor secretion, which is needed for vitamin B12 absorption.\n\nRoux-en-Y gastric bypass is a laparoscopic bariatric procedure that creates a small gastric pouch from the stomach with a gastrojejunal and jejunojejunal anastomosis. This causes small volumes of food to be consumed and decreases nutrient absorption. The small gastric pouch contains very few parietal cells, which decreases hydrochloric acid and intrinsic factor secretion. The reduced intrinsic factor secretion impairs the protection of vitamin B12 against HCl and reduces absorption in the ileum. Vitamin B12 deficiency can lead to neuropsychiatric changes (depressed mood, irritability, and insomnia) and subacute combined degeneration (weakness, sensory ataxia, peripheral neuropathy, and upper motor neuron signs). Patients who undergo Roux-en-Y gastric bypass must be given high doses of vitamin B12 (often intramuscularly) in order to prevent deficiency.\n\nAlexandrou et al. review long-term micronutrient deficiencies after sleeve gastrectomy and Roux-en-Y gastric bypass (RYGB). The authors find that vitamin B12 deficiency was significantly higher in patients with RYGB compared with sleeve gastrectomy. The authors recommend adherence to supplemental iron and vitamin intake in all patients who undergo bariatric surgery.\n\nIncorrect Answers:\nAnswer A: Poor iron absorption is a well-known complication of Roux-en-Y gastric bypass surgery due to bypass of the duodenum and proximal jejunum. Poor iron absorption leads to microcytic anemia from iron deficiency and presents with generalized fatigue and conjunctival pallor. Iron deficiency anemia would not present with brisk reflexes and impaired proprioception.\n\nAnswer B: Prior Treponema pallidum exposure can result in tabes dorsalis, a neurological complication of syphilis, and is seen in patients with a history of multiple sexual partners who do not use barrier protection. The spirochete leads to disease in the posterior columns (vibration and proprioception sensation). Patients present with sensory ataxia (similar to subacute combined degeneration), lancinating pain (in the limbs, back, or face), and an Argyll-Robertson pupil (impaired pupillary constriction to light but intact pupillary constriction to accommodation).\n\nAnswer C: Reduced central nervous system dopamine and serotonin are seen in patients with major depressive disorder which does not cause sensory ataxia with brisk reflexes.\n\nAnswer E: Subtherapeutic levothyroxine dose would result in hypothyroidism, which can present with depressed mood, cold intolerance, bradycardia, and delayed relaxation of deep tendon reflexes. Roux-en-Y is an effective means of losing weight, and as the body mass decreases with weight loss, the required dose of levothyroxine is decreased. It would not present with the abnormal neurologic findings seen in this patient.\n\nBullet Summary:\nRoux-en-Y bariatric surgery increases the risk of vitamin B12 deficiency via reduction of food intake, decreased functional parietal cell count, and reduced absorption of vitamin B12 (which may cause subacute combined degeneration).", "link": "https://bit.ly/3P8eDBc"} {"question": "A 23-year-old man presents to the emergency department after he was rescued while hiking in Alaska. The patient was found on a mountain trail non-responsive. His temperature is 91\u00b0F (32.8\u00b0C), blood pressure is 103/61 mmHg, pulse is 60/min, respirations are 11/min, and oxygen saturation is 97% on room air. The patient is placed under a warm air circulator, and his extremities are bathed in warm water. An ECG is performed as seen in Figure A. Dark bullae are noted on the patient's extremities. His mental status rapidly improves with warming. Laboratory studies are ordered and are pending. Which of the following is most likely to be found in this patient?", "choicesA": "Elevated troponin", "choicesB": "Hypocalcemia", "choicesC": "Hypomagnesemia", "choicesD": "Hypophosphatemia", "choicesE": "Red blood cells on urine microscopy", "answer_idx": "B", "answer": "Hypocalcemia", "explanation": "This patient is presenting after being found outside with hypothermia and frostbite (dark bullae noted on the extremities in the setting of being out in the cold). Frostbite can lead to rhabdomyolysis, which may cause hyperkalemia (causing ECG changes including peaked T waves and QRS widening as seen in this patient) and hypocalcemia (the most common electrolyte finding in frostbite).\n\nFrostbite occurs when there is an extensive freezing injury to tissues and can cause benign swelling, erythema, and pain, all the way to insensate extremities with necrosis and permanent dysfunction. More severe forms of frostbite can cause rhabdomyolysis or breakdown of the muscles. Breakdown of the muscle cells leads to leakage of the intracellular contents. Common electrolyte derangements include hyperkalemia and hypophosphatemia. Hypocalcemia is the most common electrolyte derangement in rhabdomyolysis. It occurs due to the deposition of calcium in the necrotic muscle cells and the sequestration of calcium in the exposed sarcoplasmic reticulum. Note that hypocalcemia requires no treatment unless it is causing symptoms or ECG changes (such as QT prolongation). The management of rhabdomyolysis is centered on fluid administration to protect the kidney from myoglobin which is released from damaged muscles and is nephrotoxic.\n\nZimmerman and Shen review rhabdomyolysis. They note that rhabdomyolysis can be commonly associated with myoglobinuria, hyperkalemia, hypocalcemia, and hyperphosphatemia. They suggest that an elevated creatine kinase level is indicated to support the diagnosis.\n\nFigure/Illustration A is an ECG demonstrating peaked T waves (blue arrows) which are associated with hyperkalemia.\n\nIncorrect Answers:\nAnswer A: Elevated troponin may be seen in rhabdomyolysis. This is a non-cardiac cause of an elevated troponin; however, this is less common than hypocalcemia in rhabdomyolysis and is neither sensitive nor specific.\n\nAnswer C: Hypomagnesemia is not a common electrolyte disturbance seen in rhabdomyolysis; however, hypomagnesemia may cause hypocalcemia (from impaired PTH release) and hypokalemia (from renal wasting of potassium). Note that this mechanism is separate from rhabdomyolysis.\n\nAnswer D: Hypophosphatemia is common in refeeding syndrome and malnourished patients (such as those with alcohol use disorder) and presents with diffuse muscle weakness. Rather, hyperphosphatemia would be seen in rhabdomyolysis as it is released from damaged muscle cells.\n\nAnswer E: Red blood cells on urine microscopy would be seen in a nephritic syndrome in addition to red blood cell casts. Note that the urine dipstick will be positive for red blood cells in rhabdomyolysis due to cross-reactivity with myoglobin; however, there are minimal/no red blood cells actually present.\n\nBullet Summary:\nFrostbite can result in rhabdomyolysis which commonly causes hyperkalemia, hyperphosphatemia, and hypocalcemia (the most common derangement).", "link": "https://step2.medbullets.com/testview?qid=216427"} {"question": "An 8-year-old boy is brought to the emergency department by his parents for evaluation. He has had headache and fever for the last 2 days, and this morning he was confused and had difficulty answering questions. He also developed a rash this morning. He recently returned from summer camp. He has no significant medical history and takes no medications. His temperature is 104\u00b0F (40\u00b0C), pulse is 120/min, blood pressure is 105/60 mmHg, and respirations are 22/min. On exam, flexion of the neck causes flexion of the hips and knees. Fundoscopic exam reveals no papilledema. Examination of the patient's rash is shown in Figure A. Which of the following is the most appropriate next step in management?", "choicesA": "Blood and urine cultures", "choicesB": "Lumbar puncture, ceftriaxone, vancomycin, and methylprednisolone", "choicesC": "MRI brain", "choicesD": "Vancomycin and ceftriaxone", "choicesE": "Vancomycin, ceftriaxone, and dexamethasone", "answer_idx": "B", "answer": "Lumbar puncture, ceftriaxone, vancomycin, and methylprednisolone", "explanation": "This patient with fever, headache, signs of meningeal irritation, and a purpuric rash likely has bacterial meningitis. The most appropriate initial step in management is a lumbar puncture as well as ceftriaxone, vancomycin, and methylprednisolone.\n\nMeningitis classically presents with fever, neck stiffness or headache, and altered mental status. Specific physical exam findings that can suggest the diagnosis include meningeal signs such as a positive Kernig sign (inability to extend the knee while the hip is flexed) and Brudzinski sign (flexion of the neck results in flexion of lower extremities). Common causative agents include Streptococcus pneumoniae and Neisseria meningitidis. The latter of which is associated with the purpuric rash seen in this patient. The best initial step in management is a lumbar puncture followed promptly by IV broad spectrum antibiotics. Once the diagnosis/infectious agent is confirmed, the antibiotic agents can be refined. Other early steps in management include early blood cultures and fluid resuscitation. Steroids should be given in the setting of suspected Waterhouse-Friderichsen syndrome and concern for adrenal failure.\n\nLinder et. al review meningococcal meningitis. They discuss the pathophysiology, clinical manifestations, and diagnosis. They describe the association of purpuric rash with meningococcal infection, and note that this is indicative of disseminated intravascular coagulation.\n\nFigure A. demonstrates a purpuric rash over the lower extremities, which is classically associated with meningococcal meningitis. However, this can occur with meningitis or bacteremia from other infectious agents, as it is indicative of disseminated intravascular coagulation.\n\nIncorrect Answers:\nAnswer A: Blood and urine cultures would be appropriate initial management for a fever of unknown origin and in this patient. Early antibiotics must be given as well to reduce mortality and improve outcomes.\n\nAnswer C: MRI brain would not be appropriate in this setting. While CNS imaging is sometimes indicated prior to lumbar puncture if signs of increased intracranial pressure such as papilledema are present, CT imaging is typically preferred. Time required to obtain MR imaging is prohibitive and would delay definitive care for this patient.\n\nAnswer D: Vancomycin and ceftriaxone are an appropriate initial antibiotic regimen for meningitis in children and adults. However, steroids are often added empirically. Lumbar puncture should be performed prior to administration of antibiotics (if possible) to avoid sterilization of the CSF.\n\nAnswer E: Vancomycin, ceftriaxone, and prednisone are an appropriate initial regimen for meningitis in adults. Steroids have been shown to reduce inflammation and prevent neurologic sequelae. However, lumbar puncture should be performed prior to administration of antibiotics if possible to avoid sterilization of the CSF. Also, IV steroids are preferred rather than oral in ill patients.\n\nBullet Summary:\nThe best initial step in management for meningitis is a lumbar puncture followed by broad-spectrum antibiotics and steroids.", "link": "https://bit.ly/46WSQly"} {"question": "A 55-year-old man presents to the emergency department with chest pressure and diaphoresis. His symptoms started at work when he was lifting boxes. He states that he currently feels crushing pressure over his chest. He is given aspirin and an ECG is performed in triage, as seen in Figure A. His past medical history is notable for diabetes and obesity. He has smoked 1 pack of cigarettes per day for the past 30 years. His temperature is 97.9\u00b0F (36.6\u00b0C), blood pressure is 155/99 mmHg, pulse is 110/min, respirations are 22/min, and oxygen saturation is 98% on room air. Physical exam reveals an uncomfortable, diaphoretic man. His heart and breath sounds are unremarkable. A repeat ECG in the emergency department is performed, also seen in Figure A. The patient\u2019s chest pain is worse when the ECG in the emergency department is taken, compared to the one taken in triage. He is given nitroglycerin, with minimal improvement in his symptoms. A troponin level is drawn. Which of the following is the most appropriate next step in management?", "choicesA": "Cardiac catheterization", "choicesB": "Chest radiograph", "choicesC": "CK-MB level", "choicesD": "Heparin", "choicesE": "Repeat ECG in 10 minutes", "answer_idx": "E", "answer": "Repeat ECG in 10 minutes", "explanation": "This patient is presenting with a pattern of chest pain (pressure and diaphoresis) that is highly concerning for an acute myocardial infarction. This patient initially has ST depressions (in triage) followed by ST segment normalization and possibly hyperacute T waves which raises concern for an acute ST elevation myocardial infarction (STEMI) requiring frequent ECGs to determine if a STEMI evolves.\n\nA STEMI presents with crushing substernal chest pain, diaphoresis, chest pressure, nausea, vomiting, and abdominal pain among many other possible presentations. When a patient presents with symptoms concerning for a STEMI, aspirin should be administered and an ECG should be performed. Early in a myocardial infarction, ST depressions with hyperacute T waves may be present. As the infarction evolves, the T waves may become more acute and the ST segment will begin to elevate. There may be a brief period of pseudonormalization where the ST segment appears to be normal. If the patient appears better, this can be reassuring. However, ongoing chest pain in the setting of these findings raises concern for pseudonormalization of the ST segment (an ST segment that is going to continue to rise). Nitroglycerin can be administered to see if the patient can become chest pain-free, and serial ECGs should be performed to determine if a STEMI is present. If symptoms resolve and the ECG is stable, this is reassuring and medical optimization and an elective catheterization may be performed. If the ECG evolves into a STEMI, then cardiac catheterization is indicated emergently.\n\nDaga et al. discuss STEMI and its workup. They note the ECG manifestations, and the pathophysiology in addition to the appropriate workup of acute coronary syndrome. They recommend aspirin and catheterization in an acute STEMI.\n\nFigure/Illustration A demonstrates an initial ECG with ST depression (blue arrows) followed by pseudonormalization of the ST segment (green arrows) and possibly hyperacute T waves (black arrows), which are concerning for ischemia.\n\nIncorrect Answers:\nAnswer A: Cardiac catheterization may be required in this patient; however, if the patient\u2019s chest pain improves or if the troponin is negative, it is unlikely he is having an acute myocardial infarction requiring immediate catheterization. Repeating the ECG will allow for triaging of the patient and will aid in determining if emergency cardiac catheterization is needed.\n\nAnswer B: Chest radiography is appropriate in the workup of chest pain as it can rule out diagnoses such as pneumothorax or may suggest diagnoses such as an aortic dissection (type A) if there is a widened mediastinum. In this patient, the ST segment changes and characteristic history make acute coronary syndrome a more likely diagnosis, and while a chest radiograph should be performed at some point, repeating the ECG is a more dire initial step in management.\n\nAnswer C: CK-MB level could be helpful in determining if a myocardial infarction has occurred as it elevates rapidly and also decreases fairly rapidly. This makes it useful when diagnosing a new myocardial infarction after a recent infarction as the troponin may still be elevated from the previous infarction. This patient has no history of recent myocardial infarction, and a repeat ECG is more dire to determine if this patient needs emergent intervention. Of note, the decision for emergent catheterization should not wait on cardiac biomarkers.\n\nAnswer D: Heparin should not be given empirically to patients in chest pain. It does not necessarily decrease mortality but does increase the risk of bleeding. It is often given to patients with an NSTEMI.\n\nBullet Summary:\nPseudonormalization occurs when an ST depression begins to elevate and appears normal in the setting of a myocardial infarction, requiring close monitoring and serial ECGs to ensure that a ST elevation myocardial infarction does not occur.", "link": "https://step2.medbullets.com/testview?qid=216617"} {"question": "A 47-year-old woman presents to the clinic with 3 weeks of increased thirst and urination. She has a history of obesity, hypertension, and depression for which she takes losartan and sertraline. She has smoked 1 pack of cigarettes per day for 30 years. Her temperature is 99.5\u00b0F (37.5\u00b0C), blood pressure is 148/90 mmHg, pulse is 84/min, and respirations are 15/min. Physical exam is unremarkable, including no costovertebral or suprapubic tenderness to palpation. Laboratory studies are performed and show:\n\nSerum:\nNa+: 140 mEq/L\nK+: 4.4 mEq/L\nHCO3-: 21 mEq/L\nBlood urea nitrogen (BUN): 38 mg/dL\nCreatinine: 1.1 mg/dL\nGlucose: 215 mg/dL\n\nWhich of the following is the most appropriate screening test for nephropathy in this patient?", "choicesA": "Creatinine clearance", "choicesB": "Hemoglobin A1c", "choicesC": "Oral glucose tolerance test", "choicesD": "Urine albumin-to-creatinine ratio", "choicesE": "Urine protein dipstick", "answer_idx": "D", "answer": "Urine albumin-to-creatinine ratio", "explanation": "This patient with a history of obesity and hypertension presenting with polyuria and polydipsia in the setting of an elevated random blood glucose (> 200 mg/dL) likely has a new diagnosis of type 2 diabetes mellitus. The urine albumin-to-creatinine ratio is the most appropriate early test for diabetic nephropathy.\n\nEarly detection of diabetic nephropathy is key to management, as the complication eventually leads to overt kidney disease and renal failure. The diagnosis of diabetic nephropathy is based on > 30 mg/g of albuminuria for > 3 months. Patients with type 2 diabetes mellitus should be screened annually with a random urine albumin-to-creatinine ratio starting at diagnosis. Elevation of urine albumin warrants treatment with ACE inhibitors or angiotensin receptor blockers. Blood pressure control slows the progression of diabetic nephropathy. Patients with type 2 diabetes mellitus and diabetic nephropathy should be started on a sodium-glucose cotransporter 2 (SGLT2) inhibitor. Patients with persistent albuminuria despite both ACE/ARB and SGLT2 inhibitor therapy should be started on the nonsteroidal selective mineralocorticoid receptor antagonist (MRA) finerenone.\n\nBakris et al. study the efficacy of finerenone, a nonsteroidal selective MRA, in the treatment of patients with albuminuria in patients with chronic kidney disease (CKD) and type 2 diabetes. The authors found in a randomized control trial that finerenone reduced the incidence of a composite outcome of kidney failure. The authors recommend the use of finerenone in patients with persistent albuminuria despite optimal medical therapy to prevent risks of CKD progression and cardiovascular events.\n\nIncorrect Answers:\nAnswer A: Creatinine clearance changes over the time course of diabetic nephropathy and can be normal or elevated (due to glomerular hyperfiltration) in early diabetic nephropathy. Therefore, it is an unreliable test for early diabetic nephropathy compared to the more sensitive urine albumin-to-creatinine ratio.\n\nAnswer B: Hemoglobin A1c is a test that reflects the average blood glucose over the past 2 to 3 months. While this test provides information on a patient's glycemic control, it does not directly assess the presence or extent of diabetic nephropathy.\n\nAnswer C: Oral glucose tolerance test is the test of choice for diagnosing gestational diabetes. It does not play a role in the detection of diabetic nephropathy.\n\nAnswer E: Urine protein dipstick is a useful test to detect proteinuria. This test is insensitive to low levels of albumin. Therefore, it is considered an inferior test to urine albumin-to-creatinine ratio in the diagnosis of diabetic nephropathy.\n\nBullet Summary:\nUrine albumin-to-creatinine ratio testing is the most sensitive test to detect moderately elevated levels of urinary albumin excretion, a known byproduct of diabetic nephropathy.", "link": "https://step2.medbullets.com/testview?qid=215044"} {"question": "A 60-year-old man presents to his primary care physician with weight gain. He states that ever since his wife died 4 months ago, he has been eating and sleeping more and no longer engages in any activities he once enjoyed such as hiking or fishing. He feels guilty for not spending more time with his wife before she died. He was recently fired for making several major bookkeeping mistakes at work as an accountant as he had trouble focusing. The patient is requesting oxycodone at this appointment as he states he has burning pain in his legs that feels worse now than it has in years past. The patient has a past medical history of obesity, poorly controlled diabetes, hypertension, and peripheral vascular disease. His temperature is 98.7\u00b0F (37.1\u00b0C), blood pressure is 129/82 mmHg, pulse is 83/min, respirations are 15/min, and oxygen saturation is 99% on room air. Physical exam reveals a stable gait and reduced sensation symmetrically over the lower extremities. He complains of electric pain when touching his lower extremities. His affect seems depressed. Which of the following is the most appropriate treatment for this patient?", "choicesA": "Amitriptyline", "choicesB": "Bupropion", "choicesC": "Escitalopram", "choicesD": "Phenelzine", "choicesE": "Venlafaxine", "answer_idx": "E", "answer": "Venlafaxine", "explanation": "This patient is presenting with sleep changes, interest loss, guilt, decrease in energy, and trouble concentrating for greater than 2 weeks concerning for major depressive disorder. In the setting of his diabetic neuropathy (lower extremity burning pain in the setting of poorly managed diabetes), a serotonin-norepinephrine reuptake inhibitor (SNRI) such as venlafaxine is the most appropriate initial treatment for this patient.\n\nMajor depressive disorder should always initially be managed with cognitive behavioral therapy and often medication. Typically, the first-line medication is a selective serotonin reuptake inhibitor (SSRI). However, SNRIs such as venlafaxine and duloxetine are viable first-line options. They are particularly preferred in patients with diabetic neuropathy as they treat both diabetic neuropathy and depression. SNRIs are also generally considered to be less sedating than SSRIs. For this reason, SNRIs are often a better initial drug in depression than SSRIs in diabetic patients with peripheral neuropathy.\n\nSansone and Sansone discuss SNRIs and their uses. They discuss how there are different efficacies and propensities of the different SNRIs. They recommend choosing an appropriate SNRI based on the patient's needs and comorbidities.\n\nIncorrect Answers:\nAnswer A: Amitriptyline is a tricyclic antidepressant that can be used to treat depression in patients who also suffer from other neuropathic pain conditions or conditions such as fibromyalgia or irritable bowel syndrome. While it may also be used in diabetic neuropathy and depression, there is higher toxicity with this medication than SNRIs. For this reason, SNRIs are preferred. A TCA might be appropriate if the patient had already failed other first-line agents.\n\nAnswer B: Bupropion is a weight-neutral antidepressant that can be used in smoking cessation and patients who are trying to avoid the sexual side effects of SSRIs (anorgasmia). It would not address this patient's diabetic neuropathy, though it could be beneficial for his obesity.\n\nAnswer C: Escitalopram is an SSRI that could be used first-line for major depressive disorder. However, if a patient has a concomitant condition, it is better to choose a single agent that addresses both problems; thus, SNRIs are better first-line medications in this patient with diabetic neuropathy.\n\nAnswer D: Phenelzine is a monoamine oxidase inhibitor that can be used in atypical depression or as a third-line or later agent in the management of depression. This agent can cause a hypertensive crisis in the setting of tyramine consumption, has many medication interactions, and a long washout period. Thus, it is not usually a first or second-line agent. However, it is highly effective in patients who have failed other first-line agents and should still be used in appropriate candidates.\n\nBullet Summary:\nSNRIs are appropriate first-line agents for major depressive disorder in patients with diabetic neuropathy.", "link": "https://step2.medbullets.com/testview?qid=216594"} {"question": "A 68-year-old woman presents to the emergency room with several days of frequent, nonbloody, watery stools. She recently completed a course of clindamycin 2 weeks ago for a tooth infection. Her medical history is notable for hypertension and hyperlipidemia for which she takes amlodipine and atorvastatin. She traveled recently and has no sick contacts. Her temperature is 37.0\u00b0C (98.6\u00b0F), blood pressure is 118/82 mmHg, pulse is 98/min, respirations are 14/min, and oxygen saturation is 98% on room air. On exam, her abdomen is soft, non-distended, and non-tender to palpation; however she has diffuse discomfort upon palpation of the abdomen with no rebound, rigidity, or guarding. Which of the following is most likely to confirm the diagnosis?", "choicesA": "Abdominal radiograph", "choicesB": "CT imaging of abdomen and pelvis", "choicesC": "Flexible sigmoidoscopy", "choicesD": "Rectal swab for selective anaerobic culture", "choicesE": "Stool toxin assay", "answer_idx": "E", "answer": "Stool toxin assay", "explanation": "This patient presenting with cramping abdominal pain and frequent watery diarrhea a few weeks after finishing a course of antibiotics most likely has Clostridioides difficile infection, for which the most appropriate confirmatory test is a stool toxin assay.\n\nC. difficile infection should be suspected in patients with acute and clinically significant diarrhea with no obvious alternative explanation. It is associated with risk factors such as recent antibiotic use, hospitalization, and advanced age. Diagnosis is established via a positive enzyme immunoassay for C. difficile toxins and C. difficile glutamate dehydrogenase (GDH), or a positive nucleic acid amplification test (NAAT) for C. difficile toxins. The GDH enzyme immunoassay is highly sensitive for detecting C. difficile, but not specific for toxin-producing C. difficile, and thus the toxin enzyme immunoassay is submitted in conjunction. If the results of both are discordant, then NAAT is used to make the diagnosis. Treatment for C. difficile infection differs by disease severity and recurrence. Commonly used medications include fidaxomicin and oral vancomycin.\n\nMcDonald et al. review the guidelines for diagnosis and treatment of C. difficile and provide the algorithm for C. difficile testing as mentioned above. The authors also note that only liquid stool from patients with clinically significant and acute diarrhea should be submitted for testing. The authors recommend that formed stool from asymptomatic patients should not be tested because the presence of the C. difficile toxin gene does not distinguish between infection and asymptomatic carriage.\n\nIncorrect Answers:\nAnswer A: Abdominal radiograph is an appropriate adjunctive diagnostic tool in patients with severe disease (e.g., severe abdominal pain and distension, fever, hypovolemia, lactic acidosis, marked leukocytosis) or fulminant colitis (marked by hypotension and ileus) to evaluate for toxic megacolon, bowel perforation, or other findings that warrant surgical intervention. In patients who are stable with low concern for severe disease or fulminant colitis, it would not be the preferred initial test for diagnosis nor would it be confirmatory.\n\nAnswer B: CT imaging of the abdomen and pelvis with oral and intravenous contrast is the preferred modality for detecting the presence of toxic megacolon, bowel perforation, or other findings that warrant surgical intervention in the setting of C. difficile colitis. Colonic dilatation of > 7 cm in diameter in the clinical setting of severe C. difficile infection is diagnostic. Other findings may include small bowel dilatation or scalloping of the bowel wall due to submucosal edema. While it would be reasonable to perform a CT scan in this patient, it would not confirm the diagnosis.\n\nAnswer C: Flexible sigmoidoscopy would be appropriate for cases in which an alternative diagnosis is suspected that requires direct visualization and/or biopsy of the bowel mucosa. It may also be helpful for patients with ileus or colitis in the absence of diarrhea as it allows for visualization of pseudomembranes on the inflamed mucosal surface, which are highly suggestive of C. difficile infection. In general, colonoscopy is a preferred and more accurate test.\n\nAnswer D: Rectal swab for selective anaerobic culture involves culture on a selective medium with toxin testing of isolated C. difficile. It is a highly sensitive test, but culture cannot distinguish toxin-producing strains from non-toxin-producing strains. A second test is required to detect toxin production by the cultured C. difficile strain (either enzyme immunoassay or nucleic acid amplification testing). Culture takes several days to finalize and as such, it is seldom used to establish a clinical diagnosis. It is most useful in patients with ileus and suspected C. difficile infection.\n\nBullet Summary:\nThe most appropriate confirmatory test for diagnosis of Clostridioides difficile infection is a stool toxin assay.", "link": "https://step2.medbullets.com/testview?qid=216371"} {"question": "A 21-year-old woman presents to her primary care physician due to a history of abdominal pain. She states that she has had recurrent bouts of mood swings, bloating, and abdominal pain that occur together approximately every 4 weeks. She states that when these symptoms occur, she gets into frequent arguments at work, is unable to concentrate, and sleeps longer than usual. Furthermore, she has episodes of extreme anxiety during these periods, leading her to take off time from work. She notes that these symptoms are causing distress in her interpersonal relationships as well. She has no past medical history and takes no medications. She experienced menarche at age 12 and has regular periods. Her temperature is 98.2\u00b0F (36.8\u00b0C), blood pressure is 100/70 mmHg, pulse is 75/min, and respirations are 12/min. Physical exam reveals a nontender abdomen. Pelvic exam reveals a closed cervix with no cervical motion or adnexal tenderness. Her cardiopulmonary and neurological exams are unremarkable. Which of the following is the most likely diagnosis?", "choicesA": "Generalized anxiety disorder", "choicesB": "Major depressive disorder", "choicesC": "Panic disorder", "choicesD": "Premenstrual dysophoric disorder", "choicesE": "Premenstrual syndrome", "answer_idx": "D", "answer": "Premenstrual dysophoric disorder", "explanation": "This patient with recurrent abdominal pain, bloating, and mood changes causing interpersonal difficulties related to the menstrual cycle likely has premenstrual dysphoric disorder. Premenstrual dysphoric disorder is a variant of premenstrual syndrome (PMS) characterized by prominent irritability and disruption of everyday life.\n\nPremenstrual dysphoric disorder is a variant of PMS characterized by recurrent physical and behavioral symptoms that manifest during the luteal phase of the menstrual cycle and resolve with menstruation. The diagnosis is made by criteria outlined by the Diagnostic and Statistical Manual of Mental Disorders (DSM-5). To make a diagnosis of premenstrual dysphoric disorder, symptoms such as marked affective lability or marked irritability or anger must be present along with signs such as decreased concentration, hypersomnia or insomnia, and decreased interest in usual activities. These symptoms must cause significant distress or interfere with work, school, or relationships and must present 1 week before the onset of menses and resolve the week following menses. The initial management of premenstrual dysphoric disorder involves a pregnancy test to rule out pregnancy and a menstrual diary to prospectively confirm symptoms and their relationship to the menstrual cycle for 2 consecutive cycles. Following confirmation of a diagnosis of premenstrual dysphoric disorder, treatment involves lifestyle changes, antidepressant/anxiolytic medications, or hormonal therapies such as oral contraceptive pills (OCPs).\n\nBhatia et al. discuss premenstrual dysphoric disorder. They note that premenstrual dysphoric disorder affects 2-10% of women of reproductive age. They note that serotonin dysregulation may be relevant to the etiology and treatment of premenstrual dysphoric disorder.\n\nIncorrect Answers:\nAnswer A: Generalized anxiety disorder (GAD) is anxiety characterized by symptoms such as restlessness, fatigue, and irritability lasting > 6 months unrelated to any specific triggers. The topics of anxiety should be unrelated to one another. This patient's anxiety and symptoms are temporally related to her menstrual cycle, making premenstrual dysphoric disorder a more likely diagnosis.\n\nAnswer B: Major depressive disorder (MDD) is characterized by depressed mood and at least 5 of 9 \"SIG E CAPS\" symptoms for 2 weeks or longer. These symptoms include sleep disturbance, anhedonia, guilt, loss of energy, loss of concentration, appetite/weight changes, psychomotor retardation or agitation, and suicidal ideation. Though this patient exhibits some of these symptoms, premenstrual dysphoric disorder is a more likely diagnosis as she exhibits these symptoms temporally related to her menstrual cycle.\n\nAnswer C: Panic disorder is characterized by 4 or more panic attacks (sweating, palpitations, shaking, paresthesias) in a 4-week period or \u2265 1 panic attack followed by at least 1 month of fear of another panic attack. Though this patient experiences anxiety that leads her to avoid her work, she does not have any of the characteristic signs of a panic attack and therefore does not meet the criteria for diagnosis of panic disorder.\n\nAnswer E: PMS and premenstrual dysphoric disorder have overlap in their symptoms; however, premenstrual dysphoric disorder is characterized by symptoms that significantly affect daily functioning. This patient's symptoms have affected her work and relationships, making premenstrual dysphoric disorder a more likely diagnosis. Note that the workup and treatment is similar between these 2 conditions.\n\nBullet Summary:\nPremenstrual dysphoric disorder (PMDD) is a variant of premenstrual syndrome (PMS) characterized by more severe physical and mood symptoms that affect daily functioning.", "link": "https://step2.medbullets.com/testview?qid=216746"} {"question": "A 15-year-old girl presents to the clinic due to concern that she may be pregnant. She had unprotected sexual intercourse earlier that morning and requests emergency contraception. She has had regular menses since menarche, and her last menstrual period ended 8 days ago. She has no significant medical history. Her temperature is 98.6\u00b0F (37\u00b0C), blood pressure is 114/71 mmHg, pulse is 81/min, and respirations are 12/min. A physical exam is unremarkable. A urine pregnancy test is negative. The patient asks for an emergency contraceptive pill and would prefer that her parents not be notified about her visit. In most states, which of the following is the most appropriate next step in the management for this patient?", "choicesA": "Do not prescribe emergency contraception given that the patient is under 18 years old", "choicesB": "Obtain parental consent before providing emergency contraception", "choicesC": "Prescribe high-dose oral contraceptives", "choicesD": "Prescribe levonorgestrel pill", "choicesE": "Prescribe misoprostol pill", "answer_idx": "D", "answer": "Prescribe levonorgestrel pill", "explanation": "This patient with recent unprotected intercourse and a negative pregnancy test is a candidate for postcoital emergency contraception. She should receive the levonorgestrel or ulipristal pill, which in most states does not require parental consent.\n\nPatients with a positive pregnancy test are not candidates for emergency contraception because implantation has already occurred, but patients with recent unprotected intercourse but a negative pregnancy test should receive the levonorgestrel pill (plan B) or ulipristal pill. These pills should be administered as soon as possible after intercourse. This is because the efficacy of the pills decrease after 72-120 hours post-intercourse. The high doses of progestin in these pills delay ovulation by blocking the LH surge, thereby preventing pregnancy. In most states, adolescents can receive emergency contraception without parental consent or notification.\n\nBatur et al. present the current options available for emergency contraception. They discuss how ulipristal acetate tends to be more efficacious compared to levonorgestrel when used more than 72 hours post-coitally. They recommend that all providers who treat young women be familiar with the options and indications for emergency contraception.\n\nIncorrect Answers:\nAnswer A: It is inappropriate to not administer emergency contraception despite the patient being under 18 years old. Adolescent patients at risk of pregnancy after unprotected intercourse who request emergency contraception should be presented with viable options.\n\nAnswer B: Obtaining parental consent before providing emergency contraception is an incorrect choice in most states. Most states allow adolescents to receive confidential care for contraception and pregnancy without parental involvement, consent, or notification.\n\nAnswer C: Prescribing high-dose oral contraceptives can be done in special circumstances to delay ovulation by patients taking multiple pills at once. However, combination oral contraceptives are less commonly used for post-coital contraception as they have significant side effects at high doses and are less effective than levonorgestrel pills.\n\nAnswer E: In patients with a positive pregnancy test (after implantation has occurred), misoprostol can be used for pregnancy termination by stimulating myometrial contractions. It does not prevent pregnancy. The use of drugs to terminate pregnancy varies significantly from state to state.\n\nBullet Summary:\nAdolescents seeking emergency contraception may do so without parental consent or involvement in most states.", "link": "https://step2.medbullets.com/testview?qid=216240"} {"question": "A 65-year-old woman presents to her family doctor to reestablish care since losing her insurance. She has not had time for regular check-ups. She exercises 3-4 times a week and consumes red meat sparingly. She drank and smoked cigarettes socially with coworkers but never at home or on vacation. She wakes up with achy wrists and elbows that she suspects is from years of using a computer keyboard. She completed menopause at age 52. Her family history is notable for coronary artery disease on her father's side and colon cancer on her mother's side. She last had a colonoscopy 5 years ago that revealed no abnormal findings. Her temperature is 99\u00b0F (37.2\u00b0C), blood pressure is 125/83 mmHg, pulse is 82/min, respirations are 12/min, and oxygen saturation is 99% on room air. Her physical exam is grossly unremarkable. What diagnostic test should this patient receive?", "choicesA": "Chest radiograph", "choicesB": "Colonoscopy", "choicesC": "Mammography", "choicesD": "Pelvic ultrasound", "choicesE": "Vitamin D level", "answer_idx": "C", "answer": "Mammography", "explanation": "This vignette describes an otherwise healthy elderly woman seeking to establish primary care. Among the answer choices, mammography is the most appropriate diagnostic test to recommend.\n\nMammography is a screening tool for breast cancer that becomes most effective in postmenopausal patients because of less glandular breast tissue compared to younger patients. According to the United States Preventive Services Task Force, all women aged 50-74 should have mammograms every 1-2 years. It is controversial whether screening mammography should begin as early as age 40. Women with 1st-degree relatives with breast cancer should begin screening 10 years before the age of the family member's diagnosis.\n\nSeely and Alhassan review the evidence regarding the screening methods used for breast cancer. They discuss how screening every 1-2 years decreases breast cancer mortality. They recommend using judicious mixed modal methods of screening.\n\nIncorrect Answers:\nAnswer A: Routine chest radiography for lung cancer screening in low-risk patients is not recommended by the USPSTF. Chest imaging can be used to screen for lung cancer in patients with a significant smoking history.\n\nAnswer B: Colonoscopy for colon cancer screening is recommended every 10 years starting at age 50. This patient already had a colonoscopy 5 years ago so she does not need another exam at this time.\n\nAnswer D: Pelvic ultrasound for ovarian cancer screening is not currently recommended by the USPSTF. Ovarian cancer detection should also be performed in patients who are symptomatic.\n\nAnswer E: Vitamin D level for osteoporosis screening is not recommended. DEXA scans would be more appropriate for testing bone density.\n\nBullet Summary\nAccording to the United States Preventive Services Task Force, all women aged 50-74 should have mammograms every 1-2 years.", "link": "https://bit.ly/3rJPzaL"} {"question": "A 27-year-old man presents to the emergency department with altered mental status. He was running a marathon but became confused halfway through the race. He arrives unable to coherently answer questions and is not sure where he is. He has a history of a seizure disorder. His temperature is 105\u00b0F (40.6\u00b0C), blood pressure is 116/68 mmHg, pulse is 167/min, respirations are 29/min, and oxygen saturation is 99% on room air. Physical exam reveals a confused man who is moving all 4 extremities and is protecting his airway. During the exam, he develops a tonic-clonic seizure, which lasts 1 minute and then terminates on its own. The patient is given intravenous fluids, and basic laboratory studies are drawn and pending. A urine dipstick is positive for red blood cells. Which of the following is most likely to be seen on laboratory testing?", "choicesA": "Hyperkalemia", "choicesB": "Hyperphosphatemia", "choicesC": "Hyperuricemia", "choicesD": "Hypocalcemia", "choicesE": "Hyponatremia", "answer_idx": "D", "answer": "Hypocalcemia", "explanation": "This patient is presenting after running a marathon with hyperthermia and altered mental status suggestive of heatstroke. In the setting of his recent exertion, tonic-clonic seizure, and urine dipstick positive for red blood cells, he is likely suffering from rhabdomyolysis, which most commonly causes hypocalcemia.\n\nRhabdomyolysis occurs when there is a breakdown of the muscle cells. It can occur with exertion (such as marathon running or weight lifting), seizures (from prolonged and intense contraction of the muscles), cocaine use, lightning strikes, electrical injuries, and crush injuries. Damage to muscle cells most commonly causes hypocalcemia from the deposition of calcium in necrotic muscle cells as well as calcium sequestration by the muscle sarcoplasmic reticulum. Symptoms of hypocalcemia include perioral numbness and tetany, altered mental status, and seizure. ECG manifestations of hypocalcemia include QT prolongation. There is no treatment needed for hypocalcemia in rhabdomyolysis unless it is severe or is causing symptoms such as seizure or ECG changes. The management of rhabdomyolysis involves aggressive IV fluid administration titrated to urine output in order to prevent myoglobin nephrotoxicity. Myoglobin causes the classic false-positive finding of red blood cells on urine dipstick given the cross-reactivity of myoglobin and hemoglobin on this test.\n\nGiannoglou et al. review the pathophysiology and diagnosis of rhabdomyolysis. The authors find that hypocalcemia is the most common electrolyte derangement seen in rhabdomyolysis. The authors recommend carefully monitoring ionized calcium in patients with rhabdomyolysis.\n\nIncorrect Answers:\nAnswer A: Hyperkalemia is a common electrolyte derangement that occurs in rhabdomyolysis and can cause life-threatening dysrhythmias. Initial ECG findings suggestive of hyperkalemia include peaked T waves and QRS widening. The management of hyperkalemia involves the administration of calcium chloride to stabilize cardiac membranes followed by insulin, albuterol, and bicarbonate. Severe/refractory hyperkalemia can be treated with dialysis, but this is usually not needed in rhabdomyolysis. Hyperkalemia is less common when compared to hypocalcemia in rhabdomyolysis and does not cause seizures.\n\nAnswer B: Hyperphosphatemia is common in rhabdomyolysis and occurs secondary to the release of phosphorus from damaged muscle cells. Hyperphosphatemia does not usually require treatment in rhabdomyolysis, though phosphate restriction and phosphate binders can be administered. Hyperphosphatemia can rarely lead to seizures, though this is uncommon.\n\nAnswer C: Hyperuricemia is possible in rhabdomyolysis, occurs secondary to the release of purines from injured muscle cells, and is worsened by acute kidney injury which is common in rhabdomyolysis. The treatment of hyperuricemia in rhabdomyolysis involves IV fluids, rasburicase, and allopurinol (which works slowly).\n\nAnswer E: Hyponatremia may occur after running a marathon where the individual sweats out sodium, chloride, and water but rehydrates with only water. While this is possible in this patient and his seizure could reflect a hyponatremic seizure (which usually would not spontaneously resolve and requires hypertonic saline), there is concrete evidence of rhabdomyolysis with the positive urine dipstick for red blood cells.\n\nBullet Summary:\nRhabdomyolysis can cause hyperkalemia, hyperphosphatemia, and hypocalcemia (the most common electrolyte disturbance).", "link": "https://step2.medbullets.com/testview?qid=214967"} {"question": "A 3-week-old boy presents to the emergency department with vomiting. His parents report that he suddenly started vomiting this morning and has shown no interest in feeding since then. They describe the vomitus as green and without any traces of blood. Prior to today, the patient was feeding, voiding, and stooling well. He was noted to have surpassed his birth weight at his last office visit 1 week ago. His temperature is 97.6\u00b0F (36.4\u00b0C), blood pressure is 78/56 mmHg, pulse is 148/min, and respirations are 44/min. On physical exam, the patient is in mild distress. He has no dysmorphic features, and his mucous membranes are dry. His abdomen is soft and distended. Bowel sounds are hypoactive. An abdominal radiograph is performed as in Figure A. Which of the following is the most appropriate next step in the diagnosis of this condition?", "choicesA": "Abdominal CT", "choicesB": "Abdominal ultrasound", "choicesC": "Contrast enema", "choicesD": "Rectal suction biopsy", "choicesE": "Upper gastrointestinal contrast series", "answer_idx": "E", "answer": "Upper gastrointestinal contrast series", "explanation": "This neonatal patient presents with sudden-onset bilious vomiting and an abdominal radiograph revealing a gasless abdomen, which is consistent with intestinal malrotation complicated by a midgut volvulus. The most appropriate next step in the diagnosis of intestinal malrotation is an upper gastrointestinal series.\n\nMalrotation classically presents with bilious vomiting due to the formation of midgut volvulus. These patients will have normal development until the development of the obstruction followed by inability to feed and abdominal distention. Findings of malrotation on an upper gastrointestinal series include an abnormally placed ligament of Treitz on the right side of the abdomen and a \"corkscrew\" appearance of the distal duodenum. If an upper gastrointestinal contrast series confirms the diagnosis, patients should undergo surgery to reduce the volvulus in order to avoid intestinal perforation or other complications.\n\nAdams and Stanton review the evidence regarding the diagnosis and treatment of neonatal bowel obstruction. They discuss how malrotation, jejunoileal atresia, duodenal atresia, and colonic atresia are common causes of this finding. They recommend surgical management of these conditions.\n\nFigure/Illustration A is an abdominal radiograph demonstrating the nonspecific finding of a paucity of air in the abdomen (red circle). This is consistent with intestinal malrotation complicated by midgut volvulus.\n\nIncorrect Answers:\nAnswer A: Although abdominal CT would detect intestinal malrotation, CT should be avoided in the pediatric population in light of the radiation exposure. A CT scan with oral contrast can be useful in the adult setting in order to determine if there is extravasation of fluid from the gastrointestinal tract.\n\nAnswer B: Abdominal ultrasound is the test of choice for diagnosing pyloric stenosis, which presents with non-bilious vomiting and a palpable olive mass in the abdomen. Treatment of this condition is with surgical release of the stenotic pyloric muscle.\n\nAnswer C: A contrast enema is used to differentiate between Hirschsprung disease and meconium ileus but would not be helpful in diagnosing malrotation as the midgut volvulus occurs in the small intestine. Both Hirschsprung disease and meconium ileus present in neonatal patients with bilious vomiting and failure to pass meconium in the first 48 hours of life. Treatment is with excision of the defective segment or removal of the meconium respectively.\n\nAnswer D: Rectal suction biopsy is the most accurate test to diagnose Hirschsprung disease. Although Hirschsprung disease commonly presents in neonatal patients with bilious vomiting, an abdominal radiograph in Hirschsprung disease would likely show dilation of the proximal colon.\n\nBullet Summary:\nThe most appropriate diagnostic test for malrotation in an infant is an upper gastrointestinal series, which will show the abnormal placement of the duodenum and ligament of Treitz on the right side of the abdomen.", "link": "https://bit.ly/44Zo5vi"} {"question": "A 6-year-old boy is brought to the emergency department by his father for elbow pain. He was riding his bicycle when he fell onto his outstretched right hand. He immediately clutched his right elbow in pain and began crying. He refuses to move his elbow. He has no medical problems and takes no medications. His temperature is 98.4\u00b0F (36.9\u00b0C), blood pressure is 108/64 mmHg, pulse is 100/min, and respirations are 20/min. On exam, the boy is holding his right wrist with his left hand and cries upon palpation of a region that is 2 cm proximal to his elbow. The radial head is palpated just distal to the lateral epicondyle. He is unable to make a pincer with his right thumb and index finger. When asked to hold a pen between his right thumb and index fingers, he drops it. The radial pulse is diminished on the right. Which of the following is the most likely diagnosis?", "choicesA": "Both bone forearm fracture", "choicesB": "Distal radius fracture", "choicesC": "Monteggia fracture-dislocation", "choicesD": "Olecranon fracture", "choicesE": "Supracondylar humerus fracture", "answer_idx": "E", "answer": "Supracondylar humerus fracture", "explanation": "This young patient presenting with elbow pain and associated signs of anterior interosseous nerve neurapraxia (inability to make pincer, weak pincer grasp) and vascular injury (diminished radial pulse) after a fall on an outstretched hand most likely has a supracondylar humerus fracture.\n\nSupracondylar humerus fractures are one of the most common fractures seen in the pediatric population. This site is particularly prone to fracture due to the thin bone separating the coronoid and olecranon fossae in the supracondylar region. Due to the proximity of the brachial artery and median nerve, which course anteriorly, these structures are at risk for injury. Neurapraxia of the anterior interosseous nerve, which is a pure motor branch of the median nerve, presents as motor deficits in interphalangeal (IP) joint flexion of the thumb and distal IP joint flexion of the index finger. The radial and ulnar arteries are terminal branches of the brachial artery, which if injured results in diminished distal pulses. The diagnosis is made with anteroposterior and lateral view elbow radiographs. Radiographs will demonstrate the \"fat pad sign\" seen as a posterior fat pad that is indicative of fracture. Management depends on fracture type but can consist of cast immobilization (if minimally displaced with no neurovascular injury) or closed reduction and percutaneous pinning (for more displaced fractures).\n\nMicheloni et al. reviews supracondylar fractures in children. They note it is the most common elbow fracture in the pediatric population. They recommend early and correct diagnosis and management to avoid early and late complications including neurovascular impairment and malunion.\n\nIncorrect Answers:\nAnswer A: Both bone forearm fracture is another common pediatric fracture sustained from a fall, involving both the radius and ulna. It can be associated with neurovascular injury depending on the plane of deformity, but the location of the pain would be in the forearm, not proximal to the elbow.\n\nAnswer B: Distal radius fracture is a common fracture after a fall on an outstretched hand mechanism of injury. Neurovascular injury is possible, though uncommon. This patient\u2019s pain is localized around his elbow, rather than near the distal radius. Often, a deformity can be visualized on exam.\n\nAnswer C: Monteggia fracture-dislocation is characterized by a proximal ulna fracture with radial head dislocation. It is commonly caused by direct trauma to the ulna. In contrast, this patient\u2019s radial head is located inferior to the lateral epicondyle.\n\nAnswer D: Olecranon fracture is an uncommon pediatric fracture caused primarily by a fall onto the elbow in flexion. Median nerve and brachial artery injury are unlikely, given that these structures pass anteriorly in the cubital fossa.\n\nBullet Summary:\nSupracondylar humerus fractures are common in the pediatric population and may present with elbow pain and concomitant median nerve and/or brachial artery injury.", "link": "https://bit.ly/3ZAMN3W"} {"question": "A 65-year-old man presents to the emergency department for evaluation of left arm and leg weakness. He was walking in the park about 45 minutes prior to arrival when the patient stumbled and fell. He had sudden weakness in his left leg and was unable to grasp a handrail with his left hand. He currently takes no medications. His temperature is 98.0\u00b0F (36.6\u00b0C), blood pressure is 128/89 mmHg, pulse is 130/min, and respirations are 18/min. On exam, he has 3/5 strength in the left upper and lower extremities and a left-sided facial droop. A non-contrast CT scan of the head is performed, which suggests parenchymal ischemia in the distribution of the right middle cerebral artery. An ECG is performed, as shown in Figure A. He is given intravenous alteplase with rapid improvement in his strength deficits and facial asymmetry. Which of the following is the most appropriate long-term therapy for secondary stroke prevention for this patient?", "choicesA": "Aspirin", "choicesB": "Atorvastatin", "choicesC": "Cardiac pacemaker", "choicesD": "Rivaroxaban", "choicesE": "Warfarin", "answer_idx": "D", "answer": "Rivaroxaban", "explanation": "This patient presents with acute onset unilateral left-sided weakness secondary to an ischemic stroke in the right MCA in the setting of an irregularly irregular rhythm without P waves (suggesting that he had an embolic stroke secondary to atrial fibrillation). First-line primary and secondary stroke prevention in patients with atrial fibrillation involves long term anticoagulation with a direct oral anticoagulant (DOAC) such as rivaroxaban.\n\nAtrial fibrillation is caused by uncoordinated atrial contractions, typically due to atrial enlargement, inflammation, or infiltrative disease. Atrial fibrillation can lead to the formation of a thrombus, typically in the left atrial appendage, which can embolize to the brain and lead to an ischemic stroke. For this reason, there is a low threshold to start patients on long term anticoagulation for both primary and secondary stroke prevention (risk stratification is done with the CHA2DS2-VASc score, which considers several cardiovascular risk factors, age, sex, and diabetes). First-line long term anticoagulation is a DOAC medication such as apixaban, dabigatran, rivaroxaban, or edoxaban. Delayed initiation of anticoagulation is recommended at 4-14 days after a stroke, due to the interim risk of hemorrhagic conversion. Further management of atrial fibrillation includes a rate control agent such as a beta blocker or a calcium channel blocker.\n\nJS Alpert reviews the latest American Heart Association/American College of Cardiology guidelines regarding atrial fibrillation. The article reports that for stroke prevention in atrial fibrillation, DOACs are first-line compared to warfarin due to a decreased risk for bleeding with DOACs. In patients with moderate to severe mitral stenosis or a mechanical heart valve, warfarin is the preferred agent.\n\nFigure/Illustration A shows an ECG with an irregularly irregular rhythm (highlighted by the spacing of blue arrows) and no discernable P waves before QRS complexes, which is characteristic of atrial fibrillation.\n\nIncorrect Answers:\nAnswer A: Aspirin exerts antiplatelet adhesion effects through the inhibition of cyclooxygenase. Patients with atrial fibrillation who are at low risk for stroke (CHA2DS2-VASc score of < 1) can use aspirin monotherapy for stroke prophylaxis. This patient presented after a stroke and will require long term anticoagulation, not anti-platelet monotherapy for stroke prevention.\n\nAnswer B: Atorvastatin is an HMG-CoA reductase inhibitor, which works to lower LDL cholesterol and triglycerides, and increase HDL cholesterol. It is most useful in secondary stroke prevention in patients who have suffered a thrombotic or thromboembolic stroke secondary to atherosclerosis. This patient may have atherosclerosis and could benefit from statin therapy; however, anticoagulation for his underlying atrial fibrillation is the highest priority.\n\nAnswer C: A cardiac pacemaker is indicated in a large variety of cardiac conditions that primarily involve sinus node dysfunction or high-grade atrioventricular block. Atrial fibrillation does not typically require a pacemaker, as it is typically managed with rate control agents such as beta-blockers, calcium channel blockers, digoxin, or with electrical cardioversion. Rate control will not lower the stroke risk, and thus long-term anticoagulation is required.\n\nAnswer E: Warfarin has historically been a mainstay anticoagulant for stroke prevention in atrial fibrillation. DOACs are non-inferior to warfarin, and have a lower risk of bleeding. Note, only the initiation of warfarin requires a heparin bridge to mitigate the transient protein C deficiency that results in a temporary hypercoagulable state (often presenting as warfarin skin necrosis).\n\nBullet Summary:\nFor patients who present with an ischemic stroke secondary to a cardiac embolus from atrial fibrillation, first-line long term anticoagulation is a direct oral anticoagulant (apixaban, dabigatran, rivaroxaban, or edoxaban).", "link": "https://bit.ly/3pOkYrN"} {"question": "A previously healthy 15-year-old boy presents to the emergency department in cardiac arrest. He was in class when he suddenly fainted and became unresponsive. His teacher began chest compressions and he has received epinephrine and defibrillation from paramedics while in transport. On arrival, the patient remains pulseless and is pronounced dead. An autopsy is performed and is notable for no clear abnormalities. The patient\u2019s laboratory studies were unremarkable from a visit to the pediatrician 3 weeks ago. A previous ECG is seen in Figure A. Which of the following is the most likely underlying etiology of this patient\u2019s death?", "choicesA": "Complete heart block", "choicesB": "Short QT syndrome", "choicesC": "Torsades des pointes", "choicesD": "Ventricular fibrillation", "choicesE": "Ventricular tachycardia", "answer_idx": "B", "answer": "Short QT syndrome", "explanation": "This healthy pediatric patient is presenting with cardiac arrest and an ECG demonstrating a short QT segment. Short QT syndrome is lethal and can lead to cardiac arrest in young, otherwise healthy patients.\n\nShort QT syndrome is an inherited condition that is caused by ion channel mutations (the most common identified mutation is a mutation in the cardiac myocyte potassium channel). There is no strict criteria for short QT syndrome, but the QT segment is often < 320 ms. Patients are usually young and healthy, and it is very common for the first presentation of this disease to be syncope or sudden cardiac death. ECG features suggestive of this condition include a short QT interval (without a clearly defined value but often < 360 ms) that is fixed (does not change with exercise or rest), peaked T waves, short or even absent ST segments, and episodes of atrial fibrillation or ventricular fibrillation in a patient in whom this would not be expected (such as a young patient). The only effective intervention is the placement of an implantable cardioverter-defibrillator.\n\nBjerregaard and Gussak discuss short QT syndrome. They note it is a heritable condition of the conduction system of the heart that can lead to lethal dysrhythmias. They discuss the management of this condition and recommend an implantable cardioverter-defibrillator.\n\nFigure/Illustration A is an ECG demonstrating a short QT segment (red arrow). Note the T waves appear peaked which can be seen in this syndrome. This may suggest hyperkalemia, but this patient\u2019s labs were normal at his appointment when the ECG was taken.\n\nIncorrect Answers:\nAnswer A: Complete heart block is more common in elderly patients with acute coronary syndrome and presents with regular P-P and R-R intervals with complete P-R dissociation. It may cause syncope but is not associated with short QT syndrome. Management involves the placement of a pacemaker.\n\nAnswer C: Torsades des pointes is associated with a prolonged QT segment that progresses to the classic twisting of the QRS complexes around an isoelectric baseline. It can lead to cardiac arrest and requires immediate cessation of QT-prolonging medications and administration of magnesium. Note that this patient\u2019s QT segment is short rather than long.\n\nAnswer D: Ventricular fibrillation presents with disorganized electrical activity with no discernible rhythm. While this patient may have arrested from ventricular fibrillation, the most likely underlying etiology for his cardiac arrest given his age, ECG, and risk factors is an underlying condition such as short QT syndrome. For this reason, short QT syndrome is a better answer.\n\nAnswer E: Ventricular tachycardia could also occur in short QT syndrome and may have caused this patient\u2019s cardiac arrest. It presents with a monomorphic wide complex tachycardia that may cause arrest. Unstable patients require cardioversion while pulseless patients require defibrillation. Note that if this patient arrested from ventricular tachycardia, it was likely secondary to short QT syndrome.\n\nBullet Summary:\nShort QT syndrome is a rare but possible diagnosis in young, healthy patients who present with recurrent syncope or cardiac arrest.", "link": "https://step2.medbullets.com/testview?qid=216625"} {"question": "A 72-year-old man presents to his primary care physician with weakness. He has felt very weak every morning with his symptoms persisting throughout the day. He notes minor improvement when he rides his bike but otherwise has not noticed any change in his symptoms with rest or ibuprofen use. The patient has lost 12 pounds recently and has had a chronic cough. The patient lives alone and drinks 7 alcoholic beverages per day and smokes 1-2 packs of cigarettes per day for the past 40 years. His temperature is 99.5\u00b0F (37.5\u00b0C), blood pressure is 177/108 mmHg, pulse is 93/min, respirations are 17/min, and oxygen saturation is 92% on room air. A chest radiograph is ordered as seen in Figure A. Which of the following diagnostic tests would most likely elucidate the cause of this patient's weakness?", "choicesA": "Blood laboratory test", "choicesB": "CT scan of the chest", "choicesC": "Electromyography", "choicesD": "Lung biopsy", "choicesE": "Muscle biopsy", "answer_idx": "C", "answer": "Electromyography", "explanation": "This patient who presents with weakness that improves with activity in the setting of a lung mass most likely has Lambert-Eaton syndrome. The most appropriate diagnostic test to confirm this syndrome is electromyography.\n\nSmall cell lung cancer presents with fatigue, weight loss, cough, and a coin lesion on chest radiograph typically in a smoker. A common paraneoplastic syndrome in small cell lung cancer is Lambert-Eaton syndrome which presents with muscle weakness that improves with activity and worsens with rest. Activity improves symptoms due to accumulation of calcium with exercise which allows for acetylcholine release. This occurs secondary to antibodies against presynaptic calcium channels which decrease acetylcholine release. The diagnosis of Lambert Eaton syndrome can be confirmed with electromyography. Treatment includes addressing the underlying lung tumor.\n\nKesner et al. review the diagnosis and etiology of Lambert-Eaton syndrome. They discuss how this disease is associated with lung cancer. They recommend making the diagnosis using electrophysiologic studies.\n\nFigure/Illustration A is a chest radiograph with a coin lesion in the left lung field (red circle). This finding is suggestive of small cell lung cancer.\n\nIncorrect Answers:\nAnswer A: Blood laboratory tests could assess for multidrug resistance 1 protein (MDR-1 aka P-glycoprotein or ABCB1) which predicts resistance to chemotherapy for small cell lung cancer; however, it would not further elucidate this patient's muscle weakness.\n\nAnswers 2 & 4: CT scan of the chest and lung biopsy would further elucidate this patient's small cell lung cancer; however, it would not explain his weakness that improves with activity. In the setting of an elderly smoker with fatigue and weight loss and a coin lesion, the diagnosis of lung cancer is essentially established.\n\nAnswer E: Muscle biopsy would be the most accurate diagnostic test for dermatomyositis, polymyositis, and inclusion body myositis; however, it would not assess for conditions such as myasthenia gravis and Lambert-Eaton syndrome.\n\nBullet Summary:\nElectromyography is the diagnostic test of choice for Lambert-Eaton syndrome.", "link": "https://bit.ly/3Mv3U1Q"} {"question": "A 28-year-old man is brought to the emergency department by helicopter after a high speed motorcycle crash. He was intubated and 2 large bore IVs were placed in the field with administration of 2 liters of lactated ringer's solution while en route. On arrival to the emergency department, his temperature is 98.6\u00b0F (37.0\u00b0C), pulse is 130/min, blood pressure is 80/50 mmHg, respirations are 22/min, and oxygen saturation is 97%. Examination reveals an ill-appearing intubated patient. Endotracheal tube placement is confirmed with end tidal CO2, and bilateral breath sounds are present. No external bleeding or wounds are noted on exam. Pulses are present in all extremities but are weak and rapid. Significant bruising is noted over the abdomen. The pelvis is stable. Infusion of packed red blood cells is initiated. Which of the following is the most appropriate next step in management?", "choicesA": "Anterior-posterior and lateral radiographs of the abdomen and pelvis", "choicesB": "CT scan of the abdomen and pelvis with contrast", "choicesC": "Diagnostic peritoneal lavage", "choicesD": "FAST exam", "choicesE": "Norepinephrine infusion", "answer_idx": "D", "answer": "FAST exam", "explanation": "This patient with hemodynamic instability and abdominal bruising after a high speed motorcycle crash likely has shock secondary to blood loss. The first step in management of patients with blunt trauma and hemodynamic instability is a focused assessment with sonography for trauma (FAST exam)\n\nFor any patient with blunt or penetrating trauma, the first step in management is an evaluation of airway, breathing, and circulation. This is referred to as the primary survey and includes the verification of endotracheal tube placement, auscultation of bilateral breath sounds, and assessment of external wounds or bleeding. This is followed by the secondary survey which involves a detailed head to toe inspection for injuries. Blunt abdominal trauma with bruising as well as hemodynamic instability warrants a FAST exam. The FAST exam visualizes the heart chambers and pericardium, left upper quadrant (spleen, splenorenal recess), right upper quadrant (Morrison pouch) and the rectovesical (males) or rectouterine (females) pouch. The extended FAST exam (eFAST) includes an examination of the lungs as well to assess for pneumothorax or hemothorax. Hemodynamically unstable patients with free fluid identified in the abdomen on initial FAST exam should proceed directly to the operating room for emergent surgical intervention with ongoing volume resuscitation. Patients with hemodynamic instability and a negative FAST exam should be evaluated for sources of bleeding elsewhere such as the pelvis.\n\nBarloon et al. review diagnostic imaging in the evaluation of blunt abdominal trauma. Blunt abdominal trauma is a common cause of death and injury, leading to hemodynamic instability. Intra-abdominal imaging is especially important in comatose or sedated (intubated) patients who are unable to provide a history or provide a reliable physical examination. In stable patients, CT of the abdomen often has a role in deciding whether to pursue observation versus surgical intervention.\n\nStengel et al. evaluate emergency ultrasound-based algorithms for diagnosing blunt abdominal trauma. The systematic review showed that ultrasound-based pathways reduced the number of CT scans obtained. Use of ultrasound can enhance the speed of the primary assessment and cut costs. However, the authors conclude that there is insufficient evidence to promote justification of ultrasound-based pathways for the evaluation of blunt abdominal trauma in terms of patient outcomes.\n\nIncorrect Answers:\nAnswer A: AP and lateral radiographs of the abdomen and pelvis may be useful for identifying pelvic fracture or intraperitoneal free air. However, this patient's presentation is suggestive of abdominal trauma warranting FAST exam.\n\nAnswer B: CT scan of the abdomen and pelvis with contrast would be appropriate for evaluation of blunt abdominal trauma in a patient that is hemodynamically stable. However, this patient's vital signs indicate hemodynamic instability.\n\nAnswer C: Diagnostic peritoneal lavage is no longer commonly used in the evaluation of blunt abdominal trauma.\n\nAnswer E: Norepinephrine infusion would not be appropriate for this patient with shock secondary to blood loss. Resuscitation for this patient should be achieved with transfusion of blood products.\n\nBullet Summary:\nThe most appropriate initial step in management for hemodynamically unstable patience with blunt abdominal trauma is a FAST exam.", "link": "https://step2.medbullets.com/testview?qid=103356"} {"question": "A 55-year-old man presents to his primary care physician with fatigue, malaise, and a painful rash. The patient states that his symptoms have been worsening over the past week. He also has experienced abdominal pain and diarrhea. He has a medical history of obesity and smokes regularly. His temperature is 97.5\u00b0F (36.4\u00b0C), blood pressure is 142/82 mmHg, pulse is 85/min, respirations are 15/min, and oxygen saturation is 98% on room air. Physical exam is notable for an erythematous rash with papules and plaques on the patient's face, torso, and extremities. He states that the rash is painful. A fingerstick blood glucose is unable to accurately read his blood glucose and gives a reading of > 500 mg/dL. Which of the following is the most likely underlying pathophysiology of this patient\u2019s condition?", "choicesA": "Alpha cell tumor", "choicesB": "Beta cell destruction", "choicesC": "Beta cell tumor", "choicesD": "Hypercortisolism", "choicesE": "Insulin resistance", "answer_idx": "A", "answer": "Alpha cell tumor", "explanation": "This patient is presenting with necrolytic migratory erythema (a painful, erythematous rash with papules and plaques) with gastrointestinal symptoms (abdominal pain and diarrhea) and hyperglycemia, which are concerning for a glucagonoma. A glucagonoma is a tumor of the alpha cells.\n\nA glucagonoma is a tumor of the alpha cells of the pancreas. This tumor pathologically secretes glucagon leading to profound hyperglycemia that is refractory to the normal treatments for diabetes. Patients also present with a classic rash termed necrolytic migratory erythema which presents with painful, erythematous papules and plaques. Other findings include gastrointestinal symptoms including anorexia, abdominal pain, and diarrhea. Hematologic findings include a normocytic, normochromic anemia. The diagnosis can be supported with a glucagon level, which will be elevated. Imaging of the pancreas by CT or MRI (more accurate) can further support the diagnosis.\n\nZandee et al. review glucagonoma. They note the presentation and symptoms associated with a glucagonoma, including the classic rash and elevated blood glucose. They recommend considering this diagnosis in patients who present with rash and hyperglycemia refractory to typical treatments.\n\nIncorrect Answers:\nAnswer B: Beta-cell destruction is the pathophysiology of type I diabetes mellitus which presents with polydipsia, polyuria, weight loss, and hyperglycemia (typically in a pediatric patient). It can progress to diabetic ketoacidosis which presents with hyperglycemia, an anion gap acidosis, altered mental status, and Kussmaul respirations.\n\nAnswer C: Beta-cell tumor describes an insulinoma, which would present with hypoglycemia refractory to glucose administration. This condition is caused by increased release of insulin. The diagnosis should be suspected in the setting of hypoglycemia, an elevated insulin level, and an elevated C-peptide level. Imaging of the abdomen/pancreas with an MRI will reveal the insulin-secreting mass.\n\nAnswer D: Hypercortisolism can cause Cushing syndrome which presents with hyperglycemia, obesity, striae, limb/muscle atrophy, mood changes, and hypertension. A buffalo hump and moon faces may also be seen. This condition can either be from endogenous ACTH production or cortisol production or can be from exogenous steroid use.\n\nAnswer E: Insulin resistance is the pathophysiology of type 2 diabetes mellitus and presents with obesity, hyperglycemia, and an elevated hemoglobin A1c. The mainstay of management is weight loss, though medications such as metformin, sulfonylureas, or insulin are often needed.\n\nBullet Summary:\nA glucagonoma is a tumor of the alpha cells of the pancreas that hypersecretes glucagon.", "link": "https://step2.medbullets.com/testview?qid=216609"} {"question": "A 67-year-old man presents with a recurrent episode of syncope. He states that this has happened to him multiple times, particularly when he changes body position from sitting to standing, exerts himself, or turns his head quickly. He says that he notes numbness, vertigo, and sometimes even trouble with speaking immediately prior to fainting. The episodes usually last 1-5 minutes. When he wakes up, he is mildly confused but typically returns to baseline within a minute. The patient has a past medical history of diabetes, dyslipidemia, and hypertension. His temperature is 98.5\u00b0F (36.9\u00b0C), blood pressure is 153/89 mmHg, pulse is 90/min, respirations are 11/min, and oxygen saturation is 97% on room air. The patient's neurological exam is unremarkable. However, when he is asked to stand up and start walking, he experiences the same numbness and tingling and feels like he may faint. Which of the following is the most likely diagnosis?", "choicesA": "Blood vessel narrowing", "choicesB": "Cardiac dysrhythmia", "choicesC": "Seizure", "choicesD": "Stroke", "choicesE": "Vagal event", "answer_idx": "A", "answer": "Blood vessel narrowing", "explanation": "This elderly patient with risk factors for vascular disease (diabetes, dyslipidemia, hypertension) is presenting with episodes of dizziness, vertigo, numbness/tingling, dysarthria, and syncope, which seem to be provoked by standing rapidly and changing head position. This is concerning for vertebrobasilar syndrome, caused by blood vessel narrowing.\n\nVertebrobasilar syndrome is caused by narrowing (via causes such as atherosclerosis, dissection, or congenital narrowing among others) of the vertebrobasilar vascular system. It is a common cause of syncope that may be worsened by changes in head position or standing rapidly. Other unique features include numbness/tingling, a sensation of vertigo, nausea, vomiting, dysphagia, and dysarthria. Note the episodes of syncope in this condition are intermittent and specifically provoked and do not have lasting/permanent neurological deficits in contrast to a stroke. Management is first centered on vascular imaging which may include a CTA head/neck or an MRA head/neck. Further care depends on the symptoms present but typically involves smoking cessation, glycemic control, blood pressure control, aspirin, and a statin.\n\nXuan-Ting et al. review posterior circulation strokes which occur more frequently in patients with atherosclerosis of the vertebrobasilar system. They note that MRI is one of the most accurate tests for assessing posterior circulation strokes which may present with dizziness and syncope. It is far superior to CT imaging. For this reason, they recommend that CT imaging alone cannot definitively rule out a posterior circulation stroke.\n\nIncorrect Answers:\nAnswer B: Cardiac dysrhythmias such as ventricular tachycardia can cause sudden onset and offset syncope without prodromal symptoms. The differentiating feature of this form of syncope is the rapid onset and offset in a high-risk patient. The diagnosis can be confirmed with an ECG; however, many patients will require prolonged observation on telemetry.\n\nAnswer C: Seizure presents with impaired consciousness, tonic-clonic activity, and a post-ictal state which gradually improves over hours. The lactate may be elevated after a seizure, and the prolactin level is a more specific indicator of a seizure. However, the diagnosis is most appropriately made based on history, exam, and EEG.\n\nAnswer D: Stroke involving the vertebrobasilar system may cause syncope in addition to ongoing dizziness and vertigo with ataxia. However, this patient's symptoms seem to be positional and have entirely resolved; thus, a stroke is unlikely. He is at higher risk of stroke given his vertebrobasilar insufficiency and an MRI should still be performed in this patient.\n\nAnswer E: Vagal event presents with a warm prodrome with palpitations, flushing, and syncope. Muscle twitching (but no tonic-clonic activity) may be seen. Patients may wake up slightly confused but return to baseline within seconds.\n\nBullet Summary:\nVertebrobasilar insufficiency may present with positional episodes of syncope caused by insufficient blood flow through the vertebrobasilar blood vessels.", "link": "https://step2.medbullets.com/testview?qid=216395"} {"question": "A 30-year-old man presents to the emergency department with diarrhea. He states he has had profuse, watery diarrhea for the past 24 hours. He notes that eating and drinking make him defecate more, thus he has been refraining from drinking. He is otherwise healthy and takes no medications. His temperature is 97.7\u00b0F (36.5\u00b0C), blood pressure is 84/64 mmHg, pulse is 130/min, respirations are 17/min, and oxygen saturation is 99% on room air. Physical exam reveals dry mucous membranes with an otherwise benign exam. The patient is given 1L of normal saline and drinks 1L of oral fluids. His blood pressure is subsequently 120/70 mmHg and pulse 95/min. Which of the following is the most appropriate next step in management?", "choicesA": "Ciprofloxacin", "choicesB": "Rest and oral rehydration", "choicesC": "Stool culture", "choicesD": "Stool ova and parasite study", "choicesE": "Stool PCR", "answer_idx": "B", "answer": "Rest and oral rehydration", "explanation": "This patient is presenting with a likely diarrheal illness given he has profuse, watery diarrhea and his vitals normalized with oral and IV hydration. Since the patient is young and healthy, oral hydration and rest are all that is needed for management.\n\nDiarrheal illness is a common chief complaint with variable workups pending on the patient's clinical status. Young, healthy, non-immunosuppressed patients who can tolerate an oral diet require only reassurance and oral hydration, as most cases of viral and bacterial diarrhea will resolve on their own. Moreover, most cases of diarrhea are caused by viral etiologies which simply require supportive therapy. This is even true in bacterial diarrhea as long as the patient's vitals normalize and they can tolerate an oral diet. It is important to consider inflammatory causes of diarrhea (such as ulcerative colitis or Crohn disease) as this would change the workup and management.\n\nSchiller et al. discuss different types of diarrhea. They review the management and diagnosis of diarrhea and its different forms, including chronic forms. They recommend noting the difference between different causes and working up and treating them appropriately.\n\nIncorrect Answers:\nAnswer A: Ciprofloxacin is an antibiotic that can be used for bacterial/infectious diarrhea. It should not be given in young, healthy patients and would only be indicated in the setting of bacterial diarrhea in ill patients, patients with persistent symptoms, or patients who are immunosuppressed. Bacterial diarrhea would present with fever, abdominal pain, and bloody/purulent diarrhea.\n\nAnswer C: Stool culture should not be empirically performed in young, well-appearing patients. It may be performed in hospitalized patients who are more ill or in patients who are going to start antibiotic therapy. It may also be performed in patients with persistent diarrhea.\n\nAnswer D: A stool ova and parasite study would not be performed unless there was a suggestive history or exposure (such as drinking unfiltered water) in the setting of fatty, foul-smelling diarrhea. This would raise suspicion for Giardia lamblia which would be treated with metronidazole.\n\nAnswer E: Stool PCR for viral etiologies of diarrhea is not useful as the management of viral diarrhea is supportive therapy in most cases. On occasion, it may be performed in critically ill patients to determine the etiology of their symptoms. It should not be performed routinely.\n\nBullet Summary:\nHealthy patients with limited diarrheal illness require supportive therapy only (in particular, oral hydration) and do not need more invasive diagnostic tests.", "link": "https://step2.medbullets.com/testview?qid=216584"} {"question": "A 23-year-old woman presents to the emergency department for evaluation of fever and chills over the last several days along with a progressive rash and diarrhea. She had nasal septoplasty performed 1 week prior and has had nasal packing in place since that time. Her rash has progressed and the skin on her hands is \"peeling.\" She has no significant medical history and takes no medications. Her temperature is 102\u00b0F (38.9\u00b0C), blood pressure is 84/54 mmHg, pulse is 160/min, respirations are 22/min, and oxygen saturation is 99% on room air. Exam reveals an ill-appearing woman. A diffuse, macular, erythematous rash is noted with desquamation over the patient's hands as shown in Figure A. Bilateral conjunctivitis is noted. The patient's nasal packing is removed revealing copious, foul-smelling, purulent discharge. Which of the following pathogens is likely responsible for this patient's condition?", "choicesA": "Borrelia burgdorferi", "choicesB": "Neisseria meningitides", "choicesC": "Rhinovirus", "choicesD": "Rickettsia rickettsii", "choicesE": "Staphylococcus aureus", "answer_idx": "E", "answer": "Staphylococcus aureus", "explanation": "This patient with fever, diarrhea, and a diffuse, erythematous rash with prolonged nasal packing in place likely has developed toxic shock syndrome. Toxic shock syndrome occurs most commonly due to infection with Staphylococcus aureus.\n\nToxic shock syndrome (TSS) occurs due to infection with Staphylococcal or Streptococcal species that produce \"super-antigen\" toxins, such as TSST-1 produced by S. aureus. These toxins bind directly to the MHC-II receptor on T cells with high affinity causing a massive, sustained cytokine release. The result is a \"cytokine storm\" causing an intense inflammatory reaction and distributive shock. These infections arise in the setting of a retained tampon or retained nasal packing following nasal surgery; however, this is not always the case. The clinical presentation of TSS is characterized by fever, nausea, vomiting, diarrhea, conjunctivitis, and a diffuse \"sunburn-like\" rash. Desquamation of the palms and soles typically occurs during recovery, 1-2 weeks after the acute phase of the illness. The initial management of TSS begins with broad spectrum antibiotic therapy. Typical regimens include vancomycin, cefepime, and clindamycin. The initial antibiotic regimen should cover methicillin-resistant Staph aureus. Clindamycin is always included due to its ability to suppress bacterial protein production, therefore limiting the production of bacterial toxins. Once a causative organism is identified, antibiotic therapy may be narrowed, though clindamycin is usually administered for the duration of treatment.\n\nFigure A is the classic generalized and erythematous macular/desquamating rash of toxic shock syndrome.\n\nIncorrect Answers\nAnswer A: Borrelia burgdorferi is a spirochetal bacteria that causes Lyme disease. A erythematous, bulls-eye shaped rash would be expected.\n\nAnswer B: Neisseria meningitidis causes meningococcemia and meningitis. Fever is common, although a purpuric rash and signs of meningeal irritation would be expected. Patients are often profoundly unstable.\n\nAnswer C: Rhinovirus is the viral pathogen most commonly responsible for the common cold that causes minor symptoms including a cough, runny nose, fever, and aches.\n\nAnswer D: Rickettsia rickettsii causes rocky mountain spotted fever which presents with a high fever, headache, malaise, and a centripetal maculopapular rash that starts on the extremities and migrates to the trunk. It is a tick-borne illness.\n\nBullet Summary:\nToxic shock syndrome classically develops in patients with prolonged use of nasal packing, and presents with fever, diarrhea, a diffuse erythematous rash, and shock.", "link": "https://bit.ly/3OvFRS0"} {"question": "Figure A is the lateral radiograph of a 44-year-old male who sustained a fall on his outstretched hand while rollerblading. The patient reports pain localized to the posterior elbow and refuses to attempt elbow extension secondary to pain and swelling. Which of the following is the most likely diagnosis in this patient?", "choicesA": "Exacerbation of elbow arthritis", "choicesB": "Triceps rupture", "choicesC": "Olecranon bursitis", "choicesD": "Lateral collateral ligament tear", "choicesE": "Anconeus avulsion fracture", "answer_idx": "B", "answer": "Triceps rupture", "explanation": "The flake sign, as seen in Figure A is pathognomonic of a triceps tendon rupture.\n\nTricep tendon ruptures are rare injuries to the elbow extensor mechanism that most commonly occur as a result of forceful elbow contraction. Diagnosis can be made clinically, as patients with complete ruptures are unable to extend the elbow against gravity. An MRI may be useful in the diagnosis in patients whom are too painful to examine or who may have partial tears of the triceps tendon. Surgical repair is indicated in patients with complete rupture or in those with >50% tears and significant weakness.\n\nYeh et al. review distal triceps ruptures. They report that these injuries are commonly caused by a fall on an outstretched hand or a direct blow. If identifiable on plain radiograph a flake sign or avulsion of the olecranon tip will be appreciated. They conclude that diagnosis may be difficult initially in a painful elbow, however, a combination of plain radiographs and MRI can assist in making the correct diagnosis.\n\nBeazley et al. review distal biceps and triceps injuries. They report that these injuries most commonly occur in middle-aged males as a result of eccentric loading of the tendon. They conclude that patients who undergo early diagnosis and surgical treatment can expect to have excellent outcomes.\n\nKeener et al. review distal tricep tendon injuries. They report that most complete tears are treated surgically in medically fit patients. Partial-thickness tears are managed according to the tear severity, functional demands, and response to conservative treatment. They conclude that anatomic footprint repair of the triceps with optimal tendon to bone healing results in an acceptable functional outcome.\n\nFigure A is the lateral radiograph of an elbow demonstrating a flake sign from the olecranon tip indicative of a triceps rupture.\n\nIncorrect Answers:\nAnswer A: The patient does not show significant elbow degenerative disease on plain radiographs.\nAnswer C: Olecranon bursitis is not likely to worsen acutely after a fall, nor are there any other indications of this diagnosis in the present patient.\nAnswer D: While lateral collateral ligament tears may occur following a fall on an outstretched arm, there is no indication that an elbow dislocation occurred here and based on current radiographs a triceps rupture is more likely.\nAnswer E: Anconeus avulsions fractures are identified as lateral epicondyle fractures.", "link": "https://bit.ly/43RQXWn"} {"question": "A 46-year-old woman presents to her primary care doctor to request a referral to ophthalmology. Her vision has been steadily declining over the past 2 years and she thinks she needs a prescription for glasses. She further complains that her lips and feet feel numb. Her medical history is notable for medullary thyroid cancer status post total thyroidectomy. Her periods are regular. She enjoys a diverse diet and takes levothyroxine. Her blood pressure is 110/70 mmHg, pulse is 80/min, and respirations are 12/min. She is alert and oriented. Her pupils are equal, round, and reactive to light, but appear opacified. Extraocular movements are intact, albeit slow. Her visual acuity is decreased bilaterally. The remainder of her physical exam is unremarkable. Her basic metabolic panel is shown below: Na+: 139 mEq/L Cl-: 100 mEq/L K+: 4.9 mEq/L HCO3-: 25 mEq/L BUN: 10 mg/dL Glucose: 110 mg/dL Creatinine: 0.8 mg/dL Thyroid-stimulating hormone: 1.5 \u00b5U/mL Ca2+: 7.0 mEq/L Phosphorus: 6.5 mEq/L Albumin: 3.6 mg/dL The patient's ECG is shown in Figure A. What is the most likely cause of this presentation?", "choicesA": "Hyperkalemia", "choicesB": "Iatrogenic hypoparathyroidism", "choicesC": "Iatrogenic hypothyroidism", "choicesD": "Multiple endocrine neoplasia", "choicesE": "Somatization", "answer_idx": "B", "answer": "Iatrogenic hypoparathyroidism", "explanation": "This patient who presents with cataracts and a prolonged QTc interval on ECG most likely has symptomatic hypocalcemia secondary to the surgical removal of all of her parathyroid glands (iatrogenic hypoparathyroidism).\n\nThyroid surgery may mandate the removal of parathyroid glands if there is evidence of malignant invasion. One parathyroid gland is enough to maintain eucalcemia, but accidental/intentional removal of all the glands is possible. Less common causes are autoimmune parathyroid destruction or parathyroid infiltration by disorders like sarcoidosis or Wilson disease. Many patients are asymptomatic, but chronic hypocalcemia is associated with increased phosphate levels, prolonged QT interval, Parkinsonian signs, cataracts, and cognitive impairment. Severe, acute hypocalcemia can present with tetany, papilledema, and seizures. Treatment is with calcium and parathyroid hormone replacement.\n\nClarke et al. review the evidence regarding the diagnosis and etiologies of hypoparathyroidism. They discuss how the most common cause is post-surgical. They recommend obtaining genetic testing in cases where the etiology is unclear.\n\nFigure/Illustration A shows an ECG with a prolonged QTc interval at approximately 500 msec (blue circle).\n\nIncorrect Answers:\nAnswer A: This patient is mildly hyperkalemic, but her symptoms and signs are not associated with hyperkalemia. Patients with hyperkalemia present with heart palpitations, shortness of breath, chest pain, nausea, or vomiting. ECG will demonstrate peaked T waves.\n\nAnswer C: The patient is on levothyroxine with appropriate TSH levels. Hypothyroidism is not associated with QT prolongation. Patients with hypothyroidism will present with cold intolerance, delayed responsiveness, and myxedema coma. Treatment is with increasing the levothyroxine dose.\n\nAnswer D: Multiple endocrine neoplasia is associated with medullary thyroid cancer, but this patient has no evidence of additional organ disease. Other signs of multiple endocrine neoplasia include pancreatic tumors, pheochromocytoma, or a Marfanoid habitus. Treatment is with surgical excision of tumors.\n\nAnswer E: This patient most likely has a biological reason for her symptoms, so somatization is less likely. Patients with somatizing disorders have psychological distress that manifests as physical symptoms. These patients will not have organic correlates of disease such as abnormal ECG results.\n\nBullet Summary:\nIatrogenic hyperparathyroidism after the removal of the thyroid gland is the most common cause of hypocalcemia.", "link": "https://bit.ly/43oZtLT"} {"question": "A 35-year-old G0P0000 woman presents to her gynecologist with complaints of irregular menstruation. She has had only 2 periods in the last year. She feels flushed without provocation and is experiencing occasional dyspareunia with post-coital spotting. She has also had more frequent headaches than usual. The patient has a medical history of Hashimoto thyroiditis and takes levothyroxine daily. Her mother has type 1 diabetes mellitus. Her temperature is 98.5\u00b0F (36.9\u00b0C), pulse is 70/min, blood pressure is 118/76 mmHg, and respirations are 13/min. Cardiopulmonary and abdominal exams are unremarkable. The patient has Tanner 5 breasts and pubic hair. A pelvic exam reveals a normal cervix, an anteverted uterus without tenderness, and no adnexal masses. The following laboratory studies are performed: Serum: Thyroid stimulating hormone (TSH): 28 \u00b5IU/mL (9-30 \u00b5IU/mL) Cycle day 3 follicle stimulating hormone (FSH): 49 mIU/mL (4.7-21.5 mIU/mL) Cycle day 3 estradiol: 8 pg/mL (27-123 pg/mL) Prolactin: 14 ng/mL (4-23 ng/mL) Testosterone: 42 ng/dL (15-70 ng/dL) Which of the following is the most appropriate next step in management?", "choicesA": "Brain MRI", "choicesB": "Combined oral contraceptive", "choicesC": "Estradiol patch with oral medroxyprogesterone", "choicesD": "Increase levothyroxine dose", "choicesE": "Vaginal estradiol gel", "answer_idx": "C", "answer": "Estradiol patch with oral medroxyprogesterone", "explanation": "This patient presents with oligomenorrhea, hot flashes, and dyspareunia, and her labs are notable for a high FSH and low estradiol, most consistent with premature ovarian insufficiency/failure (POI). The most appropriate next step in management is an estradiol patch with oral medroxyprogesterone.\n\nPOI is defined by the depletion or dysfunction of ovarian follicles, resulting in oligomenorrhea or amenorrhea with symptoms of menopause (hot flashes, vaginal atrophy and dryness, mood swings, etc.) in women under age 40. The cause is unclear and thought to be autoimmune. The main diagnostic markers are a high FSH and low estradiol at the start of the follicular phase. Patients require estrogen supplementation until around age 50, as low estrogen increases the risk of coronary artery disease, osteoporosis, and sexual dysfunction. Vaginal gel or a transdermal patch is the first-line option, but both must be used in conjunction with medroxyprogesterone to avoid unopposed estrogen and the risk of endometrial hyperplasia.\n\nTsiligiannis et al. review the evidence regarding the outcomes of patients with POI. They discuss how this diagnosis is made in patients younger than the age of 40. They recommend closely managing the cardiovascular risk factors associated with this disease.\n\nIncorrect Answers:\nAnswer A: Brain MRI would be useful for evaluating for the presence of a prolactinoma, which could cause the headaches and oligomenorrhea seen in this patient. However, her prolactin level is normal, and she has no visual disturbances (e.g., bitemporal hemianopsia), making this study less relevant.\n\nAnswer B: The combined oral contraceptive has both estrogen and progesterone, but the doses are higher than those needed for hormone replacement. For this reason, it is generally not considered first-line for the treatment of POI.\n\nAnswer D: Increasing this patient\u2019s levothyroxine dose is unnecessary, as her TSH level is within the normal range. Furthermore, while hypothyroidism may cause menstrual irregularities and temperature dysregulation, it is unlikely to cause vaginal dryness and subsequent dyspareunia.\n\nAnswer E: Vaginal estradiol gel is an effective method of estrogen replacement in women with POI. However, used alone it may increase the risk of endometrial hyperplasia and cancer, so it should only be used in combination with a systemic progesterone.\n\nBullet Summary:\nPremature ovarian insufficiency should be treated with estrogen and progesterone replacement.", "link": "https://bit.ly/45zXPbW"} {"question": "A 27-year-old man presents to his primary care physician for exposure to toxic materials. The patient states that when he left for work this morning he was certain that he had closed the door to his pantry. Upon returning home, he saw that the door to his pantry was wide open. The patient is certain that his neighbors have been tampering with his food and potentially poisoned him. He further states that he knows they have been trying to break into his house and steal his things. He has tried multiple times to get them evicted from the building to no avail. It is for this reason that he is certain that they are trying to get their revenge upon him. His temperature is 98.6\u00b0F (37\u00b0C), blood pressure is 115/71 mmHg, pulse is 72/min, and respirations are 12/min. The physician performs a physical exam and tells the patient that he thinks there is nothing to be concerned about, but that he should call him or come into the office if he experiences any symptoms. The patient is outraged at this news and requests a competent doctor who is not colluding with his enemies. He storms out of the office angrily, stating that he deserves the best in medical care. Which of the following is the most likely disorder that this patient is suffering from?", "choicesA": "Borderline personality disorder", "choicesB": "Intermittent explosive disorder", "choicesC": "Narcissistic personality disorder", "choicesD": "Paranoid personality disorder", "choicesE": "Schizoid personality disorder", "answer_idx": "D", "answer": "Paranoid personality disorder", "explanation": "This patient is presenting with a history of mistrust and suspicion of others without frankly psychotic features suggesting a diagnosis of paranoid personality disorder.\n\nParanoid personality disorder is a cluster A personality disorder that is genetically associated with schizophrenia. These patients present with a pervasive mistrust of others based on little or no evidence. These patients are inclined to believe others have bad intentions that are directed toward them. They are often odd, very emotionally cold, and typically are involved in frequent litigations. Patients can be hard to manage though they often benefit from cognitive behavioral therapy. Anxiolytics may also be helpful in calming these patients.\n\nTriebwasser et al. review the evidence regarding the diagnosis of paranoid personality disorder. They discuss how there is relatively little research on these patients because it is difficult to recruit them for studies. They recommend considering the removal of this diagnosis and replacing it with a domain of paranoia.\n\nIncorrect Answers:\nAnswer A: Borderline personality disorder presents with emotional instability, unstable relationships, and recurrent self-harm behaviors. These patients will typically demonstrate the defense mechanism of splitting (seeing things as all good or all bad). Dialectical behavioral therapy can be used in order to help these patients mitigate self-injurious behaviors.\n\nAnswer B: Intermittent explosive disorder presents with sudden episodes of aggression out of proportion to the stressor. This patient\u2019s response is a result of his paranoia rather than an excessive response to reasonable stimuli. Cognitive behavioral therapy can be used in order to help these patients control their emotional outbursts.\n\nAnswer C: Narcissistic personality disorder presents with a grand sense of self-importance and preoccupation with success and admiration. These individuals are typically selfish and lack empathy. Though this patient\u2019s demand for the best medical care may be reflective of narcissistic personality disorder, the majority of his behaviors point more toward a diagnosis of paranoid personality disorder.\n\nAnswer E: Schizoid personality disorder presents in patients with isolated and emotionally restricted behavior. These patients are typically alone but enjoy being alone and often seem disinterested in others. These patients typically do not seek relationships and do not seek therapy as they are not bothered by their behaviors.\n\nBullet Summary:\nParanoid personality disorder presents with mistrustful behavior, suspicion of others, oddness, emotional coldness, and they are typically involved in many litigations.", "link": "https://bit.ly/40Awtzi"} {"question": "A 27-year-old woman presents to the emergency department with altered mental status. Her boyfriend found her at home next to a suicide note and brought her into the emergency department. According to her boyfriend, the patient has a medical history of bipolar disorder for which she takes lithium, as well as chronic lower back pain for which she sees a pain specialist. Her temperature is 100.5\u00b0F (38.1\u00b0C), blood pressure is 102/78 mmHg, pulse is 127/min, respirations are 19/min, and oxygen saturation is 99% on room air. An ECG is performed as seen in Figure A. Which of the following findings is also likely to be present at some point in this patient\u2019s course?", "choicesA": "Diaphoresis", "choicesB": "Jaundice", "choicesC": "Miosis", "choicesD": "Urinary retention", "choicesE": "Wheezing", "answer_idx": "D", "answer": "Urinary retention", "explanation": "This patient is presenting after a suicide attempt with an unknown overdose and is confused, febrile, tachycardic, and has QRS widening on ECG. This combination of symptoms is highly concerning for a tricyclic antidepressant (TCA) overdose which can cause an anticholinergic toxidrome and urinary retention.\n\nTricyclic antidepressants (TCAs) were once a mainstay in the treatment of depression before the advent of selective serotonin reuptake inhibitors (SSRIs) and serotonin\u2013norepinephrine reuptake inhibitors (SNRIs). They are sometimes still used for depression and neuropathic pain. In overdose, TCAs block sodium channels in the heart and cause a pathognomonic QRS widening. Sodium bicarbonate should be administered until the QRS duration normalizes. TCAs also have anticholinergic properties and can cause an anticholinergic toxidrome including delirium, confusion, constipation, decreased bowel sounds, urinary retention, dry, warm, and flushed skin, and fever. Although physostigmine is typically the antidote of choice for an anticholinergic toxidrome, it is avoided in TCA toxicity due to the theoretical concern for inducing a cardiac arrest.\n\nUrquhart et al. studied the utility of tricyclic antidepressants in the treatment of chronic lower back pain. They found no significant difference in pain reduction compared to placebo. They recommend further study of the potential benefits of TCAs, independent of sleep improvement, for the treatment of lower back pain.\n\nFigure/Illustration A is an ECG demonstrating QRS widening (note how long the red lines are which demarcate the QRS duration) which is highly concerning for TCA overdose.\n\nIncorrect Answers:\nAnswer A: Diaphoresis, tachycardia, increased bowel sounds, and agitation would be seen in a sympathomimetic toxidrome from drugs like cocaine or amphetamines. Rather, dry, flushed skin would be seen in an anticholinergic toxidrome.\n\nAnswer B: Jaundice, nausea, vomiting, and abdominal pain would be seen in acetaminophen overdose which can cause fulminant liver failure. It is also possible that patients may initially be asymptomatic in acetaminophen overdose, and a high level of suspicion must be maintained for this diagnosis. A serum acetaminophen level should be drawn in any patient suspected of overdose.\n\nAnswer C: Miosis, bradypnea, decreased bowel sounds, and somnolence would be seen in an opioid toxidrome. Rather, dilated and minimally responsive pupils would be seen in an anticholinergic toxidrome.\n\nAnswer D: Wheezing, bronchorrhea, urinary incontinence, fecal incontinence, confusion, and drooling would be seen in a cholinergic toxidrome from agents such as organophosphates or nerve gases like sarin. None of these symptoms are seen in this patient.\n\nBullet Summary:\nA tricyclic antidepressant overdose can cause QRS widening on ECG as well as an anticholinergic toxidrome (dry, flushed skin, constipation, decreased bowel sounds, urinary retention, and altered mental status).", "link": "https://bit.ly/3ZzPHpE"} {"question": "A 40-year-old south asian man presents to a primary care provider with a chronic cough that is worse at night. He has had the cough for several years but it has been getting worse over the last few months. He recently moved to the United States to work in construction. He has lost 10 pounds in the last 3 months along with darker stools which he believes is caused by a Western diet. He denies any difficulty swallowing or feeling of food getting stuck in his throat. He drinks alcohol 1-2 times per week and has never smoked. He denies any family history of cancer. His temperature is 98.6\u00b0F (37\u00b0C), blood pressure is 114/72 mmHg, pulse is 82/min, and respirations are 12/min. On physical exam, his lungs are clear to auscultation bilaterally without wheezing. His abdomen is soft and non-distended. He has no tenderness to palpation and bowel sounds are present. Which of the following is the most appropriate next step in management?", "choicesA": "Barium swallow", "choicesB": "Colonoscopy", "choicesC": "Helicobacter pylori stool antigen test", "choicesD": "Trial of lansoprazole", "choicesE": "Upper endoscopy", "answer_idx": "E", "answer": "Upper endoscopy", "explanation": "This patient presents with a chronic cough that is worse at night, which is consistent with gastroesophageal reflux disease (GERD). In light of this patient\u2019s new weight loss and melena, the most appropriate next step in management would be upper endoscopy to rule out cancer.\n\nGERD may present as a chronic or nighttime cough, substernal or epigastric pain, regurgitation, nausea, or bitter taste in the mouth. Symptoms are typically worse after eating or lying down. Patients with typical GERD should undergo upper endoscopy if they have concerning symptoms, including dysphagia, odynophagia, vomiting, weight loss, gastrointestinal bleeding, or melena. They should also undergo upper endoscopy if they are men over the age of 50 with chronic symptoms (>5 years) and cancer risk factors, such as a history of tobacco use or a family history of cancer.\n\nYoung et al. review the evidence regarding the treatment of GERD. They discuss how patients with concerning symptoms or those who are failing symptomatic treatment should undergo upper endoscopy. They recommend treatment with a proton pump inhibitor in patients without alarm symptoms.\n\nIncorrect Answers:\nAnswer A: A barium swallow would be indicated in a patient who presented with symptoms of esophageal dysphagia. Possible etiologies would include Zenker diverticulum or achalasia. This patient denied any difficulty swallowing or feeling of food being stuck in the throat.\n\nAnswer B: Colonoscopy could be considered in this patient if his upper endoscopy is unrevealing, as weight loss and melena could be explained by colorectal cancer. In light of this patient\u2019s GERD, upper endoscopy should be performed first.\n\nAnswer C: Helicobacter pylori testing can be useful in the workup of gastric and duodenal ulcers but is not usually performed in the evaluation of GERD. Weight loss and melena raise concerns for carcinoma. Treatment of patients with H. pylori includes a proton pump inhibitor, clarithromycin, and metronidazole.\n\nAnswer D: A trial of lansoprazole would not be appropriate for this patient presenting with several worrisome symptoms including melena and weight loss. If the patient had presented with typical GERD without these symptoms, a trial of a PPI would be indicated. Cancer should be ruled out first prior to symptomatic treatment.\n\nBullet Summary:\nPatients with gastroesophageal reflux disease who present with alarm symptoms of weight loss, dysphagia, odynophagia, vomiting, gastrointestinal bleeding, or melena should be evaluated with upper endoscopy.", "link": "https://bit.ly/3KuUPFj"} {"question": "A 10-year-old girl is admitted to the medical floor for a respiratory infection. The patient lives in a foster home and has been admitted many times. Since birth, the patient has had repeated episodes of pain/pressure over her frontal sinuses and a chronic cough that produces mucus. She was recently treated with amoxicillin for an infection. The patient is in the 25th percentile for height and weight which has been constant since birth. Her guardians state that the patient has normal bowel movements and has been gaining weight appropriately. The patient has a history of tricuspid stenosis. She also recently had magnetic resonance imaging (MRI) of her chest which demonstrated dilation of her airways. Her temperature is 99.5\u00b0F (37.5\u00b0C), blood pressure is 90/58 mmHg, pulse is 120/min, respirations are 18/min, and oxygen saturation is 94% on room air. Physical exam is notable for bruises along the patient's shins which the guardians state are from playing soccer. The rest of the exam is deferred because the patient starts crying. Which of the following findings is associated with this patient's most likely underlying diagnosis?", "choicesA": "Diastolic murmur best heard along the right lower sternal border", "choicesB": "Hypocalcemia", "choicesC": "Increased chloride in the patient's sweat", "choicesD": "Repeat sinus infections secondary to seasonal allergies", "choicesE": "Social withdrawal and avoidance of eye contact", "answer_idx": "A", "answer": "Diastolic murmur best heard along the right lower sternal border", "explanation": "This patient is presenting with recurrent sinus infections suggestive of Kartagener syndrome. Dextrocardia is a common feature in this pathology, which results in right-sided heart sounds or a diastolic murmur best heard along the right lower sternal border for this patient's tricuspid stenosis.\n\nKartagener syndrome (primary ciliary dyskinesia) occurs secondary to a genetic defect in a patient's cilia. This results in a clinical picture of recurrent sinusitis, recurrent upper respiratory infections, bronchiectasis, and situs inversus (all major organs are a mirror image of where they should be). A physical exam will reveal right-sided heart sounds and a chest radiograph will demonstrate dextrocardia. Treatment is with respiratory support, assistance with mucociliary clearance, and treatment of infections.\n\nLeigh et al. review the evidence regarding the manifestations of Kartagener syndrome. They discuss how about 50% of patients have situs inversus. They recommend respiratory support for these patients given the overlap with other mucociliary clearance disorders such as cystic fibrosis.\n\nIncorrect Answers:\nAnswer B: Hypocalcemia would be seen in DiGeorge syndrome which presents with CATCH-22 (Cleft palate/lip, Abnormal facies, Thymic aplasia, Cardiac defects, Hypocalcemia, and chromosome 22 deletion). Treatment is with calcium repletion, surgical treatment of cardiac defects, and treatment of infections.\n\nAnswer C: Increased chloride in the patient's sweat describes cystic fibrosis which would present with weight loss, failure to thrive, and fatty/foul-smelling stools. Treatment is with respiratory support, assistance with mucociliary clearance, and treatment of infections.\n\nAnswer D: Repeat sinus infections secondary to seasonal allergies describe sinusitis, which may be present in this patient but does not explain the entire clinical picture. Treatment is with antihistamines.\n\nAnswer E: Social withdrawal and avoidance of eye contact describe child abuse which seems unlikely in this case given the constellation of symptoms that suggest Kartagener syndrome.\n\nBullet Summary:\nKartagener syndrome presents with recurrent sinusitis, bronchiectasis, and situs inversus.", "link": "https://bit.ly/40wICHc"} {"question": "A 4-day-old boy presents to the pediatrician with his mother for his 1st well visit. The patient was born at 36 weeks gestation to a 26-year-old primigravid mother via cesarean section for cervical incompetence. The patient required no resuscitation at birth and both mother and child were discharged from the hospital at 2 days of life. The patient has been exclusively breastfed since birth, and his mother reports that he feeds for 20-30 minutes every 2 hours. The patient urinates 7 times per day and has begun passing 2-3 stools per day that his mother describes as \u201cgrainy\u201d and the color of \u201cmustard.\u201d His birth weight was 3670 g (8 lb 1 oz), and his current weight is 3487 (7 lb 11 oz). His temperature is 97.1\u00b0F (36.2\u00b0C), blood pressure is 57/42 mmHg, pulse is 140/min, and respirations are 38/min. On physical exam, the patient\u2019s anterior fontanelle is soft and flat, and his eyes are moderately icteric. His abdomen is soft and non-distended. The patient has the physical exam finding seen in Figure A. His diaper can be seen in Figure B. Laboratory studies reveal the following:\n\nTotal bilirubin: 13 mg/dL\nConjugated bilirubin: 0.6 mg/dL\n\nWhich of the following is the most appropriate next step in management?", "choicesA": "Administer intravenous hydration", "choicesB": "Continue current breastfeeding regimen", "choicesC": "Order serum uric acid concentration", "choicesD": "Order spot uric acid to creatinine ratio", "choicesE": "Recommend increasing frequency of breastfeeding", "answer_idx": "B", "answer": "Continue current breastfeeding regimen", "explanation": "This patient presents at day 4 of life with weight loss of 5% of his birth weight, uric acid crystals in his diaper, desquamation of the palms and soles, and unconjugated hyperbilirubinemia, which are normal findings in a newborn. The most appropriate next step in management is continuing his current breastfeeding regimen.\n\nHealthy newborns may lose up to 7% of their birth weight in the first 5 days of life and exhibit dry, peeling skin on the palms and soles as the skin adapts to a drier environment outside the womb. Uric acid crystals that look like \u201cbrick dust\u201d are also a normal finding in the diaper during the 1st week of life, as uric acid excretion is highest at birth. All newborns should be evaluated for signs of dehydration, including sunken fontanelles, dry mucous membranes, low urine output, excessive weight loss, and tachycardia. If these findings are not present, continued breastfeeding should be encouraged.\n\nWesterfield et al. review the evidence regarding the benefits of breastfeeding. They discuss how breastfeeding is associated with decreased risk of atopic dermatitis and gastroenteritis. They recommend ensuring proper breastfeeding techniques.\n\nFigure A demonstrates flaking and peeling of the soles (red circles), which is a normal finding in the 1st week of life.\n\nFigure B demonstrates uric acid crystals (\u201cbrick dust\u201d) in the diaper (red circle), which is another normal finding that is caused by high uric acid excretion in the 1st few days of life.\n\nIncorrect Answers:\nAnswer A: Administering intravenous hydration would be appropriate for a severely dehydrated neonate who cannot tolerate hydration orally. This patient is not exhibiting signs of dehydration.\n\nAnswer C: Ordering a serum uric acid concentration may be part of an evaluation for Lesch-Nyhan syndrome, an X-linked recessive disorder of purine metabolism caused by deficiency of the enzyme hypoxanthine-guanine phosphoribosyltransferase (HGPRT) that causes hyperuricemia and hyperuricosuria. The uric acid crystals found in this patient\u2019s diapers are a normal finding in the 1st week of life. Treatment is supportive including allopurinol for gout.\n\nAnswer D: Ordering a spot uric acid to creatinine ratio would be an appropriate first step in evaluation for Lesch-Nyhan syndrome, but the uric acid crystals in this patient\u2019s diaper are not concerning for Lesch-Nyhan syndrome. Treatment is supportive as well as with allopurinol for gout.\n\nAnswer E: Recommending increasing the frequency of breastfeeding would be appropriate for a dehydrated infant with excessive weight loss, but this patient\u2019s weight loss is within the expected limits for the 4th day of life.\n\nBullet Summary:\nHealthy newborns may present with desquamation of the skin and uric acid crystals in the diaper and continued breastfeeding should be encouraged if no signs of dehydration are present.", "link": "https://step2.medbullets.com/testview?qid=109282"} {"question": "A 32-year-old G1P0 presents to the antenatal testing unit at 32 weeks of gestation for a non-stress test (NST). Yesterday she had several episodes of diarrhea and vomited once after attending a company picnic 2 days ago. She took loperamide with symptomatic relief. She is worried that she is dehydrated but otherwise feels well and has recovered from her gastrointestinal illness. The patient was diagnosed with gestational diabetes after routine screening in the 2nd trimester and has been started on insulin after unsuccessful attempts at lifestyle management. She also has migraines and gastroesophageal reflux disorder. The patient has a 15-pack-year smoking history but quit smoking in her 1st month of pregnancy. She denies alcohol or drug use. Her temperature is 98.5\u00b0F (36.9\u00b0C), blood pressure is 122/82 mmHg, pulse is 84/min, and respirations are 14/min. A representative portion of the NST is shown in Figure A. Which of the following is the most likely explanation for this patient\u2019s non-stress test (NST) results?", "choicesA": "Fetal sleep cycle", "choicesB": "Maternal dehydration", "choicesC": "Maternal medication use", "choicesD": "Maternal smoking", "choicesE": "Normal results", "answer_idx": "A", "answer": "Fetal sleep cycle", "explanation": "This patient presents with a nonreactive NST on routine testing. The most common explanation for a nonreactive NST is the fetal sleep cycle.\n\nPatients with gestational diabetes are at increased risk of fetal demise and other perinatal complications. They require closer monitoring during the 3rd trimester of pregnancy. The NST, which lasts 20 minutes, is the most common method of doing so. After 32 weeks, a reactive (i.e. \u201cnormal\u201d) NST requires at least 2 episodes of 15-beat-per-minute accelerations lasting for at least 15 seconds. An NST with no accelerations is considered nonreactive. A common cause of a nonreactive NST is the fetal sleep cycle and it is reasonable to continue the test for an additional 20 minutes to increase the chance of monitoring the fetus while awake. Otherwise, a nonreactive NST should prompt vibroacoustic stimulation or further testing such as a biophysical profile (BPP).\n\nUmana and Siccardi review the evidence regarding the use of the NST during pregnancy. They discuss how the test is a measurement of fetal cardiovascular function at the time of the test. They recommend monitoring the baseline fetal heart rate and variability as well as the presence of accelerations and decelerations when interpreting test results.\n\nFigure/Illustration A shows a representative portion of a NST where the fetal heart rate remains within a narrow band without accelerations (red circle). This pattern is characteristically seen in a nonreactive NST that can be due to the fetal sleep cycle.\n\nIncorrect Answers:\nAnswer B: Maternal dehydration generally should not affect the reactivity of an NST. Although severe dehydration may reduce amniotic fluid volume and therefore increase the baseline fetal pulse, it is unlikely to change the accelerations or variability in an NST. Though this patient had diarrhea the day before, her pulse is not elevated and any dehydration is probably not clinically significant.\n\nAnswer C: Maternal medication use can cause nonreactivity on an NST if drugs such as morphine or other sedatives are taken. Although this patient did take loperamide (an opioid), it is unlikely that this has a sedative effect on the fetus as it acts selectively on the mu-opioid receptors in the large intestine and does not cross the blood-brain barrier.\n\nAnswer D: Maternal smoking in close proximity to an NST has been linked to nonreactivity, but this patient has not smoked in several months. Mothers who smoke are counseled to avoid smoking on the day of their NST to avoid the effects of nicotine on their test results.\n\nAnswer E: A normal result is known as a reactive NST and would present with accelerations and no concerning or late decelerations. This is not a reactive NST due to the lack of appropriate accelerations.\n\nBullet Summary:\nThe most common cause of a non-reactive non-stress test is the fetal sleep cycle.", "link": "https://bit.ly/3Waen6G"} {"question": "A 25-year-old man presents to his primary care physician with a chief complaint of \"failing health.\" He states that he typically can converse with animals via telepathy but is having trouble right now due to the weather. He has begun taking an assortment of Peruvian herbs to little avail. Otherwise, he is not currently taking any medications. The patient lives alone and works in a health food store. He states that his symptoms have persisted for the past 8 months. When obtaining the patient's medical history, there are several times he attempts to telepathically connect with the animals in the vicinity. His temperature is 98.6\u00b0F (37\u00b0C), blood pressure is 115/71 mmHg, pulse is 72/min, and respirations are 12/min. On physical exam, you note a healthy young man who is dressed in an all-burlap ensemble. Which of the following is the most likely diagnosis?", "choicesA": "Brief psychotic disorder", "choicesB": "Schizoid personality disorder", "choicesC": "Schizophrenia", "choicesD": "Schizophreniform disorder", "choicesE": "Schizotypal personality disorder", "answer_idx": "E", "answer": "Schizotypal personality disorder", "explanation": "This patient has several odd and eccentric beliefs without frankly psychotic features and he is able to function in society. This suggests a diagnosis of schizotypal personality disorder.\n\nSchizotypal personality disorder presents with eccentric behavior, magical thinking, odd beliefs, and perceptual disturbances. These patients may seem to have symptoms of schizophrenia; however, they do not respond to internal stimuli and do not experience visual or auditory hallucinations. They are able to function in society and care for themselves. Patients with schizotypal personality disorder typically struggle in maintaining close relationships or making friends. Patients are typically not distressed about their thought patterns but can undergo treatment with psychotherapy if they seek to change their behaviors.\n\nSher reviews the evidence regarding the treatment of patients with schizotypal personality disorder. He discusses how this disorder often goes undiagnosed and is difficult to treat. He recommends providing suicide prevention measures as this disease is associated with suicidal ideation.\n\nIncorrect Answers:\nAnswer A: Brief psychotic disorder presents with either visual or auditory hallucinations with symptoms that have lasted for less than 1 month. Patients have frankly psychotic features and should be closely monitored for the development of schizophrenia.\n\nAnswer B: Schizoid personality disorder presents with social isolation. These patients are alone but do not desire social contact and are content in their isolation. A typical example is an individual who works in a solitary job on the graveyard shift. Patients do not have strange or magical beliefs.\n\nAnswer C: Schizophrenia presents with either visual or auditory hallucinations with symptoms that have lasted for 6 months or longer. These patients typically have trouble caring for themselves and struggle to function in society. Though this patient is attempting to connect/communicate with animals, this likely represents an odd belief rather than responding to internal stimuli. Schizophrenia should be treated with antipsychotic medications.\n\nAnswer D: Schizophreniform disorder presents with either visual or auditory hallucinations with symptoms that have lasted for 1-6 months. Patients have symptoms with frankly psychotic features and should be treated with antipsychotic medications.\n\nBullet Summary:\nSchizotypal personality disorder presents with eccentric behavior, magical thoughts, and odd beliefs but patients are able to function within society.", "link": "https://bit.ly/43nVOhs"} {"question": "A 55-year-old man presents to the emergency department with chest pain and shortness of breath. He has a medical history of hypertension, diabetes, and obesity. His temperature is 98.6\u00b0F (37.0\u00b0C), blood pressure is 177/118 mmHg, pulse is 127/min, respirations are 11/min, and oxygen saturation is 98% on room air. An ECG is performed and notable for ST elevation in leads II, III, and aVF. The patient is treated appropriately and transferred to the medical floor. On the 2nd day of his hospitalization, the patient has abdominal pain. His serum lipase is 272 U/L and his creatinine is 1.6 mg/dL. A physical exam is notable for the finding in Figure A. He is requesting pain medication for his abdominal pain. Which of the following is the most likely underlying diagnosis?", "choicesA": "Cholesterol embolism", "choicesB": "Heart failure and reinfarction", "choicesC": "Medication side effect", "choicesD": "Pancreatitis", "choicesE": "Renal failure", "answer_idx": "A", "answer": "Cholesterol embolism", "explanation": "This patient is presenting with abdominal pain and livedo reticularis after a catheterization procedure. These symptoms are suggestive of a cholesterol embolism.\n\nA cholesterol embolism typically presents after any vascular procedure due to a dislodged cholesterol emboli. Symptoms include livedo reticularis, gastrointestinal problems such as mesenteric ischemia or pancreatitis, and acute kidney injury. Patients who have undergone cardiac catheterization should be monitored for the development of this complication. Treatment is centered around treating the current complications, often using supportive therapy as they self-resolve. In addition, patients should be treated carefully to prevent recurrent embolism.\n\nManingding and Kermani review the evidence regarding the presentation of vasculitis mimics. They discuss how cholesterol embolism can cause diverse systemic symptoms and livedo reticularis. They recommend carefully considering the history of each patient.\n\nFigure/Illustration A is a clinical photograph demonstrating a netlike reticular bluish discoloration of the skin (red circle). These findings are consistent with livedo reticularis.\n\nIncorrect Answers:\nAnswer B: Heart failure and reinfarction could present with renal failure but would likely present with other symptoms of heart failure as well (pulmonary crackles, edema, and jugular venous distension). Treatment of a repeat infarct involves catheterization and fluid overload involves volume optimization.\n\nAnswer C: Medication side effects could explain this patient's livedo reticularis, but the drugs that may cause this (certain antivirals and amantadine) would not have been given to this patient. These medications also typically do not cause abdominal pain.\n\nAnswer D: Pancreatitis presents with abdominal pain and an elevated lipase; however, it is not the underlying pathology for this patient's presentation but rather is a complication of the cholesterol embolism. Treatment of pancreatitis is with gastrointestinal rest and fluid support.\n\nAnswer E: Renal failure is another complication in this patient secondary to the cholesterol embolism as revealed by the elevated creatinine, but is not the underlying pathology. Treatment is with renal replacement therapy if renal function does not return promptly.\n\nBullet Summary:\nCholesterol embolism presents after a vascular procedure with livedo reticularis, gastrointestinal complications, and acute kidney injury.", "link": "https://step2.medbullets.com/testview?qid=210075"} {"question": "A 25-year-old man is brought to the emergency department by paramedics for altered mental status. He was found lying down at a bus stop unresponsive. On arrival, the patient has slurred speech and is somnolent. Further is unable to be elicited. His temperature is 98.0\u00b0F (36.7\u00b0C), blood pressure is 130/80 mmHg, pulse is 80/min, and respirations are 12/min with a saturation of 97% on room air. Physical exam reveals a sleeping man with miotic pupils. The patient is easily arousable to voice and sternal rub but falls asleep rapidly. He has decreased bowel sounds. He is able to state his name and knows that he is at the hospital and has an otherwise normal neurologic exam. Which of the following is the most appropriate management?", "choicesA": "Atropine", "choicesB": "Disulfiram", "choicesC": "Flumazenil", "choicesD": "Naloxone", "choicesE": "Observation", "answer_idx": "E", "answer": "Observation", "explanation": "This patient's presentation with altered mental status, pinpoint pupils, and decreased bowel sounds is most consistent with an opioid overdose. Given his stable respiratory status and oxygen saturation without notable bradypnea, observation alone until the patient is clinically sober is the most appropriate management.\n\nOpioids are commonly used to treat painful conditions but have the potential to become drugs of abuse. Opioid overdose presents with bradypnea, somnolence, decreased bowel sounds, and pinpoint pupils. Any patient who is not protecting their airway, failing to oxygenate, or failing to ventilate should immediately be given naloxone which rapidly reverses the effect of opioids. However, this precipitates withdrawal which is unpleasant for patients and leads to nausea, vomiting, agitation, and diaphoresis among other findings. For this reason, patients who have overdosed on opioids and are stable can be observed until clinically sober.\n\nDietze et al. studied the effect of intranasal versus intramuscular naloxone for opioid overdose in a randomized control trial. The authors found that patients who received intranasal naloxone needed a second dose of naloxone more frequently compared to patients who received intramuscular naloxone. The authors recommend further study of optimal dosing and concentration of intranasal naloxone to respond to opioid overdoses outside of the hospital.\n\nIncorrect Answers:\nAnswer A: Atropine is an appropriate treatment for a cholinergic toxidrome. This would manifest with bronchorrhea, bronchospasm, bradycardia, urinary incontinence, diarrhea, and drooling. Atropine should be administered until respiratory symptoms improve; large doses may be required.\n\nAnswer B: Disulfiram inhibits the enzyme acetaldehyde dehydrogenase causing severe nausea, vomiting, and a very ill feeling when a patient drinks alcohol from the accumulation of byproducts. This drug is a second-line agent used to reduce drinking in chronic alcohol use disorder but has no indication in managing acute alcohol intoxication as it is a preventive medication.\n\nAnswer C: Flumazenil is the reversal agent for benzodiazepines. Its use has fallen out of favor as it can precipitate seizures in chronic benzodiazepine users. Any patient with a suspected benzodiazepine overdose who is protecting their airway should be observed until clinically sober as there is no need to rapidly reverse them and possibly cause seizures.\n\nAnswer D: Naloxone is the antidote of choice in opioid overdose in patients who are not adequately oxygenating, ventilating, or protecting their airway. It is not indicated in opioid overdose in a clinically stable patient as the patient can merely be observed without the unpleasant side effects of immediate withdrawal from naloxone (including vomiting which in some cases could lead to aspiration). This patient is oxygenating well, protecting his airway, and does not have profound bradypnea.\n\nBullet Summary:\nOpioid overdose presents with somnolence, pinpoint pupils, decreased bowel sounds, and bradypnea; stable patients who are adequately breathing and protecting their airway require only observation until clinically sober.", "link": "https://step2.medbullets.com/testview?qid=215094"} {"question": "A 32-year-old man presents to the emergency department with pain upon defecation. His symptoms started a few weeks ago but have been worsening. Sometimes he notes blood on the toilet paper. He has to strain to have bowel movements and has trouble defecating secondary to pain. He is generally healthy and does not smoke. The patient is a software engineer and enjoys hiking, going to nightclubs, and dancing. His temperature is 98.1\u00b0F (36.7\u00b0C), blood pressure is 112/83 mmHg, pulse is 83/min, respirations are 13/min, and oxygen saturation is 98% on room air. Physical exam reveals a small, superficial fissure that is lateral to the anus and lateral to the midline. Which of the following is the most appropriate management for this patient?", "choicesA": "CT scan of the abdomen and pelvis", "choicesB": "Incision, drainage, ciprofloxacin, and metronidazole", "choicesC": "Surgical excision", "choicesD": "Testing for HIV, fecal occult blood, and sexually transmitted diseases", "choicesE": "Topical lidocaine, sitz baths, and fiber supplements", "answer_idx": "D", "answer": "Testing for HIV, fecal occult blood, and sexually transmitted diseases", "explanation": "This patient is presenting with pain upon defecation, constipation, and a fissure lateral to the midline which is suggestive of an anal fissure. Anal fissures lateral to the midline are associated with inflammatory and infectious etiologies; thus, further workup (testing for HIV, fecal occult blood testing, and sexually transmitted disease testing) is needed rather than merely symptomatic treatment.\n\nAnal fissures present with exquisite pain upon defecation which may lead to fecal retention and constipation. Commonly, anal fissures are caused by constipation, local trauma (such as anal intercourse), or a tight anal sphincter. Anal fissures that occur secondary to trauma are usually midline. On the other hand, lateral anal fissures are a harbinger of more serious alternative diagnoses and require further workup. Lateral anal fissures can be caused by infections (HIV being one of the most concerning as well as syphilis or tuberculosis), Crohn disease, granulomatous disease, malignancy, as well as foreign bodies. For this reason, lateral anal fissures should not only be treated symptomatically but require testing for sexually transmitted diseases, inflammatory conditions, and malignancy to determine the underlying etiology based on the patient\u2019s risk factors and associated symptoms.\n\nSteele and Madoff review anal fissures. They note that while both operative and non-operative interventions may be indicated in the management of an anal fissure after alternative diagnoses have been ruled out, advances in medical management may spare patients sphincter-dividing surgery. They recommend considering all modalities.\n\nIncorrect Answers:\nAnswer A: CT scan of the abdomen and pelvis followed by an MRI may be the most appropriate testing if a perirectal abscess is suspected. These abscesses can track very deep. For this reason, further imaging and drainage in the operating room followed by antibiotics are indicated rather than a simple bedside incision and drainage. Imaging may be indicated in this patient after medical workup fails to elucidate an etiology.\n\nAnswer B: Incision, drainage, ciprofloxacin, and metronidazole may be indicated for a perianal abscess (note, this is not a perirectal abscess), which presents with a tender, fluctuant mass right next to the anus. The diagnosis can be made clinically or supported by an ultrasound.\n\nAnswer C: Surgical excision may be performed in the management of a perianal fistula, which presents with a draining tract next to the anus that releases purulence or fecal contents causing staining of the patient\u2019s underwear and irritation of the skin. The diagnosis is made with an exam and a CT, MRI, or fistulogram followed by a fistulotomy.\n\nAnswer E: Topical lidocaine, sitz baths, and fiber supplements are the appropriate management of an anal fissure that is midline and secondary to a clear cause such as trauma or constipation. It would be inappropriate to only treat this patient symptomatically with a lateral anal fissure as this is a sign of a more serious diagnosis.\n\nBullet Summary:\nAnal fissures that are lateral to the midline require workup for sexually transmitted infections, cancer, and autoimmune disease.", "link": "https://step2.medbullets.com/testview?qid=216438"} {"question": "A 31-year-old man presents to the emergency department for fever, malaise, and cough. For the last week, his cough has been progressively worsening, and he has been noticing blood in his sputum. He denies any recent travel or sick contacts and infrequently smokes cigarettes. Approximately 5 weeks ago, he presented with similar symptoms and was found to have a right upper lobe lung infiltrate, a CD4 count of 40/mm^3, and an HIV viral load of 115,000 copies/mL. He was appropriately treated and discharged home. Four weeks after initiation of treatment, his CD4 count was over 400/mm^3 and HIV viral load was negligible. His temperature today is 102\u00b0F (38.9\u00b0C), blood pressure is 130/90 mmHg, pulse is 100/min, and respirations are 20/min. A radiograph of the chest demonstrates new nodules in the left upper lobe and hilar adenopathy. Which of the following is the most appropriate next step in management?", "choicesA": "Continue current anti-retroviral treatment, antimicrobial, and anti-tuberculoid therapies", "choicesB": "Determine drug sensitivities against the patient\u2019s pulmonary infection", "choicesC": "Discontinue antimicrobial treatment", "choicesD": "Obtain a D-dimer level", "choicesE": "Start high-dose glucocorticoid treatment and discontinue antiretroviral therapy", "answer_idx": "A", "answer": "Continue current anti-retroviral treatment, antimicrobial, and anti-tuberculoid therapies", "explanation": "This patient with recently diagnosed HIV and tuberculosis infection (upper lung lobe infiltrate) is presenting with worsening symptoms after being appropriately treated. Worsening of symptoms after initiation of these therapies is concerning for immune reconstitution inflammatory syndrome (IRIS), which is managed by continuing current combined antiretroviral medications, antibiotics for opportunistic infections and symptom management.\n\nIRIS describes a paradoxical worsening of a patient's underlying infection after initiating antiretroviral therapy for patients with HIV. IRIS can also occur in HIV-uninfected patients with tuberculosis or leprosy who are started on antimycobacterial treatment. The pathogenesis of IRIS is unclear, but in patients with HIV, antiretroviral therapy increases the amount of CD4+ T-helper cells, which can lead to an exaggerated inflammatory response (e.g., increased cytokine release) towards the infection. Thus, there is a paradoxical worsening of the infection symptoms. When patients have a paradoxical worsening of their symptoms in the setting of starting their antiretroviral therapy, the therapy should be continued with symptomatic management (acetaminophen for fever).\n\nNelson et al review IRIS. They note it is seen in patients with HIV. They recommend antiretroviral therapy and antibiotics for bacterial infections.\n\nIncorrect Answers:\nAnswer B: Determining drug sensitivities against the patient\u2019s pulmonary infection would be of low diagnostic utility since IRIS best explains his current clinical presentation.\n\nAnswer C: Discontinuing antimicrobial treatment would not be appropriate since the patient has findings concerning for tuberculosis (upper lobe lung infiltrate), and lack of treatment can lead to progression of his infection.\n\nAnswer D: Obtaining a D-dimer level is only indicated in low-risk patients for pulmonary embolism by Wells criteria.\n\nAnswer E: Starting high-dose glucocorticoid treatment and discontinuing antiretroviral therapy would not be appropriate since discontinuing his antiretroviral therapy would not control his HIV infection, which would place him at greater risk of acquiring opportunistic infections. Corticosteroid use is part of symptom management as it decreases the inflammatory response.\n\nBullet Summary:\nImmune reconstitution inflammatory syndrome (IRIS) can result in worsening of infectious symptoms and is managed by continuing antiviral therapy with symptomatic treatment.", "link": "https://step2.medbullets.com/testview?qid=214791"} {"question": "A 55-year-old man presents to the emergency department with chest pain and shortness of breath. He has a medical history of hypertension, diabetes, and obesity. His temperature is 98.6\u00b0F (37.0\u00b0C), blood pressure is 177/118 mmHg, pulse is 127/min, respirations are 11/min, and oxygen saturation is 98% on room air. An ECG is performed and notable for ST elevation in leads II, III, and aVF. The patient is treated appropriately and transferred to the medical floor. On the 2nd day of his hospitalization, the patient has abdominal pain. His serum lipase is 272 U/L and his creatinine is 1.6 mg/dL. A physical exam is notable for the finding in Figure A. He is requesting pain medication for his abdominal pain. Which of the following is the most likely underlying diagnosis?", "choicesA": "Cholesterol embolism", "choicesB": "Heart failure and reinfarction", "choicesC": "Medication side effect", "choicesD": "Pancreatitis", "choicesE": "Renal failure", "answer_idx": "A", "answer": "Cholesterol embolism", "explanation": "This patient is presenting with abdominal pain and livedo reticularis after a catheterization procedure. These symptoms are suggestive of a cholesterol embolism.\n\nA cholesterol embolism typically presents after any vascular procedure due to a dislodged cholesterol emboli. Symptoms include livedo reticularis, gastrointestinal problems such as mesenteric ischemia or pancreatitis, and acute kidney injury. Patients who have undergone cardiac catheterization should be monitored for the development of this complication. Treatment is centered around treating the current complications, often using supportive therapy as they self-resolve. In addition, patients should be treated carefully to prevent recurrent embolism.\n\nManingding and Kermani review the evidence regarding the presentation of vasculitis mimics. They discuss how cholesterol embolism can cause diverse systemic symptoms and livedo reticularis. They recommend carefully considering the history of each patient.\n\nFigure/Illustration A is a clinical photograph demonstrating a netlike reticular bluish discoloration of the skin (red circle). These findings are consistent with livedo reticularis.\n\nIncorrect Answers:\nAnswer B: Heart failure and reinfarction could present with renal failure but would likely present with other symptoms of heart failure as well (pulmonary crackles, edema, and jugular venous distension). Treatment of a repeat infarct involves catheterization and fluid overload involves volume optimization.\n\nAnswer C: Medication side effects could explain this patient's livedo reticularis, but the drugs that may cause this (certain antivirals and amantadine) would not have been given to this patient. These medications also typically do not cause abdominal pain.\n\nAnswer D: Pancreatitis presents with abdominal pain and an elevated lipase; however, it is not the underlying pathology for this patient's presentation but rather is a complication of the cholesterol embolism. Treatment of pancreatitis is with gastrointestinal rest and fluid support.\n\nAnswer E: Renal failure is another complication in this patient secondary to the cholesterol embolism as revealed by the elevated creatinine, but is not the underlying pathology. Treatment is with renal replacement therapy if renal function does not return promptly.\n\nBullet Summary:\nCholesterol embolism presents after a vascular procedure with livedo reticularis, gastrointestinal complications, and acute kidney injury.", "link": "https://bit.ly/481p4x3"} {"question": "A 32-year-old man presents to his primary care provider for a headache. He reports that he has headaches at night several times a week. He first developed these headaches over 1 year ago, but they started up again 3 weeks ago. The episodes start suddenly and feel like a stabbing, electrical pain over his left eye. He also reports tearing of the left eye during these episodes. The headaches self-resolve over the course of 2-3 hours, but the patient complains that he is avoiding going to sleep for fear of waking up in pain. His medical history includes type 1 diabetes mellitus and an episode of herpes zoster on his right flank 1 year ago. His only home medication is insulin. His temperature is 98.6\u00b0F (37\u00b0C), blood pressure is 112/69 mmHg, pulse is 61/min, and respirations are 14/min. On physical exam, his extraocular muscles are intact and his eyes are not injected. A CT of the head and sinuses shows no acute abnormalities. Which of the following is most likely to prevent future episodes of headache in this patient?", "choicesA": "Carbamazepine", "choicesB": "High-flow oxygen", "choicesC": "Sumatriptan", "choicesD": "Topiramate", "choicesE": "Verapamil", "answer_idx": "E", "answer": "Verapamil", "explanation": "This patient presents with an episodic, stabbing pain in the periorbital region that occurs at night, which is consistent with a diagnosis of cluster headaches. Verapamil is used as prophylaxis against cluster headaches.\n\nCluster headaches usually occur at night and may wake patients out of sleep. Cluster headaches may occur several times a week to multiple times daily and the attacks may be followed by a period of remission. Cluster headaches also typically involve the periorbital region and are unilateral. Patients also classically describe their headaches as \u201cstabbing\u201d. Tension-type headaches are more commonly described as dull and migraines are usually described as pulsating or throbbing. Treatment is with high-flow oxygen during acute episodes and prophylaxis against long-term headaches includes calcium channel blockers such as verapamil.\n\nHoffman and May review the evidence regarding the treatment of cluster headaches. They discuss how high-flow oxygen and triptans are the most effective abortive treatment for acute attacks. They recommend using chronic prophylactic medications such as verapamil.\n\nIncorrect Answers:\nAnswer A: Carbamazepine is the treatment of choice for trigeminal neuralgia, which presents as shooting pain in the face. They have exquisite tenderness to palpation over the affected neurologic distribution.\n\nAnswer B: High-flow oxygen is an abortive treatment for cluster headaches. Cluster headaches typically present in men with unilateral pain around the eye that is worse at night. Associated symptoms include spasms of the unilateral face as well as tearing and visual abnormalities in the eye. This treatment is not effective for long-term prophylaxis.\n\nAnswer C: Sumatriptan is used as an abortive treatment for cluster headaches and migraines, but it is not used as prophylaxis for either. Migraines present as unilateral, throbbing headaches that may be associated with nausea and/or vomiting, photophobia, phonophobia, and aura.\n\nAnswer D: Topiramate is used as prophylaxis for migraines rather than cluster headaches. Migraines present as unilateral, throbbing headaches that may be associated with nausea and/or vomiting, photophobia, phonophobia, and aura.\n\nBullet Summary:\nCluster headaches classically present as sharp or stabbing periorbital headaches that occur at night, with verapamil being the prophylactic treatment of choice and high-flow oxygen being the most effective abortive treatment.", "link": "https://bit.ly/3n1kBse"} {"question": "A 62-year-old man presents to the emergency department with skin lesions. He felt itchy recently, then noticed skin lesions that broke out prompting him to come to the emergency department. He recently went camping and hiking in the woods. The patient drinks 1-2 beers per day and smokes 1 pack of cigarettes per day. He is currently sexually active with multiple partners and does not use condoms. The patient was discharged 1 week ago for pneumonia which was treated with azithromycin; however, he feels that his symptoms may be returning. His temperature is 99.5\u00b0F (37.5\u00b0C), blood pressure is 157/78 mmHg, pulse is 90/min, respirations are 15/min, and oxygen saturation is 98% on room air. Physical exam reveals the findings in Figure A on the patient's arm. The skin lesions are thick and do not break when pressure is applied. Which of the following most appropriately describes the most likely diagnosis?", "choicesA": "IgG in a fish-net pattern", "choicesB": "Linear IgG along the basement membrane", "choicesC": "Multi-nucleated giant cells", "choicesD": "Panniculitis", "choicesE": "Type IV hypersensitivity reaction", "answer_idx": "B", "answer": "Linear IgG along the basement membrane", "explanation": "This patient is presenting with pruritic bullous skin lesions with a thick/tough covering suggesting a diagnosis of bullous pemphigoid. Linear IgG binding of anti-hemidesmosome antibodies along the basement membrane is the underlying cause of this disease.\n\nBullous pemphigoid presents with a urticarial prodrome in elderly patients followed by the formation of tense bullae that do not break when pressure is applied to them. IgG antibodies bind the hemidesmosome along the basement membrane and appear linear on immunofluorescence. Risk factors that are associated with this condition include the initiation of new medications such as loop diuretics, metformin, and neuroleptics. Steroids are the treatment of choice.\n\nMiyamoto et al. review the evidence regarding the diagnosis and treatment of bullous pemphigoid. They discuss how high-potency corticosteroids are the mainstay of treatment for this disease. They recommend considering adjuvant therapy with doxycycline, dapsone, and immunosuppressants.\n\nFigure/Illustration A is a clinical photograph showing bullous skin lesions (red circles). These lesions are tense and do not separate/break when pressure is applied to them and are classically seen in bullous pemphigoid.\n\nIncorrect Answers:\nAnswer A: IgG in a fish-net pattern suggests a diagnosis of pemphigus vulgaris, which presents with skin lesions that are flaccid and break easily when pressure is applied. Treatment is with corticosteroids.\n\nAnswer C: Multi-nucleated giant cells suggests a diagnosis of HSV or herpes zoster virus, which presents with potentially painful vesicles distributed in a dermatomal pattern (for zoster). Treatment is with antivirals such as valacyclovir.\n\nAnswer D: Panniculitis suggests a diagnosis of erythema nodosum, which presents with painful, raised nodules that are associated with inflammatory disorders and infection. Treatment is centered on addressing the underlying pathology.\n\nAnswer E: Type IV hypersensitivity reaction suggests a diagnosis of contact dermatitis, which presents with pruritic lesions after exposure to a particular antigen such as poison ivy, poison oak, or nickel. Treatment is avoidance of the precipitating agent and steroids as well as symptomatic control.\n\nBullet Summary:\nBullous pemphigoid occurs secondary to anti-hemidesmosome antibodies which appear linear along the basement membrane on immunofluorescence.", "link": "https://bit.ly/3qLbsG6"} {"question": "A 45-year-old man presents to his primary care physician with a rash. He first noticed the rash 2 weeks prior and he endorses intermittent itchiness from the rash. His medical history is significant for chronic hepatitis C infection, hypertension, type 2 diabetes mellitus, and heart failure. The patient works as a butcher at a local meat processing shop and is sexually active with several partners. He does not use barrier protection. His temperature is 98.6\u00b0F (37.0\u00b0C), pulse is 90/min, blood pressure is 155/95 mmHg, and respirations are 12/min. On physical exam, his skin has the finding shown in Figure A. Fine, lace-like white lines are also noted on the patient\u2019s buccal mucosa. Which of the following is the most likely diagnosis?", "choicesA": "Atopic dermatitis", "choicesB": "Lichen planus", "choicesC": "Prurigo nodularis", "choicesD": "Psoriasis", "choicesE": "Scabies", "answer_idx": "B", "answer": "Lichen planus", "explanation": "This patient presents with several purple, polygonal, pruritic papules and plaques. These findings are most consistent with a diagnosis of lichen planus.\n\nLichen planus is an inflammatory skin disorder most commonly characterized by an eruption of Purple, Polygonal, Planar, Pruritic, Papules and Plaques (the 6 P\u2019s), often on extensor surfaces such as the wrist. Development of fine white lines on the surfaces of skin lesions, as well as on the buccal mucosa (Wickham striae), are also characteristically seen in this disease. Concurrent hepatitis C infection is a risk factor for lichen planus. A skin biopsy will show a sawtooth infiltrate of lymphocytes at the dermal-epidermal junction. Treatment is with antihistamines for pruritus and topical steroids for localized disease as well as addressing the underlying cause.\n\nTziotzios et al. review the evidence regarding the diagnosis and treatment of lichen planus. They discuss how the molecular basis of this disorder is relatively unknown. They recommend making a clear diagnosis to allow for treatment.\n\nFigure/Illustration A shows multiple purple, polygonal, papules and plaques on the wrist (red circles). These findings are characteristic of lichen planus.\n\nIncorrect Answers:\nAnswer A: Atopic dermatitis most commonly presents with pruritic, lichenified plaques on the flexural surfaces. The mucous membranes are not involved. Treatment is with topical moisturizers and antihistamines for pruritus. This disease is also associated with other atopic conditions such as asthma and seasonal allergies.\n\nAnswer C: Prurigo nodularis is characterized by hypertrophic, dome-shaped papules that develop secondary to chronic scratching. The mucous membranes are not involved. Treatment is with avoidance of the triggering behavior such as scratching and antihistamines for pruritus.\n\nAnswer D: Psoriasis is characterized by silver, scaly plaques that bleed easily when the scale is physically removed (Auspitz sign). Patients with cutaneous psoriasis are at increased risk for the development of psoriatic arthritis. Psoriasis can be treated with topical corticosteroids as well as calcipotriene.\n\nAnswer E: Scabies is a cutaneous infection with the mite Sarcoptes scabei that presents with severe pruritus, especially in the intertriginous areas. Patients may have serpiginous scaling lesions that represent the burrowing of the mite through the skin, but mucous membrane lesions are absent. Treatment can include topical permethrin first line.\n\nBullet Summary:\nLichen planus presents with the 6 P\u2019s \u2013 Purple, Pruritic, Polygonal, Planar, Papules and Plaques.", "link": "https://bit.ly/3JrjtWO"} {"question": "A 27-year-old woman presents to her primary care physician with new hair growth on her face and lower abdomen over the last month. She has started to develop pimples on her face and back over the last several months. Her last menstrual period was over 3 months ago and her periods have been irregular over the last year. She has been gaining weight recently. The patient has a medical history of obesity and prediabetes with a hemoglobin A1c of 6.0% last year. Her temperature is 98.5\u00b0F (36.9\u00b0C), pulse is 80/min, blood pressure is 139/88 mmHg, and respirations are 13/min. Cardiopulmonary exam is unremarkable, and the patient\u2019s abdomen appears slightly distended but exhibits no tenderness to palpation. The patient\u2019s face has coarse stubble along the jawline and on the upper lip, and there is similar hair along the midline of her lower abdomen. A pelvic exam reveals mild clitoromegaly, a normal anteverted uterus, and a large left adnexal mass that is mildly tender. Her laboratory test results are shown below:\n\nHemoglobin: 13.9 g/dL\nLeukocyte count: 8,000 cells/mm^3\nPlatelet count: 142,000/mm^3\n\nDHEAS: 73 ug/dL (Normal: 145-395 ug/dL)\nTestosterone: 256 ng/dL\n17-hydroxyprogesterone: 214 ng/dL (Normal: < 200 ng/dL)\n\nUltrasound findings are shown in Figure A. Which of the following is the most likely diagnosis in this patient?", "choicesA": "Adrenal tumor", "choicesB": "Congenital adrenal hyperplasia", "choicesC": "Ovarian hyperthecosis", "choicesD": "Ovarian tumor", "choicesE": "Polycystic ovary syndrome", "answer_idx": "D", "answer": "Ovarian tumor", "explanation": "This young woman presents with rapid virilization, elevated testosterone, and a large left adnexal mass that is primarily solid on ultrasound. These findings are most consistent with an ovarian androgen-secreting tumor.\n\nSertoli-Leydig cell tumors are the most common androgen-secreting tumor of the ovaries and classically result in rapid virilization of affected women. Testosterone levels are often over 200 ng/dL, and 17-hydroxyprogesterone may also be slightly elevated as it is an androgen precursor. The androgen excess may result in symptoms such as facial and abdominal hirsutism, oligomenorrhea, deepening voice, and acne. Treatment involves staging the tumor and surgical resection of the tumor if possible.\n\nDurmus et al. review the evidence regarding the diagnosis and treatment of Sertoli-Leydig cell tumors. They discuss how this tumor generally has a favorable prognosis compared to malignant epithelial tumors. They recommend using fertility-preserving surgeries in younger patients.\n\nFigure/Illustration A shows an ultrasound of the left ovary, which has a predominantly solid mass with vascularity and some cystic spaces (red circle). These findings are consistent with an ovarian tumor.\n\nIncorrect Answers:\nAnswer A: Adrenal tumors may secrete androgens and cause similar symptoms of virilization, but the presence of a malignant-appearing ovarian mass in this patient is more consistent with ovarian pathology. Furthermore, adrenal tumors typically secrete DHEAS, which can be a helpful test for localizing the source of androgen excess. Treatment is with surgical excision.\n\nAnswer B: Congenital adrenal hyperplasia (CAH) results from one of several possible enzyme deficiencies in the steroid pathway and may appear later in life as virilization of a woman. While 17-hydroxyprogesterone elevation is a strong indicator of the most common subtype of CAH, this patient\u2019s ultrasound findings suggest ovarian pathology instead, which could also elevate this androgen precursor. Treatment is with hormone replacement therapy.\n\nAnswer C: Ovarian hyperthecosis is hyperplasia of the theca interna of the ovary, which produces androgens. Testosterone levels would be elevated and virilization may occur, but ultrasound findings would show a bilateral increase in stromal tissue as opposed to a unilateral solid mass. Treatment is with hormonal therapy.\n\nAnswer E: Polycystic ovary syndrome (PCOS) can also cause elevated testosterone, though levels are typically lower than 100 ng/dL. Frank virilization with clitoromegaly and voice deepening would not be expected with PCOS, nor would an ovarian mass. Treatment is with metformin and lifestyle modifications.\n\nBullet Summary:\nSertoli-Leydig cell tumors of the ovary can produce androgens and result in rapid onset of virilization with elevated testosterone levels and an adnexal mass on ultrasound.", "link": "https://bit.ly/3qYGGdc"} {"question": "A study is performed that looks at individuals who suffer from opioid use disorder and the effect the disorder has on their daily life and functionality. Individuals are surveyed using a validated method and followed up in 6-month intervals for 5 years. At the end of the study period, 40% of individuals have replied to the survey throughout the entire period. Analyzing only those subjects who responded throughout the entire period, it is determined that opioid use disorder only minimally impairs individuals in their daily functioning. Which of the following most likely occurred in this study?", "choicesA": "Attrition bias", "choicesB": "Confounding variable", "choicesC": "Late-look bias", "choicesD": "Recall bias", "choicesE": "Volunteer bias", "answer_idx": "A", "answer": "Attrition bias", "explanation": "This study following individuals who abuse opioids with a 40% follow-up response rate suggests that opioid use disorder (known for impairing daily functioning) has only a minor effect on patient functionality. This is most likely secondary to attrition bias, or a loss to follow-up of participants in a non-random fashion whereby more severe cases stopped responding and were thus left out.\n\nBias in studies distorts outcomes and can make interpreting results challenging. For this reason, it is important to be meticulous when designing a study with every effort to minimize bias. The attrition bias can occur when individuals are lost to follow-up in a non-random fashion. For example, consider the assessment of daily functionality via a validated method in individuals with coronary artery disease. Those who are severely impaired from disease progression may not follow up in this study, thus altering results and removing more severe cases from the analysis. Having a higher response rate can help mitigate this bias, but is more costly as it may be difficult to ensure that every participant follows up.\n\nNunan et al. discuss the attrition bias. They note how it can affect studies and the data collected. They recommend mitigating the attrition bias for more accurate results and clinical application.\n\nIncorrect Answers:\nAnswer B: Confounding variables are variables that are correlated with the studied variable that better explain the difference between 2 groups. For example, it may be found that alcohol is associated with lung cancer when in reality, the confounding variable of smoking (more common in individuals who drink alcohol) explains this difference.\n\nAnswer C: Late-look bias occurs when patients with severe disease are not studied because they are dead, critically ill, or unavailable to be studied, making the disease overall appear less severe. For example, patients with HIV may not have increased morbidity and mortality in a study. However, this may be a result of all the critically ill cases being too sick or dead and thus unable to participate thus distorting the outcome.\n\nAnswer D: Recall bias occurs when individuals with disease or an adverse outcome are more likely to recall an exposure given the occurred outcome, skewing their memory and thus the results. For example, pregnant mothers who have a newborn with a defect or condition may be more likely to recall certain exposures (such as exposure to pollution).\n\nAnswer E: Volunteer bias occurs when people who volunteer for a study are fundamentally different from the general population or those who do not volunteer. An example may be individuals volunteering in a study involving exercise as those who are interested in participating in an exercise study may be more motivated or healthier at baseline.\n\nBullet Summary:\nAttrition bias occurs when individuals are lost to follow-up in a non-random fashion leading to skewed results.", "link": "https://step2.medbullets.com/testview?qid=216600"} {"question": "A 62-year-old woman is brought to a physician by her husband for fluctuating changes in behavior and hallucinations over the past 2 years. On occasion, she becomes difficult to understand and her speech becomes disorganized. She also frequently sees \"small people\" and animals on the dining room table, which is disturbing to her. Her medical problems include hypertension and chronic obstructive pulmonary disease for which she takes amlodipine, albuterol, ipratropium, and a fluticasone inhaler. She was formerly an executive at a software company but had to retire due to frequent memory lapses. Her temperature is 98.6\u00b0F (37\u00b0C), blood pressure is 120/80 mmHg, pulse is 80/min, and respirations are 12/min. Physical exam is notable for slowed movements and difficulty with balance at rest, which were not present at her wellness visit 1 year ago. Which of the following pathological findings (Figures A-E) would most likely be seen if a brain biopsy were performed?", "choicesA": "Figure A", "choicesB": "Figure B", "choicesC": "Figure C", "choicesD": "Figure D", "choicesE": "Figure E", "answer_idx": "C", "answer": "Figure C", "explanation": "This patient with fluctuating changes in behavior and cognition, visual hallucinations, memory loss, and extrapyramidal symptoms (bradykinesia, postural instability) most likely has Lewy body dementia. Lewy body dementia is characterized histologically by Lewy bodies, which are intracytoplasmic eosinophilic inclusions of alpha-synuclein (shown in Figure C).\n\nAlpha-synuclein is normally abundantly present in its unfolded form in neurons. Abnormal folding of alpha-synuclein leads to aggregation into insoluble fibrils. These intracytoplasmic aggregates (Lewy bodies) are seen in Parkinson disease, Lewy body dementia, and multiple system atrophy, which are collectively referred to as alpha-synucleinopathies. Lewy body dementia is the second most common degenerative dementia and presents with fluctuating cognition, visual hallucinations, parkinsonism (bradykinesia, postural instability, rigidity, and a resting tremor), rapid eye movement (REM) sleep disorder, autonomic dysfunction, and sensitivity to antipsychotics. Lewy body dementia can be distinguished from Parkinson disease with dementia by the timing of symptoms. In Lewy body dementia, cognitive symptoms begin at the same time as or 1 year prior to the onset of movement symptoms whereas in Parkinson disease the cognitive symptoms classically occur over 1 year after motor symptoms. There is no cure for Lewy body dementia, and treatment is aimed at alleviating symptoms.\n\nArnaoutoglou et al. reviewed the epidemiology, pathology, treatment, and future directions for research of Lewy body dementia. The authors found potential genetic bases for Lewy body dementia, including mutations in the GBA gene. They recommended that further research be conducted on the therapeutic implications of these genetic findings.\n\nFigure/Illustration A shows a hematoxylin and eosin (H&E) stained brain biopsy of a patient with Creutzfeldt-Jakob disease. Characteristic spongiform changes in neurons (yellow boxes) and agglomerated prions (orange arrows) are seen in this slide.\n\nFigure/Illustration B shows an H&E stained brain biopsy of a patient with a small lacunar infarct on low magnification. A central area of tissue loss (yellow outline) and surrounding residual gliosis (orange arrows) are seen in this slide.\n\nFigure/Illustration C shows a brain biopsy of a patient with Lewy body dementia stained with mouse anti-alpha-synuclein antibody and counterstained with hematoxylin. Lewy bodies (intracytoplasmic eosinophilic inclusions of alpha-synuclein) are indicated by orange arrows. Note that Lewy bodies are always found intracellularly (neurons highlighted by blue boxes).\n\nFigure/Illustration D shows an H&E stained brain biopsy of a patient with Alzheimer disease. Extracellular neurofibrillary tangles found in Alzheimer disease are indicated by black arrows.\n\nFigure/Illustration E shows an H&E stained brain biopsy of a patient with frontotemporal dementia. A characteristic Pick cell (red box), or ballooned neuron, with dissolved chromatin (yellow arrow), is seen in this slide.\n\nIncorrect Answers:\nAnswer A: Figure A shows an H&E stained brain biopsy of a patient with Creutzfeldt-Jakob disease. Creutzfeldt-Jakob disease is a rapidly progressive dementia that manifests with poor concentration, declining executive function, startle myoclonus, 14-3-3 protein on lumbar puncture, and synchronous sharp wave complexes on electroencephalogram (EEG). Clinical decline occurs over the course of weeks and months, culminating in death within a year. Creutzfeldt-Jakob disease shows spongiform vacuolation due to the accumulation of abnormal prion protein (PrPsc).\n\nAnswer B: Figure B shows an H&E stained brain biopsy of a patient with a small lacunar infarct. Lacunar infarcts may be asymptomatic or cause focal neurologic deficits such as motor hemiparesis, pure sensory stroke, ataxic hemiparesis, or dysarthria-clumsy hand syndrome depending on the territory involved. Lacunar strokes will show diffuse eosinophilia of the affected area, followed by liquefactive necrosis with neutrophilic infiltration, followed by macrophages with surrounding reactive gliosis, finally leading to an area of tissue loss with residual gliosis.\n\nAnswer D: Figure D shows an H&E stained brain biopsy of a patient with Alzheimer disease. Alzheimer disease presents with a gradual cognitive decline with difficulty completing activities of daily living. Memory is the most prominently affected domain. Visual hallucinations, fluctuating levels of cognition, and extrapyramidal symptoms are not common in Alzheimer disease. Amyloid plaques and neurofibrillary tangles may be seen on brain biopsy.\n\nAnswer E: Figure E shows an H&E stained brain biopsy of a patient with frontotemporal dementia (Pick disease). The behavioral variant of frontotemporal dementia is associated with early-onset personality and behavioral changes, and it may be associated with extrapyramidal symptoms. Unlike Lewy body dementia, memory is usually not affected in frontotemporal dementia until late in the disease course. frontotemporal dementia is characterized by Pick cells (ballooned neurons with dissolved chromatin) and Pick bodies, round intracellular aggregates of hyperphosphorylated tau protein.\n\nBullet Summary:\nLewy body dementia presents with alterations in consciousness, disorganized speech, visual hallucinations, extrapyramidal symptoms, early compromise of executive functions, and eosinophilic intracytoplasmic inclusions of alpha-synuclein (Lewy bodies) on biopsy.", "link": "https://step2.medbullets.com/testview?qid=217097"} {"question": "A 6-month-old boy is brought to the physician by his mother for abnormal movements. Several times a day, he experiences paroxysmal episodes of crying, bending at the waist, and jerking of the arms and legs. These episodes last 1-2 seconds, but occur in clusters every 20-30 seconds for several minutes. His mother's pregnancy, labor, delivery, and his neonatal course have been unremarkable. He is rolling over less frequently than before, has worse head control, and no longer smiles socially. His temperature is 98.6\u00b0F (37\u00b0C), blood pressure is 92/46 mmHg, pulse is 115/min, and respirations are 34/min. On examination, no dysmorphic features are noted. A 24-hour electroencephalography (EEG) is performed, during which the patient has one of these spells. A chaotic pattern of slowing and multifocal epileptiform charges is seen. Which of the following is the pathophysiology of this patient's condition?", "choicesA": "Abnormalities in T-type calcium channels", "choicesB": "Anti-N-methyl-D-aspartate (NMDA) receptor antibodies", "choicesC": "Decreased gamma aminobutyric acid (GABA) levels", "choicesD": "Excess of corticotropin releasing hormone", "choicesE": "Vitamin B6 deficiency", "answer_idx": "D", "answer": "Excess of corticotropin releasing hormone", "explanation": "This patient with infantile spasms (bending at the waist and jerking out the arms and legs), developmental regression (rolling over less frequently, worsening head control, loss of social smile), and hypsarrhythmia on video-EEG (slowing and multifocal epileptiform discharges) are characteristic of West syndrome. West syndrome is caused in part by an excess of corticotropin-releasing hormone which leads to increased neuronal excitability.\n\nThe pathogenesis of West syndrome is incompletely understood. Infantile spasms are often idiopathic, but may also be secondary to tuberous sclerosis, hypoxic-ischemic encephalopathy, congenital brain malformations, aminoacidopathies, and organic acidopathies. Brain insults may activate a neuroendocrine stress response, which in turn leads to abnormalities of the hypothalamic-pituitary-adrenal axis. Disturbed immune activation may also play a role. Patients will present with infantile spasms, developmental regression, and slowing and multifocal epileptiform discharges known as hypsarrhythmia. Many affected infants are responsive to ACTH therapy, with cessation of spasms and resolution of hypsarrhythmia in many cases.\n\nD\u2019Alonzo et al. review the epidemiology, clinical presentation, pathophysiology, and treatment of West syndrome. They discuss how patients will present with the classic symptoms of spasms and regression. They recommend early diagnosis of this syndrome so that patients can be treated appropriately.\n\nIncorrect Answers:\nAnswer B: Abnormalities in T-type calcium channels may contribute to absence seizures. Absence seizures are generalized seizures that involve brief, sudden lapses of consciousness and are more common in children. This child\u2019s developmental regression, infantile spasms, and EEG findings are more consistent with West syndrome.\n\nAnswer C: Anti-N-methyl-D-aspartate (NMDA) receptor antibodies may cause anti-NMDA encephalitis. Anti-NMDA encephalitis presents with seizures, psychiatric manifestations, and decreased level of consciousness. Hypsarrhythmia is characteristic of West syndrome and is not consistent with anti-NMDA encephalitis.\n\nAnswer D: Decreased gamma-aminobutyric acid (GABA) levels were once theorized to be the basis of epilepsy syndromes. This theory is less accepted today. Moreover, West syndrome is likely not mediated by disturbances in GABA. Nonetheless, vigabatrin, which in part works by inhibiting GABA metabolism, is an effective medication for patients with West syndrome.\n\nAnswer E: Vitamin B6 deficiency has not been documented in West syndrome. Some studies have suggested vitamin B6 may be of some benefit in the treatment of infantile spasms; however, there is no evidence that response rates to B6 are different from the spontaneous remission rates predicted from natural history data. Therefore, the pathophysiology of West syndrome is more likely CRH-excess mediated rather than due to a deficiency of vitamin B6.\n\nBullet Summary:\nWest syndrome, which may be caused by CRH excess, presents with hypsarrhythmia, developmental regression, and infantile spasms.", "link": "https://bit.ly/3rSAzaC"} {"question": "A 23-year-old woman is admitted to the inpatient psychiatry unit after her boyfriend reported she was \u201cacting funny and refusing to talk.\u201d The patient\u2019s boyfriend states that he came home from work and found the patient sitting up in bed staring at the wall. When he said her name or waved his hand in front of her, she did not respond. When he tried to move her, she would remain in whatever position she was placed. Her temperature is 99\u00b0F (37.2\u00b0C), blood pressure is 122/79 mmHg, pulse is 68/min, and respirations are 12/min with an oxygen saturation of 98% on room air. The patient is lying on the bed with her left arm raised and pointing at the ceiling. She resists any attempt to change her position. The patient remains mute and ignores any external stimuli. The patient\u2019s medical history is significant for depression. She was recently switched from phenelzine to fluoxetine. Which of the following is the most appropriate initial therapy?", "choicesA": "Benztropine", "choicesB": "Cyproheptadine", "choicesC": "Electroconvulsive therapy", "choicesD": "Haloperidol", "choicesE": "Lorazepam", "answer_idx": "E", "answer": "Lorazepam", "explanation": "The patient has a history of depression and is presenting with decreased motor activity, waxy flexibility, lack of response to external stimuli, and mutism, suggesting the diagnosis of catatonia. Catatonia should initially be treated with benzodiazepines like lorazepam.\n\nCatatonia is a state of psychomotor immobility and apathy that is associated with a mood disorder or psychosis. The diagnosis is made clinically by having at least 3 of the following symptoms: stupor (no psychomotor activity), negativism (no response to external stimuli), catalepsy (muscle rigidity in a position against gravity), waxy flexibility (maintaining positioning placed by the examiner), mutism, echolalia (mimicking speech), echopraxia (mimicking movements), stereotypy, grimacing, and non-purposeful agitation or mannerisms. Initial treatment involves benzodiazepines for symptomatic relief. If the patient is unresponsive to benzodiazepines, electroconvulsive therapy (ECT) can be used.\n\nFrancis reviews the evidence regarding the treatment of patients with catatonia. He discusses how benzodiazepines can be used with rapid improvement in symptoms. He recommends electroconvulsive therapy in refractory cases.\n\nIncorrect Answers:\nAnswer A: Benztropine and IV diphenhydramine can be used to treat an acute dystonic reaction. Dystonic reactions present as involuntary spasms or sustained contractions of the face, neck, torso, pelvis, extremities, and larynx. Medications that block dopamine D2 receptors, such as neuroleptics and antiemetics, carry the greatest risk.\n\nAnswer B: Cyproheptadine is a serotonin antagonist that may be part of the treatment for serotonin syndrome. Serotonin syndrome may occur as a result of the simultaneous use of monoamine oxidase inhibitors (MAOIs), selective serotonin reuptake inhibitors (SSRIs), or serotonin-norepinephrine reuptake inhibitors (SNRIs). It presents with myoclonus, hyperthermia, flushing, tachycardia, and delirium.\n\nAnswer C: Electroconvulsive therapy is an effective treatment for catatonia but would be considered 2nd line to benzodiazepines for refractory cases.\n\nAnswer D: Haloperidol is an antipsychotic, which may worsen catatonia. It can cause neuroleptic malignant syndrome, which can mimic catatonia.\n\nBullet Summary:\nCatatonia can present in patients with a history of mood disorders and psychosis with symptoms of decreased motor activity, rigid posture, and mutism, and should be treated initially with a benzodiazepine.", "link": "https://step2.medbullets.com/testview?qid=210373"} {"question": "A 30-year-old man presents to the emergency room with a laceration over the 4th metacarpophalangeal (MCP) joint of his right hand. He reports that he is a mailman, and his closed fist was bitten by a dog while he was delivering mail yesterday. He reports pain but denies fevers, chills, drainage, or any other symptoms. The dog is up to date on vaccinations. His last tetanus vaccine was 3 years ago. He has no medical history and takes no medications. His temperature is 98.6\u00b0F (37.0\u00b0C), pulse is 80/min, blood pressure is 125/75 mmHg, and respirations are 16/min. Examination of the patient's right hand is shown in Figure A. Which of the following is the most appropriate next step in management?", "choicesA": "Amoxicillin-clavulanate", "choicesB": "Cephalexin", "choicesC": "Clindamycin", "choicesD": "Clindamycin plus doxycycline", "choicesE": "Wound closure and amoxicillin-clavulanate", "answer_idx": "A", "answer": "Amoxicillin-clavulanate", "explanation": "This patient has suffered a clenched fist injury which are generally managed with irrigation and amoxicillin-clavulanate.\n\nAmoxicillin-clavulanate is the usual first-line antibiotic for patients with a human or dog bite. Lacerations sustained from animal or human bites should not be sutured due to the risk of infection. Amoxicillin-clavulanate is first-line therapy for prophylaxis and treatment of human bites and dog bites, in general. Closed fist injuries put the patient at a high risk for osteomyelitis, septic arthritis, and tenosynovitis; therefore, prophylactic antibiotics are appropriate. Amoxicillin-clavulanate provides essential coverage against Pasturella multocida, commonly found in the mouths of dogs, cats, and other mammals. In patients with a penicillin allergy, agents such as TMP-SMX, doxycycline, or cefdinir paired with anaerobic coverage (clindamycin or metronidazole) is appropriate.\n\nPresutti discusses the prevention and treatment of dog bites. Appropriate treatment consists of immediate, copious irrigation, assessment for risk of tetanus and rabies, and administration of prophylactic antibiotics as discussed above.\n\nFigure A shows a dog bite wound over the 4th MCP of the right hand.\n\nIncorrect Answers:\nAnswer B: Cephalexin does not cover species that are typically present in the mouths of dogs and other mammals and is therefore not an appropriate antibiotic choice for this patient as monotherapy.\n\nAnswer C: Clindamycin provides coverage of anaerobic species, but is not sufficient alone for treatment of this patient's injury.\n\nAnswer D: Clindamycin plus doxycycline may be appropriate coverage for a dog bite in a patient with an allergy to amoxicillin-clavulanate. However, this patient has no stated allergies and should receive amoxicillin-clavulanate.\n\nAnswer E: Wound closure and amoxicillin-clavulanate would not be appropriate. Lacerations sustained from dog or cat bites are not sutured closed due to the risk of infection.\n\nBullet Summary:\nAmoxicillin-clavulanate is the first line antibiotic choice for patients with dog or cat bites.", "link": "https://bit.ly/46gXevP"} {"question": "A 59-year-old woman presents to her primary care physician for trouble sleeping. When she goes to bed at night, she has an urge to get up out of bed and walk around. The patient often wakes her husband when she does this which irritates him. There is a perpetual uneasiness and feeling of a need to move at night which is relieved by getting up and walking around. The patient denies symptoms during the day. She works as a mail carrier and is nearing retirement. She has a medical history of anxiety, depression, irritable bowel syndrome, and dysmenorrhea. She is not currently taking any medications. Her temperature is 99.5\u00b0F (37.5\u00b0C), blood pressure is 157/98 mmHg, pulse is 80/min, respirations are 17/min, and oxygen saturation is 98% on room air. Physical exam reveals 5/5 strength in the upper and lower extremities, 2+ reflexes in the upper and lower extremities, a stable gait pattern, and normal sensation. Cardiopulmonary and abdominal exams are within normal limits. Which of the following is the most appropriate initial step in management?", "choicesA": "Alprazolam", "choicesB": "Ferrous sulfate", "choicesC": "Iron studies", "choicesD": "Pramipexole", "choicesE": "Supportive therapy and an exercise routine", "answer_idx": "C", "answer": "Iron studies", "explanation": "This patient is presenting with the need to get up and walk around at night, which is suggestive of restless leg syndrome. The most appropriate initial step in management is collecting iron studies to assess for iron deficiency.\n\nRestless leg syndrome presents with a sustained sense of restlessness and a feeling of a need to move one's legs. Symptoms typically occur at night and are relieved by activities such as walking. Iron deficiency can be a precipitating factor in restless leg syndrome as iron is a cofactor in the production of dopamine. Prior to administering iron, it is important to first confirm or rule out the diagnosis of iron deficiency with laboratory values which should include a CBC and iron studies. If iron deficiency exists, iron should be administered. If iron deficiency is not present, then dopamine agonists such as pramipexole or ropinirole can be administered.\n\nGossard et al. review the evidence regarding the diagnosis and treatment of restless leg syndrome. They discuss how iron deficiency is associated with the development of this disorder. They recommend iron repletion in deficient patients.\n\nIncorrect Answers:\nAnswer A: Alprazolam could be an appropriate step in management if the patient suffers from episodic panic attacks/anxiety. It is not an optimal long-term therapy for anxiety or depression. Benzodiazepines are not first-line in treating restless leg syndrome.\n\nAnswer B: Ferrous sulfate would not be indicated until iron deficiency is confirmed. Iron deficiency is associated with restless leg syndrome as iron is a cofactor in the production of dopamine.\n\nAnswer D: Pramipexole is a dopamine agonist that is an effective therapy for restless leg syndrome. However, it is more important to treat any underlying causes for restless leg syndrome such as iron deficiency prior to starting a dopamine agonist.\n\nAnswer E: Supportive therapy and an exercise routine ignore the potential underlying pathology and do not offer any specific treatment for this patient's restless leg syndrome. Exercise can improve symptoms of restless leg syndrome but iron deficiency must still be ruled out.\n\nBullet Summary:\nPatients with restless leg syndrome should be assessed for iron deficiency and treated if they are iron deficient.", "link": "https://bit.ly/3Mv8f5p"} {"question": "A 35-year-old man presents to the emergency department with altered mental status. He was found by his roommate to be disoriented this morning. He had complained of a right-sided headache for the past 5 days. He has a history of human immunodeficiency virus (HIV) infection being treated with bictegravir, emtricitabine, and tenofovir. His last CD4 cell count was 500/mm^3. He does not smoke tobacco and drinks alcohol socially. His temperature is 104.2\u00b0F (40.1\u00b0C), blood pressure is 110/70 mmHg, pulse is 110/min, and respirations are 22/min. Examination reveals 3/5 strength to right shoulder abduction and elbow flexion/extension and 4/5 strength to right hip flexion/extension. The remainder of the strength exam on the right and left sides is normal. Neck flexion does not elicit pain, and there are no tongue lacerations. During the exam, the patient\u2019s eyes are seen to roll backward with repeated flexion/extension of his extremities and loss of urine. This stops after 1 minute. Which of the following is the most likely diagnosis?", "choicesA": "Bacterial meningitis", "choicesB": "Brain abscess", "choicesC": "Cytomegalovirus encephalitis", "choicesD": "Epilepsy", "choicesE": "Fungal meningitis", "answer_idx": "B", "answer": "Brain abscess", "explanation": "This HIV-positive patient presents with fever, unilateral headache, altered mental status, focal neurological deficits (3/5 strength to right shoulder abduction, elbow flexion/extension, hip flexion/extension), and a generalized tonic-clonic seizure, suggestive of a brain abscess.\n\nA brain abscess typically presents in patients with underlying risk factors for infection, such as positive HIV status or bacterial endocarditis with hematogenous bacterial seeding. Patients present with fever, unilateral headache, altered mental status, focal neurological deficits, and fevers. Work-up consists of blood cultures, CT scan, and confirmatory magnetic resonance imaging with gadolinium contrast, showing a focal, hyperintense lesion with restricted diffusion. Treatment is with empiric broad-spectrum antibiotic therapy (e.g., vancomycin, metronidazole, and cefepime for gram-positive, gram-negative, and Pseudomonas coverage in immunocompromised patients) and surgical drainage.\n\nBrouwer et al. review the causes, diagnosis, and treatment of brain abscesses. They discuss the importance of rapid initiation of antimicrobial therapy in these patients.\n\nIncorrect Answers:\nAnswers 1 & 5: Bacterial or fungal meningitis can present with fever, headache, focal neurological deficits, and seizures but patients typically have meningeal signs such as pain with neck flexion. Cryptococcal meningitis is commonly seen in HIV-positive patients with CD4 counts < 100/mm^3.\n\nAnswer C: Cytomegalovirus encephalitis can present with fever, altered mental status, headache, and focal neurological deficits in patients with HIV infection. It typically presents in patients with severe immunodeficiency, often with CD4 cell count < 50/mm^3. This patient\u2019s CD4 cell count is 500/mm^3, making this less likely than an abscess.\n\nAnswer D: Epilepsy is a primary neurological disorder that can cause generalized tonic-clonic seizures but would not explain this patient\u2019s fever or unilateral headache. While Todd paralysis (focal weakness after a seizure) can occur after seizures, this patient\u2019s focal weakness predates his seizure. A primary seizure disorder is not diagnosed when seizures occur secondary to another process.\n\nBullet Summary:\nBrain abscess presents with fever, altered mental status, unilateral headache, seizures, and focal neurological deficits in patients with risk factors for infection (for example HIV, bacterial endocarditis).", "link": "https://bit.ly/3PAgPQX"} {"question": "A 24-year-old man with epilepsy refractory to valproic acid, phenytoin, and levetiracetam undergoes magnetic resonance imaging of his brain while monitored by anesthetic care. He wakes up screaming in pain due to an electrocardiogram lead having caused a significant thermal burn circumferentially around his left leg. He is admitted to the medical intensive care unit for continuous electroencephalogram monitoring while on a midazolam infusion for seizure suppression and supportive care for his burn. Overnight, the nurse continued to increase the patient's midazolam infusion rate, but she also noticed that his left toes were cold to the touch with significant edema. His temperature is now 100\u00b0F (37.8\u00b0C), blood pressure is 110/75 mmHg, pulse is 80/min, respirations are 10/min and oxygen saturation is 95% on 2 liters nasal cannula. No dorsalis pedis or posterior tibial pulses are detected on the left lower extremity. A delta pressure of 25 mmHg is obtained in the left leg. What is the most appropriate next step in management?", "choicesA": "Amputation", "choicesB": "Escharotomy", "choicesC": "Fasciotomy", "choicesD": "Intravenous fluid infusion based on Parkland formula", "choicesE": "Transfer to burn center", "answer_idx": "B", "answer": "Escharotomy", "explanation": "This patient with a circumferential burn due to an ECG cable wrapped around his leg in an MRI machine now has clear evidence of compartment syndrome due to the eschar. The most appropriate next step in management is emergent escharotomy to restore blood flow.\n\nBurns can be caused by chemical, thermal, electrical, and radiation sources. The initial inflammation causes vasodilatation and hyperemia, which can progress to edema and local microvascular thrombosis. If not promptly addressed, tissue necrosis and irreversible damage can occur. Circumferential burns can evolve into eschars that constrict blood flow and cause distal compartment syndrome. A delta pressure (diastolic pressure minus local compartment pressure) lower than 30 mmHg is diagnostic. In the case of eschar-induced compartment syndrome, the most appropriate next step is escharotomy to alleviate pressure.\n\nOrmiston and Marappa-Ganeshan review the evidence regarding the diagnosis and treatment of patients with compartment syndrome. They discuss how this condition can be caused by many different pathologies including burns. They recommend the emergent release of constrictive structures.\n\nIncorrect Answers:\nAnswer A: Amputation may be necessary if the distal limb is completely necrotic and not salvageable. Timely release of the eschar can allow for reperfusion of the limb and prevent this outcome.\n\nAnswer C: Fasciotomy is the treatment of choice for compartment syndrome not due to a circumferential eschar, but in this case the least invasive surgical intervention should be performed first without releasing all the compartments of the lower leg. If escharotomy does not relieve the compartment syndrome then fasciotomy may be required.\n\nAnswer D: Intravenous fluid infusion based on the Parkland formula is part of comprehensive burn management, but would not address the underlying compartment syndrome, which is more urgent at this time.\n\nAnswer E: Transfer to a specialized burn center can be considered since circumferential burns are one of the admission criteria, but early escharotomy should be performed first.\n\nBullet Summary:\nCircumferential burns that cause eschars and compartment syndrome require an escharotomy.", "link": "https://bit.ly/3RSgtYc"} {"question": "A 27-year-old woman presents for her first prenatal visit at an estimated gestational age of 16 weeks and 4 days by last menstrual period. She is presenting late to prenatal care due to significant anxiety about her pregnancy, as she and her husband had struggled with infertility for several years prior to conceiving. She has had nausea and vomiting for about 2 months, tender and swollen breasts, and cravings for foods she typically does not eat. She took a pregnancy test about 10 weeks ago and was too nervous to read the result immediately. After 24 hours, she saw that it was positive. The patient has a past medical history of bulimia nervosa but has not been symptomatic in 2 years. She also had an appendectomy at age 15 for appendicitis. She is a Ph.D. student and her husband is a physician. Her temperature is 98.5\u00b0F (36.9\u00b0C), pulse is 75/min, blood pressure is 122/76 mmHg, and respirations are 13/min. The patient\u2019s affect is normal and she is pleasant and excited about her pregnancy. A physical exam is notable for abdominal distension but is otherwise unremarkable. Ultrasound is shown in Figure A, and urine human chorionic gonadotropin is negative in the office. Which of the following is the most likely diagnosis?", "choicesA": "Couvade syndrome", "choicesB": "Delusion of pregnancy", "choicesC": "Feigned pregnancy", "choicesD": "Pseudocyesis", "choicesE": "Recent miscarriage", "answer_idx": "D", "answer": "Pseudocyesis", "explanation": "This patient presents with signs and symptoms of pregnancy but a nongravid uterus on ultrasound and a negative pregnancy test. The most likely diagnosis is pseudocyesis.\n\nPseudocyesis, or false pregnancy, most commonly presents with abdominal distension, breast tenderness, nausea, and other typical symptoms of pregnancy. Patients truly believe they are pregnant and may report positive pregnancy tests at home (perhaps due to false positives from waiting too long before reading the result) but office testing is negative. Ultrasound will be normal and show the absence of a developing fetus. The mainstay of treatment is explaining the diagnosis in a therapeutic manner and offering counseling to help patients cope.\n\nSmall reviews the evidence regarding the diagnosis and treatment of pseudocyesis. He discusses how this disease is a heterogenous entity involving neuroendocrine changes. He recommends clearly defining the cause of this disease in patients.\n\nFigure/Illustration A shows a transabdominal ultrasound with an empty cavity without signs of pregnancy (red circle). These findings are consistent with pseudocyesis.\n\nIncorrect Answers:\nAnswer A: Couvade syndrome refers to a \u201csympathetic pregnancy,\u201d in which a patient experiences pregnancy symptoms when someone close to her is pregnant. This does not apply to this patient because she believes that she is pregnant herself.\n\nAnswer B: Delusion of pregnancy is a false and fixed belief about being pregnant, despite factual evidence to suggest otherwise. Patients typically do not experience actual pregnancy symptoms and may have a history of psychotic disorders, both of which are inconsistent with this patient\u2019s presentation. Treatment is with cognitive behavioral therapy.\n\nAnswer C: Feigned pregnancy is the intentional falsifying of symptoms for a clear external motive and is a subtype of factitious disorder or malingering. This patient does not express any desire for secondary gain and seems to truly believe that she is pregnant, which is more consistent with pseudocyesis.\n\nAnswer E: A recent miscarriage is unlikely, as urine hCG testing often takes up to a month to become negative after pregnancy loss. Furthermore, the patient reports that her symptoms have been consistently present, whereas a miscarriage would typically cause a cessation of pregnancy symptoms.\n\nBullet Summary:\nPseudocyesis is a somatic symptom disorder in which women believe they are pregnant and experience pregnancy symptoms without an actual pregnancy.", "link": "https://bit.ly/3DDvy7X"} {"question": "A 42-year-old man presents to the urgent care clinic with low back pain. He was working on a home improvement project the day prior to presentation when the pain started. He describes the pain as \"achy and sore.\" It is not positional and does not radiate. He denies fevers, chills, paresthesias, and bowel or bladder incontinence. He has a history of a distal radius fracture 2 years ago from falling off a ladder. He drinks 3 alcoholic beverages weekly and denies illicit drug use. The patient\u2019s temperature is 98.4\u00b0F (36.9\u00b0C), blood pressure is 124/80 mmHg, pulse is 90/min, and respirations are 16/min. His body mass index (BMI) is 22.4 kg/m^2. There is tenderness to palpation of his paravertebral lumbar region bilaterally. Perineal and dermatomal sensation is symmetric and intact. Strength is 5/5 to knee flexion/extension and ankle dorsiflexion/plantarflexion. Patellar and Achilles reflexes are 2+ bilaterally. Raising either leg while the patient is in the supine position does not elicit any pain. Which of the following is the most likely diagnosis?", "choicesA": "Disc herniation", "choicesB": "Lumbar stenosis", "choicesC": "Lumbar strain", "choicesD": "Osteoarthritis", "choicesE": "Vertebral compression fracture", "answer_idx": "C", "answer": "Lumbar strain", "explanation": "This patient presents with low back pain after activity with paravertebral tenderness and without red flag signs/symptoms, such as history of trauma or malignancy, intravenous drug use, saddle anesthesia (reduced perineal sensation), or neurological deficits. The most likely diagnosis is a lumbar strain.\n\nThe differential diagnosis of low back pain is broad and includes mechanical causes such as lumbar strain, osteoarthritis, spondylolisthesis, disc herniation, spinal stenosis, and fractures as well as non-mechanical etiologies such as osteomyelitis and malignancy. History and physical should focus on eliciting red flag symptoms, including traumatic etiology, constitutional symptoms (e.g., fever, weight loss), history of malignancy, intravenous drug use, steroid use, and neurological deficits (e.g., saddle anesthesia, incontinence). The presence of any red flags should prompt further evaluation with imaging. Lumbar strain typically presents with acute low back pain after a precipitating event (e.g., lifting weight) with no red flag signs/symptoms and responds well to conservative management including activity modification, ice, and non-steroidal anti-inflammatory drugs. Prevention measures include strengthening of core muscles and education on proper lifting techniques. Physical therapy is a core component of treating patients with muscle strains.\n\nKnezevic et al. review lower back pain. They note the many potential causes and symptoms that may present. They recommend a multimodal interdisciplinary approach to back pain given the many mechanisms, especially in complex cases.\n\nIncorrect Answers:\nAnswer A: Disc herniation presents with radicular symptoms due to compression of spinal nerve roots as they exit the spinal canal, manifesting as a burning or shooting pain that radiates down either leg. Dermatomal sensory changes or hyporeflexia may be present depending on the spinal level affected (e.g., reduced Achilles reflex with S1 radiculopathy). The straight leg raise reproduces pain radiating down the leg on the affected side.\n\nAnswer B: Lumbar stenosis characteristically causes low back pain or lower extremity paresthesias with walking or standing that resolves when leaning forward (\u201cshopping cart sign\u201d). This is referred to as neurogenic claudication. Involvement of spinal nerve roots can lead to sensory loss and weakness in the lower extremities.\n\nAnswer D: Osteoarthritis can cause low back pain and stiffness, but is an age-related degenerative disease that is less likely in this patient who presents acutely after activity. The patient also does not have the typical risk factors for osteoarthritis, which include advanced age, female sex, or obesity.\n\nAnswer E: Vertebral compression fracture occurs mainly in osteoporotic patients, whose reduced bone mineral density increases their risk for vertebral body collapse under stress. Vertebral compression fractures present with point tenderness at the midline over the site of fracture. Additionally, this patient has no risk factors for osteoporosis (e.g., advanced age, post-menopausal women, low BMI).\n\nBullet Summary:\nLumbar strain presents as low back pain without red flag signs or symptoms (e.g., constitutional symptoms, neurologic deficits) and is treated conservatively with activity modification, ice, and core strengthening.", "link": "https://step2.medbullets.com/testview?qid=216255"} {"question": "A 57-year-old man presents to the emergency department with several days of malaise, weakness, and night sweats. Today, he experienced a headache with blurry vision, thus prompting his presentation. The patient has a history of diabetes and is followed closely by an endocrinologist. Otherwise, he has lost 10 pounds over the past month. His temperature is 100\u00b0F (37.8\u00b0C), blood pressure is 122/90 mmHg, pulse is 84/min, respirations are 16/min, and oxygen saturation is 99% on room air. Physical exam is notable for a thin man but is otherwise unremarkable. His visual acuity is 20/100 in both eyes, though he says that he normally has 20/20 vision. His gait is mildly ataxic as well, but his neurological exam is otherwise nonfocal. Laboratory studies are ordered as seen below.\n\nHemoglobin: 10 g/dL\nHematocrit: 31%\nLeukocyte count: 57,500/mm^3 with 35% blasts\nPlatelet count: 109,000/mm^3\n\nSerum:\nNa+: 139 mEq/L\nCl-: 100 mEq/L\nK+: 4.1 Eq/L\nHCO3-: 22 mEq/L\nBUN: 20 mg/dL\nGlucose: 99 mg/dL\nCreatinine: 1.1 mg/dL\nCalcium: 10.0 mEq/L\n\nWhich of the following is the most likely diagnosis?", "choicesA": "Acute lymphoblastic leukemia", "choicesB": "Acute myelogenous leukemia", "choicesC": "Chronic lymphocytic leukemia", "choicesD": "Chronic myelogenous leukemia", "choicesE": "Multiple myeloma", "answer_idx": "D", "answer": "Chronic myelogenous leukemia", "explanation": "This patient is presenting with symptoms of malignancy (malaise, weakness, and night sweats) and has a white blood cell count > 50,000/mm^3 in the setting of > 20% blasts, which are concerning for chronic myelogenous leukemia (CML). Given the blurry vision and ataxia in the setting of increased blasts, this patient may be suffering from a hyperviscosity syndrome secondary to a blast crisis, which can be seen in CML.\n\nCML is a neoplastic proliferation of myeloid stem cells. Patients are often asymptomatic and may present with a white blood cell count > 50,000/mm^3. There are many different presentations. The chronic phase presents with < 10% blasts and is often asymptomatic. The accelerated phase has increased blasts, and patients present with fatigue, weight loss, malaise, night sweats, abdominal pain, and hepatosplenomegaly. Finally, the blast phase presents similar to the accelerated phase but with more severe symptoms. The blast phase may evolve into a blast crisis (> 20% blasts) which could cause a hyperviscosity syndrome. Hypervicosity syndrome may require IV fluids and leukapheresis. The most common translocation in CML is the 9;22 translocation which forms the BCR:ABL gene product.\n\nJain et al. review CML. They note the outcomes in patients in the blast phase in CML and how there are more poor outcomes associated with this phase. They recommend paying close attention and obtaining a CBC with a differential to determine the percent blasts.\n\nIncorrect Answers:\nAnswer A: Acute lymphoblastic leukemia is more common in pediatric patients and presents with pancytopenia, fatigue, recurrent infections (from leukopenia), and bleeding (from thrombocytopenia). Other findings include systemic symptoms, malaise, fever, bone pain, lymphadenopathy, splenomegaly, and hepatomegaly.\n\nAnswer B: Acute myelogenous leukemia is a malignancy of myeloblasts and presents in the elderly. Patients will present with fatigue, malaise, weight loss, anorexia, fever, leukocytosis, anemia (causing fatigue), thrombocytopenia (causing bleeding), and neutropenia (leading to life-threatening infections). Auer rods can be seen on peripheral smear.\n\nAnswer C: Chronic lymphocytic leukemia is a monoclonal proliferation of incompetent mature B cells that occurs in elderly men and is the most common form of leukemia in the west. Patients are often asymptomatic and may have a white blood cell count > 50,000/mm^3. When symptomatic, patients may experience chills, weight loss, and weakness (note these symptoms are nonspecific). Peripheral smear will show smudge cells.\n\nAnswer E: Multiple myeloma is common in the elderly and presents with CRAB: hyperCalcemia, Renal insufficiency, Anemia, and lytic Bone lesions/Back pain. The diagnosis is supported with protein electrophoresis and confirmed when biopsy demonstrates increased plasma cells.\n\nBullet Summary:\nChronic myelogenous leukemia presents with increased white blood cells which may be >50,000/mm^3 (possibly with elevated blasts).", "link": "https://step2.medbullets.com/testview?qid=216612"} {"question": "A 35-year-old man presents to the emergency department for evaluation of chest pain. The pain starts in his chest and moves into his back, arms, and abdomen. He describes the pain as tearing. He has no known medical history and takes no medications. His temperature is 37.0\u00b0C (98.6\u00b0F), pulse is 130/min, blood pressure is 210/145 mmHg, respirations are 22/min, and pulse oximetry is 98% on room air. Exam reveals a diaphoretic, anxious-appearing man. Pulses are diminished over the left wrist when compared to the right. A chest radiograph is obtained as shown in Figure A. Which of the following is the most appropriate next step in management?", "choicesA": "Aspirin", "choicesB": "CT angiography of the chest, abdomen, and pelvis", "choicesC": "Echocardiography", "choicesD": "Lorazepam", "choicesE": "Serum troponin testing", "answer_idx": "B", "answer": "CT angiography of the chest, abdomen, and pelvis", "explanation": "This patient with \"tearing\" chest pain, hypertension, and a chest radiograph demonstrating mediastinal widening likely has an aortic dissection. The most appropriate next step in management would be to obtain CT angiography of the chest, abdomen, and pelvis to confirm the diagnosis.\n\nAortic dissections develop due to a tear in the intimal layer of the aorta, causing a second blood-filled lumen to form. Risk factors include hypertension, trauma, Marfan syndrome, Turner syndrome, and pregnancy. Prognosis depends on the location and severity of the intimal tear. Aortic dissections are often described by using the Stanford classification, where Type A involves the ascending aorta and Type B involves tears distal to the left subclavian artery. Patients classically present with chest pain described as \"tearing\" or \"ripping\" with radiation into the back, arms, or abdomen. Hypertension is common. Chest radiography characteristically demonstrates widening of the mediastinum due to accumulation of blood between layers of the wall of the aorta. Type A dissections are managed with heart rate control, blood pressure control, and surgery. Type B dissections are managed with blood pressure and heart rate control only.\n\nCooper et al. report on aortic dissections in adolescence. Although rare, aortic dissections have been seen in children with congenital heart disease, connective tissue disorders, or severe traumatic accidents. Severe, migrating abdominal pain should heighten suspicion for this diagnosis. The patient will often look worse clinically than can be explained by physical findings.\n\nFigure A depicts a chest radiograph demonstrating a widened superior mediastinum.\n\nIncorrect Answers:\nAnswer A: Aspirin is administered for chest pain that is thought to be due to acute coronary syndrome. This patient's presentation, exam, and imaging findings suggest aortic dissection. Aspirin is contraindicated in aortic dissection due to the increased risk for bleeding.\n\nAnswer C: Echocardiography offers information on cardiac contractility, valve function, and the presence of effusions. It would not be an appropriate initial diagnostic test in aortic dissection. Transesophageal echocardiography may be used to make the diagnosis in patients who cannot undergo a CTA.\n\nAnswer D: Lorazepam would be appropriate for patients presenting with chest pain that is secondary to use of cocaine or other sympathomimetic drugs. Tachycardia, tachypnea, diaphoresis, hyperactive bowel sounds, and dilated pupils suggest a sympathomimetic toxidrome.\n\nAnswer E: Serum troponin testing is indicated for risk stratification of patients with suspected acute coronary syndrome.\n\nBullet Summary:\nThe most appropriate next step in management for patients with suspected aortic dissection and mediastinal widening on chest radiograph is CT angiography of the chest, abdomen, and pelvis.", "link": "https://bit.ly/48elxeS"} {"question": "A 36-year-old man presents to the office for follow up after initiating antidepressant therapy. Two months ago, he was started on citalopram after several weeks of depressed mood, loss of interest in activities, depressed appetite, and inability to sleep. Today, he reports minimal improvement in his symptoms. He had some nausea after starting the medication, but this has resolved. He denies thoughts of suicide or self-harm. His temperature is 98.6\u00b0F (37.0\u00b0C), pulse is 80/min, blood pressure is 120/80 mmHg, respirations are 22/min, and oxygen saturation is 97% on room air. Exam reveals a depressed, tired appearing man. Which of the following is the most appropriate next step in management?", "choicesA": "Add mirtazapine", "choicesB": "Electroconvulsive therapy", "choicesC": "Switch to amitriptyline", "choicesD": "Switch to paroxetine", "choicesE": "Switch to phenelzine", "answer_idx": "A", "answer": "Add mirtazapine", "explanation": "This patient is being treated for depression without improvement in his symptoms after 6 weeks of medication use. In the setting of his reduced appetite and trouble sleeping, mirtazapine is an appropriate medication to add.\n\nSSRIs (including paroxetine, citalopram, fluvoxamine, sertraline, and fluoxetine) are the first line treatment for major depression. For patients in whom an adequate trial of SSRI therapy has been ineffective, the first step in management is a trial of augmentation with a separate agent prior to switching to a different primary antidepressant medication. Mirtazapine, a noradrenergic and specific serotonergic antidepressant, acts by inhibiting pre-synaptic alpha-2 receptors as well as 5HT2 and 5HT3 serotonin receptors. Notably, its side effect profile includes increased appetite and mild sleepiness, making it a good option for depressed patients with insomnia or decreased appetite. Mirtazapine is commonly used to augment primary antidepressant therapy. Augmentation with an additional agent should generally be attempted prior to switching to a different SSRI or other first line medication.\n\nDavies et. al review the pharmacologic management of depression. They note that while evidence is limited, there is some data to suggest benefit in augmentation with a secondary agent prior to switching to another first line therapy.\n\nIncorrect Answers:\nAnswer B: Electroconvulsive therapy, while often extremely effective, is typically reserved for patients that are actively suicidal, unable to eat, or displaying psychotic symptoms. It is more commonly used after medications have failed.\n\nAnswer C: Switching to amitriptyline, a tricyclic antidepressant, would not be appropriate. Tricyclic antidepressants have a negative side effect profile and potential for severe toxicity.\n\nAnswer D: Switching to paroxetine, another SSRI medication, may be appropriate. However, this patient's particular constellation of symptoms (decreased appetite and insomnia) suggest that he would likely respond well to mirtazapine.\n\nAnswer E: Switching to phenelzine, a monoamine oxidase inhibitor, would not be appropriate. Monoamine oxidase inhibitors have a negative side effect profile and potential for severe drug-drug interaction.\n\nBullet Summary:\nMirtazapine (a noradrenergic and serotonergic antidepressant) is an appropriate next step for depressed patients with significant insomnia and decreased appetite who fail to respond to a trial of SSRI monotherapy.", "link": "https://bit.ly/3Lm4zC8"} {"question": "A 65-year-old man presents to his primary care physician for stiffness in his arm. He has been having trouble combing his hair and reaching objects that are high on the shelf. The patient has a medical history of type 2 diabetes mellitus, obesity, and hypertension. His current medications include metformin, insulin, lisinopril, and hydrochlorothiazide. The patient leads a sedentary life in which he tends to stay home and watch television. He does not engage in any physical or strenuous activity. His temperature is 98.6\u00b0F (37\u00b0C), blood pressure is 138/85 mmHg, pulse is 75/min, and respirations are 12/min. On physical exam, the patient has decreased passive and active range of motion of his shoulder. The strength of the patient's upper extremity is 4/5 limited by pain. Which of the following is the most likely diagnosis?", "choicesA": "Adhesive capsulitis", "choicesB": "Biceps tendinopathy", "choicesC": "Glenohumeral osteoarthritis", "choicesD": "Rotator cuff impingement", "choicesE": "Subacromial bursitis", "answer_idx": "A", "answer": "Adhesive capsulitis", "explanation": "This patient is presenting with stiffness and a decreased range of motion in his shoulders suggestive of a diagnosis of adhesive capsulitis.\n\nAdhesive capsulitis classically presents in elderly, sedentary patients with a chief complaint of stiffness. On physical exam, the key findings are a loss of active, but more importantly, passive range of motion. It is the loss of passive range of motion that suggests a diagnosis of adhesive capsulitis over rotator cuff injuries, biceps tendinopathy, glenohumeral osteoarthritis, and subacromial bursitis. The diagnosis of adhesive capsulitis is made clinically; however, radiography, MRI, and ultrasound can be used to rule out other diagnoses. Treatment includes NSAIDs and physical therapy and can include steroids.\n\nRamirez reviews the evidence regarding the diagnosis and treatment of adhesive capsulitis. He discusses how non-operative management consists of nonsteroidal anti-inflammatory drugs, short-term oral corticosteroids, intra-articular corticosteroid injections, and physiotherapy. He recommends manipulation under anesthesia and arthroscopic capsule release in patients who fail conservative management.\n\nIncorrect Answers:\nAnswer B: Biceps tendinopathy presents with pain and weakness overlying the long head of the biceps tendon. This tendon runs in the intertubercular groove on the anterior aspect of the proximal humerus. Patients will often present with pain with resisted flexion of the elbow. Treatment is surgical debridement in patients who fail conservative measures.\n\nAnswer C: Glenohumeral osteoarthritis presents with vague and diffuse pain that is exacerbated by activity in addition to a loss of range of motion of the shoulder. Though this patient is experiencing a decreased range of motion, the decreased passive range of motion is more specific for adhesive capsulitis. Treatment is with a total shoulder replacement in patients who fail conservative measures.\n\nAnswer D: Rotator cuff impingement presents with pain and weakness of the upper extremity, but also can present with a decreased range of motion as this patient experienced. Of note, the passive range of motion will not be significantly affected by this disease. Treatment is with surgical reconstruction or debridement of the rotator cuff if initial measures fail.\n\nAnswer E: Subacromial bursitis presents with localized tenderness, decreased range of motion, pain with motion, and sometimes erythema/edema. Though this patient is experiencing a decreased range of motion and pain with motion, the decreased passive range of motion suggests a diagnosis of adhesive capsulitis. Treatment is with rest and oral anti-inflammatory medications.\n\nBullet Summary:\nAdhesive capsulitis presents with pain and a decreased passive and active range of motion.", "link": "https://step2.medbullets.com/testview?qid=108971"} {"question": "A 27-year-old man presents to the emergency department after a bar fight. He was punched in the eye and is having pain and blurry vision. The patient is otherwise healthy and does not take any medications. His temperature is 98.1\u00b0F (36.7\u00b0C), blood pressure is 132/84 mmHg, pulse is 103/min, respirations are 17/min, and oxygen saturation is 98% on room air. Physical exam is notable for conjunctival injection of the patient\u2019s left eye as shown in Figure A. Visual acuity reveals 20/20 vision in his right eye and 20/40 vision in his left. His left pupil is sluggish to constrict when compared to the right. A fluorescein stain is unremarkable and tonometry reveals a pressure of 45 mmHg in the left eye. Which of the following is the most appropriate next step in management?", "choicesA": "CT scan of the head and facial bones", "choicesB": "Cyclopentolate", "choicesC": "Lateral canthotomy", "choicesD": "Orbital ultrasound", "choicesE": "Topical timolol and pilocarpine", "answer_idx": "C", "answer": "Lateral canthotomy", "explanation": "This patient is presenting after blunt trauma to the eye with eye pain, blurry vision, conjunctival injection, a sluggish pupil, and elevated intraocular pressure (> 20 mmHg) suggesting a diagnosis of a retrobulbar hematoma/hemorrhage causing orbital compartment syndrome. The most important initial step in management is a lateral canthotomy to alleviate retrobulbar pressure.\n\nA retrobulbar hematoma (or hemorrhage) occurs after blunt trauma to the eye. Presenting symptoms may include eye pain, photophobia, proptosis (an eye that appears to be anteriorly displaced), decreased visual acuity, and conjunctival injection. It is important to perform a thorough exam including a slit lamp exam and a fluorescein stain to rule out other traumatic injuries such as a globe rupture. Tonometry can be used to assess for increased intraocular pressure. A pressure > 20 mmHg suggests a diagnosis of orbital compartment syndrome, a vision-threatening condition. Immediate management with a lateral canthotomy is indicated in order to relieve pressure on the optic nerve and retina and save the patient\u2019s vision. Further management may require drainage of the hematoma in the operating room.\n\nBurkat et al. review the technique for performing a lateral canthotomy to relieve pressure in retrobulbar hematoma. The authors first excise the skin and orbicularis muscle at the lateral canthal angle in a horizontal direction for approximately 10 mm. The authors then recommend performing an inferior cantholysis by incising the inferior crus (a thick fibrous band connecting the lateral lower eyelid to the orbital rim) near the orbital rim.\n\nFigure A demonstrates proptosis on physical exam which raises concern for a retrobulbar hematoma that is likely increasing orbital compartment pressure.\n\nIncorrect Answers:\nAnswer A: CT scan of the head and facial bones would be appropriate if there was a concern for facial fracture. It would delay necessary and vision-saving interventions to perform a CT scan in the setting of orbital compartment syndrome. However, cross-sectional imaging may be performed after the pressure is relieved.\n\nAnswer B: Cyclopentolate is a cycloplegic that can be used to reduce pain in traumatic iritis. This typically presents after blunt trauma to the eye with erythema, photophobia, blurry vision, and cell/flare in the anterior chamber on slit lamp exam. A sluggish pupil may also be seen; however, increased intraocular pressure would not be seen in this condition.\n\nAnswer D: Orbital ultrasound could be used for diagnoses such as a retinal detachment which presents with flashes of light, floaters, and visual field loss like a \u201ccurtain coming down over the eye.\u201d An ultrasound would demonstrate the detached retina in the back of the eye. Management involves elevating the head of the bed and surgical reattachment.\n\nAnswer E: Topical timolol and pilocarpine in addition to brimonidine, acetazolamide, and possible iridotomy would be the appropriate management of acute closed-angle glaucoma which presents with a rock-hard eye, blurry vision, a mid-dilated and fixed pupil, as well as a headache. This diagnosis can be supported by elevated intraocular pressure on tonometry. This is unlikely for this patient in the context of trauma.\n\nBullet Summary:\nA retroorbital hematoma can cause orbital compartment syndrome which requires immediate treatment with a lateral canthotomy.", "link": "https://step2.medbullets.com/testview?qid=215169"} {"question": "A 23-year-old woman presents to the emergency department with severe abdominal pain. The pain has been dull and progressive, but became suddenly worse while she was exercising. Her medical history is notable for depression, anxiety, and gonococcal urethritis that was appropriately treated. She is sexually active and does not use condoms. She admits to drinking at least 5 standard alcoholic drinks a day. The patient also recently lost a large amount of weight for a fitness show she planned on entering. The patient's current medications include oral contraceptive pills, fluoxetine, alprazolam, ibuprofen, acetaminophen, and folate. Her temperature is 99.5\u00b0F (37.5\u00b0C), blood pressure is 80/40 mmHg, pulse is 110/minute, and respirations are 15/minute with an oxygen saturation of 96% on room air. On physical exam, you note an athletic young woman with burly shoulders, a thick neck, and acne on her forehead and back. On abdominal exam you note diffuse tenderness with 10/10 pain upon palpation of the right upper quadrant. Intravenous fluids are started and labs are sent. A urinary \u00df-hCG has been ordered. Which of the following is the most likely the diagnosis?", "choicesA": "Ectopic implantation of a blastocyst", "choicesB": "Inflammation of the pancreas", "choicesC": "Obstruction of blood flow through the hepatic vein", "choicesD": "Obstruction of the common bile duct by radio-opaque stones", "choicesE": "Vascular ectasia within the liver", "answer_idx": "E", "answer": "Vascular ectasia within the liver", "explanation": "This patient is presenting with constant right upper quadrant pain in the setting of oral contraceptives (OCP) and likely anabolic steroid use, suggesting a diagnosis of a ruptured hepatic adenoma (HA). The pathophysiology of HA is vascular ectasia from exposure to OCPs and androgens.\n\nA hepatic adenoma typically occurs secondary to exposure to OCPs and androgens (anabolic steroids) causing a generalized vascular ectasia. Both of these drugs should be discontinued in patients with hepatic adenoma. Symptoms include right upper quadrant abdominal pain. Sudden worsening of pain with unstable vitals suggests a ruptured hepatic adenoma. Patients should be managed with fluids and an ultrasound should be performed to support the diagnosis. Additional imaging tests include an abdominal contrast-enhanced CT to better characterize the pathologic anatomy in this disease. The definitive treatment in this disease is surgery in order to remove the hepatic adenoma. In cases of a ruptured hepatic adenomas, patients require transfusion and operative management.\n\nBioulac-Sage et al. present the current data that are available for classifying hepatic adenomas. They discuss how many of these lesions involve alterations of the HNF1A gene, the JAK/STAT3 pathway, and the CTNNB1 gene. They recommend performing molecular analysis in order to better understand which lesions are likely to undergo malignant transformation.\n\nIncorrect Answers:\nAnswer A: Ectopic implantation of a blastocyst (ectopic pregnancy) that has ruptured could present similarly to what this patient is experiencing. Symptoms would include lower abdominal pain and unstable vitals. However, this patient's pain is more pronounced in the right upper quadrant and is occurring in the setting of OCP and possible anabolic steroid use.\n\nAnswer B: Inflammation of the pancreas represents a diagnosis of pancreatitis. This would present with epigastric pain after a fatty meal or alcohol consumption. Though this patient does drink more than recommended and should be counseled to reduce her consumption, the pain began after exercising at the gym and there are no other symptoms of pancreatitis.\n\nAnswer C: Obstruction of blood flow through the hepatic vein causes Budd-Chiari syndrome, which presents with a non-specific triad of abdominal pain, ascites, and hepatomegaly in the setting of a hypercoagulable state (factor V Leiden, polycythemia vera, OCP use, and paroxysmal nocturnal hemoglobinuria). Though this patient has risk factors for this pathology, they do not have any classic symptoms.\n\nAnswer D: Obstruction of the common bile duct with radio-opaque stones would present with colicky right upper quadrant pain in a patient that is \"fat, female, fertile, and forty.\" Though the pain presented in this case is localized to the right upper quadrant, it is not colicky and there is not a positive Murphy sign described. OCP's are a risk factor for cholesterol stones as are rapid weight loss, both of which this patient has. This is a strong possibility on the differential; however, the symptoms presented suggest a diagnosis of ruptured hepatic adenoma given the demographics of the patient.\n\nBullet Summary:\nA hepatic adenoma presents with right upper quadrant pain in the setting of OCP and/or androgen use and can rupture leading to unstable vitals.", "link": "https://step2.medbullets.com/testview?qid=108515"} {"question": "A 56-year-old man presents from prison with 1 month of subjective fevers, chills, cough, and night sweats. He noticed a gradual weight loss over the past year despite no changes to his diet or physical activity. He has never smoked cigarettes but does have a history of opiate use prior to incarceration. He does not take any medications. His temperature is 100.8\u00b0F (38.2\u00b0C), blood pressure is 142/88 mmHg, pulse is 78/min, and respirations are 12/min. Physical exam reveals a thin, ill-appearing man. Pulmonary auscultation reveals faint, inspiratory crackles in the right upper lobe. His chest radiograph is shown in Figure A. Which of the following is the most appropriate next step in management?", "choicesA": "Ampicillin-sulbactam", "choicesB": "Biopsy", "choicesC": "Itraconazole", "choicesD": "Rifampin, isoniazid, pyrazinamide, and ethambutol (RIPE) therapy", "choicesE": "Vancomycin", "answer_idx": "D", "answer": "Rifampin, isoniazid, pyrazinamide, and ethambutol (RIPE) therapy", "explanation": "This patient likely has active tuberculosis (TB), as evidenced by his symptoms (fever, night sweats, cough, and weight loss), inmate status, and apical cavitary lesion on chest radiograph. He should be treated with RIPE therapy.\n\nAn active TB infection should be suspected in patients with a febrile illness, cough, night sweats, and weight loss along with epidemiological risk factors for TB infection. Risk factors include residence in or previous visits to endemic regions, work in hospitals or nursing homes, imprisonment, and immunosuppression. Treatment typically requires 2 phases: an intensive phase involving 2 months of rifampin, isoniazid, pyrazinamide, and ethambutol (RIPE) therapy and a continuation phase involving 4 additional months of just isoniazid and rifampin. Vitamin B6 (pyridoxine) should be taken with isoniazid to prevent peripheral neuropathy. Rifampin, isoniazid, and pyrazinamide use should be monitored for hepatotoxicity. Ethambutol can cause optic neuropathy.\n\nConradie et al. discuss the use of a bedaquiline-pretomanid-linezolid (BPal) regimen in the treatment of extensively drug-resistant tuberculosis. The authors find that most patients treated with BPal have a favorable outcome (no treatment failure or disease relapse) after 6 months. The authors recommend the use of this regimen in highly drug resistant TB.\n\nFigure/Illustration A demonstrates a right upper lobe cavitating lesion (arrow) that resulted in extensive necrosis and fibrosis. These lesions are usually localized to the upper regions of the lung due to favorable oxygen concentrations in the apex.\n\nIncorrect Answers:\nAnswer A: Ampicillin-sulbactam can be used as empiric treatment for a lung abscess to cover anaerobic pathogens and facultative anaerobic streptococci. A lung abscess is associated with aspiration risk factors (such as alcohol use) or poor dentition.\n\nAnswer B: Biopsy may be indicated for a solitary cavitary nodule that is suggestive of malignancy. In this case, the patient's inmate status, along with his fever, night sweats, and cough are more suggestive of an infectious rather than malignant etiology.\n\nAnswer C: Itraconazole can be used to treat chronic pulmonary aspergillosis. These patients typically have prior or current lung disease such as chronic obstructive pulmonary disease or tuberculosis. The classic air crescent sign may be seen on imaging in a patient with an aspergilloma, a form of pulmonary aspergillosis in which a fungal ball develops within an existing cavitary lesion.\n\nAnswer D: Vancomycin can be used to treat a lung abscess due to methicillin-resistant Staphylococcus aureus. S. aureus pneumonia classically presents as a secondary infection that follows an influenza infection.\n\nBullet Summary:\nActive tuberculosis infection should be treated with rifampin, isoniazid, pyrazinamide, and ethambutol (RIPE) therapy.", "link": "https://step2.medbullets.com/testview?qid=215176"} {"question": "A resident physician places a patient\u2019s home medications in for their admission orders during an overnight admission. They order the patient\u2019s home medications including clobazam; however, they accidentally order clonazepam. Subsequently, the patient is over-sedated and has to be transferred to the ICU for airway monitoring. Which of the following is the most appropriate method for preventing future similar occurrences?", "choicesA": "Add an alert in the electronic medical system for medications that sound similar", "choicesB": "Enter medications only on morning rounds with multiple providers", "choicesC": "Have nursing perform the medication reconciliation", "choicesD": "Reprimand and educate the resident to check medications more carefully", "choicesE": "Verify the patient\u2019s home medications with the patient", "answer_idx": "A", "answer": "Add an alert in the electronic medical system for medications that sound similar", "explanation": "A medical error occurred where 2 similar-sounding medications were confused, leading to an adverse event. The most appropriate way to prevent this error from occurring is a system-based approach, such as adding an alert in the electronic medical system for medications that sound similar.\n\nMedical errors commonly occur in the medical system given the complexity of delivering patient care. The best way to prevent future errors from occurring is a systems-based approach where the system is built to catch errors rather than relying on the diligence of staff. In a systems-based approach, there is an algorithmic method set in place for situations where errors are common. Examples of systems-based approaches to avoiding medical errors include time-outs for procedures, use of the electronic medical record and ordering system to generate stops and checks, and built-in procedures such as a pharmacist verifying all medical orders. The best way to prevent future errors, when possible, is a systems-based approach as it does not rely solely on individuals given that individuals are fallible and may not always avoid mistakes.\n\nCondren et al. review a systems based approach in the medical system. They note that a systems-based approach reduces medication errors. For this reason, systems-based approaches are recommended whenever possible.\n\nIncorrect Answers:\nAnswer B: Entering medications only on morning rounds with multiple providers is not a plausible strategy as patients will not receive medications they may need at other times, such as overnight. It is always a good idea to verify medications with the team on morning rounds to catch any possible errors that may be missed, but this would not necessarily fix errors that occur at other times of the day.\n\nAnswer C: Having nursing perform the medication reconciliation is not the best solution in this case, as the core problem was that the wrong medication was entered due to similar-sounding medication names. However, it is always a good idea to have multiple providers (e.g., nurses, pharmacists) verify information, check this with the physician team, and then verify the dose and route of administration when giving the medication.\n\nAnswer D: Reprimanding and educating the resident to check medications more carefully is not a solution to this problem. It does not seem that the resident does not understand the pharmacology or medication prescribed but made an understandable error. Explaining the need for care is important; reprimanding the resident is not.\n\nAnswer E: Verifying the patient\u2019s home medications with the patient should always be performed, but it would not fix this issue with similar-sounding medications. It would catch errors such as new medications that are not on the patient\u2019s medication list or out-of-date medications remaining on the list.\n\nBullet Summary:\nA systems-based approach is the most appropriate way to prevent future errors from occurring.", "link": "https://step2.medbullets.com/testview?qid=216598"} {"question": "A 33-year-old man presents to the emergency department after slamming his finger in the car door 5 hours ago. He initially experienced pain which is currently well controlled with acetaminophen. The patient is otherwise healthy and does not take any medications. His temperature is 98.5\u00b0F (36.9\u00b0C), blood pressure is 123/79 mmHg, pulse is 90/min, respirations are 11/min, and oxygen saturation is 97% on room air. Physical exam is notable for the finding in Figure A. A radiograph of the affected digits shows a smooth cortex without disruption of the bone. Which of the following is the most appropriate management for this patient?", "choicesA": "Excision and histological examination", "choicesB": "Fingernail removal", "choicesC": "Observation", "choicesD": "Operative repair", "choicesE": "Trephination", "answer_idx": "E", "answer": "Trephination", "explanation": "This patient is presenting after trauma to his finger with a subungual hematoma. Given that he has no other associated injuries, trephination is the only management indicated.\n\nA subungual hematoma occurs after trauma to the finger leading to a collection of blood under the fingernail. Depending on the mechanism, this can be associated with a distal phalanx fracture. Evaluation for a fracture should include a thorough history, exam, and radiography. If there are no other signs of injury including nailbed dislocation or an open fracture, then management requires only trephination (putting a hole in the nail to allow blood to drain) as this alleviates pressure on the nailbed matrix thus preventing possible irreversible damage.\n\nRsoer et al. compare nail bed repair to nail trephination for subungual hematomas in children. The authors find that there was no difference in complications or outcomes between nail bed repair and trephination; however, the cost in the nail trephination group was lower. The authors recommend nail trephination for children with a subungual hematoma and an intact nail and nail margin.\n\nFigure/Illustration A shows the physical exam finding of a subungual hematoma with a collection of blood underneath the fingernail bed (green circle).\n\nIncorrect Answers:\nAnswer A: Excision and histological examination could be appropriate management of melanoma, which may present under the fingernails with a hyperpigmented, irregular skin lesion, or a \u201cbarcode\u201d appearance if it is at the base of the nail. A spontaneous subungual hematoma could be suggestive of melanoma; however, in the setting of trauma, it is a much less likely diagnosis.\n\nAnswer B: Fingernail removal should only be performed if there is also nail avulsion or nail fold disruption to allow for inspection of the nail bed and repair of any laceration. Failure to repair a nailbed laceration could lead to permanent deformity of the fingernail when it regrows.\n\nAnswer C: Observation is inappropriate as the accumulation of blood under the nail in a subungual hematoma could lead to ischemia and permanent injury to the nailbed. When a subungual hematoma is present, trephination is indicated.\n\nAnswer D: Operative repair may be necessary for fractures; however, this patient has no fracture and only has a subungual hematoma which can be drained easily.\n\nBullet Summary:\nA subungual hematoma is common after trauma, presents with a collection of blood under the fingernail, and should be treated with trephination.", "link": "https://bit.ly/3sorW7P"} {"question": "A 27-year-old woman presents to her primary care physician for an abnormal vaginal discharge. The patient has engaged in unprotected intercourse with 12 different partners. She has a medical history of asthma, IV drug abuse, and depression. She has 1 to 2 alcoholic drinks per day. She feels ashamed of her behavior and is requesting treatment for her condition and advice for safe sex. The patient is given antibiotics and is advised on safe sex practices. The patient presents to the emergency department 3 days later with a complaint of a rash. She states that every time she has sex a rash has emerged on her skin. Her temperature is 99.5\u00b0F (37.5\u00b0C), blood pressure is 127/68 mmHg, pulse is 91/min, respirations are 14/min, and oxygen saturation is 98% on room air. A physical exam is notable for the finding in Figure A. Which of the following is the most likely cause of this patient's symptoms?", "choicesA": "Antigen exposure", "choicesB": "Drug reaction", "choicesC": "Infection", "choicesD": "IV drug use", "choicesE": "Photosensitivity", "answer_idx": "A", "answer": "Antigen exposure", "explanation": "This patient is presenting with urticaria whenever she engages in sexual intercourse suggesting a latex allergy (found in condoms). These allergies are due to antigen exposure.\n\nAnaphylaxis and urticaria are immunologically mediated responses to environmental antigens that are typically IgE-mediated. This results in crosslinking of antigen receptors and the subsequent release of histamine and other inflammatory signals. Patients can present with a spectrum of symptoms from urticaria (hives alone) all the way to full-blown anaphylaxis (hypotension, tachycardia, and bronchoconstriction). Minor cases can be treated with avoidance of antigen exposure. Anaphylaxis is treated with intramuscular epinephrine (1:1,000).\n\nGawchik reviewed the evidence regarding the diagnosis of latex allergy. She discusses how the diagnosis is primarily made by observing the relationship between exposure and symptoms. She recommends avoidance of latex in these patients.\n\nFigure/Illustration A is a clinical photograph that demonstrates dermal edema and urticaria (red circles). This can occur secondary to allergen exposure such as from a new drug or latex.\n\nIncorrect Answers:\nAnswer B: Drug reaction could be possible if this patient has an antibiotic allergy (ceftriaxone is a treatment for gonorrhea); however, a rash that only occurs during intercourse is more likely related to latex exposure.\n\nAnswer C: Infection is possible given this patient\u2019s IV drug abuse and risky behavior; however, the hives present in this patient are likely not related to an infection. Infectious rashes tend to be painful, ulcerative, or vesicular.\n\nAnswer D: IV drug use could expose this patient to any number of foreign substances that could induce an allergic reaction; however, the symptoms that are associated with sexual intercourse point toward latex exposure.\n\nAnswer E: Photosensitivity would not present as hives but could occur in this patient given that doxycycline is an appropriate drug to treat her initial sexually transmitted infection and offers coverage for Chlamydia trachomatis. Patients would present with a photosensitive rash.\n\nBullet Summary:\nAnaphylaxis and urticaria can occur secondary to latex exposure (such as from condoms or gloves).", "link": "https://bit.ly/4918Jtt"} {"question": "A 46-year-old farmer presents to the office for a follow-up visit. He was seen 4 days ago for a new lesion on his arm that had been present for 1 week prior to that visit. A photograph of the lesion is shown in Figure A. At that time, he was prescribed treatment for the lesion and instructed to follow up as necessary. Today, he reports that he became sunburned this morning within an hour of beginning his workday, despite wearing appropriate sunscreen. He has a medical history significant for type 2 diabetes mellitus that is controlled with metformin. He has no known medical allergies. He smokes 1 pack of cigarettes per day and denies alcohol or illicit drug use. His temperature is 98.6\u00b0F (37.0\u00b0C), blood pressure is 125/70 mmHg, pulse is 65/min, and respirations are 12/min. On exam, there is a blistering erythematous rash on the arms and shoulders. The previous skin lesion appears unchanged. Which of the following is the most likely explanation for the patient's new rash?", "choicesA": "Adverse effect of ciprofloxicin", "choicesB": "Adverse effect of doxycycline", "choicesC": "Allergic reaction to prescribed therapy", "choicesD": "Progression of disease and alternative therapy is indicated", "choicesE": "Progression of disease and surgical debridement is necessary", "answer_idx": "B", "answer": "Adverse effect of doxycycline", "explanation": "This farmer has a cutaneous black eschar most consistent with cutaneous anthrax for which doxycycline is an appropriate treatment. A new blistering, erythematous rash after spending an hour outside that appears 4 days after starting treatment is consistent with a photosensitivity reaction, a known adverse effect of doxycycline.\n\nCutaneous anthrax is the most common form of infection caused by Bacillus anthracis. The infection is contracted during contact with infected livestock. The first-line treatment is doxycycline, ciprofloxacin, levofloxacin, or moxifloxacin. Progression to systemic anthrax would result in pulmonary (flu-like symptoms and hemoptysis) or gastrointestinal (nausea, vomiting, and dysentery) symptoms and would require treatment with clindamycin plus ciprofloxacin. Patients with meningitis due to anthrax require antibiotic therapy with ciprofloxacin plus meropenem plus linezolid. Additionally, antitoxins such as raxibacumab and adjunctive therapies such as glucocorticoids should be considered. In addition to photosensitivity, adverse effects of doxycycline include gastrointestinal upset (most common), teeth discoloration, and inhibition of bone growth in children. Doxycycline is teratogenic and should not be prescribed during pregnancy.\n\nMigone et al. discuss the use of raxibacumab, a monoclonal antibody against a component of the anthrax toxin, in treating inhalational anthrax. The authors found that in monkeys, raxibacumab significantly increased survival after exposure to anthrax. The authors recommend the use of antitoxins such as raxibacumab in any patient suspected of having systemic anthrax.\n\nFigure/Illustration A demonstrates the characteristic black eschar (blue circle) of cutaneous anthrax.\n\nIncorrect Answers:\nAnswer A: Adverse effects of ciprofloxacin do not include photosensitivity, though it is an appropriate antibiotic choice for cutaneous anthrax treatment. Adverse effects of ciprofloxacin include gastrointestinal upset, risk of tendon rupture, and QT prolongation.\n\nAnswer C: Allergic reactions to prescribed therapy would be characterized by urticaria and wheals. In severe cases, anaphylaxis may be present.\n\nAnswer D: Progression of disease and alternative therapy is indicated does not fit this case. The patient's symptoms (blistering, erythematous rash) are not concerning for the progression of cutaneous anthrax to systemic disease, which would result in pulmonary or gastrointestinal symptoms. Alternative therapy for systemic anthrax is not indicated in this case.\n\nAnswer E: Progression of disease and surgical debridement is not indicated. The patient's symptoms (blistering, erythematous rash) are not concerning for the progression of cutaneous anthrax to systemic disease, which would result in pulmonary or gastrointestinal symptoms. Additionally, surgical debridement is not indicated for cutaneous anthrax.\n\nBullet Summary:\nCutaneous anthrax presents with a black eschar and is treated with antitoxin and either doxycycline or ciprofloxacin, the former of which can cause a photosensitivity reaction.", "link": "https://step2.medbullets.com/testview?qid=215046"} {"question": "A 16-year-old boy presents to an ophthalmologist with blurry vision. Over the past several months, he has had increasing difficulty seeing the board from the back of the classroom at school. The patient is otherwise doing well in school and enjoys playing basketball. His medical history is otherwise significant for scoliosis which is managed by an orthopedic surgeon. His family history is significant for a mother with type 2 diabetes mellitus and a father who underwent aortic valve replacement last year. His temperature is 98.6\u00b0F (37\u00b0C), blood pressure is 115/71 mmHg, pulse is 72/min, and respirations are 12/min. On physical exam, the patient is tall for his age and has long arms. He has 20 degrees of thoracic scoliosis, which is stable from previous exams. On slit-lamp examination, the patient is found to have bilateral upward lens subluxation and is prescribed corrective lenses. Which of the following is the most likely cause of this patient\u2019s symptoms?", "choicesA": "Defective metabolism of methionine", "choicesB": "Extra copy of sex chromosome", "choicesC": "Mutation of chromosome 15", "choicesD": "Mutation of COL5A1 or COL5A2", "choicesE": "Mutation of RET proto-oncogene", "answer_idx": "C", "answer": "Mutation of chromosome 15", "explanation": "This patient presents with tall stature, upward lens subluxation, and a family history of aortic valve disease, which suggests a diagnosis of Marfan syndrome. Marfan syndrome is caused by an autosomal dominant mutation in the fibrillin-1 gene on chromosome 15.\n\nThe fibrillin-1 gene codes for a glycoprotein in the extracellular matrix of connective tissue. Patients with Marfan syndrome classically present with a triad of musculoskeletal, ocular, and cardiovascular symptoms. The musculoskeletal findings include tall stature, long limbs, arachnodactyly, pectus excavatum, joint hypermobility, skin hyperelasticity, and scoliosis. Patients with Marfan syndrome also often present with upward lens subluxation, and they are at increased risk of cardiac disease, including aortic regurgitation and aortic dissection. Treatment is symptomatic with avoidance of contact sports, beta-blockers, and valve repair to prevent cardiac complications.\n\nBitterman and Sponseller review the evidence regarding the diagnosis and treatment of Marfan syndrome. They discuss how skeletal manifestations are often the initial presentation of this condition leading to diagnosis. They recommend screening for patients who are suspected to have this disease.\n\nIncorrect Answers:\nAnswer A: Defective metabolism of methionine describes homocystinuria, an autosomal recessive deficiency of cystathionine synthase. Although patients with homocystinuria may also have tall stature, arachnodactyly, and pectus deformity, the lens dislocation in homocystinuria is downward rather than upwards. Patients with homocystinuria also tend to have a fair complexion, intellectual disability, and thrombophilia. Treatment includes vitamin B6 supplementation.\n\nAnswer B: Extra copy of a sex chromosome describes Klinefelter syndrome (47,XXY). Although patients with Klinefelter syndrome classically present with tall stature, they do not have the other clinical features found in this patient, including joint hypermobility and lens subluxation. Patients with Klinefelter also present with hypogonadism and azoospermia. Treatment is supportive\n\nAnswer D: Mutation of COL5A1 or COL5A2 leads to Ehlers-Danlos syndrome, a collagen disorder that commonly presents with joint hypermobility, skin hyperextensibility, and skin fragility with easy bruising. Ehlers-Danlos would not explain this patient\u2019s lens subluxation. Treatment may include bracing of unstable joints.\n\nAnswer E: Mutation of RET proto-oncogene describes multiple endocrine neoplasia type 2B (MEN2B). Patients with MEN2B may present with Marfanoid habitus and are predisposed to medullary thyroid cancer, pheochromocytoma, and mucosal neuromas. MEN2B would not explain lens subluxation. Treatment is with surgical resection of tumors.\n\nBullet Summary:\nMarfan syndrome is caused by an autosomal dominant mutation in the fibrillin-1 gene on chromosome 15 and presents with a triad of musculoskeletal, ocular, and cardiovascular findings.", "link": "https://bit.ly/42Cge5V"} {"question": "A 62-year-old man presents to the emergency department with increased fatigue and changes in his vision. For the past month, he has felt abnormally tired and today noticed his vision is blurry. The patient also endorses increased sweating at night and new-onset headaches. He currently feels dizzy. The patient has a medical history of diabetes and hypertension. His current medications include insulin, metformin, and lisinopril. His temperature is 99.5\u00b0F (37.5\u00b0C), blood pressure is 157/98 mmHg, pulse is 90/min, respirations are 17/min, and oxygen saturation is 98% on room air. A cardiopulmonary exam is within normal limits. HEENT exam reveals non-tender posterior and anterior chain lymphadenopathy. Abdominal exam reveals splenomegaly and hepatomegaly. There are large, non-tender palpable lymph nodes in the patient's inguinal region. A neurological exam is notable for decreased sensation in the patient's hands and feet. He also complains of numbness and tingling pain in his extremities that has been persistent during this time. The dermatologic exam is notable for multiple bruises on his upper and lower extremities. Which of the following is most likely to be abnormal in this patient?", "choicesA": "Calcium", "choicesB": "IgA and IgG", "choicesC": "IgM", "choicesD": "Natural killer cells", "choicesE": "T-cells", "answer_idx": "C", "answer": "IgM", "explanation": "This patient is presenting with symptoms of hyperviscosity, fatigue, and organomegaly suggestive of a diagnosis of Waldenstrom macroglobulinemia (WM). WM is characterized by increased IgM production.\n\nWM occurs secondary to a clonal B-cell that pathologically overproduces IgM. High levels of IgM can lead to symptoms of hyperviscosity including blurred vision, dizziness, headaches, and peripheral neuropathy. Organomegaly is a unique finding in WM and can present with hepatomegaly and splenomegaly. Other non-specific findings include fatigue, anemia, and lymphadenopathy. This disease can be diagnosed using protein electrophoresis, which would demonstrate a spike in the levels of IgM protein. Treatment can include plasmapheresis and rituximab.\n\nGertz et al. review the evidence regarding the diagnosis and treatment of patients with WM. They discuss how patients often present with organomegaly as well as symptoms of hyperviscosity. They recommend rituximab treatment in symptomatic patients.\n\nIncorrect Answers:\nAnswers 1 & 2: Calcium and IgA/IgG can be elevated in multiple myeloma (MM). MM presents with fatigue, weight loss, bone pain (back pain in particular), and other non-specific findings. MM less commonly presents with symptoms of hyperviscosity and organomegaly and this phenomenon has been described primarily with the IgA subtype. This is a late phenomenon and would be unlikely without preceding symptoms of bone pain and weight loss as a substantial protein concentrations (>5 g/dL) are required for this phenomenon. Treatment is with systemic chemotherapeutic agents.\n\nAnswers 4 & 5: Natural killer cells and T-cells could be elevated in non-Hodgkin lymphoma which would present with fatigue, lymphadenopathy, and \"B-symptoms\" (e.g., night sweats and chills). It would not present with symptoms of hyperviscosity. Treatment is with multi-agent chemotherapy as well as possible local radiation therapy.\n\nBullet Summary:\nWaldenstrom macroglobulinemia (hyper IgM syndrome) presents with symptoms of hyperviscosity, fatigue, and organomegaly.", "link": "https://step2.medbullets.com/testview?qid=109769"} {"question": "A 26-year-old medical student presents to occupational health after sustaining a needlestick injury. She was drawing blood from an HIV-positive patient when she stuck herself percutaneously while capping the needle. She immediately washed the puncture wound with saline. The medical student has had a negative HIV serology from the beginning of medical school 2 years ago. She is monogamous with one male partner and denies any intravenous drug use. The source patient was recently diagnosed with HIV and has a CD4 count of 550 cells/\u00b5L. His most recent viral load is 1,800,000 copies/mL, and he was started on HAART 3 days ago. Which of the following is the most appropriate next step in management?", "choicesA": "Immediately initiate triple antiretroviral therapy", "choicesB": "Perform genotype testing on source patient and initiate antiretroviral therapy tailored to results", "choicesC": "Repeat HIV serology and initiate triple antiretroviral therapy if negative", "choicesD": "Repeat HIV serology and initiate triple antiretroviral therapy if positive", "choicesE": "Repeat HIV serology and initiate triple antiretroviral therapy immediately", "answer_idx": "E", "answer": "Repeat HIV serology and initiate triple antiretroviral therapy immediately", "explanation": "This medical student has sustained a hollow-bore needlestick injury. The most appropriate next step is to repeat HIV serology and immediately initiate triple antiretroviral therapy with tenofovir, emtricitabine, and raltegravir.\n\nHealthcare exposures to HIV are divided into 2 categories based on the level of contact with the source patient. If the healthcare worker\u2019s mucous membrane or nonintact skin comes into contact with an HIV-positive patient\u2019s blood, semen, or vaginal secretions, immediate prophylaxis is recommended. This category also includes all needlestick exposures. If the healthcare worker\u2019s mucous membrane or nonintact skin comes into contact with an HIV-positive patient\u2019s urine, feces, or bodily secretions, prophylaxis is not recommended. If the source patient\u2019s HIV status is unknown, it should be assumed that the patient is HIV positive until testing proves otherwise. The triple antiretroviral drug combination of tenofovir, emtricitabine, and raltegravir is favored due to the low incidence of side effects. Prophylactic treatment should be initiated immediately (ideally within 1-2 hours) and continued for 4 weeks at which time the healthcare worker\u2019s serology should be repeated.\n\nParikh et al. studied technologies for monitoring HIV infection. They discuss how high throughput methods are required in order to surveil for infection on a population level. They recommend using next-generation sequencing methods to improve surveillance methods.\n\nIncorrect Answers:\nAnswer A: Immediately initiating triple antiretroviral therapy is appropriate after drawing the medical student\u2019s HIV serology. The patient\u2019s serology at the time of exposure must be established to compare to future testing. Despite her previous negative serology and low-risk lifestyle, it would still be best to repeat the testing at the same time prophylactic treatment is initiated.\n\nAnswer B: Perform genotype testing on source patient and initiate antiretroviral therapy tailored to results is incorrect because this option would delay the initiation of prophylaxis. Empiric treatment with a triple antiretroviral drug combination is appropriate for all patients with high-risk exposure.\n\nAnswer C: Waiting to hear that the medical student\u2019s HIV serology is negative would delay the initiation of prophylaxis. Triple antiretroviral drug treatment should be initiated without waiting for the results. Empiric treatment of all patients with high-risk exposure is recommended in all cases.\n\nAnswer D: Waiting for the results of the medical student\u2019s serology would delay treatment for her exposure. If her serology is already positive, she will need to be treated past the 4-week period of prophylaxis. Even though she does not engage in high-risk behaviors, a positive serology would demonstrate that she was already infected with HIV.\n\nBullet Summary:\nAfter a needlestick injury from a HIV-positive source patient, a healthcare worker\u2019s HIV serology should be drawn and they should be immediately initiated on triple antiretroviral drug therapy.", "link": "https://bit.ly/3M2S7YO"} {"question": "A 51-year-old man presents to the emergency department complaining of fever and a cough with yellow-green sputum for the past 3 days. He denies any hemoptysis, facial pain, rhinorrhea, or night sweats. The patient reported similar symptoms 2 months ago. Chest radiograph performed at the time was notable for a right middle lobe consolidation. He was diagnosed with lobar pneumonia and completed a 7-day course of antibiotics with complete resolution of his symptoms. He has no other significant medical history and does not take any medications. He has smoked 1 pack of cigarettes a day for 35 years and drinks 3 beers per week. He is sexually active with his wife. His temperature is 101.7\u00b0F (38.7\u00b0C), blood pressure is 127/85, pulse is 102/min, and respirations are 22/min. Physical exam is notable for decreased breath sounds on the right side without wheezing and increased dullness to percussion on the right side. A chest radiograph is ordered and reveals a consolidation in the right middle lobe. Which of the following is the most appropriate next step in management?", "choicesA": "Barium swallow", "choicesB": "CT of the chest without contrast", "choicesC": "Human immunodeficiency virus (HIV) antibody assay", "choicesD": "Interferon-gamma release assay", "choicesE": "Serum immunoglobulin levels", "answer_idx": "B", "answer": "CT of the chest without contrast", "explanation": "This patient's presentation with recurrent pneumonia in the same anatomical location is concerning for an anatomic abnormality. Given his history of heavy smoking, lung cancer should be ruled out with a chest CT with possible subsequent bronchoscopy for tissue biopsy.\n\nRecurrent pneumonia can be classified either as those confined to 1 anatomical location of the lung or those involving different portions of the lung. If recurrent pneumonia is limited to a single region, the patient should be evaluated for an anatomic obstruction, such as a mass or mucous plug. Obstruction will hinder mucociliary clearance and allow for the proliferation of bacteria, leading to recurrent infections. The most concerning etiology of such an obstruction is a tumor causing external bronchial compression. Thus, patients should be screened with a chest CT to rule out lung cancer in these cases. If lesions are identified on CT, subsequent bronchoscopy or CT-guided biopsy may be warranted. Treatment may involve radiotherapy, chemoimmunotherapy, or surgery depending on the underlying malignancy. Other causes of recurrent pneumonia limited to an anatomical region include recurrent aspiration secondary to tracheoesophageal disorders or seizures.\n\nPurysko et al. discuss the role of contrast enhancement in chest CTs. The authors find common indications for contrast-enhanced chest CTs include investigation for pulmonary embolism, pleural pathology, or hilar pathology. The authors recommend against the regular use of contrast enhancement in chest CTs outside of selected indications as they rarely provide additional diagnostic benefits while subjecting the patient to the risk of adverse effects with contrast agents.\n\nIncorrect Answers:\nAnswer A: Barium swallow is often done to investigate anatomical abnormalities of the upper gastrointestinal tract such as tracheoesophageal fistula or Zenker diverticulum. These can increase the risk of aspiration and lead to recurrent pneumonia. In a patient with positive risk factors for cancer, chest CT to rule out malignancy would be the most appropriate next step.\n\nAnswer C: Human immunodeficiency virus (HIV) can cause secondary immunodeficiency, which can predispose patients to recurrent infections. These recurrent pneumonias would be expected to be in different regions of the lung and not anatomically confined.\n\nAnswer D: Interferon-gamma release assay can be used to diagnose tuberculosis (TB). Reactivation of TB will present clinically with fever, cough, hemoptysis, and night sweats. Chest radiography will commonly reveal an upper lobe infiltrate.\n\nAnswer E: Serum immunoglobulin levels would be an appropriate next step to diagnose primary immunodeficiencies such as common variable immunodeficiency or other antibody defects. This would present with multiple lung infections in different regions of the lung. Moreover, these patients would be expected to have other infections such as sinusitis, otitis, and gastrointestinal infections.\n\nBullet Summary:\nRecurrent pneumonia in the same anatomical region should be worked up with a CT with subsequent bronchoscopy and biopsy to rule out malignancy.", "link": "https://step2.medbullets.com/testview?qid=214961"} {"question": "A 15-year-old boy is brought to his pediatrician for a follow-up visit. His parents report that he is doing well in school and has many friends. Their only concern is that he has a \u201cdifficult stomach.\u201d Every few months, he complains of intermittent abdominal pain that takes several days to pass. He occasionally misses school due to the pain. There is blood in his stool during these episodes. These symptoms have persisted for many years. The parents have so far managed the condition using herbal teas and essential oils but have not sought formal medical care. His temperature is 98.2\u00b0F (36.8\u00b0C), blood pressure is 106/61 mmHg, pulse is 88/min, and respirations are 12/min. He has lost 10 pounds since his annual visit the year before and is the same height. On physical exam, the patient is thin with conjunctival pallor. His sclerae are anicteric and his abdomen is soft and non-tender without hepatosplenomegaly. He also has the findings seen in Figure A. He undergoes an abdominal CT that shows disease affecting the terminal ileum. Which of the following is most likely to be present in this patient?", "choicesA": "Decreased mean corpuscular volume", "choicesB": "Decreased serum homocysteine level", "choicesC": "Increased serum methylmalonic acid level", "choicesD": "Microcytes", "choicesE": "Prolonged partial thromboplastin time", "answer_idx": "C", "answer": "Increased serum methylmalonic acid level", "explanation": "This patient with episodic abdominal pain, bloody stool, growth failure, and erythema nodosum most likely has Crohn disease. Disease involving the terminal ileum predisposes patients to vitamin B12 deficiency, which would cause anemia with an elevated serum methylmalonic acid level.\n\nCrohn disease is an inflammatory bowel disease that presents with abdominal pain, bloody stools, and growth failure in children. Extra-intestinal manifestations of Crohn disease include fistula formation and erythema nodosum, which appear as multiple tender erythematous nodules that fade after 1-2 weeks into bluish plaques or macules that can be confused with bruises. Complications of Crohn disease when the disease involves the terminal ileum is vitamin deficiency and malabsorption. These deficiencies can include vitamin A, vitamin D, vitamin E, vitamin K, zinc, and vitamin B12. Vitamin B12 deficiency presents with a macrocytic, megaloblastic anemia with hypersegmented neutrophils on peripheral blood smear. Lab findings include elevated serum homocysteine and methylmalonic acid levels. Vitamin B12 deficiency is also associated with peripheral neuropathy and should be treated with intramuscular vitamin repletion.\n\nAkbulut presents the current state of evidence regarding vitamin deficiency in patients with Crohn disease. They found that vitamin B12 and folate levels are often low in patients with this disease. They recommend monitoring vitamin levels and providing repletion when necessary.\n\nFigure/Illustration A is a clinical photograph showing erythematous nodules on the extensor surfaces of the legs (red circles). This finding is consistent with the latter stage of erythema nodosum.\n\nIncorrect Answers:\nAnswer A: Decreased mean corpuscular volume may be seen in iron deficiency anemia, which can be co-morbid with Crohn disease. While this patient does have recurrent gastrointestinal blood loss, the very low frequency of his symptoms is unlikely to result in iron deficiency, making a decreased MCV less likely compared to an increased MCV given the underlying terminal ileum pathology.\n\nAnswer B: Decreased serum homocysteine level is not seen in any common human disease. An elevated (not decreased) serum homocysteine level is seen in vitamin B12 deficiency in addition to folate deficiency. This increase is due to the impaired processing of homocysteine in the absence of these cofactors.\n\nAnswer D: Microcytes would not be seen in vitamin B12 deficiency or folate deficiency, which instead present with macro-ovalocytes. Microcytic anemia would be seen in patients with iron deficiency anemia, which can present with fatigue and pallor in patients with occult bleeding. Iron supplementation and treatment of the underlying source of bleeding is appropriate care.\n\nAnswer E: A prolonged PTT could be seen in this patient as vitamin K absorption is likely affected. This patient is not presenting with a chief complaint of bruising/bleeding (the physical exam finding is erythema nodosum rather than ecchymosis), and symptoms point toward a diagnosis of anemia which would be associated with poor B12 absorption.\n\nBullet Summary:\nCrohn disease may affect the terminal ileum, resulting in vitamin B12 deficiency (increasing the serum methylmalonic acid level) that would present with macrocytic, megaloblastic anemia with hypersegmented neutrophils on a peripheral blood smear.", "link": "https://step2.medbullets.com/testview?qid=108740"} {"question": "A 55-year-old woman presents to the emergency department for evaluation of headache. She had a headache that was present for a few hours before subsiding 2 days ago, and then today she developed a headache that was sudden in onset and at maximal intensity within minutes. She has never had a headache like this in the past. Her temperature is 99.0\u00b0F (37.2\u00b0C), blood pressure is 157/99 mmHg, pulse is 90/min, respirations are 16/min, and oxygen saturation is 99% on room air. On exam, she has pain with passive and active movement of the neck. No focal neurologic deficits are appreciated. A non-contrast head CT is obtained as shown in Figure A. Which of the following is the most likely underlying etiology of this patient's headache?", "choicesA": "CNS vascular endothelial dysfunction", "choicesB": "Dehydration and stress", "choicesC": "Hypersensitivity of the trigeminal nerve", "choicesD": "Muscular pain and strain", "choicesE": "Rupture of a berry aneurysm", "answer_idx": "E", "answer": "Rupture of a berry aneurysm", "explanation": "This patient with sudden-onset headache, neck pain, and a CT scan showing blood in the basal cisterns likely has a subarachnoid hemorrhage. The most common cause of non-traumatic subarachnoid hemorrhage (SAH) is rupture of a saccular (berry) aneurysm.\n\nSAH typically presents with a sudden onset \"thunderclap\" headache that is often described as the \"worst headache of my life.\" Hypertension is a common risk factor. The diagnosis can be confirmed with a non-contrast head CT. Non-contrast CT scan of the head is sufficient to rule out subarachnoid hemorrhage if performed within 6 hours of symptom onset. If the head CT is negative but SAH is still suspected, a lumbar puncture can help confirm the diagnosis (demonstrating xanthochromia) or CT angiography can be performed. Patients should be admitted for neurosurgery and neuro-IR consultation and may require calcium channel blockers and possible intervention.\n\nMacdonald et. al review spontaneous subarachnoid hemorrhage. They discuss the pathophysiology, clinical manifestations, and diagnosis. They note the typical finding of accumulation of blood in the basal cisterns and sylvian fissures on CT imaging.\n\nFigure A shows a non-contrast head CT demonstrating blood in the subarachnoid space supporting the diagnosis of SAH. Note the hyperdensity in the sylvian fissures and basal cisterns.\n\nIncorrect Answers:\nAnswer A: CNS vascular endothelial dysfunction results in posterior reversible encephalopathy syndrome, which typically presents with hypertension and encephalopathy.\n\nAnswer B: Dehydration and stress could explain a migraine headache or a tension headache. Migraine headaches are often pulsating and may be associated with aura.\n\nAnswer C: Hypersensitivity of the trigeminal nerve is thought to be involved in the pathophysiology of migraine headache, and also could describe trigeminal neuralgia which presents with sudden and severe electrical pain in the distribution of the trigeminal nerve with minor stimulation.\n\nAnswer D: Muscular pain and strain describes a tension headache which presents with diffuse head pain that is mild and not localized. CT imaging is normal in tension headache.\n\nBullet Summary:\nSpontaneous subarachnoid hemorrhages are most commonly caused by rupture of a berry aneurysm.", "link": "https://bit.ly/3QRYhxm"} {"question": "A 55-year-old man presents with a 2-day history of mono-articular joint pain. He is otherwise healthy and denies fever, chills, or recent trauma. His medical history is notable for a kidney stone 6 months ago. He underwent a dental procedure to extract an infected wisdom tooth 3 weeks ago. The patient does not take any medications. He is a non-smoker and does not drink alcohol or use recreational drugs. His family history is significant for osteoarthritis in his father who is 78. His temperature is 98.6\u00b0F (37\u00b0C), blood pressure is 133/84 mmHg, pulse is 84/min, and respirations are 15/min. His body mass index is 27 kg/m^2. Physical examination shows a swollen and tender joint with overlying erythema. Diagnostic arthrocentesis is performed and shows a leukocyte count of 30,000/mm^3 with 85% neutrophils. Polarized microscopy of the synovial fluid is shown in Figure A. Which of the following would most likely be found on this patient\u2019s radiograph?", "choicesA": "Boutonniere deformity", "choicesB": "Chondrocalcinosis", "choicesC": "Heberden nodes", "choicesD": "Juxta-articular osteoporosis", "choicesE": "Tophi", "answer_idx": "B", "answer": "Chondrocalcinosis", "explanation": "This patient is presenting with a joint aspiration positive for calcium pyrophosphate dehydrate crystals, suggesting a diagnosis of pseudogout, which may show chondrocalcinosis on radiography.\n\nPseudogout presents classically with mono- or oligo-articular arthritis that most often affects the knee and proximal joints of elderly patients (> 60 years of age). The disease can be associated with hyperparathyroidism, Gitelman syndrome, familial hypocalciuric hypercalcemia, and hemochromatosis. The patient will have a leukocyte count < 50,000/mm^3 on joint aspiration and positively birefringent, rhomboid-shaped cells on microscopy, which represent calcium pyrophosphate dehydrate (CPPD) crystals. On radiography, chondrocalcinosis can be found, which is the calcification of adjacent cartilaginous structures. Treatment is with indomethacin.\n\nSidari and Hill review the evidence regarding the diagnosis and treatment of gout and pseudogout. They discuss how diagnosis can be made with fluid microscopy. They recommend treatment with non-steroidal anti-inflammatory drugs.\n\nFigure/Illustration A is polarized microscopy of joint fluid showing rhomboid-shaped crystals that are positively birefringent (red circles). These represent calcium pyrophosphate dehydrate crystals seen in pseudogout.\n\nIncorrect Answers:\nAnswer A: Boutonniere deformity is characterized by proximal interphalangeal (PIP) joint flexion and distal interphalangeal (DIP) joint extension and is a common hand deformity of rheumatoid arthritis. Rheumatoid arthritis is characterized by insidious onset of morning stiffness and pain that usually affects symmetric PIP and metacarpophalangeal (MCP) joints first. On joint aspiration, leukocyte count ranges from 5,000-50,000/mm^3, but rheumatoid arthritis would not explain the CPPD crystals. Treatment may include methotrexate.\n\nAnswer C: Heberden nodes are bony protuberances in the DIP joints, which are characteristic of osteoarthritis. Osteoarthritis presents with insidious onset of joint pain that worsens with activity and weight-bearing. On joint aspiration, the leukocyte count is < 2,000/mm^3.\n\nAnswer D: Juxta-articular osteoporosis is decreased bone density of the bone surrounding the joint space. It is a nonspecific sign that can be seen with many inflammatory joint diseases such as rheumatoid arthritis or septic arthritis. Septic arthritis presents with mono-articular joint pain, erythema, and swelling. Joint aspiration will show a leukocyte count > 50,000/mm^3 and Gram stain may be positive.\n\nAnswer E: Tophi are deposits of urate crystals in the soft tissue, which are nearly pathognomonic for gout. Gout presents as acute, mono-articular joint pain with physical examination showing a red, inflamed joint (usually the first metatarsophalangeal, knee, or ankle joint). Needle-shaped, negatively birefringent urate crystals are seen in the joint fluid aspirate. Prevention may include allopurinol.\n\nBullet Summary:\nPseudogout is associated with positively birefringent, rhomboid-shaped crystals and chondrocalcinosis.", "link": "https://bit.ly/48MbdvT"} {"question": "A 1-year-old girl is brought to a neurologist due to increasing seizure frequency over the past 2 months. She recently underwent a neurology evaluation which revealed hypsarrhythmia on electroencephalography (EEG) with a mix of slow waves, multifocal spikes, and asynchrony. Her parents have noticed the patient occasionally stiffens and spreads her arms at home. She was born at 38-weeks gestational age without complications. She has no other medical problems. Her medications consist of lamotrigine and valproic acid. Her temperature is 98.3\u00b0F (36.8\u00b0C), blood pressure is 90/75 mmHg, pulse is 94/min, and respirations are 22/min. Physical exam reveals innumerable hypopigmented macules on the skin and an irregularly shaped, thickened, and elevated plaque on the lower back. Which of the following is most strongly associated with this patient's condition?", "choicesA": "Cardiac rhabdomyoma", "choicesB": "Glaucoma", "choicesC": "Optic glioma", "choicesD": "Polyostotic fibrous dysplasia", "choicesE": "Renal cell carcinoma", "answer_idx": "A", "answer": "Cardiac rhabdomyoma", "explanation": "This patient with a seizure disorder, ash-leaf spots (innumerable hypopigmented macules), Shagreen patch (elevated irregular plaque on the lower back), and West syndrome (hypsarrhythmia on EEG and movements consistent with infantile spasms) likely has tuberous sclerosis. Tuberous sclerosis is associated with cardiac rhabdomyomas.\n\nTuberous sclerosis is a neurocutaneous disorder that is inherited in an autosomal dominant fashion. Mutations in TSC1 and TSC2 lead to unregulated cell growth, leading to the formation of hamartomas in various locations. Manifestations of tuberous sclerosis include seizures (secondary to subependymal nodules and cortical dysplasia), mental retardation, renal angiomyolipomas, facial angiofibromas, mitral regurgitation, and hypomelanotic macules (ash-leaf spots). Cardiac rhabdomyoma is strongly associated with tuberous sclerosis and can lead to arrhythmias. West syndrome, which is characterized by infantile spasms, intellectual disability, and hypsarrhythmia (a chaotic mixture of high-amplitude slow waves, multifocal spikes, and intrahemispheric-interhemispheric asynchrony) on EEG, is also associated with tuberous sclerosis. Diagnosis of tuberous sclerosis is made with genetic testing. Advanced imaging (CT or MRI) may reveal cortical tubers and lesions in the third and fourth ventricles. Abdominal imaging may reveal renal angiomyolipomas. The management of seizures in tuberous sclerosis includes anti-seizure medications, though surgery may also be pursued in select cases.\n\nHinton et al. studied the cardiac manifestations of tuberous sclerosis. They found that cardiac rhabdomyomas can lead to arrhythmia later in life. They recommended that infantile spasms in tuberous sclerosis be treated with vigabatrin but other seizure types should be treated similar to other epileptic disorders.\n\nIncorrect Answers:\nAnswer B: Glaucoma may occur in patients with Sturge-Weber syndrome, which is characterized by \"tram-track\" calcifications in the brain, port-wine stains of the face, mental retardation, and epilepsy. Diagnosis is with MRI of the brain with contrast to demonstrate leptomeningeal vascular malformations that are characteristic of the disease.\n\nAnswer C: Optic glioma is associated with neurofibromatosis type 1 (NF1), which presents with hyperpigmented macules (cafe-au-lait spots), axillary freckling, and cutaneous neurofibromas. Neurofibromas in NF1 may also grow on peripheral nerves, leading to malignant peripheral nerve sheath tumors. Lisch nodules, or benign growths of the iris, may also be seen in NF1. Diagnosis of NF1 is clinical, but can be aided by genetic testing.\n\nAnswer D: Polyostotic fibrous dysplasia is associated with McCune-Albright syndrome. Polyostotic fibrous dysplasia causes fibrous tissue in bones, leading to growth abnormalities and pathologic fractures. McCune-Albright syndrome is also associated with cafe-au-lait macules, which have irregular borders, and endocrine abnormalities such as precocious puberty and hyperthyroidism. Diagnosis is supported with endocrine tests as well as genetic testing.\n\nAnswer E: Renal cell carcinoma is associated with Von-Hippel-Lindau syndrome (VHL), which presents with tumors arising in multiple organs. Patients with VHL present with hemangioblastomas in the brain, spinal cord, cerebellum, and retina, and pheochromocytoma. Pheochromocytomas release catecholamines, leading to episodic hypertension, diaphoresis, and palpitations. Diagnosis of VHL is with genetic testing.\n\nBullet Summary:\nTuberous sclerosis has numerous manifestations including seizures, angiofibromas, mitral regurgitation, renal angiomyolipoma, ash-leaf spots, mental retardation, and cardiac rhabdomyoma.", "link": "https://bit.ly/47NX9Au"} {"question": "A 55-year-old man presents to urgent care for weakness and weight loss. For the past several months, he has felt progressively weaker and has lost 25 pounds with intermittent abdominal pain. The patient has not seen a physician in 30 years and recalls being current on most of his vaccinations. A few years ago, he went to the emergency department due to abdominal pain and was found to have increased liver enzymes due to excessive alcohol use and incidental gallstones. The patient has a 50 pack-year smoking history. His temperature is 99.5\u00b0F (37.5\u00b0C), blood pressure is 161/108 mmHg, pulse is 90/min, respirations are 17/min, and oxygen saturation is 95% on room air. Physical exam reveals an emaciated man. The patient has a negative Murphy sign and his abdomen is non-tender. Cardiopulmonary exam is within normal limits. Which of the following is the most appropriate next step in management?", "choicesA": "CT scan of the abdomen and pelvis", "choicesB": "CT scan of the chest", "choicesC": "HIDA scan", "choicesD": "Right upper quadrant ultrasound", "choicesE": "Smoking cessation advice and primary care follow up", "answer_idx": "A", "answer": "CT scan of the abdomen and pelvis", "explanation": "This patient with weight loss and painless liver dysfunction has symptoms that are suggestive of pancreatic cancer. The most appropriate next diagnostic test is a CT scan of the abdomen and pelvis.\n\nPancreatic cancer presents with weight loss, jaundice, epigastric pain, and fatigue/malaise typically in an older male smoker who uses alcohol. Laboratory tests will demonstrate an elevated alkaline phosphatase and bilirubin. If a patient is presenting with the classic \"painless jaundice\" then the most appropriate next step in management could be an ultrasound to evaluate for obstruction. Otherwise, the best diagnostic test for this condition is a CT scan of the abdomen to characterize the pancreas and any metastasis to local organs. Treatment is with excision of the tumor through a Whipple procedure in appropriate candidates.\n\nLee and Lee review the evidence regarding the diagnosis of pancreatic cancer. They discuss how abdominal CT or MRI are important in making this diagnosis. They recommend having a high clinical index of suspicion given this diagnosis carries a high mortality rate.\n\nIncorrect Answers:\nAnswer B: CT scan of the chest would be appropriate to screen a smoker for lung cancer. If this patient had concern for malignancy with a smoking history, then a CT scan of the chest would be appropriate. It would be appropriate at a later time to assess for pulmonary nodules and lung cancer.\n\nAnswer C: HIDA (hepatobiliary iminodiacetic acid) scan is an appropriate diagnostic test to characterize a gallstone blockage of the common bile duct if a right upper quadrant ultrasound is not revealing in the diagnosis of acute cholecystitis.\n\nAnswer D: A right upper quadrant ultrasound can assess the gallbladder for stones and the liver for an abscess, but it will not assess the pancreas for pancreatic cancer (at least in a reliable manner that assesses the entire pancreas).\n\nAnswer E: Smoking cessation and primary care follow up would be appropriate management for this patient if he only presented requesting general advice to improve his health or stop smoking. His current symptoms are too alarming for malignancy. He should stop smoking regardless.\n\nBullet Summary:\nThe most appropriate initial test for pancreatic cancer is an abdominal CT.", "link": "https://bit.ly/3GigfDn"} {"question": "A 26-year-old woman presents to the emergency department with 1 day of vaginal bleeding and lower abdominal pain. The pain is most pronounced in the lower abdomen and has gotten worse over the past day. Her last menstrual period was 7 weeks ago. She denies any recent abdominal or vaginal trauma. She has no other past medical history and takes no medications. She uses a copper intrauterine device and is in a monogamous relationship. Her temperature is 99\u00b0F (37.2\u00b0C), blood pressure is 79/50 mmHg, pulse is 135/min, and respirations are 20/min. Physical examination is notable for diffuse abdominal tenderness upon palpation. Pelvic examination demonstrates uterine and adnexal tenderness and uterine bleeding. Laboratory studies are shown below:\n\nHemoglobin: 9.5 g/dL\nLeukocyte count: 6,000/mm^3\nSerum beta-human chorionic gonadotropin (hCG): 2,700 IU/L\n\nThe patient is started on intravenous fluids. Which of the following is the most appropriate next step in management?", "choicesA": "CT of the abdomen and pelvis", "choicesB": "Laparoscopy", "choicesC": "Methotrexate therapy", "choicesD": "Repeat beta-hCG in 48 hours", "choicesE": "Transvaginal ultrasound", "answer_idx": "B", "answer": "Laparoscopy", "explanation": "This patient\u2019s acute onset abdominal pain with uterine bleeding and tenderness, positive pregnancy test, anemia, and hemodynamic instability are concerning for a ruptured ectopic pregnancy. Emergency laparoscopy is the most appropriate next step in management.\n\nAn ectopic pregnancy, due to implantation of a fertilized ovum outside the uterine cavity, most commonly in the fallopian tube, can be life-threatening if it ruptures leading to intra-abdominal hemorrhage and shock. Initial laboratory studies should include a complete blood count and a pregnancy test. A patient\u2019s hemodynamic stability should be ascertained immediately as it determines the most appropriate next step in management. In patients who are hemodynamically unstable (hypotensive and tachycardic), a Focused Assessment with Sonography for Trauma (FAST) exam should be immediately performed to evaluate for intraperitoneal bleeding, and ob/gyn should be immediately consulted for emergent surgical intervention (e.g., salpingectomy or salpingotomy). In patients who are hemodynamically stable, transvaginal ultrasound should be performed to determine the gestational sac location, and medical therapy can be initiated (e.g., methotrexate therapy, depending on the size and location of the pregnancy).\n\nCheng et al. compare fertility outcomes of salpingotomy and salpingectomy in women with tubal pregnancies. The authors found that there was no difference in intrauterine pregnancy rates or repeat ectopic pregnancy rates between patients that underwent salpingotomy or salpingectomy. The authors recommend that both salpingectomy and salpingotomy are reasonable options for the surgical management of tubal pregnancies.\n\nIncorrect Answers:\nAnswer A: CT of the abdomen and pelvis is useful in determining if there is an intra-abdominal hemorrhage in patients who are hemodynamically stable. Unstable patients should generally not undergo CT.\n\nAnswer C: Methotrexate therapy is an effective treatment option in patients who are hemodynamically stable. with smaller ectopic pregnancies and lower hCG levels.\n\nAnswer D: Repeat beta-hCG in 48 hours is reserved for hemodynamically stable patients with non-diagnostic findings on transvaginal ultrasound. If the beta-hCG level is below the discriminatory zone (beta-hCG level 1,500-2,500 IU/L), then it should be repeated in 48 hours.\n\nAnswer E: Transvaginal ultrasound is reserved for hemodynamically stable patients who are presenting with abnormal uterine bleeding with abdominal/pelvic pain.\n\nBullet Summary:\nPatients who present with a ruptured ectopic pregnancy with hemodynamic instability are managed with emergent surgical intervention.", "link": "https://bit.ly/42mJcXw"} {"question": "A 24-year-old man presents to his primary care physician with left groin pain. The pain started a year ago and has progressed. He now has 8/10 pain when climbing stairs or rising from a seated position. He localizes the pain to the front of his groin. He denies any erythema or swelling. He has a history of hypertension, sickle cell anemia, and a splenectomy. He drinks a glass of wine with dinner every night and took up scuba diving 2 months ago but otherwise does not exercise. His temperature is 97.5\u00b0F (36.4\u00b0C), blood pressure is 124/68 mmHg, pulse is 82/min, and respirations are 16/min. His body mass index (BMI) is 18.1 kg/m^2. Physical exam reveals left groin pain at the terminal ranges of hip abduction and internal rotation. There is no point tenderness. A radiograph is obtained as in Figure A. Which of the following components of this patient\u2019s history most strongly predisposed him to this condition?", "choicesA": "Alcohol use", "choicesB": "History of sickle cell disease", "choicesC": "Lack of physical activity", "choicesD": "Low BMI", "choicesE": "Scuba diving", "answer_idx": "B", "answer": "History of sickle cell disease", "explanation": "This patient with sickle cell disease (SCD) presents with anterior groin pain, especially with weight-bearing, and radiographic findings of amorphous lytic and sclerotic lesions of the femoral head, consistent with avascular necrosis. SCD is a predisposing factor for avascular necrosis.\n\nAvascular necrosis has a multitude of direct causes, including irradiation, trauma (history of femoral neck fracture), hematologic diseases such as leukemia, SCD, and idiopathic etiology. In traumatic causes, injury to the medial femoral circumflex artery supplying the femoral head causes vascular disruption with resulting osteonecrosis. In SCD, the propensity for sickle-shaped erythrocytes to stick to one another and occlude blood vessels results in loss of circulation to the femoral head with consequent osteonecrosis. Avascular necrosis of the femoral head presents with insidious onset of anterior groin pain. The diagnosis can be secured with plain radiographs, although magnetic resonance imaging has the highest sensitivity and specificity for uncertain cases. Treatment consists of bisphosphonates with crutches (non-weight-bearing) and/or surgery depending on the degree of femoral head collapse and etiology.\n\nAkinyoola et al. studied the risk factors associated with avascular necrosis of the femoral head in patients with sickle cell disease. The authors found that the rate of pain crises and annual hospitalizations are associated with avascular necrosis. The authors recommend further study to explain differences in the prevalence of osteonecrosis in different populations with sickle cell disease.\n\nFigure/Illustration A shows a plain radiograph of the left hip in a skeletally mature individual, demonstrating amorphous cortical lesions (red arrows) in the femoral head and cortical irregularity (blue arrow) typical of avascular necrosis.\n\nIncorrect Answers:\nAnswer A: Excess alcohol use has been associated with the risk of avascular necrosis. This patient uses alcohol in moderation (2 drinks or fewer per day for men) and has no history of alcohol use disorder.\n\nAnswer C: Lack of physical activity is a predisposing factor for osteoporosis but is not associated with avascular necrosis. Osteoporosis may increase the risk of a hip fracture, which would present with a painful, shortened, and externally rotated lower extremity.\n\nAnswer D: Low BMI is not a predisposing factor for avascular necrosis. In contrast, obesity has been associated with an increased risk of avascular necrosis.\n\nAnswer E: Scuba diving can indirectly predispose patients to avascular necrosis through decompression sickness (Caisson Disease). In decompression, dissolved gases in the arterial supply form gas emboli, leading to microvascular occlusion and dysbaric osteonecrosis. However, this patient\u2019s symptoms have been present for 1 year whereas he only recently began scuba diving.\n\nBullet Summary:\nSickle cell disease is a predisposing factor for avascular necrosis of the femoral head, which presents as insidious anterior groin pain.", "link": "https://step2.medbullets.com/testview?qid=216499"} {"question": "A 30-year-old man is evaluated in the post-anesthesia care unit for neck pain. He underwent a laparoscopic appendectomy and was treated for post-operative nausea and vomiting with metoclopramide. Approximately 20 minutes after receiving the medication, he developed pain and stiffness in his neck and eventually was unable to move his neck. His temperature is 98.6\u00b0F (37.0\u00b0C), pulse is 90/min, blood pressure is 130/80 mmHg, respirations are 16/min, and oxygen saturation is 98% on room air. Examination reveals an uncomfortable appearing man. His neck is rotated to the right and is unable to return to midline. Examination of the patient is shown in Figure A. Which of the following is the most appropriate next step in management?", "choicesA": "Botulinum toxin injection", "choicesB": "Diphenhydramine", "choicesC": "Haloperidol", "choicesD": "Observation", "choicesE": "Ondansetron", "answer_idx": "B", "answer": "Diphenhydramine", "explanation": "This patient is suffering from a metoclopramide-induced acute dystonic reaction. Treatment of this complication includes IV diphenhydramine or benztropine.\n\nDystonic reactions are reversible extrapyramidal symptoms that may include involuntary spasms or sustained contractions of the face, neck, torso, pelvis, extremities, and larynx. Medications that block dopamine D2 receptors carry the greatest risk for development of a dystonic reaction, most notably including neuroleptics, antidepressants, and antiemetics. A family history of dystonia increases the risk of developing a reaction (there is likely an underlying genetic predisposition). Diagnosis is typically based on history and clinical presentation. Intravenous anticholinergic agents are the first-line treatment, and symptoms typically resolve rapidly (within 1 hour) after initiation of these agents.\n\nDressler et al. discuss the diagnosis and management of medication-induced acute movement disorders, notably dystonic reactions. These reactions are most commonly caused by dopamine-receptor blocking agents, including antipsychotics and antiemetics. Cranial, pharyngeal, and cervical muscles are typically affected within the first 4 days of initiating treatment.\n\nFigure A shows a patient with dystonia and torticollis; note the posturing of the extremities as well as how the head is tilted to the patient's right side while the chin is directed up and towards the left.\n\nIncorrect Answers:\nAnswer A: Intramuscular botulinum toxin injection may be appropriate for chronic torticollis or other forms of chronic dystonia. This patient has acute dystonia that is generally able to be easily reversed with intravenous diphenhydramine.\n\nAnswer C: Intravenous haloperidol, a D2 receptor antagonist would not be appropriate. This patient's condition is due to the dopamine antagonist action of metoclopramide and would therefore be worsened with haloperidol.\n\nAnswer D: Observation would not be appropriate. This patient is experiencing an acute dystonic reaction that is uncomfortable and potentially dangerous. Acute dystonia is not likely to self resolve in a reasonable amount of time.\n\nAnswer E: Intravenous ondansetron may be a more appropriate option to treat this patient's nausea given his reaction to metoclopramide. However, it will not reverse the acute dystonic reaction and is not the most appropriate first step.\n\nBullet Summary:\nAcute dystonic reaction may occur with administration of medications that block dopamine receptors (such as metoclopramide), and can be treated with intravenous diphenhydramine.", "link": "https://bit.ly/3LH7lCu"} {"question": "A 48-year-old man presents to the emergency room with a 2-hour history of severe abdominal pain, nausea, and vomiting. He states that he has not passed gas or had a bowel movement in 4 days and his pain has worsened and become constant over the past 2 hours. His only medical history includes an appendectomy that he underwent as a child, and he takes no daily medications. His temperature is 38.5\u00b0C (101.3\u00b0F), blood pressure is 92/60 mmHg, pulse is 138/min, and respirations are 25/min. His pulse oximetry is 99% on room air.There are no cardiopulmonary abnormalities on auscultation. His abdomen is distended and tender in all quadrants, with guarding and rebound present. He also has increased bowel sounds throughout. Laboratory results are as follows:\n\nHemoglobin: 11 g/dL\nLeukocyte count: 16,500/mm^3 with normal differential\nPlatelets: 250,000/mm^3\n\nSerum:\nCreatinine: 1.0 mg/dL\nGlucose: 95 mg/dL\nLipase: 45 U/L\nTotal bilirubin: 0.8 mg/dL\nAlkaline phosphatase: 74 U/L\nAspartate aminotransferase (AST, GOT): 32 U/L\nAlanine aminotransferase (ALT, GPT): 45 U/L\nLactate: 7.0 mmol/L\n\nWhich of the following is the most appropriate next step in management?", "choicesA": "Broad-spectrum antibiotics and serial abdominal radiographs", "choicesB": "CT angiography of the abdomen and pelvis", "choicesC": "Nasogastric tube placement", "choicesD": "Supportive care, NPO, and intravenous fluids", "choicesE": "Urgent surgical intervention", "answer_idx": "E", "answer": "Urgent surgical intervention", "explanation": "This patient with abdominal pain, vomiting, and obstipation is now hemodynamically unstable with fever, leukocytosis, rising lactate, and worsening pain. He most likely has a complicated small-bowel obstruction (SBO) and should undergo immediate surgical intervention.\n\nSBO occurs when the normal progression of intraluminal contents in the intestines is interrupted, with most cases stemming from mechanical blockages. A history of abdominal surgery is a risk factor for SBO due to the formation of adhesions. Patients with SBO can typically be managed conservatively including nasogastric tube suction (if discomfort and active vomiting), bowel rest, and intravenous fluid resuscitation. However, patients with findings of a complicated SBO (changes in abdominal pain, fever, leukocytosis, guarding, and hemodynamic instability) require emergent surgical intervention. Delay in abdominal exploration may lead to necrosis, perforation, infection and a significant risk of mortality.\n\nBower et al. review small bowel obstruction. They note the need for operative intervention only in those who fail conservative management. They recommend nonoperative management for most cases of small bowel obstruction.\n\nIncorrect Answers:\nAnswer A: Broad-spectrum antibiotics and serial abdominal radiographs alone would not be appropriate for this patient with a deteriorating clinical picture (hemodynamically unstable, worsening abdominal pain, fever, leukocytosis, high lactate). Antibiotics are not indicated in patients with uncomplicated SBO but may be useful in reducing infection risk in patients with complicated SBO. This patient should undergo emergent abdominal exploration in addition to receiving broad-spectrum antibiotics as it is possible he has already experienced bowel perforation.\n\nAnswer B: Computed tomography angiography (CTA) is useful in the diagnosis of acute mesenteric ischemia, which classically presents with abdominal pain out of proportion to exam (severe pain without much tenderness), vomiting, abdominal distention, and decreased bowel sounds. However, severe peritonitis and obstipation are more characteristic of SBO, and this patient is too hemodynamically unstable to undergo further imaging before proceeding to surgery.\n\nAnswers 3 & 4: Nasogastric tube placement with supportive care can be used to conservatively manage patients with uncomplicated SBO. This hemodynamically unstable patient has a fever, guarding, peritoneal signs, and newly worsened abdominal pain, indicative of a complicated SBO. He should be taken emergently to the operating room as the next step in management.\n\nBullet Summary:\nPatients with intestinal obstruction who have a surgical abdomen or hemodynamic instability should undergo immediate surgical intervention.", "link": "https://step2.medbullets.com/testview?qid=216246"} {"question": "A 60-year-old woman presents to the emergency department with back pain after gardening. Her pain is 7/10 in severity, non-radiating, and not relieved by rest. She has never experienced this pain in the past and denies fever, night sweats, unintentional weight loss, and bowel or bladder incontinence. She has hypertension for which she takes hydrochlorothiazide and had a recent asthma flare requiring a prednisone taper. She does not drink alcohol or smoke. Her temperature is 98.6\u00b0F (37.0\u00b0C), blood pressure is 120/80 mmHg, pulse is 90/min, and respirations are 18/min. Physical exam reveals an uncomfortable middle-aged woman in no acute distress. There is no tenderness to palpation of the spinous processes. Flexion of the hip with the knee extended while the patient is supine does not elicit any pain, nor does forced dorsiflexion of the foot at terminal hip extension. She has 5/5 strength to hip flexion, extension, abduction, and adduction; knee flexion and extension; and ankle dorsiflexion and plantarflexion bilaterally. Bilateral patellar and Achilles reflexes are 2+. Serum laboratory results are as follows: Hemoglobin: 12.0 g/dL Creatinine: 1.1 mg/dL Ca2+: 10.6 mg/dL Which of the following is the most likely diagnosis?", "choicesA": "Herniated disc", "choicesB": "Lumbosacral strain", "choicesC": "Multiple myeloma", "choicesD": "Spondylolisthesis", "choicesE": "Vertebral compression fracture", "answer_idx": "B", "answer": "Lumbosacral strain", "explanation": "This patient presents with acute low back pain after physical exertion without radicular signs (e.g., motor or sensory changes in a nerve root distribution) and a negative straight leg raise, which are most consistent with lumbosacral strain.\n\nThe evaluation of acute low back pain is focused on eliciting clinical history and signs that would indicate further work-up. In patients without significant trauma, the presence of signs of spinal cord compression (e.g., urinary retention, bowel incontinence, saddle anesthesia, focal neurologic deficits), history of or strong risk factors for cancer, signs of infection, or risk of vertebral compression fracture should prompt additional work-up with imaging such as radiographs or magnetic resonance imaging. If none of these findings are present (most patients with acute low back pain in the primary care setting), then conservative therapy for 4-6 weeks consisting of non-steroidal anti-inflammatory drugs with or without physical therapy is indicated.\n\nPatel and Ogle review the management of acute low back pain in the primary care setting. They outline the Waddell signs, which indicate the presence of a functional (signs of excessive pain behavior) aspect of pain. The authors recommend surgical evaluation only in patients with worsening neurologic deficits or intractable pain resistant to conservative treatment.\n\nIncorrect Answers:\nAnswer A: Herniated discs occurs when the nucleus pulposus herniates through a weakened part of the annulus fibrosus of an intervertebral disc, resulting in spinal nerve root compression. This causes radicular findings at the level of compression, including weakness in hip abduction and ankle dorsiflexion with L5 involvement and weakness in ankle plantarflexion and reduced Achilles reflex with S1 involvement, and radiation of pain into the ipsilateral lower extremity. The straight leg raise test would be positive, as it exacerbates the nerve root compression.\n\nAnswer C: Multiple myeloma presents with hypercalcemia, anemia, renal failure, and lytic bone lesions causing bone pain. This patient has no constitutional symptoms to suggest a malignant process and has a mild elevation in serum calcium, which is likely due to her thiazide diuretic. The clear association of her back pain with exertion makes lumbosacral strain more likely.\n\nAnswer D: Spondylolisthesis occurs when a vertebral body is forwardly subluxated relative to the adjacent vertebral body. This presents as back pain alleviated by rest. Since this can narrow the spinal canal, it may also present with neurogenic claudication, manifesting as buttock or leg pain with walking that can be alleviated by bending forward. This patient\u2019s back pain is not alleviated by rest and is associated with exertion.\n\nAnswer E: Vertebral compression fractures can present as back pain in a patient with minor or no trauma. These patients have osteoporosis and other risk factors like advanced age or chronic corticosteroid use and would have point tenderness at the site of fracture. This patient does not have point tenderness or risk factors.\n\nBullet Summary:\nLumbosacral strain is a common cause of acute low back pain that presents after physical exertion with no radicular signs and a negative straight leg raise test.", "link": "https://bit.ly/46vZtvp"} {"question": "A 4-week-old boy presents with his parents to the pediatrician for a well-child visit. His mother reports he was eating well until 1 week ago when he began vomiting after breastfeeding. His mother has tried increasing the frequency of feeds and decreasing the amount of each feed, but the vomiting seems to be getting worse. The patient now vomits after every feed. The vomitus looks like breast milk. The patient\u2019s mother is exclusively breastfeeding and would prefer not to switch to formula but worries that the patient is not getting the nutrition he needs. Two weeks ago, the patient was in the 75th percentile for weight and 70th for height. He is now in the 60th percentile for weight and 68th percentile for height. His temperature is 98.2\u00b0F (36.8\u00b0C), blood pressure is 58/37 mmHg, pulse is 144/min, and respirations are 34/min. On physical exam, the patient has dry mucous membranes. His abdomen is soft and non-distended. Which of the following is the most appropriate next step in management?", "choicesA": "Abdominal radiograph", "choicesB": "Abdominal ultrasound", "choicesC": "Supplement breastfeeding with formula", "choicesD": "Trial of dairy-free diet", "choicesE": "Trial of empiric proton pump inhibitor", "answer_idx": "B", "answer": "Abdominal ultrasound", "explanation": "This patient presents with a history of nonbilious, forceful vomiting, poor weight gain, and signs of dehydration, which is consistent with pyloric stenosis. The most appropriate next step in management is abdominal ultrasound to confirm the diagnosis.\n\nPyloric stenosis presents in 4- to 6-week-old infants with nonbilious, post-prandial projectile vomiting. Although pyloric stenosis classically presents with an \u201colive-shaped\u201d mass in the epigastrium, the mass cannot be palpated in some patients. Any infant who develops new, persistent vomiting in the first 2 months of life and has associated dehydration or failure to thrive should undergo an abdominal ultrasound to evaluate for pyloric stenosis. Patients should undergo resuscitation prior to the definitive treatment, which is a pyloromyotomy.\n\nAdams and Stanton review the evidence regarding the diagnosis and treatment of neonatal bowel obstruction. They discuss how malrotation, jejunoileal atresia, duodenal atresia, and colonic atresia are common causes of this finding. They recommend surgical management of these conditions.\n\nIncorrect Answers:\nAnswer A: Abdominal radiograph is not helpful in evaluating for pyloric stenosis. It is more useful in the workup of bilious vomiting, as several etiologies of bilious vomiting (such as duodenal atresia and Hirschsprung disease) can be seen on an abdominal radiograph. Treatment of these conditions is surgical excision of the defective segment of colon.\n\nAnswer C: Supplementing breastfeeding with formula would not be appropriate for this patient as his failure to thrive is driven by gastric outlet obstruction. In the case of pyloric stenosis, formula would cause vomiting just as breastmilk does.\n\nAnswer D: A trial of a dairy-free diet is used to improve gastroesophageal reflux, but in the setting of this patient\u2019s age and the description of vomiting, he should undergo an abdominal ultrasound to evaluate for pyloric stenosis. Treatment of reflux also involves positional changes during feeding.\n\nAnswer E: A trial of empiric proton pump inhibitor is indicated for patients with gastroesophageal reflux disease, but it would not address this patient\u2019s pyloric stenosis. Patients can also undergo conservative treatments such as changing positioning during feeding.\n\nBullet Summary:\nThe diagnostic test of choice for pyloric stenosis in infants is an abdominal ultrasound.", "link": "https://bit.ly/3D2V2LR"} {"question": "A 75-year-old man presents to the emergency department with fatigue. He has had severe nausea, vomiting, and diarrhea for the past 3 days. Today, his wife noticed that he was confused. The patient has a history of coronary artery disease with a drug-eluting stent placed 1 year ago, obesity, hypertension, and dyslipidemia. He denies chest pain or dyspnea, but complains of diffuse abdominal pain. His temperature is 99.0\u00b0F (37.2\u00b0C), blood pressure is 100/60 mmHg, pulse is 105/min, and respirations are 22/min. Physical exam reveals a confused man with dry mucous membranes. His abdomen is diffusely tender to palpation. Laboratory studies are ordered and notable for the following:\n\nSerum:\nNa+: 147 mEq/L\nCl-: 112 mEq/L\nK+: 3.5 mEq/L\nHCO3-: 14 mEq/L\nBUN: 80 mg/dL\nGlucose: 100 mg/dL\nCreatinine: 2.1 mg/dL\nTroponin: 0.10 mcg/L (normal < 0.01 mcg/L)\n\nAn ECG is ordered and is pending. Which of the following is the most appropriate management of this patient's elevated troponin?", "choicesA": "Aspirin", "choicesB": "Aspirin, heparin, clopidogrel, and cardiac catheterization", "choicesC": "Metoprolol and aspirin", "choicesD": "Obtain a urinalysis and creatine phosphokinase level", "choicesE": "Ringer lactate bolus", "answer_idx": "E", "answer": "Ringer lactate bolus", "explanation": "This patient with likely gastroenteritis (given his nausea, vomiting, and diarrhea) is dehydrated, as indicated by his low blood pressure, tachycardia, dry mucous membranes, and prerenal kidney injury (elevated BUN and creatinine with BUN:creatinine ratio > 20:1) in the setting of an anion gap (anion gap = [Na+] - [Cl-] - [HCO3-] = 147 - 112 - 14 = 21). His elevated troponin is likely secondary to poor renal clearance and dehydration, which is most appropriately addressed with IV fluids such as Ringer lactate.\n\nTroponin is a lab marker classically thought to be elevated when there is an injury to the heart causing leakage of this enzyme from the cardiac myocyte. However, there are many possible causes of an elevated troponin that may not be directly cardiac in nature or related to myocardial ischemia. Acute kidney injury is a common cause of an elevated troponin. For this reason, patients with an elevated troponin with kidney dysfunction without any cardiac symptoms or ECG changes should have their renal etiology addressed, and this will often clear the troponin. In the setting of a prerenal injury (dehydration causing poor kidney perfusion and a BUN:creatinine ratio > 20:1), IV fluids will restore perfusion and treat the underlying cause. Patients with chronic kidney disease or kidney failure should have an ECG obtained and their current troponin should be compared to previous troponin levels to ensure it is not higher than their baseline. If there is uncertainty regarding a troponin level, it can be trended to see if it is rising, falling, or staying the same. Correlation with clinical symptoms is important when evaluating an elevated troponin level.\n\nMasri et al. review the causes of an elevated troponin. They note that a troponin elevation can be seen in myopericarditis, renal failure, heart failure, pulmonary embolism, septic shock, rhabdomyolysis, stroke, and many other possible causes that are not related to myocardial infarction. They recommend keeping a broad differential and correlating the troponin with the patient's clinical presentation.\n\nIncorrect Answers:\nAnswer A: Aspirin would be appropriate management of an elevated troponin secondary to acute coronary syndrome as its antiplatelet activity reduces mortality. It should be administered prior to performing an ECG in patients with chest pain thought to be from a cardiac etiology.\n\nAnswer B: Aspirin, heparin, clopidogrel, and cardiac catheterization is appropriate management of a ST elevation myocardial infarction, which presents with chest pain, diaphoresis, dyspnea, and ST elevation within a vascular distribution with reciprocal changes on ECG. This patient has no cardiac symptoms and has a better alternative explanation for their troponin elevation.\n\nAnswer C: Metoprolol and aspirin may be indicated in acute coronary syndrome, and metoprolol would reduce this patient's heart rate. However, note that this patient's tachycardia is secondary to volume depletion; thus, beta-blockade for sinus tachycardia is inappropriate and may make the patient hemodynamically unstable.\n\nAnswer D: Obtaining a urinalysis and creatine phosphokinase level may be performed in rhabdomyolysis which presents in the setting of seizures, electrical shock, heatstroke, or with vigorous exertion. Patients will complain of muscle pain, oliguria, and dark urine. Management is centered on the administration of large volumes of fluid to protect the kidney and clear the toxic myoglobin. Note that rhabdomyolysis may elevate the troponin; however, there is nothing in this patient's history that suggests rhabdomyolysis.\n\nBullet Summary:\nPrerenal injury causing an acute kidney injury may elevate the troponin due to decreased renal clearance and should be addressed with the administration of fluids.", "link": "https://step2.medbullets.com/testview?qid=216358"} {"question": "A girl presents to her pediatrician for a well-child visit. Her mother reports that she is eating well at home and sleeping well throughout the night. She can jump and walk up and down stairs with both feet on each step. In the doctor\u2019s office, the patient builds a 6-cube tower and imitates a circle. She seems to have a vocabulary of over 50 words that she uses in 2-word sentences. The patient enjoys playing near other children and sometimes argues over toys with her older brother. Her temperature is 98.6\u00b0F (37\u00b0C), blood pressure is 92/56 mmHg, pulse is 106/min, and respirations are 23/min. On physical exam, she appears well developed and well nourished, and she is following along her growth curves. The child is assessed as developmentally normal. Which of the following is an additional milestone associated with this child\u2019s age?", "choicesA": "Balances on one foot", "choicesB": "Cuts with scissors", "choicesC": "Follows two-step commands", "choicesD": "Points to one body part", "choicesE": "Turns pages in book", "answer_idx": "C", "answer": "Follows two-step commands", "explanation": "This patient is able to walk up and down stairs, can build a 6 cube tower, has a 50+ word vocabulary, and uses 2-word phrases, which suggests she is 2 years of age. Another milestone at 2 years of age is following 2-step commands.\n\nIn the gross motor category, a child at 2 years of age should be able to jump and walk up and down stairs with both feet on each step. In the fine motor category, the child should be able to copy a line or circle and build a tower of 6 cubes. In the language category, the child should be using over 50 words and putting them together in 2-word phrases. In the social category, a child should be participating in parallel play. In the receptive language category, a child should be able to follow a 2-step command. Persistent developmental delays should prompt investigation for correctable causes such as hearing loss.\n\nScharf et al. review the evidence regarding the use of developmental milestones in assessing children. They discuss how early identification of delays can allow for referral to required services. They recommend paying special attention to sensory function to avoid missing a correctable hearing deficit.\n\nIncorrect Answers:\nAnswer A: Balancing on 1 foot is a milestone at 3 years of age. A 2-year-old child would not be expected to maintain balance in that position.\n\nAnswer B: Cutting with scissors is a milestone at 3 years and 6 months of age. A 2-year-old child would not be expected to be able to hold or use scissors correctly.\n\nAnswer D: Pointing to one body part is a milestone at 15 months of age. A 2-year-old child should be able to point to 5-6 body parts.\n\nAnswer E: Turning pages in a book is a milestone at 16 months of age. It would not be considered a milestone for a 2-year-old child.\n\nBullet Summary:\nAt 2 years of age, a child should be able to walk up and down stairs, build a tower of 6 cubes, use a vocabulary of over 50 words in 2-word phrases, and follow a 2-step command.", "link": "https://bit.ly/3Q5BZZe"} {"question": "A 41-year-old man presents to his primary care provider with a 2-month history of abdominal pain. He says that the pain \u201ccomes and goes\u201d throughout the day and usually lasts 20-30 minutes per episode. The pain is above his umbilicus. He denies any feeling of regurgitation or nighttime cough but endorses nausea. He used to eat 3 large meals per day but has found that eating smaller meals more frequently improves his pain. He tried ibuprofen with food and thinks it helped. He has gained 4 pounds since his last appointment 3 months ago. He denies any diarrhea or change in his stools. He has no known medical history. He drinks 5-6 beers on the weekend and has a 20 pack-year smoking history. He denies any family history of cancer. His temperature is 98.6\u00b0F (37\u00b0C), blood pressure is 117/72 mmHg, pulse is 72/min, and respirations are 13/min. On physical exam, he is tender to palpation above the umbilicus. Bowel sounds are present. A stool guaiac test is positive. The patient undergoes an endoscopy with a biopsy to diagnose his condition. Which of the following is most likely to be found on histology?", "choicesA": "Crypt abscesses in the large intestine", "choicesB": "Mucosal defect in the stomach", "choicesC": "Noncaseating granulomas in the small intestine", "choicesD": "PAS-positive material in the small intestine", "choicesE": "Urease-producing organism in the small intestine", "answer_idx": "E", "answer": "Urease-producing organism in the small intestine", "explanation": "This patient presents with episodic epigastric pain and nausea that improves with the consumption of food, which suggests a diagnosis of a duodenal ulcer. Duodenal ulcers are often caused by infection with Helicobacter pylori, a urease-producing organism.\n\nDuodenal ulcers are erosions and defects in the mucosal lining of the small intestines that allow for damage to the underlying tissues. Patients will classically present with abdominal pain that is localized to the epigastric region. Duodenal ulcers improve upon eating due to the secretion of bicarbonate in the small intestines, whereas gastric ulcers are exacerbated by food due to the release of gastric acid in the stomach. Duodenal ulcers are most commonly caused by infection with H. pylori. If the patient tests positive for H. pylori infection, treatment involves triple therapy with amoxicillin, clarithromycin, and a proton pump inhibitor. Patients who fail to improve should undergo esophagogastroduodenoscopy, which remains the gold standard for diagnosis.\n\nCiociola et al. studied the prevalence of H. pylori infection in patients with gastric ulcers. They found that about many had confirmed infections. They recommend against empiric treatment without prior confirmation of infection.\n\nIncorrect Answers:\nAnswer A: Crypt abscesses are found on histology in ulcerative colitis. Ulcerative colitis presents with bloody diarrhea and abdominal pain. Patients can also experience musculoskeletal pain, uveitis, and dermatologic findings such as erythema nodosum or pyoderma gangrenosum. Treatment may include mesalazine.\n\nAnswer B: A mucosal defect in the stomach is the pathologic description of a gastric ulcer. This patient\u2019s presentation of epigastric pain that improves with the consumption of food is more consistent with an ulcer in the duodenum. Treatment of a gastric ulcer may include proton pump inhibitors.\n\nAnswer C: Noncaseating granulomas are found on histology in Crohn disease. This patient denies any diarrhea (often associated with pain), which is the primary feature of Crohn disease. Other findings include abscesses, fistulas, and strictures. Treatment may include mesalamine and infliximab.\n\nAnswer D: PAS-positive material is found in the lamina propria on small intestinal biopsy in Whipple disease. In addition to abdominal pain, Whipple disease is characterized by diarrhea with flatulence and steatorrhea, fever, arthralgias, and weight loss. Treatment is with ceftriaxone or penicillin.\n\nBullet Summary:\nDuodenal ulcers typically present with episodic epigastric abdominal that is improved after eating and are associated with H. pylori infection.", "link": "https://bit.ly/3OrGevt"} {"question": "A 62-year-old man arrives at the emergency room complaining of chest pain and difficulty breathing. He reports that the dyspnea started 2 months ago after he had the flu. At first, the difficulty breathing occurred whenever he went up and down 1 flight of stairs, but the dyspnea progressively worsened since then. This morning, he developed chest pain and difficulty breathing while sitting at the kitchen table. The patient\u2019s medical history is significant for hypertension, type 2 diabetes mellitus, and rheumatoid arthritis. His medications include aspirin, lisinopril, metformin, and sulfasalazine. His temperature is 97\u00b0F (36.1\u00b0C), blood pressure is 130/78 mmHg, pulse is 88/min, and respirations are 14/min with an oxygen saturation of 97% O2 on room air. On physical exam, jugular venous distension during both inspiration and expiration, mild abdominal distention, and 2+ bilateral lower extremity edema is noted. A chest radiograph is obtained and the result is shown in Figure A. Which of the following is the most likely cause of this patient's symptoms?", "choicesA": "Cardiac tamponade", "choicesB": "Cirrhosis", "choicesC": "Constrictive pericarditis", "choicesD": "Pulmonary arterial hypertension", "choicesE": "Restrictive cardiomyopathy", "answer_idx": "C", "answer": "Constrictive pericarditis", "explanation": "This patient is presenting with peripheral edema, ascites, elevated jugular venous distension, and pericardial calcifications on chest radiograph, which are all consistent with a diagnosis of constrictive pericarditis.\n\nConstrictive pericarditis is the result of scarring and loss of elasticity of the pericardial sac. Common etiologies include idiopathic, post-viral, post-cardiac surgery, post-radiation, tuberculosis-related, or connective tissue disorder-related. It presents as right heart failure including symptoms such as dyspnea, chest pain relieved by leaning forward, progressive peripheral edema, and ascites. A physical exam will identify an early diastolic pericardial knock, pericardial calcifications on chest radiograph, and jugular venous distension (JVD). Kussmaul sign can also occur where the JVD paradoxically rises during inspiration instead of falling. The diagnosis is supported with echocardiography. Management may require pericardiectomy for severe cases.\n\nWelch and Oh review the evidence regarding the diagnosis and treatment of constrictive pericarditis. They discuss how this disease can be evaluated using a cardiac MRI. They recommend surgical pericardiectomy in many cases.\n\nFigure/Illustration A is a frontal chest radiograph with pericardial calcifications (red circle). These findings are consistent with constrictive pericarditis.\n\nIncorrect Answers:\nAnswer A: Cardiac tamponade can present similarly to constrictive pericarditis. Hypotension and pulsus paradoxus (a greater than 10 mmHg decrease in the systolic blood pressure during inspiration), will commonly present in patients with cardiac tamponade. A chest radiograph will show an enlarged cardiac silhouette rather than calcifications in some cases. Treatment is with emergent pericardial decompression.\n\nAnswer B: Cirrhosis can present with ascites and peripheral edema, but will not have a marked elevation in jugular venous pressure. Cirrhosis is the end-stage result of many hepatic pathologies. Treatment is with the control of sequelae such as hepatic encephalopathy and venous congestion.\n\nAnswer D: Pulmonary arterial hypertension, like constrictive pericarditis, can present with exertional dyspnea, chest pain, and peripheral edema if it progresses to right ventricular failure. A chest radiograph may show enlarged central pulmonary arteries and peripheral pulmonary vessels. Treatment is with reduction of pulmonary pressure such as with endothelin inhibitors.\n\nAnswer E: Restrictive cardiomyopathy can also present as right heart failure with similar symptoms to constrictive pericarditis. A history of an infiltrative disease (e.g., amyloidosis and sarcoidosis) often favors restrictive cardiomyopathy, as does an audible S3 on a physical exam. Calcification of the pericardium on the chest radiograph is more consistent with constrictive pericarditis.\n\nBullet Summary:\nConstrictive pericarditis presents with symptoms of right heart failure, progressive peripheral edema, ascites, jugular venous distension, pericardial knock, and pericardial calcifications on a chest radiograph.", "link": "https://step2.medbullets.com/testview?qid=109135"} {"question": "A 25-year-old woman presents to her primary care physician with joint and muscle pain. She has a general aching/pain in her joints that lasts all day. The pain is constant and is not relieved by rest or activity. She also has pain in her hands at times and occasionally notices trouble swallowing when she is eating. She has a medical history of anxiety and is not currently taking any medications. Her temperature is 98.6\u00b0F (37\u00b0C), blood pressure is 115/72 mmHg, pulse is 74/min, and respirations are 11/min. On physical exam, the patient is a healthy young woman with a sunburn. Her hands are mildly edematous with the findings in Figure A. Laboratory values are below:\n\nSerum:\nNa+: 145 mEq/L\nK+: 4.4 mEq/L\nCl-: 100 mEq/L\nHCO3-: 24 mEq/L\nBUN: 9 mg/dL\nGlucose: 70 mg/dL\nCreatinine: .7 mg/dL\nCa2+: 10 mg/dL\nMg2+: 1.8 mEq/L\n\nWhich of the following antibodies is most likely to aid in making the diagnosis in this patient?", "choicesA": "Anti-dsDNA", "choicesB": "Anti-IgG Fc region", "choicesC": "Anti-nuclear antibody", "choicesD": "Anti-ribonuleoprotein", "choicesE": "Anti-Ro", "answer_idx": "D", "answer": "Anti-ribonuleoprotein", "explanation": "This patient is presenting with diffuse muscle/joint pain, Raynaud phenomenon, and dysphagia suggesting a diagnosis of mixed connective tissue disease. Anti-ribonucleoprotein (U1) antibodies are found in this disease.\n\nMixed connective tissue disease can present with a vast array of symptoms. The most common symptoms are Raynaud phenomenon (vascular hyper-reactivity that presents with burning/pain in the hands where the hands change color from white to blue to red, seen in Figure A), arthralgias, myalgias, dysphagia, and edema of the hands. Anti-U1 ribonucleoprotein antibodies are commonly found in mixed connective tissue disease and are a specific marker. In this disease, there is typically an absence of renal involvement. Treatment involves steroids as well as calcium channel blockers for symptomatic control of Raynaud phenomenon.\n\nBatu et al. review the evidence regarding patients with mixed connective tissue disease. They found that many patients have overlapping symptoms with systemic lupus erythematosus and systemic sclerosis. They recommend studying patients with these mixed characteristics in order to better understand which treatment modalities are effective.\n\nFigure/Illustration A is a clinical photograph showing the blanching of the distal fingers (red box). This finding demonstrates the Raynaud phenomenon during the early phase where the vessels have spasmed.\n\nIncorrect Answers:\nAnswer A: Anti-dsDNA is a specific marker in systemic lupus erythematosus (SLE). SLE presents with SOAP BRAIN MD - Serositis, Oral ulcers, Arthritis, Photosensitivity, Blood disorders (hemolytic anemia, etc.), Renal involvement, ANA, Immune phenomena (anti-dsDNA, Smith), Neurological symptoms, Malar rash, and Discoid rash. Four of these criteria must be met to suspect a diagnosis of SLE. Treatment is with hydroxychloroquine.\n\nAnswer B: Anti-IgG Fc region is representative of rheumatoid factor which is found in rheumatoid arthritis. Rheumatoid arthritis can present with fever, weight loss, and joint pain that worsens in the morning and improves with activity. It is common for the joints of the hands to become deformed due to the disease process. Treatment is with methotrexate.\n\nAnswer C: Anti-nuclear antibodies (ANA) are likely positive in this patient; however, they are not more likely to aid in making the diagnosis than anti-ribonucleoprotein antibodies are. ANA are non-specific and can be found in many rheumatological conditions such as MCTD, lupus, and rheumatoid arthritis.\n\nAnswer E: Anti-Ro antibodies are found in Sjogren syndrome which presents with dry eyes, dry mouth, dental caries, and dyspareunia. A salivary gland biopsy can be used to confirm the diagnosis. Treatment includes artificial tears, vitamin D supplementation, and pilocarpine or cevimeline.\n\nBullet Summary:\nMixed connective tissue disease is associated with anti-ribonuleoprotein antibodies and presents with Raynaud phenomenon, arthralgia/arthritis, swollen hands, sclerodactyly, and myositis.", "link": "https://bit.ly/3WPL8WV"} {"question": "A 32-year-old man presents to the emergency department for evaluation of headache. His symptoms started last night and have persisted. He occasionally has migraine headaches, but he has not had a headache like this in the past. He reports associated nausea, vomiting, fevers, and neck pain. He has a medical history of asthma. His temperature is 100.4\u00b0F (38.0\u00b0C), blood pressure is 110/60 mmHg, pulse is 95/min, respirations are 17/min, and oxygen saturation is 98% on room air. On physical exam, he appears uncomfortable. Neurologic exam reveals no focal deficits. There is pain with passive neck flexion. No edema or rashes are noted. Lungs are clear to auscultation bilaterally. Lumbar puncture is performed and CSF results are obtained as shown below. Cerebrospinal fluid (CSF): Cell count: 175/mm^3 RBCs: 0/mm^3 Cl-: 119 mEq/L Glucose: 49 mg/dL Pressure: 150 mmH2O Total protein: 55 mg/dL Based on these results, which of the following is the most appropriate treatment regimen for this patient?", "choicesA": "Acyclovir", "choicesB": "Amphotericin and 5-flucytosine", "choicesC": "Ceftriaxone and vancomycin", "choicesD": "Ceftriaxone, vancomycin and steroids", "choicesE": "Supportive care and monitoring", "answer_idx": "E", "answer": "Supportive care and monitoring", "explanation": "This patient with headache, fever, and neck stiffness has a presentation and CSF studies consistent with a diagnosis of viral or aseptic meningitis. Supportive care is the most appropriate management strategy for patients with viral meningitis.\n\nAseptic (viral) meningitis presents similarly to bacterial meningitis with neck stiffness, photophobia, fever, malaise, and flu-like symptoms. Patients should be worked up and treated as if they have bacterial meningitis until proven otherwise. CSF findings in aseptic meningitis show an elevated cell count (though typically < 300/mm^3 in contrast to bacterial meningitis), a normal or elevated protein, and a normal glucose. CSF should be sent for Gram stain and culture to confirm the diagnosis and rule out bacterial meningitis. Once the diagnosis has been confirmed, management of viral meningitis consists of supportive care and monitoring for complications such as seizures, cerebral edema, or SIADH.\n\nKohil et. al review viral meningitis. They discuss the pathophysiology, clinical manifestation and diagnosis. They recommend that most patients with viral meningitis be treated with supportive care.\n\nIncorrect Answers:\nAnswer A: Acyclovir would be appropriate for suspected HSV or varicella encephalitis or meningitis. This patient with no RBCs in the CSF and no changes in mental status is not likely to have infection with HSV or varicella.\n\nAnswer B: Amphotericin and 5-flucytosine followed by long-term fluconazole is the appropriate management of cryptococcal meningitis. Cryptococcal meningitis presents similarly to bacterial and viral meningitis, almost exclusively in immunocompromised patients.\n\nAnswer C: Ceftriaxone and vancomycin would be appropriate if bacterial meningitis was suspected. Steroids are generally added if a bacterial source is likely. However, this patient has a presentation that is less concerning for bacterial meningitis, and CSF studies that are more consistent with aseptic meningitis.\n\nAnswer D: Ceftriaxone, vancomycin and steroids are an appropriate initial regimen for bacterial meningitis. Dexamethasone reduces inflammatory injury while patients are being treated with antibiotics.\n\nBullet Summary:\nViral or aseptic meningitis is managed with supportive care and monitoring.", "link": "https://step2.medbullets.com/testview?qid=210854"} {"question": "A 44-year-old man presents to the emergency department with palpitations. He has a history of anxiety managed with fluoxetine and lorazepam. He also has hypertension but does not take any medications. Today, his symptoms have been persistent despite taking his medications. He says that he feels lightheaded and can\u2019t walk. His temperature is 98.7\u00b0F (37.1\u00b0C), blood pressure is 120/72 mmHg, pulse is 183/min, respirations are 25/min, and oxygen saturation is 99% on room air. Physical exam reveals a somnolent and uncomfortable man. His pulse is rapid and regular. An ECG is performed as seen in Figure A. During the exam, the patient states that he feels lightheaded and appears occasionally somnolent. Which of the following is the most appropriate treatment for this patient?", "choicesA": "Adenosine", "choicesB": "Amiodarone", "choicesC": "Cardioversion", "choicesD": "Defibrillation", "choicesE": "Procainamide", "answer_idx": "C", "answer": "Cardioversion", "explanation": "This patient is presenting with palpitations and an ECG demonstrating a wide complex tachycardia with a delta wave which are concerning for a tachydysrhythmia secondary to Wolff Parkinson White syndrome. Given his poor CNS perfusion (somnolence during his tachycardia despite a \u201cnormal\u201d blood pressure), the most appropriate next step in management is cardioversion.\n\nWolff Parkinson White syndrome (WPW) is a pre-excitation syndrome where an accessory electrical pathway exists between the atria and ventricles that bypasses the AV node. WPW can demonstrate antidromic conduction where the accessory pathway leads to anterograde conduction causing a wide-complex tachycardia that is often mistaken for ventricular tachycardia. In this circumstance, the QRS is wide and a delta wave is often present but may be difficult to discern. If ever uncertain as to whether the diagnosis is ventricular tachycardia or WPW, cardioversion is a safe and effective intervention for both rhythms. Unstable patients with either WPW or ventricular tachycardia should undergo cardioversion immediately. Other treatments for WPW in stable patients may include procainamide. Ablation may present future episodes.\n\nBartlett and Friedman review the pathophysiology of WPW. They discuss the congenital presence of impulse-conducting fascicles leading to this condition. They also delve into the management and recommend rhythm agents and ablation as a modality of treatment.\n\nFigure/Illustration A is an ECG demonstrating a wide complex (red arrow) tachycardia with a delta wave (blue arrow). These ECG findings are characteristically seen in WPW syndrome.\n\nIncorrect Answers:\nAnswer A: Adenosine would be indicated after vagal maneuvers in the management of supraventricular tachycardia (SVT). SVT presents as a narrow complex tachycardia without P waves. It is regular (in contrast to atrial fibrillation). Failure to respond to vagal maneuvers or adenosine warrants cardioversion. If this patient was perfusing his CNS, a trial of adenosine could be appropriate given it has a short half-life and may differentiate ventricular tachycardia from SVT with aberrancy or a bundle branch block.\n\nAnswer B: Amiodarone may be used in stable ventricular tachycardia to control the rhythm and convert it to sinus. It would not be appropriate in a hemodynamically unstable patient. It can be used in other dysrhythmias as well including atrial fibrillation and atrial flutter.\n\nAnswer D: Defibrillation is only indicated in pulseless, shockable rhythms including ventricular fibrillation (disorganized electrical activity), pulseless ventricular tachycardia (wide complex tachycardia), and pulseless torsades des pointes (twisting of the QRS complexes around an isoelectric baseline). It could cause cardiac arrest if performed on a patient with a pulse.\n\nAnswer E: Procainamide is an antidysrhythmic that can be used in tachydysrhythmias that occur in WPW. Given that this patient is unstable, it is more appropriate to perform cardioversion.\n\nBullet Summary:\nUnstable tachyarrhythmias in the setting of Wolff Parkinson White syndrome require cardioversion.", "link": "https://step2.medbullets.com/testview?qid=216511"} {"question": "A 53-year-old African-American woman presents to her primary care doctor with increasing difficulty climbing the stairs. She noticed the weakness approximately 3 months ago. It has gotten progressively worse since that time. She has also noticed increasing difficulty combing her hair, standing from a seated position, and experiences muscle aches. Her medical history is significant for hypertension treated with lifestyle modification and gastroesophageal reflux disease (GERD). Her temperature is 98.6\u00b0F (37.0\u00b0C), blood pressure is 130/65 mmHg, pulse is 80/min, and respirations are 16/min. She has 3/5 strength to shoulder abduction and hip flexion bilaterally. Her strength is 5/5 to wrist extension and ankle plantar flexion. No skin rashes are noted. Which of the following is the most accurate test to confirm this patient\u2019s diagnosis?", "choicesA": "Electromyography", "choicesB": "MRI", "choicesC": "Muscle biopsy", "choicesD": "Serum aldolase", "choicesE": "Serum creatine kinase", "answer_idx": "C", "answer": "Muscle biopsy", "explanation": "This patient with insidious, subacute, progressive, symmetric proximal muscle weakness (difficulty climbing the stairs, combing her hair, standing from a seated position) without associated skin lesions most likely has polymyositis. The most accurate diagnostic test for polymyositis is muscle biopsy.\n\nPolymyositis is caused by an autoimmune reaction that leads to cell-mediated cytotoxicity against unidentified skeletal muscle antigens, primarily affecting the endomysium. It is more often found in women, patients between 30-60 years of age, and African-Americans. Clinical features of polymyositis include symmetric proximal muscle weakness that may be accompanied by myalgia, dysphagia (due to pharyngeal muscle weakness), restrictive lung disease (due to respiratory muscle weakness), cardiac involvement (myocarditis, heart failure), interstitial lung disease, and increased risk of malignancy. There are no accepted diagnostic criteria for polymyositis; however, diagnosis is usually secured with a combination of clinical features and other tests. Routine initial studies can include a complete blood count, complete metabolic panel, C-reactive protein, erythrocyte sedimentation rate, lactate dehydrogenase, aldolase, creatine kinase, autoantibodies (anti-nuclear antibody, anti-Jo-1), and electromyography (EMG). Muscle biopsy of an affected muscle is the gold standard for diagnosis and will typically show muscle fiber damage, CD8+ T cells in the endomysium, and overexpression of MHC-I on the sarcolemma. First-line treatment of polymyositis includes glucocorticoids and a steroid-sparing agent (methotrexate, azathioprine). Subsequent treatments can include intravenous immunoglobulins, rituximab, or plasmapheresis.\n\nAmato et al. review the evaluation and treatment of inflammatory myopathies such as polymyositis, dermatomyositis, and inclusion body myositis. They found that polymyositis typically presents in adult life with symmetric proximal leg greater than arm weakness. They recommend that a muscle biopsy be done on any patient considered to have polymyositis, preferably prior to starting immunosuppressive treatment.\n\nIncorrect Answers:\nAnswer A: Electromyography (EMG) can help distinguish myopathic weakness (e.g., polymyositis) from neuropathic causes of weakness (e.g., amyotrophic lateral sclerosis, myasthenia gravis). Characteristic EMG findings include fibrillations, early recruitment of motor unit potentials, and abnormal low-amplitude motor unit potentials. EMG is not a confirmatory test and is less sensitive than muscle biopsy for diagnosing myositis.\n\nAnswer B: MRI can be used to demonstrate areas of edema, atrophy, fatty replacement, and calcification in patients with suspected polymyositis. MRI can assess large regions of muscle, thus avoiding sampling error with muscle biopsy. The findings in MRI are nonspecific and require correlation with muscle biopsy for definitive diagnosis. MRI can also be used to help select the site of muscle biopsy.\n\nAnswer D: Serum aldolase can occasionally be used in the diagnostic investigation of suspected polymyositis if creatine kinase is normal. Aldolase elevation is nonspecific and may be seen in fasciitis, drug-induced myositis, and connective tissue disease-associated interstitial lung disease.\n\nAnswer E: Serum creatine kinase (CK) is the most sensitive muscle enzyme exam and should be part of the initial diagnostic investigation of any patient with suspected polymyositis. CK elevation is nonspecific and may be seen in myriad conditions such as inflammatory insult, metabolic injury, drug-induced injury, and infection.\n\nBullet Summary:\nMuscle biopsy is the gold standard for the diagnosis of polymyositis.", "link": "https://step2.medbullets.com/testview?qid=217661"} {"question": "A 16-year-old girl who recently immigrated to the United States from Bolivia presents to her primary care physician with a chief complaint of inattentiveness in school. The patient's teacher describes her as occasionally \"daydreaming\" for periods of time during where the patient does not respond or participate in school activities. Nothing has helped the patient change her behavior, including parent-teacher conferences or punishment. The patient has no other concerns. The only other concern that the patient's mother has is that, upon awakening, she notices that sometimes the patient's arm will jerk back and forth. The patient states she is not doing this intentionally. The patient has an unknown medical history and is currently not on any medications. She is not sexually active and does not intend to become pregnant in the near future. Her temperature is 98.6\u00b0F (37\u00b0C), blood pressure is 112/74 mmHg, pulse is 63/min, and respirations are 11/min. On physical exam, you note a young, healthy girl whose neurological exam is within normal limits. Which of the following is the most appropriate initial treatment?", "choicesA": "Carbamazepine", "choicesB": "Cognitive behavioral therapy", "choicesC": "Ethosuximide", "choicesD": "Lamotrigine", "choicesE": "Valproic acid", "answer_idx": "E", "answer": "Valproic acid", "explanation": "This patient is presenting with both absence seizures (periods of inattentiveness in school) and juvenile myoclonic epilepsy (abnormal, involuntary spasms of her upper extremity). The most appropriate treatment for these conditions is valproic acid.\n\nAbsence seizures classically present with cessation of motor activity and speech with a blank facial expression and occasionally automatisms such as flickering of the eyelids. They can be mistaken for inattentiveness in school-aged children. Juvenile myoclonic epilepsy presents with a \"jerky\" movement of the patient's extremity that typically occurs in the morning and begins around adolescence. The only medication that can be used to treat both absence seizures and juvenile myoclonic epilepsy is valproic acid. Valproic acid inhibits GABA transaminase, blocks thalamic T-type calcium channels, and blocks voltage-gated sodium channels. In post-pubertal female patients, it is important to counsel them on the potential teratogenic effects of this medication and also to consider alternative treatments in patients who are considering becoming pregnant or who cannot guarantee reliable birth control practices.\n\nKessler and McGinnis discuss the treatment of childhood absence seizures. They discuss how the treatment of choice is ethosuximide due to its favorable side effect profile. They recommend using valproic acid if there are concurrent seizure conditions.\n\nIncorrect Answers:\nAnswer A: Carbamazepine would be the appropriate therapy for a partial seizure (tonic or clonic movements in addition to an absence of a post-ictal period) or for trigeminal neuralgia (sudden pain in the distribution of the trigeminal nerve). It would not address both absence seizures and juvenile myoclonic epilepsy.\n\nAnswer B: Cognitive behavioral therapy is a useful treatment that helps a patient develop coping strategies and change unhelpful patterns of thought. It could be useful if this girl was having trouble focusing in school for another reason (such as obsessive-compulsive disorder); however, it is not useful in managing seizures.\n\nAnswer C: Ethosuximide is the most appropriate initial therapy for absence seizures. In this patient, she is suffering from both absence seizures and juvenile myoclonic epilepsy which could not be treated by ethosuximide alone, in contrast to valproic acid which could treat both conditions.\n\nAnswer D: Lamotrigine is an anti-epileptic that can also be used as a mood stabilizer. It carries the potentially severe side effect of Stevens-Johnson syndrome. It is not a first-line treatment for absence seizures and juvenile myoclonic epilepsy. Stevens-Johnson syndrome will present with painful desquamation.\n\nBullet Summary:\nAbsence seizures present with periods of inattentiveness and subtle automatisms and are most appropriately treated with ethosuximide or with valproic acid if there is concurrent juvenile myoclonic epilepsy.", "link": "https://step2.medbullets.com/testview?qid=108997"} {"question": "A 31-year-old man presents to the emergency department for fever, malaise, and cough. For the last week, his cough has been progressively worsening, and he has been noticing blood in his sputum. He denies any recent travel or sick contacts and infrequently smokes cigarettes. Approximately 5 weeks ago, he presented with similar symptoms and was found to have a right upper lobe lung infiltrate, a CD4 count of 40/mm^3, and an HIV viral load of 115,000 copies/mL. He was appropriately treated and discharged home. Four weeks after initiation of treatment, his CD4 count was over 400/mm^3 and HIV viral load was negligible. His temperature today is 102\u00b0F (38.9\u00b0C), blood pressure is 130/90 mmHg, pulse is 100/min, and respirations are 20/min. A radiograph of the chest demonstrates new nodules in the left upper lobe and hilar adenopathy. Which of the following is the most appropriate next step in management?", "choicesA": "Continue current anti-retroviral treatment, antimicrobial, and anti-tuberculoid therapies", "choicesB": "Determine drug sensitivities against the patient\u2019s pulmonary infection", "choicesC": "Discontinue antimicrobial treatment", "choicesD": "Obtain a D-dimer level", "choicesE": "Start high-dose glucocorticoid treatment and discontinue antiretroviral therapy", "answer_idx": "A", "answer": "Continue current anti-retroviral treatment, antimicrobial, and anti-tuberculoid therapies", "explanation": "This patient with recently diagnosed HIV and tuberculosis infection (upper lung lobe infiltrate) is presenting with worsening symptoms after being appropriately treated. Worsening of symptoms after initiation of these therapies is concerning for immune reconstitution inflammatory syndrome (IRIS), which is managed by continuing current combined antiretroviral medications, antibiotics for opportunistic infections and symptom management.\n\nIRIS describes a paradoxical worsening of a patient's underlying infection after initiating antiretroviral therapy for patients with HIV. IRIS can also occur in HIV-uninfected patients with tuberculosis or leprosy who are started on antimycobacterial treatment. The pathogenesis of IRIS is unclear, but in patients with HIV, antiretroviral therapy increases the amount of CD4+ T-helper cells, which can lead to an exaggerated inflammatory response (e.g., increased cytokine release) towards the infection. Thus, there is a paradoxical worsening of the infection symptoms. When patients have a paradoxical worsening of their symptoms in the setting of starting their antiretroviral therapy, the therapy should be continued with symptomatic management (acetaminophen for fever).\n\nNelson et al review IRIS. They note it is seen in patients with HIV. They recommend antiretroviral therapy and antibiotics for bacterial infections.\n\nIncorrect Answers:\nAnswer B: Determining drug sensitivities against the patient\u2019s pulmonary infection would be of low diagnostic utility since IRIS best explains his current clinical presentation.\n\nAnswer C: Discontinuing antimicrobial treatment would not be appropriate since the patient has findings concerning for tuberculosis (upper lobe lung infiltrate), and lack of treatment can lead to progression of his infection.\n\nAnswer D: Obtaining a D-dimer level is only indicated in low-risk patients for pulmonary embolism by Wells criteria.\n\nAnswer E: Starting high-dose glucocorticoid treatment and discontinuing antiretroviral therapy would not be appropriate since discontinuing his antiretroviral therapy would not control his HIV infection, which would place him at greater risk of acquiring opportunistic infections. Corticosteroid use is part of symptom management as it decreases the inflammatory response.\n\nBullet Summary:\nImmune reconstitution inflammatory syndrome (IRIS) can result in worsening of infectious symptoms and is managed by continuing antiviral therapy with symptomatic treatment.", "link": "https://bit.ly/47Zw1Ph"} {"question": "A 57-year-old woman presents to her primary care physician with weakness for the past 6 months with symptoms that have worsened recently. She feels fatigued, depressed, and has gained 10 pounds which she attributes to feeling too weak to go to the gym. She was recently treated for an ear infection with an antibiotic which she completed 3 weeks ago, and experienced a rash which she believes was an allergic reaction to her treatment. The patient has a medical history of diabetes mellitus that is well-controlled with insulin and metformin. She has been admitted multiple times for hypoglycemia secondary to missing meals. Her temperature is 99.5\u00b0F (37.5\u00b0C), blood pressure is 127/68 mmHg, pulse is 80/min, respirations are 17/min, and oxygen saturation is 98% on room air. Physical exam is notable for 2/5 strength in the patient's upper and lower extremities, sparse fine hair on her body, and a rash on her face and arms. Laboratory values are ordered as seen below. Serum: Na+: 139 mEq/L K+: 3.3 mEq/L Cl-: 100 mEq/L HCO3-: 25 mEq/L BUN: 20 mg/dL Glucose: 70 mg/dL Creatinine: 1.1 mg/dL Ca2+: 10.2 mg/dL AST: 12 U/L ALT: 10 U/L A muscle biopsy is obtained as seen in Figure A. Which of the following is the most likely explanation for this patient's presentation?", "choicesA": "Dermatomyositis", "choicesB": "Hypokalemia", "choicesC": "Hypothyroidism", "choicesD": "Inclusion body myositis", "choicesE": "Polymyositis", "answer_idx": "D", "answer": "Inclusion body myositis", "explanation": "This patient who presents with weakness and a rash has a biopsy suggestive of inclusion body myositis.\n\nInclusion body myositis presents with an insidious onset of weakness, a mildly elevated CRP and CK, and a rash. A muscle biopsy in this disease will demonstrate endomysial inflammation with basophilic-rimmed vacuoles within the muscle fiber sarcoplasm. Electromyography is typically abnormal, and the most accurate/confirmatory test is a muscle biopsy. There is no cure for this disease and no standard course of treatment. Corticosteroids are not usually effective but intravenous immunoglobulins can be effective.\n\nNaddaf et al. review the evidence regarding the diagnosis and treatment of patients with inclusion body myositis. They discuss how this disease is still refractory to treatment. They recommend a muscle biopsy to establish a definitive diagnosis.\n\nFigure/Illustration A is a muscle biopsy demonstrating endomysial inflammation with basophilic-rimmed vacuoles (red circle) within the muscle fiber sarcoplasm. These findings are classically seen in patients with inclusion body myositis.\n\nIncorrect Answers:\nAnswers 1 & 5: Dermatomyositis and polymyositis present similarly with muscle weakness and an elevated CK, CRP, and aldolase. However, dermatomyositis presents with dermatologic findings. A biopsy would demonstrate inflammation surrounding the muscle fascicle (dermatomyositis) or within the muscle fascicle (polymyositis). Treatment may include azathioprine and methotrexate.\n\nAnswer B: Hypokalemia could present with weakness and cardiac abnormalities. Though this patient is hypokalemic, her biopsy demonstrates inclusion body myositis. Treatment of hypokalemia is with potassium repletion.\n\nAnswer C: Hypothyroidism presents with depression, fatigue, scarce lanugo, myopathy, and weight gain (as this patient has). Though myopathy from hypothyroidism is plausible, her symptoms of weakness have a confirmatory biopsy. Treatment of hypothyroidism is with thyroid hormone replacement.\n\nBullet Summary:\nInclusion body myositis presents with weakness and an elevated CRP/CK with a muscle biopsy demonstrating endomysial inflammation with basophilic-rimmed vacuoles within the muscle fiber sarcoplasm.", "link": "https://bit.ly/3SyN6Lf"} {"question": "A 6-month-old girl is brought to the pediatrician by her father for a scheduled check-up. She was in her usual state of health until 6 weeks ago when she was hospitalized for bacterial pneumonia. She received amoxicillin and had a resolution of her symptoms. Her father reports that she seems \u201cback to normal.\u201d She can roll over by herself and can sit up without support. She often babbles but sometimes does make identifiable sounds like \u201cdada\u201d and \u201cbaba.\u201d They recently started feeding her mushy cereal, which she seems to like but occasionally spits up. She has started teething and sometimes becomes \u201cfussy.\u201d Vital signs are within normal limits. A physical examination is unremarkable. The patient\u2019s father brought a 4-week follow-up chest radiograph taken at the hospital, as shown in Figure A. Which of the structures labeled in Figure A would be most likely to change in response to another infectious illness?", "choicesA": "A", "choicesB": "B", "choicesC": "C", "choicesD": "D", "choicesE": "E", "answer_idx": "A", "answer": "A", "explanation": "The thymus gland (labeled in A) is visible on a chest radiograph in infants and will change in response to illness due to its immunologic role.\n\nThe thymus gland is the site of T-cell maturation. The thymus is visible on a chest radiograph in infants and continues to grow throughout childhood. It is located in the anterior part of the superior mediastinum. On a chest radiograph, the thymus may appear as a \u201cwidened mediastinum,\u201d continuous with the superior border of the heart (Figure A) or as a triangular-shaped opacity known as the thymic \u201csail sign,\" located towards the right of the mediastinum. At puberty, the gland will decrease in size and eventually be replaced by fatty tissue.\n\nAskin and Young review the evidence regarding the history and function of the thymus. They discuss how this structure is involved in immune function. They recommend monitoring for the absence of this structure in immunodeficient patients.\n\nFigure A shows a normal chest radiograph of an infant with a normal cardiothymic contour.\n\nIncorrect Answers:\nAnswer B: B points to the trachea. Deviation of this structure can be seen in patients with tension pneumothorax. Treatment is with needle thoracostomy followed by chest tube placement.\n\nAnswer C: C points to the apex of the heart. This structure can be enlarged in congenital heart conditions. Treatment is with surgical correction of these conditions.\n\nAnswer D: D points to the stomach. This structure can be abnormal in patients with pyloric stenosis. Treatment is with pyloromyotomy.\n\nAnswer E: E points to the diaphragm. This structure can be abnormal in patients with diaphragmatic hernias. Treatment is with surgical closure of the defect.\n\nBullet Summary:\nThe thymus is normally visible on a pediatric chest radiograph, but it is replaced by fatty tissue after puberty.", "link": "https://bit.ly/3v12XZt"} {"question": "A 57-year-old man presents to the emergency department for weight loss and abdominal pain. The patient has felt steadily more fatigued over the past month and has lost 22 pounds without effort. Today, he fainted prompting his presentation. The patient has no significant medical history. He does have a 33-pack-year smoking history and drinks 4 to 5 alcoholic drinks per day. His temperature is 99.5\u00b0F (37.5\u00b0C), blood pressure is 100/58 mmHg, pulse is 100/min, respirations are 17/min, and oxygen saturation is 98% on room air. The patient is very thin and appears pale. Stool fecal occult blood testing is positive. A CT scan of the abdomen is performed demonstrating a mass in the colon with multiple metastatic lesions scattered throughout the abdomen. The patient is informed of his diagnosis of metastatic colon cancer. When the patient conveys the information to his family, he focuses his efforts on discussing the current literature in the field and the novel therapies that have been invented. He demonstrates his likely mortality outcome which he calculated using the results of a large multi-center study. Which of the following is this patient most likely demonstrating?", "choicesA": "Dissociation", "choicesB": "Intellectualization", "choicesC": "Optimism", "choicesD": "Pessimism", "choicesE": "Rationalization", "answer_idx": "B", "answer": "Intellectualization", "explanation": "This patient is using facts and logic to emotionally distance himself from his stressful diagnosis. This ego defense method is suggestive of intellectualization.\n\nIntellectualization typically occurs in patients who have received a very stressful diagnosis or bad news for a close loved one. They will often focus on the science and research of their disease rather than focus on emotionally processing this stressful event. This is a way of distancing themselves from the diagnosis and their circumstances. These patients may have difficulty connecting with others because they become extremely focused on data.\n\nArnold reviews the evidence regarding the diagnosis of intellectualization. He discusses how this term can either refer to the isolation of affect or the general use of reason. He recommends a better understanding of how this ego defense works.\n\nIncorrect Answers:\nAnswer A: Dissociation is the detachment of the mind from the emotional state/body. A more specific diagnosis for this patient would be intellectualization as he is detaching himself from the emotional state of the diagnosis and instead focusing on the science behind his condition.\n\nAnswer C: Optimism is trying to find the best in a bad situation. This patient's focus on treatment outcomes and absence of emotion suggests intellectualization as the employed defense mechanism. Patients who are optimistic will still be able to emotionally process information.\n\nAnswer D: Pessimism is finding the worst in most situations. Pessimism would be a possible answer if the patient had a very treatable disease but thought that poor outcomes would occur regardless.\n\nAnswer E: Rationalization is the use of logic or reasoning (that can be inappropriate) in order to substantiate one's behavior even when it is inappropriate. An example is justifying theft because a person was in need.\n\nBullet Summary:\nIntellectualization is the use of facts and logic to distance oneself from a stressful situation.", "link": "https://bit.ly/3OGVR2b"} {"question": "A 55-year-old man presents to his primary care physician with persistent symptoms of depression. He started taking fluoxetine 3 weeks ago when he was diagnosed with major depressive disorder after a divorce. Since then, he notes that his symptoms are roughly unchanged. His other medications include albuterol. His temperature is 99.3\u00b0F (37.4\u00b0C), blood pressure is 132/88 mmHg, pulse is 77/min, respirations are 14/min, and oxygen saturation is 99% on room air. Physical exam reveals a depressed affect. His neurologic exam is non-focal. A serum thyroid stimulating hormone (TSH) resulted from his last visit and is noted to be 4.8 \u00b5U/mL (normal: 0.5-5 \u00b5U/mL). Which of the following is the most appropriate next step in management?", "choicesA": "Administer levothyroxine", "choicesB": "Change medication to bupropion", "choicesC": "Change medication to escitalopram", "choicesD": "Increase dose of current medication", "choicesE": "Maintain dose of current medication", "answer_idx": "E", "answer": "Maintain dose of current medication", "explanation": "This patient is presenting with major depressive disorder but has not given his current selective serotonin reuptake inhibitor (SSRI) at least 4 weeks to begin working. Thus, he should be reassured and maintained on the current dose of his SSRI, with follow-up after the 4 week mark to see if his symptoms have improved.\n\nMajor depressive disorder is managed initially with cognitive behavioral therapy and pharmacotherapy. The typical first-line medication is an SSRI. SSRIs may take at least 4 weeks or more to begin working, so patients that return before this time period should be reassured and encouraged to let the medication take effect as long as they are not experiencing suicidal thoughts or intent. If the medication is not working after at least 4 weeks, the next steps in management could include increasing the dose of the current medication, changing SSRIs, adding another antidepressant agent, or adding a low dose of levothyroxine. All patients with suicidal ideation and either intent or a plan to hurt themselves should be hospitalized involuntarily.\n\nGautam et al. discuss the current management algorithms for major depressive disorder. They note different options including CBT and pharmacotherapy. They recommend appropriate treatment with a first-line agent based on the patient and their circumstances.\n\nIncorrect Answers:\nAnswer A: Administering levothyroxine is appropriate as an augmenting agent in patients who do not have an improvement in symptoms on SSRIs or other antidepressants even if their TSH is within the normal range. However, it would not be attempted at this point given this patient has not given his current medication sufficient time to begin working. There is limited evidence to support its use overall. Note that this patient's TSH is within normal limits. In some cases, T3 administration is used in refractory depression.\n\nAnswer B: Changing the medication to bupropion could be appropriate in patients who want to avoid weight gain, those who want to quit smoking, or those who are trying to avoid sexual side effects. It also is an appropriate second-line agent or could be added to a first-line agent; however, this patient\u2019s current medication should be tried for at least 4 weeks before changing medications.\n\nAnswers 3 & 4: Changing medication to escitalopram or increasing the dose of the current medication are both viable strategies if a patient has failed to improve on their first medication after at least 4 weeks on the medication. Increasing the dose early may lead to symptoms of serotonin excess when the drug takes effect.\n\nBullet Summary:\nSSRIs are the first-line agents in the management of major depressive disorder and should be tried for at least 4 weeks before changing the treatment regimen.", "link": "https://step2.medbullets.com/testview?qid=216593"} {"question": "A 24-year-old man presents to the emergency department with abdominal pain, nausea, and vomiting. The symptoms began towards the end of running a marathon and persisted after finishing the race. The patient is otherwise healthy and has no other medical problems. His temperature is 98.0\u00b0F (36.7\u00b0C), blood pressure is 105/71 mmHg, pulse is 113/min, respirations are 19/min, and oxygen saturation is 98% on room air. Physical exam reveals a distended and tender abdomen. A radiograph is performed as seen in Figure A. Which of the following is the most likely diagnosis?", "choicesA": "Cecal volvulus", "choicesB": "Ileus", "choicesC": "Ogilvie syndrome", "choicesD": "Sigmoid volvulus", "choicesE": "Small bowel obstruction", "answer_idx": "A", "answer": "Cecal volvulus", "explanation": "This young, healthy patient presents with abdominal pain, nausea, vomiting, and a distended, tender abdomen with a radiograph demonstrating the \"coffee bean sign,\" likely has a cecal volvulus.\n\nCecal volvulus is the second most common cause of volvulus after sigmoid volvulus and presents with abdominal pain, constipation, vomiting, abdominal distention, and a tympanitic abdomen. It is more common in young patients who are active (e.g. marathon runners). The classic conventional radiographic finding in cecal volvulus is the coffee bean sign, in which closed loops of intestine distend with gas, making visible the medial and lateral walls of dilated bowel. Diagnosis of cecal volvulus can be confirmed with a CT scan. Management involves analgesia, intravenous fluids, and surgical detorsion.\n\nHasbahceci et al. review the presentation, diagnosis, and management of cecal volvulus. The authors find that abdominal radiography has a low sensitivity for cecal volvulus. The authors recommend surgical management of cecal volvulus with resection with right hemicolectomy and primary ileocolic anastomosis.\n\nFigure/Illustration A is a radiograph demonstrating the \"coffee bean\" sign which is suggestive of a cecal volvulus.\n\nIncorrect Answers:\nAnswer B: Ileus presents after surgery or manipulation of the intestines with decreased peristalsis of the intestine leading to a failure to pass gas or have bowel movements. Treatment involves treating any electrolyte abnormalities, encouraging ambulation, and avoiding medications that decrease peristalsis (such as opioids).\n\nAnswer C: Ogilvie syndrome occurs when the colon wall is not appropriately undergoing peristalsis leading to a dilated colon with abdominal distension. It is more common in the elderly and critically ill patients. Treatment involves the placement of a rectal tube to decompress the intestines and treatment of any underlying medical conditions.\n\nAnswer D: Sigmoid volvulus is more common in elderly and critically ill patients and presents with constipation, an absence of bowel movements, and a distended, tympanitic abdomen. Although the coffee bean sign may also be seen in patients with sigmoid volvulus, patients tend to be older. In addition, plain radiography typically demonstrates multiple air-fluid levels in a sigmoid volvulus versus usually only a single air-fluid level in a cecal volvulus. Treatment involves flexible sigmoidoscopy and surgery.\n\nAnswer E: Small bowel obstruction presents with a distended and tympanitic abdomen with nausea, vomiting, and an absence of flatus or bowel movements. Radiography may demonstrate distended bowel and air-fluid levels, and the most accurate test is a CT scan of the abdomen. It occurs secondary to adhesions entrapping the bowel. Management involves the placement of a nasogastric tube, intravenous fluids, and surgery.\n\nBullet Summary:\nCecal volvulus presents in younger patients with abdominal pain, distention, constipation, nausea, vomiting, and the \"coffee bean\" sign on radiography.", "link": "https://bit.ly/49ckvkP"} {"question": "A 26-year-old woman presents to the emergency department because she feels as though her heart is \"beating out of her chest.\" She states that she feels well apart from the discomfort of that sensation. Her temperature is 97.9\u00b0F (36.6\u00b0C), blood pressure is 124/84 mmHg, pulse is 180/min, respirations are 22/min, and oxygen saturation is 98% on room air. An ECG is obtained as seen in Figure A. After unsuccessful attempts at vagal maneuvers, the emergency physician administers an IV medication. The effect of the medication wears off in seconds. Which of the following is a potential side effect of this medication?", "choicesA": "Disulfiram-like reaction", "choicesB": "Flushing", "choicesC": "Photosensitivity", "choicesD": "Seizure", "choicesE": "Tachycardia", "answer_idx": "B", "answer": "Flushing", "explanation": "This patient presents with a narrow complex tachycardia that is regular which is suggestive of supraventricular tachycardia (SVT) and was pharmacologically cardioverted with adenosine (as it has a very short half-life). Side effects of adenosine include flushing, hypotension, bronchospasm, and heart block.\n\nSVT presents with a narrow-complex, regular tachycardia. The diagnosis is made with these findings on ECG. Management in an unstable patient is centered on cardioversion. Otherwise, the first step in management is centered on vagal maneuvers. If these fail, adenosine is the first-line pharmacologic treatment. While a relatively safe drug, side effects of adenosine include hypotension, heart block, flushing, and bronchospasm. Patients may also report a \"sense of impending doom\" following administration, thought to be the result of the transient asystole adenosine produces. Persistent SVT refractory to adenosine may be further managed with calcium channel blockers, beta blockers, or cardioversion.\n\nColucci et al. review the diagnosis and management of common types of supraventricular tachycardias. The most common types of supraventricular tachycardia are caused by a reentry phenomenon producing accelerated heart rates. Symptoms may include palpitations, chest pain, lightheadedness or dizziness, and dyspnea. If Wolff-Parkinson-White syndrome is present, expedient referral to a cardiologist is warranted because ablation is a potentially curative option.\n\nFigure A is an ECG demonstrating SVT. Note the regular narrow complex tachycardia that is a defining feature of this condition.\n\nIncorrect Answers:\nAnswer A: A disulfiram-like reaction was once thought to be associated with medications such as metronidazole. Metronidazole is an antibiotic that covers anaerobes. When alcohol is consumed in a patient on disulfiram, headache, malaise, nausea, and vomiting may be experienced.\n\nAnswer C: Photosensitivity is associated with medications such as sulfonamides, amiodarone, and tetracycline. Patients should be advised to stay out of the sun on these medications.\n\nAnswer D: Seizures are not associated with adenosine. Isoniazid, bupropion, imipenem/cilastatin, tramadol, enflurane, and metoclopramide are all known to increase the risk of seizures. Seizures often present with tonic-clonic activity, tongue biting, urinary incontinence, and a postictal phase.\n\nAnswer E: Tachycardia can be associated with medications such as atropine which reduce vagal tone on the heart increasing the heart rate. Medications that block the AV node would be expected to cause bradycardia.\n\nBullet Summary:\nAdenosine is a first-line pharmacological agent in supraventricular tachycardia and can cause hypotension, heart block, flushing, and bronchospasm.", "link": "https://step2.medbullets.com/testview?qid=106849"} {"question": "A 69-year-old man presents to his primary care physician for a general checkup. The patient works as a farmer and gardens in his spare time. He has a medical history of poorly managed type 2 diabetes mellitus and irritable bowel syndrome. His current medications include lisinopril, metformin, insulin, fiber supplements, and a multivitamin. On physical exam, there is a calm, obese gentleman. Inspection of the patient's scalp reveals the finding shown in Figure A. The patient is sent home and instructed to keep a blood glucose journal. Three months later, the patient returns with his glucose journal. In addition, he complains of a cut on his scalp that is not seeming to heal. The patient's glucose journal reveals an average blood glucose of 175 mg/dL. His temperature is 98.8\u00b0F (37.1\u00b0C), blood pressure is 134/86 mmHg, pulse is 80/min, and respirations are 13/min. On inspection of the patient's scalp, the finding in Figure B is noted. Which of the following is the most likely diagnosis?", "choicesA": "Actinic keratosis", "choicesB": "Basal cell carcinoma", "choicesC": "Melanoma", "choicesD": "Poor wound healing secondary to diabetes", "choicesE": "Squamous cell carcinoma", "answer_idx": "E", "answer": "Squamous cell carcinoma", "explanation": "This patient with a non-healing ulcer on the site of a previous actinic keratosis most likely has sustained progression of the lesion to squamous cell carcinoma.\n\nSquamous cell carcinoma (SCC) is a common dermatological malignancy. Patients can present with actinic keratosis, which is a pre-malignant lesion in sun-exposed areas. These lesions can progress to squamous cell carcinoma if risk factors such as sun exposure continue. The presentation of SCC is typically an ulcerative lesion in a sun-exposed area. A biopsy will demonstrate atypical keratinocytes and malignant cells, invasion into the dermis, and keratin \u201cpearls\u201d on histology. Treatment is with wide local surgical excision with histologic confirmation of negative margins.\n\nWaldman and Schmults present a review of the evidence regarding the epidemiology and treatment of squamous cell skin cancer. They discuss how the incidence of this disease continues to increase due to an aging population. They recommend local excision and treatment due to the risk of metastasis.\n\nFigure/Illustration A is a clinical photograph demonstrating a rough scaly patch on the skin (red circle). This finding is consistent with actinic keratosis, which is a pre-malignant lesion typically found on sun-exposed areas.\n\nFigure/Illustration B demonstrates an ulcerative lesion with wound formation on the skin (red circle). This finding is consistent with squamous cell carcinoma, a malignant lesion that can progress from actinic keratosis.\n\nIncorrect Answers:\nAnswer A: Actinic keratosis represents a pre-malignant condition that this patient initially presented with (as demonstrated in Figure A). His current presentation reflects SCC, a progression from actinic keratosis given the new ulceration and non-healing wound.\n\nAnswer B: Basal cell carcinoma presents with a shiny or pearly appearance, rather than the ulcerative appearance that is seen in SCC. This disease should be treated with surgical excision.\n\nAnswer C: Melanoma presents with a pigmented lesion that has an irregular border, an uneven shape, and areas that differ in pigmentation. Patients with melanoma should undergo surgical staging as this disease can become metastatic and should be treated with surgical excision.\n\nAnswer D: Poor wound healing secondary to diabetes would be possible if there was a history of trauma to the scalp. The history of actinic keratosis and the current ulcerative lesion, point more toward a diagnosis of SCC. Non-healing wound in patients with diabetes are usually located on the foot rather than on the scalp.\n\nBullet Summary:\nActinic keratosis is a pre-malignant lesion that occurs in sun-exposed areas that can progress to squamous cell carcinoma.", "link": "https://bit.ly/3CB9Cdn"} {"question": "A 25-year-old woman presents for a new patient physical exam. Aside from occasional shin splints, she has a relatively unremarkable medical history. She takes oral contraceptive pills and a multivitamin daily. All of her age appropriate immunizations are up to date. Her periods have been regular, occurring once every 28 to 30 days with normal flow. She is sexually active with 2 partners, who use condoms routinely. Her prior pap smear was performed last year and was normal. Her mother has diabetes and coronary artery disease, and her father passed away at age 45 after being diagnosed with colon cancer at age 40. Her grand-aunt underwent bilateral mastectomies after being diagnosed with breast cancer at age 60. Her temperature is 36.7\u00b0C (98\u00b0F), blood pressure is 115/80 mmHg, pulse is 65/min, respirations are 12/min, and oxygen saturation is 100% on room air. Her physical exam is within normal limits. Which of the following is the most appropriate recommendation for this patient?", "choicesA": "Colonoscopy in 5 years", "choicesB": "Colonoscopy in 10 years", "choicesC": "Mammogram now", "choicesD": "Pap smear in 5 years", "choicesE": "Pap smear with human papillomavirus (HPV) DNA testing now", "answer_idx": "A", "answer": "Colonoscopy in 5 years", "explanation": "This patient is presenting for a health evaluation with a family history significant for her father's death due to colon cancer at age 45. She should thus get a colonoscopy when she reaches the age 10 years prior to her father's age of diagnosis or in 5 years (at age 30).\n\nThe age threshold for colon cancer screening varies based on the patient's risk factors, including family history of colon cancer, inflammatory bowel disease, and/or genetic disorders. In the absence of significant risk factors, colonoscopies can be used for screening starting at age 45 and repeated every 10 years if results are normal. If there is family history of colon cancer (one or more first degree relatives, including parent, sibling, or child), colonoscopies should be recommended starting at age 40 or 10 years prior to the age of the family member's diagnosis, whichever is earlier. Polyps or grossly abnormal colon tissue may be biopsied for grading and guide further diagnostic imaging workup.\n\nAasma et al. review the clinical guidelines for colorectal cancer screening. They found that recent epidemiological data shows an increase in the incidence of colorectal cancer in individuals younger than 50 years. They recommend starting colonoscopy screening at 45 years of age in the average-risk adult population.\n\nIncorrect Answers:\nAnswer B: Colonoscopy in 10 years is reasonable, but it is not the earlier of the 2 age thresholds mentioned above.\n\nAnswer C: Pap smear with HPV DNA screening for cervical cancer is indicated every 5 years for women aged 30 to 65 who wish to extend the screening interval from the 3 year interval for pap smear alone.\n\nAnswer D: Pap smear alone is indicated every 3 years for women of ages 21 to 65.\n\nAnswer E: Mammogram to screen for breast cancer is recommended every 2 years for women aged 50 to 74.\n\nBullet Summary:\nFor patients with a family history of colon cancer, screening should start at age 40 or 10 years prior to the age of the family member's diagnosis, whichever is earlier.", "link": "https://step2.medbullets.com/testview?qid=109573"} {"question": "A 67-year-old man is seen on the surgical floor after a transplant procedure. The previous day, the patient had a renal transplant from a matched donor. He is currently recovering and doing well. The patient has a medical history of IV drug use, diabetes mellitus, oral cold sores, hypertension, renal failure, and dyslipidemia. The patient's current medications include lisinopril, atorvastain, insulin, and aspirin. Prior to the procedure, he was also on dialysis. The patient is started on cyclosporine. The patient successfully recovers over the next few days. His temperature is 98.5\u00b0F (36.9\u00b0C), blood pressure is 121/76 mmHg, pulse is 70/min, and respirations are 13/min. Which of the following medications should be started in this patient?", "choicesA": "Azithromycin", "choicesB": "Low dose acyclovir", "choicesC": "Penicillin", "choicesD": "TMP-SMX", "choicesE": "Valacyclovir", "answer_idx": "D", "answer": "TMP-SMX", "explanation": "This patient is presenting status post renal transplant and is currently on immunosuppressive therapy. This patient should be started on TMP-SMX as prophylaxis against Pneumocystis jiroveci pneumonia (PJP).\n\nAfter a transplant procedure, patients are started on immunosuppressive therapy in order to avoid rejection of the transplanted organ. The most important interventions for a patient after a transplant procedure are to receive all of their vaccinations and to receive TMP-SMX as prophylaxis against PJP. Other appropriate but less dire measures include giving ganciclovir as prophylaxis against cytomegalovirus (CMV). Patients should be closely monitored to avoid transplant rejection as well as infection.\n\nWeyant et al. review the evidence regarding the treatment of patients with PJP. They discuss how TMP-SMX has been the classic prophylaxis and treatment for this disease. They recommend considering echinocandins for patients who experience significant side effects.\n\nIncorrect Answers:\nAnswer A: Azithromycin is appropriate prophylaxis for patients with HIV with CD4 counts < 50. Patients on immunosuppressive drugs should receive some of the same prophylactic medications but they do not require azithromycin upon initiation of immunosuppressive medications.\n\nAnswer B: Low-dose acyclovir would be an inappropriate intervention in this patient after a transplant procedure. Ganciclovir and high-dose acyclovir are the appropriate options for prophylaxis of CMV.\n\nAnswer C: Penicillin can be used as prophylaxis for patients with rheumatic heart disease or asplenia; however, it would not be the most appropriate initial measure in a transplant patient.\n\nAnswer E: Valacyclovir is the treatment for herpes simplex virus (HSV). It is not routinely indicated as prophylaxis in transplant patients. HSV will present with vesicular rashes developing on the affected areas.\n\nBullet Summary:\nPatients on immunosuppressive therapy after a transplant should be vaccinated (pneumovax, hepatitis A and B, influenza, and tetanus-diphtheria) and started on TMP-SMX.", "link": "https://bit.ly/3s3wiAZ"} {"question": "An 83-year-old man is brought to the emergency department by his daughter due to confusion over the last 2 weeks. The patient initially became less talkative than usual before starting to confuse her with her sister. The patient also could not remember what day of the week it was and he was out of groceries and likely had been for several days when visited. While she was talking to him, he fell asleep and was difficult to rouse. The patient is unable to provide a clear history. He has a medical history of hypertension, glaucoma, and osteoarthritis in his left knee that requires him to walk with a cane. He takes hydrochlorothiazide, timolol eyedrops, and occasional acetaminophen. His temperature is 98.5\u00b0 F (36.9\u00b0 C), blood pressure is 122/74 mmHg, pulse is 75/min, and respirations are 14/min. The patient is lethargic but arousable, oriented only to person, and disheveled. Neurologic exam reveals intact cranial nerves, 1+ deep tendon reflexes throughout, and 4/5 strength in all extremities. There is a 3 x 5 cm green-blue area of ecchymosis on the lateral aspect of his left knee. A CT of the head without contrast is performed and shown in Figure A. Which of the following factors plays the earliest role in the progression towards this patient\u2019s presentation?", "choicesA": "Aneurysmal rupture", "choicesB": "Cerebral atrophy", "choicesC": "Elderly abuse", "choicesD": "Hypertension", "choicesE": "Increased cerebrospinal fluid", "answer_idx": "B", "answer": "Cerebral atrophy", "explanation": "This patient is an elderly man with progressive confusion over weeks, a nonfocal neurologic exam, and a CT of the head showing a hypodense crescent, most consistent with chronic subdural hematoma (SDH). Cerebral atrophy plays the earliest role in this presentation, resulting in exposure of the bridging veins and subsequent rupture with even minor trauma.\n\nChronic subdural hematomas present gradually, most often with headaches, somnolence, light-headedness, cognitive impairment, and rarely, seizures. Typically, the neurologic exam is largely normal but rarely focal deficits such as contralateral hemiparesis from compression of the cortex underlying the hematoma or ipsilateral hemiparesis from the lateral displacement of the opposite cortex may occur. Chronic SDH most often occurs in the setting of cerebral atrophy related to increased age, chronic alcohol use, and prior traumatic brain injury. Exposure of the bridging veins allows even minor trauma to precipitate subdural bleeding. A non-contrast CT of the head is indicated in any elderly patient with decreased mentation and concern for chronic SDH. Since old blood appears hypodense, imaging will allow for the determination of whether the bleed is acute or chronic. Supportive care is usually sufficient for treatment but surgical evacuation of the hematoma can be considered in patients with focal deficits or an expanding hematoma.\n\nMehta et al. review the evidence regarding the diagnosis and management of chronic subdural hematomas in elderly patients. They discuss how crescentic layering of fluid in the subdural space on a non-contrast computed tomography scan is diagnostic of this condition. They recommend close follow-up as many patients will develop a recurrence of this condition.\n\nFigure/Illustration A shows a non-contrast CT scan of the head with a hypodense crescent left side (red circle). This finding is consistent with a chronic subdural hematoma.\n\nIncorrect Answers:\nAnswer A: Aneurysmal rupture is an extremely uncommon cause of SDH and typically causes subarachnoid hemorrhage instead. It often presents with \u201cthe worst headache of my life\u201d or \u201cthunderclap headache,\u201d and would show hyperdensity in the subarachnoid space on head CT. Surgical clipping or endovascular treatment can prevent rebleeding.\n\nAnswer C: Elder abuse can be subtle and should always be considered when an elderly patient presents with confusion, bruising, or dishevelment. In children, abuse is an important cause of SDH, which may occur in the setting of \u201cshaken baby syndrome.\u201d However, abuse is not especially associated with chronic SDH in the elderly population, and there is another plausible reason for this patient\u2019s presentation (trauma from a fall as evidenced by the resolving ecchymosis on his knee).\n\nAnswer D: Hypertension is a major risk factor for stroke, which typically presents with focal neurologic deficits such as hemiparesis, aphasia, or sensory loss. Although this patient does have a history of hypertension, he is normotensive in the emergency room and has non-focal neurologic decline that has been slowly progressive, which is less consistent with stroke. Furthermore, in contrast to this patient\u2019s CT scan, acute ischemic stroke would likely present with a normal head CT and prior infarct would cause a hypodensity in the parenchyma itself. Treatment is with risk factor optimization when a patient presents outside the window for thrombolytic agents.\n\nAnswer E: Increased CSF due to poor reabsorption by arachnoid granules is the cause of normal pressure hydrocephalus (NPH). Patients typically present with the \u201cwet, wacky, and wobbly\u201d triad \u2013 urinary incontinence, mental status changes, and magnetic gait. Head CT would show dilated ventricles and management includes CSF shunting.\n\nBullet Summary:\nChronic subdural hematomas may occur in elderly patients due to cerebral atrophy and subsequent exposure of bridging veins that may tear with minor trauma.", "link": "https://step2.medbullets.com/testview?qid=108758"} {"question": "A 45-year-old African American woman presents to her primary care physician for not feeling well. She states she has had a cough for the past week. She also has abdominal pain and trouble focusing that has been worsening and has lost 5 pounds recently. Her gastroesophageal reflux disease (GERD) has been poorly controlled recently. The patient is a non-smoker and has a history of GERD for which she takes antacids. Her temperature is 98.6\u00b0F (37\u00b0C), blood pressure is 115/71 mmHg, pulse is 72/min, and respirations are 12/min. A physical exam is within normal limits. Laboratory studies are ordered and are below:\n\nSerum:\nNa+: 139 mEq/L\nK+: 4.1 mEq/L\nCl-: 101 mEq/L\nHCO3-: 24 mEq/L\nUrea nitrogen: 12 mg/dL\nGlucose: 70 mg/dL\nCreatinine: 0.9 mg/dL\nCa2+: 12.5 mg/dL\nAlkaline phosphatase: 150 U/L\nPhosphorus: 2.0 mg/dL\n\nUrine:\nColor: amber\nNitrites: negative\nSodium: 5 mmol/24 hours\nRed blood cells: 0/hpf\n\nWhich of the following is the most likely explanation of this patient's current presentation?", "choicesA": "Antacid overuse", "choicesB": "Increased 1,25-dihydroxyvitamin D", "choicesC": "Increased parathyroid hormone", "choicesD": "Malignancy", "choicesE": "Viral illness", "answer_idx": "C", "answer": "Increased parathyroid hormone", "explanation": "This patient is presenting with hypercalcemia and low phosphate levels suggesting a diagnosis of hyperparathyroidism.\n\nHyperparathyroidism can cause hypercalcemia which typically presents with \"bones, stones, moans, and groans,\" which corresponds to symptoms of bone pain (increased bone resorption), kidney stones, GI pain, and change in mental status/trouble focusing. The increased PTH results in renal phosphate wasting, increased production of 1,25-dihydroxy vitamin D, and increased renal calcium reabsorption. It also results in increased osteoclast activity resulting in increased bone breakdown. This will lead to laboratory changes of hypercalcemia and hypophosphatemia.\n\nTaniegra reviews the evidence regarding hyperparathyroidism. She discusses how this disease is usually discovered asymptomatically with routine blood work. She recommends surgical treatment with intractable cases as success rates are high.\n\nFigure A is a chest radiograph that is within normal limits. No masses are seen which suggests that malignancy is a less likely diagnosis.\n\nIncorrect Answers:\nAnswer A: Antacid overuse for this patient's GERD can lead to a milk-alkali syndrome which presents with hypercalcemia and metabolic alkalosis. This patient does not have metabolic alkalosis. Treatment is with the discontinuation of the offending agent.\n\nAnswer B: Increased 1,25-dihydroxy vitamin D occurs in sarcoidosis which presents with cough, fatigue, and weight loss in an African American woman. Increased vitamin D by itself will result in hypercalcemia, and hyperphosphatemia rather than hypophosphatemia. Treatment is supportive and/or with corticosteroids.\n\nAnswer D: Malignancy (lung or renal cancer) is possible given this patient's presentation. Hypercalcemia of malignancy is a common feature with PTH-related protein effects; however, the patient's normal urinalysis and normal chest radiograph as well as non-smoking history point more toward a diagnosis of increased PTH.\n\nAnswer E: Viral illness is possible given this patient's systemic symptoms; however, it does not explain this patient's hypercalcemia and hypophosphatemia. Treatment of seasonal viruses is generally supportive though specific viruses such as influenza have agents such as oseltamivir that may be helpful.\n\nBullet Summary:\nHyperparathyroidism presents with \"bones, stones, moans, and groans,\" as well as laboratory findings of hypercalcemia and hypophosphatemia.", "link": "https://step2.medbullets.com/testview?qid=108995"} {"question": "A 57-year-old man is brought to a neurologist by his wife for abnormal behavior. He was normal until around 3 months ago when he started forgetting important meetings and misplacing items around the house. One month ago, the patient was fired from his job as a software engineer after attempting to grope a coworker. The patient expresses frustration that he is no longer able to concentrate enough to enjoy his hobby of solving puzzles. His medical history is significant for gastroesophageal reflux disease for which he takes omeprazole. He has no family history of neurologic disease. His temperature is 98.6\u00b0F (37.0\u00b0C), blood pressure is 120/75 mmHg, pulse is 70/min, and respirations are 16/min. During the examination, the patient\u2019s cell phone rings unexpectedly causing the patient to suddenly jerk his arms and legs. Further diagnostic investigation is performed which reveals an elevated 14-3-3 protein on cerebrospinal fluid analysis. Which of the following is the most likely diagnosis?", "choicesA": "Alzheimer disease", "choicesB": "Creutzfeldt-Jacob disease", "choicesC": "Dementia with Lewy bodies", "choicesD": "Frontotemporal dementia", "choicesE": "Subcortical leukoencephalopathy", "answer_idx": "B", "answer": "Creutzfeldt-Jacob disease", "explanation": "This patient with rapidly progressive dementia over 3 months, declining executive function, impaired concentration, decreased memory, startle myoclonus (sudden jerking of extremities in response to unexpected stimulus), and elevated 14-3-3 protein levels on cerebrospinal fluid (CSF) analysis most likely has Creutzfeldt-Jakob disease (CJD).\n\nCJD is a human prion disease in which the normal prion protein (PrPc) converts to a beta-pleated sheet disease-associated form (PrPsc) which is resistant to proteases. CJD is usually spontaneous. Rarely, CJD may be familial or acquired (from iatrogenic transmission or dietary exposure). Aggregates of PrPsc lead to spongiform vacuolation. Clinically, this manifests with rapidly progressive mental deterioration with prominent executive dysfunction (declining concentration, judgment, and memory) and startle myoclonus. Further diagnostic investigation may reveal 14-3-3 protein on CSF analysis and synchronous sharp wave complexes on electroencephalogram (EEG). CJD usually leads to death within 1 year. There is no known effective treatment for this disease.\n\nUttley et al. review the diagnosis, clinical presentation, and epidemiology of Creutzfeldt-Jakob disease (CJD). They discuss how CJD in a global context is increasing in prevalence and how incubation periods as long as 40 years have been observed for iatrogenic cases. They recommend heightened awareness of this disease in order to mitigate iatrogenic transmission.\n\nIncorrect Answers:\nAnswer A: Alzheimer disease is characterized by gradual cognitive decline with difficulty completing activities of daily living. Impaired executive function early in the disease course, rapidly progressive disease, elevated 14-3-3 protein on CSF analysis, and startle myoclonus are not found in Alzheimer disease.\n\nAnswer C: Lewy body dementia (LBD) presents with alterations in consciousness, disorganized speech, visual hallucinations, extrapyramidal symptoms, and early compromise of executive function. Although this patient\u2019s dementia is characterized by early compromise of executive function, the absence of fluctuating levels of consciousness, absence of extrapyramidal symptoms, and presence of startle myoclonus make CJD more likely than LBD.\n\nAnswer D: The behavioral variant of frontotemporal dementia (FTD) is associated with early-onset personality and behavioral changes, and it may be associated with extrapyramidal symptoms. Memory is not usually affected until late in the disease course.\n\nAnswer E: Subcortical leukoencephalopathy (Binswanger disease) is a type of small vessel vascular disease that presents with prominent subcortical dementia. This manifests clinically as slowed mental processing, impaired cognition, apathy, and depression. This disease almost always presents in the context of chronic hypertension and is usually seen in older patients.\n\nBullet Summary:\nCreutzfeldt-Jacob disease is a fatal prion disease that presents with sudden-onset and rapidly progressive dementia and startle myoclonus or seizures.", "link": "https://bit.ly/3My1xLG"} {"question": "A 24-year-old man presents to the emergency room after a motor vehicle collision. He was the driver in a head-on collision and was not wearing a seatbelt. The patient is acutely intoxicated and is attempting to attack the nurses. Soon after presentation, he is sedated, paralyzed, and intubated. His medical history is unknown. His temperature is 97.6\u00b0F (36.4\u00b0C), blood pressure is 74/34 mmHg, pulse is 180/min, respirations are 32/min, and oxygen saturation is 98% on room air. Physical exam is notable for jugular venous distension. An electrocardiogram (ECG) is performed as seen in Figure A. Which of the following is the most appropriate next step in management?", "choicesA": "Needle thoracostomy", "choicesB": "Packed red blood cells", "choicesC": "Pericardial window", "choicesD": "Pericardiocentesis", "choicesE": "Tube thoracostomy", "answer_idx": "D", "answer": "Pericardiocentesis", "explanation": "This patient is presenting after trauma with hypotension, tachycardia, jugular venous distension, and electrical alternans on ECG, which is concerning for cardiac tamponade. The most important initial step in management is pericardiocentesis.\n\nCardiac tamponade can occur after trauma, infections, or malignancy. It can occur if there is an accumulation of fluid in the pericardial sac with inadequate time for the pericardial sac to stretch and for the heart to adapt to the compression that causes diastolic dysfunction. Patients will present with hypotension, tachycardia, jugular venous distension, muffled heart sounds, and pulsus paradoxus (blood pressure drop > 10 mmHg with inspiration). The diagnosis can be supported with ultrasonography (in the clinical setting a point of care ultrasound will often be performed first); however, the initial step in management is a pericardiocentesis which will stabilize the patient. Subsequent management requires the placement of a pericardial drain or window, as well as treatment of the underlying cause.\n\nPrabhakar et al. discuss pericardial decompression syndrome, an infrequent complication of pericardiocentesis. The authors find that pericardial decompression syndrome presents with paradoxical hemodynamic instability and/or pulmonary edema following pericardiocentesis, especially in patients with underlying myocardial dysfunction. The authors recommend careful monitoring of patients after pericardiocentesis for the development of this complication.\n\nFigure/Illustration A is an ECG demonstrating electrical alternans. Note the alternating large QRS complexes (blue arrows) and small QRS complexes (green arrows) which are suggestive of the heart swinging in a fluid-filled pericardium.\n\nIncorrect Answers:\nAnswers 1 & 5: Needle thoracostomy followed by tube thoracostomy is the appropriate management of a tension pneumothorax. A tension pneumothorax would also present with hypotension, tachycardia, and jugular venous distension; however, it would not present with muffled heart sounds and electrical alternans on ECG. On exam, there would be an absence of unilateral breath sounds and the presence of tracheal deviation.\n\nAnswer B: Packed red blood cells are appropriate management of a hypotensive and tachycardic trauma patient when it is suspected that their unstable vitals are secondary to hemorrhage. In this patient, there is conclusive evidence (jugular venous distension and electrical alternans) that this patient has cardiac tamponade.\n\nAnswer C: Pericardial window is the appropriate long-term treatment of recurrent pericardial effusions as it allows for a permanent conduit for the drainage of fluid. An acutely unstable patient should first be stabilized with a pericardiocentesis before going to the operating room for this procedure.\n\nBullet Summary:\nThe most important initial step in the management of cardiac tamponade in a hemodynamically unstable patient is pericardiocentesis.", "link": "https://step2.medbullets.com/testview?qid=215186"} {"question": "A 68-year-old woman presents to her primary care provider for dizziness. She often feels lightheaded and like she may \u201cpass out\u201d but never actually loses consciousness. She denies dyspnea on exertion or orthopnea. The patient had a myocardial infarction 6 months ago, and she now has multiple drug-eluting stents. Her ECG upon discharge from that hospitalization was normal. The patient has been taking all of her medications as instructed. Her medical history is otherwise significant for rheumatoid arthritis and migraines. Her home medications include methotrexate, atorvastatin, clopidogrel, and aspirin. Her temperature is 98.6\u00b0F (37\u00b0C), blood pressure is 118/75 mmHg, pulse is 92/min, and respirations are 12/min. On physical exam, she has an irregular heartbeat with normal cardiac sounds. Her abdomen is non-distended and she has mild lower extremity edema. Her current ECG can be seen in Figure A. Which component of the cardiac conduction system is most likely affected in this patient?", "choicesA": "Atrioventricular node", "choicesB": "Bundle branches", "choicesC": "Bundle of His", "choicesD": "Purkinje fibers", "choicesE": "Sinoatrial node", "answer_idx": "A", "answer": "Atrioventricular node", "explanation": "This patient who presents with lightheadedness and is found to have progressive prolongation of the PR interval on ECG most likely has Mobitz type 1 (Wenckebach) 2nd-degree heart block. Mobitz type 1 heart block is usually caused by atrioventricular node dysfunction.\n\nSecond degree heart block can be divided into Mobitz type 1 and Mobitz type 2 patterns. The Mobitz type 1 pattern is usually a result of dysfunction at the atrioventricular (AV) node, whereas the Mobitz type 2 pattern is usually caused by dysfunction below the AV node in the His-Purkinje system. In Mobitz type 1 2nd-degree heart block, the PR interval becomes progressively longer with each beat before the dropped QRS complex. In Mobitz type 2 2nd-degree heart block, the PR intervals are consistent in length and the dropped QRS complexes occur at regular intervals. The easiest way to determine whether the PR interval is getting longer is to compare the PR interval just before the dropped beat to the PR interval immediately following the dropped beat. Patients with Mobitz type 1 blocks generally do not require treatment unless symptomatic whereas those with Mobitz type 2 blocks generally require pacemaker placement.\n\nClark and Prystowsky review the pathophysiology of heart block. They discuss how a type 1 2nd-degree heart block is caused by dysfunction of the atrioventricular node. They recommend symptomatic treatment of this condition.\n\nFigure/Illustration A is an ECG that demonstrates the progressive prolongation of the PR interval (blue lines) until a beat is dropped. This progressive prolongation is characteristically seen in Mobitz type 1 (Wenckebach) heart block.\n\nIncorrect Answers:\nAnswer B: A bundle branch block would produce a classic right or left bundle branch block pattern on ECG (RSR') described colloquially as \"bunny ears.\" Bundle branch blocks are defined in part by a QRS duration > 120 ms and can be seen in lead V1. Treatment is with control of underlying risk factors such as hypertension.\n\nAnswer C: The bundle of His conducts the electrical impulse from the atrioventricular node to the ventricles. Dysfunction at the level of the bundle of His would more likely cause Mobitz type 2 2nd-degree heart block or 3rd-degree heart block rather than the Mobitz type 1 heart block seen in this patient. Treatment is with pacemaker placement.\n\nAnswer D: Purkinje fibers rapidly conduct along the ventricular walls to create synchronized contraction of the ventricles. Aberrant conduction of the Purkinje fibers leads to premature ventricular complexes. Treatment is with beta-blockers if patients are symptomatic.\n\nAnswer E: The sinoatrial node is the heart\u2019s primary pacemaker. Dysfunction at this level can lead to sick sinus syndrome. Sick sinus syndrome can manifest as many types of arrhythmias, including bradycardia-tachycardia syndrome, in which there is alternating bradycardia with paroxysmal tachycardia. Sinoatrial node dysfunction can be treated with pacemaker placement.\n\nBullet Summary:\nMobitz type 1 (Wenckebach) 2nd-degree heart block is characterized by progressive prolongation of the PR interval before a dropped QRS complex and is usually caused by dysfunction of the atrioventricular node.", "link": "https://bit.ly/3P1O8fD"} {"question": "A 69-year-old man presents to his primary care provider for a general checkup. He has a medical history of type 2 diabetes mellitus, hypertension, depression, obesity, and a myocardial infarction 7 years ago. The patient's prescribed medications are metoprolol, aspirin, lisinopril, hydrochlorothiazide, fluoxetine, metformin, and insulin. The patient has not been filling his prescriptions regularly and can not remember what medications he has been taking. His temperature is 99.5\u00b0F (37.5\u00b0C), pulse is 96/min, blood pressure is 180/120 mmHg, respirations are 18/min, and oxygen saturation is 97% on room air. Laboratory values are obtained and the results are shown below:\n\nSerum:\nNa+: 139 mEq/L\nK+: 4.3 mEq/L\nCl-: 100 mEq/L\nHCO3-: 24 mEq/L\nBUN: 7 mg/dL\nGlucose: 170 mg/dL\nCreatinine: 1.2 mg/dL\n\nWhich of the following cardiac findings would be expected on physical exam?", "choicesA": "Fixed splitting of S1 and S2", "choicesB": "Heart sound after S2", "choicesC": "Heart sound before S1", "choicesD": "Holosystolic murmur at the apex", "choicesE": "Normal S1 and S2", "answer_idx": "C", "answer": "Heart sound before S1", "explanation": "This patient who presents with poorly managed hypertension over a long period of time most likely has compensatory hypertrophy of the heart. The most likely heart sound in this patient is an S4 gallop that occurs immediately before S1.\n\nChronic hypertension that is poorly controlled can lead to concentric hypertrophy of the heart. This adaptive mechanism creates a stiff and non-compliant left ventricle. When blood enters this stiff ventricle, the typical sound heard is an S4 heart sound which is heard right before S1. This sound represents the atrial kick that pushes blood against a non-compliant left ventricle in the last phase of diastole. Treatment is by meticulous control of the underlying cardiac pathophysiology such as hypertension.\n\nKino et al studied the relationship between the S1 and S4 heart sounds in aging patients. They found that an S4 heart sound can be missed in cases of prominent S1 splitting. They recommend studying heart sounds in order to not accidentally miss an S4 finding.\n\nIncorrect Answers:\nAnswer A: Fixed splitting of S1 and S2 describes an atrial septal defect. The atrial septic defect allows blood to communicate between both sides of the heart and equalizes atrial pressures. Asymptomatic patients may not need treatment but surgical repair of the defect should be performed in symptomatic cases.\n\nAnswer B: A heart sound after S2 suggests a S3 heart sound which would be found in fluid-overloaded states such as heart failure. This post-systolic gallop is the result of incomplete emptying of the ventricle prior to the start of diastole so that the blood from the atria falls against an already full chamber. Treatment is with diuresis and treatment of the underlying heart failure.\n\nAnswer D: A holosystolic murmur heard at the apex describes mitral valve regurgitation. This would be possible if a papillary muscle had ruptured in an acute myocardial infarction. Treatment is with mitral valve replacement in symptomatic patients.\n\nAnswer E: A normal S1 and S2 is possible but is less likely given this patient's long-standing history of poorly managed hypertension. Hypertension usually causes concentric hypertrophy of the ventricle and thereby leads to increased resistance to diastolic filling.\n\nBullet Summary:\nAn S4 heart sound is created by blood entering a stiff and non-compliant left ventricle and is often heard in patients with long-standing hypertension.", "link": "https://step2.medbullets.com/testview?qid=109092"} {"question": "A 66-year-old man presents to the clinic with a 6-week history of bloody stools. He describes the blood as bright red wisps that float on the surface of the stool. He has no pain with bowel movements but notes on occasion he may strain. He does not endorse fatigue, fever, constipation, diarrhea, nausea, vomiting, abdominal pain, weight loss, or dyspnea. His last colonoscopy was 3 years prior to presentation and was benign. He has a medical history significant for hypertension that is well-controlled with lisinopril. He had an uncomplicated appendectomy when he was 23 years old. He has no family history of gastrointestinal malignancy. His temperature is 98.6\u00b0F (37.0\u00b0C), blood pressure is 140/90 mmHg, pulse is 76/min, and respirations are 16/min. On physical exam, the patient's abdomen is soft, non-distended, and non-tender with normal bowel sounds and without organomegaly or masses. The anus is patent without fissures, hemorrhoids, or skin tags. Which of the following is the most appropriate next step in management?", "choicesA": "Colonoscopy", "choicesB": "Computerized tomography scan of abdomen and pelvis", "choicesC": "Increase fiber in diet", "choicesD": "Observation", "choicesE": "Sigmoidoscopy", "answer_idx": "A", "answer": "Colonoscopy", "explanation": "This patient is presenting with painless bright red blood per rectum. Because he is over 50 years old and having rectal bleeding, he should be evaluated for malignancy with a colonoscopy.\n\nBright red blood per rectum has a broad differential diagnosis including anal fissures, hemorrhoids, angiodysplasia, inflammatory bowel disease, diverticular bleed, infection, polyps, and colon cancer. However, as patients age, their risk for malignancy increases significantly. Therefore, stable patients older than 50 years of age with rectal bleeding, regardless of their symptoms or family history, should receive a colonoscopy to rule out malignancy. Patients younger than 50 with warning signs of colon cancer (abnormal bowel habits, weight loss, family history of colon cancer, iron deficiency anemia) should also be evaluated with a colonoscopy. Often, colonoscopy is performed routinely at the age of 50 or 10 years prior to the age of presentation for malignancy in family members (whichever is sooner). Of note, even if there is an apparent cause for a lower GI bleed in patients 50 or older (such as hemorrhoids) colonoscopy is still indicated to rule out malignancy.\n\nBarry et al. note the guidelines for colon cancer screening. They note the different age cutoffs. The USPSTF recommends screening for colorectal cancer in all adults aged 50-75 years.\n\nIncorrect Answers:\nAnswer B: Computerized tomography scan of the abdomen and pelvis is incorrect because colonoscopy is the gold standard for diagnosis of suspected colon cancer. A CT scan may detect larger masses and a CTA may be performed if there is active bleeding and concern for an arterial source of bleeding that can be intervened upon.\n\nAnswer C: Increasing fiber in the diet may help patients with constipation or hemorrhoidal bleeding. Constipation presents with straining to have bowel movements and hard stools. However, because of this patient's age, he needs a colonoscopy to rule out malignancy.\n\nAnswer D: Observation is not appropriate at this patient's age. Patients greater than 50 years old with bright red blood per rectum should receive a colonoscopy to rule out malignancy despite any recent negative colonoscopy results.\n\nAnswer E: Sigmoidoscopy is less preferred to colonoscopy, as a smaller portion of the colon would be visualized, and colonic polyps or malignancies may be missed. For this reason, it is not the preferred study in this patient.\n\nBullet Summary:\nPatients with bright red bleeding per rectum over 50 years old or with warning signs of colonic malignancy (weight loss, abnormal bowel habits, positive family history, or anemia) should receive a colonoscopy.", "link": "https://step2.medbullets.com/testview?qid=216275"} {"question": "A 6-week-old girl presents with her parents to the pediatrician for blood-streaked diapers. The patient is breastfed every 2-3 hours and voids 10-12 times daily. The patient previously had several soft stools per day that ranged in color from mustard yellow to dark green. Over the past 2 days, the patient\u2019s stools have become looser and streaked with blood. The patient has also regurgitated several of her feedings in the last few days. Neither the vomit nor bloody stools have seemed to bother the patient. The patient\u2019s weight continues to trend along the 50th percentile. The patient\u2019s temperature is 98.4\u00b0F (36.9\u00b0C), blood pressure is 66/51 mmHg, pulse is 127/min, and respirations are 26/min. On physical exam, the patient appears well nourished and is cooing. Her abdomen is soft and non-tender. Physical exam reveals the finding seen in Figure A. Which of the following is the most likely etiology of this patient\u2019s presentation?", "choicesA": "Congenital enzyme deficiency", "choicesB": "Hypertrophy of sphincter muscle", "choicesC": "Non-IgE-mediated immunologic reaction", "choicesD": "Passage of gastric contents into esophagus", "choicesE": "Telescoping of the intestine", "answer_idx": "C", "answer": "Non-IgE-mediated immunologic reaction", "explanation": "This patient presents with new regurgitation, blood-streaked stools, and eczema, which is consistent with proctocolitis secondary to milk-protein allergy. This type of delayed onset milk-protein allergy is a non-IgE mediated immunologic reaction.\n\nAllergy to cow\u2019s milk is the most common food allergy in children and encompasses both IgE-mediated and non-IgE-mediated reactions. IgE-mediated reactions occur minutes to 2 hours after ingestion of the allergen and range from mild symptoms to anaphylaxis. Milk protein-induced enterocolitis and proctocolitis are non-IgE-mediated reactions that present by 6 months of life with regurgitation or vomiting and bloody stools. Management of milk protein-induced enterocolitis and proctocolitis involves the elimination of cow\u2019s milk and soy from the mother\u2019s diet if the infant is breastfed or switching to a hydrolyzed formula if the infant is formula fed.\n\nGiannetti et al. review the evidence regarding the diagnosis and treatment of milk protein allergy in infants. They discuss how the selective elimination of certain proteins from the diet is usually effective. They recommend considering oral immunotherapy as an adjuvant in the treatment of this disease.\n\nFigure/Illustration A is a clinical photograph demonstrating the physical exam finding of eczema (red circle), which is commonly found in infants with an allergy to milk protein.\n\nIncorrect Answers:\nAnswer A: Congenital enzyme deficiency describes lactose intolerance, in which there is an absence of lactase activity in the small intestine. Lactose intolerance classically presents with abdominal pain, bloating, flatulence, and diarrhea. Treatment is with diet modification.\n\nAnswer C: Hypertrophy of the sphincter muscle describes the pathogenesis of pyloric stenosis, which presents in patients between 3- and 6-weeks of age with non-bilious, projectile vomiting. Treatment is with surgical pyloromyotomy.\n\nAnswer D: Passage of gastric contents into the esophagus describes gastroesophageal reflux, which may present with regurgitation or vomiting. Treatment is with positioning but can include proton pump inhibitors in refractory cases.\n\nAnswer E: Telescoping of the intestine describes intussusception, which presents with sudden-onset severe, crampy, abdominal pain with non-bilious vomiting and \u201ccurrant jelly\u201d stools. Intussusception is rare before 3 months of age and presents with intermittent episodes of pain. Initial treatment is with air or contrast enema.\n\nBullet Summary:\nMilk-protein allergy presents in infants between 2- to 8-weeks of age with regurgitation or vomiting, blood-streaked stools, and eczema, and management involves the elimination of cow\u2019s milk and soy from the infant\u2019s diet.", "link": "https://step2.medbullets.com/testview?qid=109255"} {"question": "A 22-year-old man presents to the emergency department with difficulty breathing. He has experienced similar symptoms before. Today, he was hit in the face with a football, followed by significant swelling of his lips and tongue. The patient speaks with a muffled voice. His temperature is 98.1\u00b0F (36.7\u00b0C), blood pressure is 120/83 mmHg, pulse is 80/min, respirations are 16/min, and oxygen saturation is 98% on room air. Physical exam reveals swollen lips and a protruding, enlarged tongue. His breath sounds are clear. He demonstrates no rash and his abdomen is soft and nontender. Which of the following is the most effective therapy for this patient?", "choicesA": "Dexamethasone", "choicesB": "Diphenhydramine", "choicesC": "Epinephrine", "choicesD": "Fresh frozen plasma", "choicesE": "Tranexamic acid", "answer_idx": "D", "answer": "Fresh frozen plasma", "explanation": "This patient is presenting with recurrent episodes of significant swelling of his lips and tongue with minor trauma, which are concerning for hereditary angioedema. Fresh frozen plasma is the most effective therapy for hereditary angioedema of the options listed.\n\nAngioedema is a potentially life-threatening condition that presents with severe edema of the gastrointestinal and respiratory tracts. It can be caused by antigens, medications, and trauma or may occur spontaneously. It often presents with notable edema of the lips and tongue and may threaten the airway. There are 3 subtypes. Hereditary and acquired angioedema occur secondary to a deficiency of C1-esterase inhibitor. As a result, the most effective therapy is to administer fresh frozen plasma which has C1-esterase inhibitor. On the other hand, drug-induced angioedema (often from angiotensin-converting enzyme inhibitors and angiotensin receptor blockers) does not respond well to C1-esterase inhibitor replacement and supportive therapy; discontinuing the offending agent is the mainstay of therapy. Other effective but costly treatments that can be used in hereditary angioedema include: icatibant, ecallantide, and C1 inhibitor.\n\nPaige and Anderson note the management of hereditary angioedema. They review the different treatments and discuss C1-esterase inhibitor replacement. They recommend this treatment for acute episodes.\n\nIncorrect Answers:\nAnswer A: Dexamethasone and other steroids can be given in acute allergic reactions and may reduce the recurrence of symptoms but are unlikely to be effective in angioedema, nor are they the most effective therapy. Steroids can take up to 4 hours to begin having any effect.\n\nAnswer B: Diphenhydramine is an antihistamine that could be given for anaphylaxis or urticaria and works immediately; however, it is not the mainstay of therapy for angioedema nor is it the most effective therapy. It is the mainstay of therapy for urticaria. It can also be given in anaphylaxis.\n\nAnswer C: Epinephrine is the treatment of choice for anaphylaxis which presents with urticaria, upper airway swelling, hypotension, nausea/vomiting, and wheezing. It is often given in angioedema but has limited efficacy when compared to fresh frozen plasma.\n\nAnswer E: Tranexamic acid is effective in treating angioedema and current evidence supports its use; however, the mainstay of therapy is still fresh frozen plasma. Tranexamic acid is also used in hemorrhagic shock to stop fibrinolysis and reduce bleeding.\n\nBullet Summary:\nA possible treatment in hereditary or acquired angioedema is the administration of fresh frozen plasma to replace C1-esterase inhibitor.", "link": "https://step2.medbullets.com/testview?qid=216588"}